Paediatrics Flashcards

1
Q

Define appendicitis

A

Acute appendicitis is an acute inflammation of the vermiform appendix, most likely due to obstruction of the lumen of the appendix (by faecolith, normal stool, infective agents, or lymphoid hyperplasia).

How well did you know this?
1
Not at all
2
3
4
5
Perfectly
2
Q

Explain RFs for appendicitis

A

<6m breast feeding
Low fibre
Improved hygeine?
Passive smoking

How well did you know this?
1
Not at all
2
3
4
5
Perfectly
3
Q

Aetiology of appendicitis

A

Obstruction of the lumen of the appendix is the main cause of acute appendicitis. Faecolith (a hard mass of faecal matter), normal stool, or lymphoid hyperplasia are the main causes for obstruction. Faecolith alone causes simple appendicitis in 40%, gangrenous non-perforated appendicitis in 65%, and perforated appendicitis in 90% of cases.

There is evidence suggesting a neuroimmune aetiology in some cases, but this is still being investigated.

How well did you know this?
1
Not at all
2
3
4
5
Perfectly
4
Q

Sx of appendicitis

A

Abdominal pain
- Mid -> RIF
- Worse on movement
Anorexia

RIF tenderness
N+V
Fever
Dec bowel sounds
Tachycardia
Fetor 

ROSVINGs
- Pressing on LIF illicits pain in RIF

PSOAS
- Extending the right thigh on left lateral position elicits pain in right lower quadrant.

OBTURATOR
- Pain is elicited at right lower quadrant of abdomen by internal rotation of the flexed right thigh.

How well did you know this?
1
Not at all
2
3
4
5
Perfectly
5
Q

What are the relevant Ix for appendicitis

A

FBC -> inc WCC
ABDO/PELV CT - abnormal appendix (diameter >6 mm) identified or calcified appendicolith seen in association with peri-appendiceal inflammation

URINE PREG TEST NEG

Abdo USS - aperistaltic or non-compressible structure with outer diameter >6 mm

NEGATIVE URINALYSIS

How well did you know this?
1
Not at all
2
3
4
5
Perfectly
6
Q

Management of appendicitis

A

Open or LAP Appendectomy
Supportive care - IV fluids

Given for 24 hours for uncomplicated appendicitis: IV ABx - CEFOXITIN 1-2g prior to surgery, 1-2g 8hrly post surgery

PERFORATION/ABSCESS:

  • Maintenance of BP/pulse
  • Begin ABx immediately
  • Abscess - may need drainage first THEN INTERVAL appendectomy
How well did you know this?
1
Not at all
2
3
4
5
Perfectly
7
Q

Complications of appendicitis

A
Perforation
Peritonitis
Appendicular mass
Appendicular abscess
Surgical wound infection
How well did you know this?
1
Not at all
2
3
4
5
Perfectly
8
Q

Prognosis of appendicitis

A

If patients are treated in a timely fashion, the prognosis is good. Wound infection and intra-abdominal abscess are potential complications associated with appendectomy. Laparascopic appendectomy has been shown to decrease the incidence of overall complications.

How well did you know this?
1
Not at all
2
3
4
5
Perfectly
9
Q

Define ADHD

A

ADHD is a triad of inattention, hyperactivity, and impulsivity.

A key element of the definition is functional impairment across TWO OR MORE domains, most often in school and at home.

ADHD can limit academic, interpersonal, and occupational success and can also lead to greater risk-taking and accidents.

In addition, patients with ADHD are more likely to have co-existing psychiatric disorders such as oppositional defiant disorder (ODD), conduct disorder, substance abuse, and possibly mood disorders, such as depression and mania.

How well did you know this?
1
Not at all
2
3
4
5
Perfectly
10
Q

Aetiology ADHD

A

Genetic predisposition: there is substantial evidence for a genetic contribution to ADHD, with the mean heritability for ADHD shown to be 76% based on twin studies.

Environmental factors: these account for 12% to 40% of the variance in twin ADHD scores. Low birth weight and maternal smoking have the strongest evidence for association with ADHD. Other risk factors include poverty, lead exposure, iron deficiency, maternal alcohol drinking during pregnancy, and psychosocial adversity.

How well did you know this?
1
Not at all
2
3
4
5
Perfectly
11
Q

RFs ADHD

A

STRONG

FHx
Male
LBW (<2500)
Epilepsy
Maternal nicotine use during pregnancy

WEAK

Maternal alcohol preg
Stress during preg
Psychosocial adversity - severe marital discord, low social class, large family size, paternal criminality, maternal mental disorder, and foster placement
Lead exposure
Traumatic brain injury
Severe early deprivation
Iron deficiency
How well did you know this?
1
Not at all
2
3
4
5
Perfectly
12
Q

Sx of ADHD

A
Careless mistakes/missing detail
Attention deficit
Listening deficit
Instructions deficit
Organisational difficulties
Reluctance to engage in long activities
Loses things necessary for tasks
Easily distracted
Forgetful
Fidgeting
Failure to remain seated
Excessive talking
Blurts out answers
Cannot wait turn
Interrupts others
Mild mood symptoms
Difficult peer interactions
Low self-esteem
Working memory impairment
Processing speed impairment
How well did you know this?
1
Not at all
2
3
4
5
Perfectly
13
Q

Ix of ADHD

A

Clinical diagnosis
BUT
Behavioural rating scales EG:
The ADHD Rating Scale: an 18-item scale [DSM]

The Vanderbilt Scale: a 55-item scale, which assesses ADHD, comorbid conditions, and performance.

SNAP-IV is included in many research trials, including the Multimodal Treatment Study of AD/HD (MTA). It is a 90-item scale that screens for ADHD and other diagnoses.

Neuropsychological testing by child psychiatrist

How well did you know this?
1
Not at all
2
3
4
5
Perfectly
14
Q

Rx of ADHD

A

Methylphenidate -> can be immediate release / extended release / transdermal

Dexamfetamine (immediate/extended/TD)

Behavioural therapy

  • parent training
  • teacher training

2nd line: change stimulant

3rd: Atomoxetine
- CONSIDER CHILD PSYCH REFERRAL

3rd: Guanfacine or Clonidine
- NB also antihypertensives: SE hypo/bradyC/rebound HTN

4th: Nortriptyline / Bupropion / Imipramine
AKA ANTIDEPRESSANTS

NB if child has tick:
- Guanfacine or clonifidine or atomoxetine used

How well did you know this?
1
Not at all
2
3
4
5
Perfectly
15
Q

Complications of ADHD

A

SEs of meds:

Anorexia
Insomnia
Cardiac
Mood lability
Headache
Psychotic symptoms 
Tics
Substance abuse
Growth delay
How well did you know this?
1
Not at all
2
3
4
5
Perfectly
16
Q

Prognosis of ADHD

A

Between 60% to 85% of patients with ADHD will continue to meet criteria in adolescence, and significant functional impairment often persists into adulthood.

Over time, symptoms of hyperactivity tend to remit, while impairments in attention persist. In fact, patients with the predominantly inattentive type of ADHD often present later (e.g., middle school, high school) because their lack of hyperactivity and impulsivity makes them less disruptive in primary school than children with combined type.

Adolescents and adults with symptoms of ADHD have higher risk for academic and professional difficulties, development of conduct disorder and antisocial behaviours, maladaptive relationships, increased injuries and car accidents, and teen pregnancies.

How well did you know this?
1
Not at all
2
3
4
5
Perfectly
17
Q

Define bronchiolitis

A

Acute viral infection of the lower respiratory tract.

Although it can affect individuals of any age, the term is most often used to refer to infection in infancy. It is characterised by epithelial cell destruction, cellular oedema, and airway obstruction by inflammatory debris and mucus.

The clinical manifestations include cough, wheeze, and laboured breathing.

Respiratory syncytial virus (RSV) accounts for the majority of cases, although rhinovirus, human metapneumovirus, influenza, parainfluenza, and adenovirus can all cause bronchiolitis as well.

How well did you know this?
1
Not at all
2
3
4
5
Perfectly
18
Q

Aetiology of bronchiolitis

A

The most common cause is respiratory syncytial virus (RSV). In one cohort study of bronchiolitis, RSV was responsible for 76%, rhinovirus for 18%, influenza virus for 10%, coronavirus for 2%, and human metapneumovirus for 3%, and 1% had parainfluenza. Bronchiolitis caused by RSV and other respiratory viruses begins as an upper respiratory tract infection, which then spreads to the lower respiratory tract in 1 to 3 days.

RSV infection occurs in almost all infants by 3 years of age, but only a minority develop bronchiolitis. This observation has led to the hypothesis that host and possibly environmental factors play a role in disease pathogenesis. Birth cohort studies have shown that diminished lung function at birth is a risk factor for wheezing in early infancy, but this mechanism cannot completely explain the variability of clinical manifestations of RSV infection. Environmental tobacco smoke exposure may also contribute to disease severity. Another area of focus has been the role of the host immune response in determining the effects of RSV infection.

How well did you know this?
1
Not at all
2
3
4
5
Perfectly
19
Q

RFs bronchiolitis

A
STRONG
<3yo
Nov-May
Prematurity
Bronchopulmonary dysplasia
Passive smoking
Impaired airway clearance/function eg PCD
Congenital HD
Immunodeficiency
How well did you know this?
1
Not at all
2
3
4
5
Perfectly
20
Q

Epidemiology of bronchiolitis

A

Bronchiolitis is one of the most common acute illnesses in infancy and the leading cause of hospitalisation in this age group. Approximately 1 in every 30 infants will be diagnosed with bronchiolitis in their first year of life.

Bronchiolitis is almost exclusively an infantile disease, and by 3 years of age essentially all children have serological evidence of having been infected with RSV. However, primary infection with RSV in infants does not confer protective immunity, so repeat infections are common. Although in most infants the disease is mild and self-limited, severe disease can occur, especially in infants under 6 months of age. Infants with underlying risk factors for severe infection, such as prematurity, congenital heart disease, or chronic lung disease, have a greater risk of hospitalisation, but the majority of hospitalisations are in infants with no underlying risk factors.

In addition to the acute effects of bronchiolitis, studies have demonstrated that a significant proportion of infants with RSV bronchiolitis go on to develop recurrent wheezing; rhinovirus has been increasingly studied and shown to have an association with recurrent wheezing and a diagnosis of asthma.
Risk factors such as family history of asthma increase the risk of a future asthma diagnosis.

How well did you know this?
1
Not at all
2
3
4
5
Perfectly
21
Q

Sx of bronchiolitis

A
Cough - dry/wet/croupy
Tachypnoea
Wheezing
Retractions, grunting, flaring 
Rhinitis
Fever <40deg
Apnoea

A hallmark of bronchiolitis is fluctuating clinical findings, often within short time periods.

How well did you know this?
1
Not at all
2
3
4
5
Perfectly
22
Q

Ix bronchiolitis

A

Hypoxaemia (<90%)
CONSIDER

Elisa rapid antigen detection
RT-PCR
CxR - ONLY PERFORMED IF V SEVERE - hyperinflation, interstitial inflammation, atelectasis

How well did you know this?
1
Not at all
2
3
4
5
Perfectly
23
Q

Rx bronchiolitis

A

Preventative - palivizumab

Supportive - maintain O2
Mechanical ventilation is indicated for respiratory failure.

RIBAVIRIN

? SOME STUDIES SHOW corticosteroids MAY benefit if Hx of wheeze

How well did you know this?
1
Not at all
2
3
4
5
Perfectly
24
Q

Complications bronchiolitis

A

Bacterial Pneumonia
Recurrent wheeze
Paed asthma

How well did you know this?
1
Not at all
2
3
4
5
Perfectly
25
Q

Prognosis bronchiolitis

A

In general, viral bronchiolitis is a mild condition that is self-limiting and resolves within a few days.

How well did you know this?
1
Not at all
2
3
4
5
Perfectly
26
Q

Define Cerebral Palsy

A

Cerebral palsy (CP) is an umbrella term referring to a non-progressive disease of the brain originating during the antenatal, neonatal, or early postnatal period when brain neuronal connections are still evolving.

Secondary effects of spasticity on growth may, however, be progressive. There may be additional disturbances of sensation, perception, cognition, communication, and behaviour; neurogenical bladder/bowel; GORD; sialorrhoea (excessive secretion of saliva); feeding and swallowing difficulties; and/or epilepsy. CP is also known as Little’s disease.

How well did you know this?
1
Not at all
2
3
4
5
Perfectly
27
Q

What is the aetiology of Cerebral Palsy

A

Antenatal:

Prematurity
Multiple birth
TORCH (toxo, rubella, CMV, Herpes)
Factor Va leiden
Chorioamniotnitis
Teratogen exposure
Genetic/metabolic disorders
Fetal brain malformation

Perinatal:

<10% of cases - birth asphyxia due to instrumental delivery, non-vertex presentation, birth trauma, placental abruption, rupture of the uterus or prolonged/obstructed labour, and postmaturity

Postnatal:

Hyperbilirubinaemia, neonatal sepsis, respiratory distress, early-onset meningitis, intraventricular haemorrhage, and head injuries prior to 3 years (including child abuse and shaken baby syndrome).
Around 25% of infants who survive neonatal seizures have CP

How well did you know this?
1
Not at all
2
3
4
5
Perfectly
28
Q

Discuss epidemiology of Cerebral Palsy

A

Most common cause of childhood disability affecting 2.5 per 1000 individuals in the industrialised world.

How well did you know this?
1
Not at all
2
3
4
5
Perfectly
29
Q

RF for Cerebral Palsy

A
STRONG
Prem
Fetal birth asphyxia
MultiGest
Maternal illness
Fetal brain malformation
Major birth defects
Familial metabolic disorder
Neonatal complications
Maternal teratogen exposure
Low socio

WEAK
Non-vertex presentation
Postmaturity
Head injury

How well did you know this?
1
Not at all
2
3
4
5
Perfectly
30
Q

Sx for Cerebral Palsy

A

All patients present with motor impairment; 80% have spasticity. Other movement disorders observed are dystonia, athetosis, chorea, and ataxia.

Other common problems include feeding difficulties, speech impairment, intellectual deficits, urinary incontinence, and a variable sensory/proprioceptive loss.

DELAY IN MOTOR/SPEECH?COGNITIVE

Retention of primitive reflexes
Lack of age appropriate reflexes
Spasticity/clonus 
Toe walking/knee hyperextension 
Scissoring
Crouched gait
Contractures
Weakness
Joint instability
Dystonia Chorea
Athetosis (changing writhing contorting movements)
Ataxia
Neonatal hypotonia
Scoliosis
How well did you know this?
1
Not at all
2
3
4
5
Perfectly
31
Q

Ix for Cerebral Palsy

A

MRI BRAIN
- periventricular leukomalacia, congenital malformation, stroke or haemorrhage, cystic lesions

CONSIDER - ie for causes
Coagulation studies ?5a Leiden
Genetic testing
Metabolic screen
Xray of joint
How well did you know this?
1
Not at all
2
3
4
5
Perfectly
32
Q

Rx for Cerebral Palsy

A

Management is multidisciplinary and includes occupational, physical, and speech therapy; neurology and neurosurgery; psychiatry; urology; ophthalmology; and orthopaedic, paediatric, dietary, and psychosocial services. Educational and vocational support is also needed.

Spasticity can be treated with oral medications, botulinum toxin and other injections; intrathecal baclofen; or selective posterior rhizotomy.

Physical, occupational, and speech therapy address motor function, communication, and activities of daily living to prevent deformity and optimise independence and quality of life. ADAPTIVE EQUIPMENT

Orthopaedic interventions address contractures, scoliosis, subluxing hips, and extremity deformity.

How well did you know this?
1
Not at all
2
3
4
5
Perfectly
33
Q

Prognosis for Cerebral Palsy

A

A child with CP becomes an adult with CP. Except for the most severely involved patients, people with CP survive to senior age status, though some require lifelong help and accommodation.

Adults with CP have an increased risk of non-communicable diseases such as stroke, COPD, or cardiac conditions and an increased risk of related deaths.

Reflexes and reactions in early infancy that are poor prognostic factors for the development of independent walking include:

Retention of asymmetric and symmetric tonic neck reflexes
Retention of Moro (startle) reflex
Retention of neck righting reflex
Presence of lower-extremity extensor thrust response
Lack of parachute reaction
Lack of foot placement reaction.

How well did you know this?
1
Not at all
2
3
4
5
Perfectly
34
Q

Complications for Cerebral Palsy

A
Feeding difficulties
Microcephaly 
Hydrocephalus
Drooling
Osteoporosis
Hearing impairment 
Cervical myelopathy
Behaviour
Aspiration
Poor growth 
Epilepsy 
Visual impairment 
Incontinence
GORD
Sleep disturbance 
Osteopenia
How well did you know this?
1
Not at all
2
3
4
5
Perfectly
35
Q

An 18-month-old child with a history of prematurity (28 weeks’ gestation, 1200 grams) presents with failure to meet developmental milestones. The child sat independently at 1 year, has few words vocally, does not pull to stand, and exhibits increased deep tendon reflexes in the lower extremities and sustained clonus at both ankles. There is good upper-extremity function. A magnetic resonance imaging scan of the brain reveals periventricular leukomalacia.

A

The child is diagnosed with spastic diplegic CP

How well did you know this?
1
Not at all
2
3
4
5
Perfectly
36
Q

A 2-year-old boy, born after a normal pregnancy and delivery, presents with an asymmetric gait. Examination reveals mild spasticity of the left upper and lower extremity, hyperactive left knee and ankle deep tendon reflexes, and decreased dorsiflexion of the left ankle compared with the right. When walking, the patient walks on his left toes, and his left arm is held mildly flexed at the elbow with the palm facing the floor (pronated forearm). The left calf is smaller in girth than the right leading to the diagnosis of…

A

hemiplegic CP.

How well did you know this?
1
Not at all
2
3
4
5
Perfectly
37
Q

Define Neonatal Encephalopathy

A

Defined by signs and symptoms of abnormal neurological function in the first few days of life in an infant born at term:

Difficulty initiating and maintaining respirations, a subnormal level of consciousness, and associated depression of tone, reflexes, and possibly seizures.

Encephalopathy is a nonspecific response of the brain to injury which may occur via multiple methods, but is commonly caused by birth asphyxia, leading to cerebral hypoxia.

How well did you know this?
1
Not at all
2
3
4
5
Perfectly
38
Q

Epidemiology of Neonatal Encephalopathy

A

Overall, the relative incidence of neonatal encephalopathy is estimated to be between 2 and 9 per 1000 term births.

40% to 60% of affected infants die by 2 years old or have severe disabilities.

In 2013 it was estimated to have resulted in 644,000 deaths down from 874,000 deaths in 1990.

How well did you know this?
1
Not at all
2
3
4
5
Perfectly
39
Q

Sx of Neonatal Encephalopathy

A

Reduced level of consciousness
Seizures (which peak at 48 hours)
Difficulty initiating and maintaining respiration
Depression of tone and reflexes

How well did you know this?
1
Not at all
2
3
4
5
Perfectly
40
Q

Ix for Neonatal Encephalopathy

A

Cord blood gas analysis can be used to determine if there is perinatal hypoxia/asphyxia, which are potential causes of hypoxic-ischemic encephalopathy or cerebral palsy, and give insight into causes of intrapartum fetal distress.

Cord blood gas analysis is indicated for high-risk pregnancies, in cases where C-sections occurred due to fetal compromise, if there were abnormal fetal heart rate patterns, Apgar scores of 3 or lower, intrapartum fever, or multifetal gestation.

Evidence of brain injury related to the hypoxic-ischemic events that cause neonatal encephalopathy can be seen with brain MRIs, CTs, magnetic resonance spectroscopy imaging or ultrasounds.[5][6]

How well did you know this?
1
Not at all
2
3
4
5
Perfectly
41
Q

Rx of Neonatal Encephalopathy

A

Currently, neonatal encephalopathy is treated using hypothermia therapy.
This has been shown to reduce brain damage, reduce future disability, and improve survival by a 2013 Cochrane review.

How well did you know this?
1
Not at all
2
3
4
5
Perfectly
42
Q

Prognosis + Complications of Neonatal Encephalopathy

A

HIE is a major predictor of neurodevelopmental disability in term infants. 25 percent have permanent neurological deficits.

It can result in developmental delay or periventricular leukomalacia.

How well did you know this?
1
Not at all
2
3
4
5
Perfectly
43
Q

Define Croup

A

Croup, also known as laryngotracheobronchitis, is a common respiratory disease of childhood, characterised by the sudden onset of a seal-like barky cough, often accompanied by stridor, voice hoarseness, and respiratory distress. The symptoms are a result of upper-airway obstruction due to generalised inflammation of the airways, as a result of viral infection (typically parainfluenza virus types 1 or 3).

How well did you know this?
1
Not at all
2
3
4
5
Perfectly
44
Q

Aetiology of Croup

A

The illness is due to viral infection (typically parainfluenza virus types 1 or 3). Several other viral pathogens have been recognised, including influenza A and B, adenovirus, respiratory syncytial virus, metapneumovirus, coronavirus HCoV-NL63, and rarely measles.

The symptoms result from upper-airway obstruction due to generalised inflammation and oedema of the airways. At the cellular level this progresses to necrosis and shedding of the epithelium. The narrowed subglottic region is responsible for the symptoms of seal-like barky cough, stridor (from increased airflow turbulence), and sternal/intercostal indrawing. If the upper-airway obstruction worsens, respiratory failure can result, leading to asynchronous chest and abdominal wall motion, fatigue, hypoxia, and hypercarbia

How well did you know this?
1
Not at all
2
3
4
5
Perfectly
45
Q

RFs for Croup

A

STRONG
6 months - 6 years old

WEAK
Male
Prior intubation

How well did you know this?
1
Not at all
2
3
4
5
Perfectly
46
Q

Epidemiology of Croup

A

Croup is a frequent cause of acute respiratory distress in young children. Typically, it affects those between 6 months and 3 years of age, peaking in the second year of life. It has been reported to occur in infants younger than 6 months, in adolescents, and, more rarely, in adults.

M>F

A peak in clinical presentations is correlated with parainfluenza virus epidemics. These peaks typically occur in alternating years and result in a 50 % increase in the number of children admitted with croup.

How well did you know this?
1
Not at all
2
3
4
5
Perfectly
47
Q

Sx of Croup

A
COMMON
Symptoms increases with agititation
Seal-like bark
Peak late autumn [SEP-DEC]
Prodromal symptoms
Abrupt onset symptoms
Symptoms worse at night 
Hoarse voice 
UNCOMMON
Resp distress
Persistent agitiation 
Lethargy 
synchronous chest wall / abdominal movement 
Fatigue
Hypoxia - cyanosis/pallor
Dec consciousness
How well did you know this?
1
Not at all
2
3
4
5
Perfectly
48
Q

Ix of Croup

A

CLINICAL DIAG

Could do Xray lateral neck - see STEEPLE sign
BUT DON’T NORMALLY DO!!

How well did you know this?
1
Not at all
2
3
4
5
Perfectly
49
Q

Rx of Croup

A

Orally administered corticosteroids are the mainstay for all levels of severity, combined with nebulised epinephrine (adrenaline) in moderate to severe croup to provide temporary relief of the symptoms of upper-airway obstruction.

NO STRIDOR:
Dexamethasone: 0.15 to 0.6 mg/kg orally as a single dose

Can use IM dex, or inhaled budesonide (2mg)

SUPPLEMENTAL O2 IF IMPENDING RESP FAILURE

INTUBATE IF NEEDED

How well did you know this?
1
Not at all
2
3
4
5
Perfectly
50
Q

Prognosis of Croup

A

Mild-Mod = reasonable outlook but may appear frightening

Severe: Since combination treatment with dexamethasone and nebulised epinephrine (adrenaline) became standard care, prognosis for severe croup has been excellent.

1-3% resp failure

How well did you know this?
1
Not at all
2
3
4
5
Perfectly
51
Q

Complications of Croup

A

Bacterial Tracheitis

Pneumonia

How well did you know this?
1
Not at all
2
3
4
5
Perfectly
52
Q

A 2-year-old boy is brought to the emergency department by his parents in the middle of the night. He has had mild symptoms of an upper respiratory infection for 48 hours, awoke with a sudden onset of seal-like barky cough, and has had inspiratory stridor when crying. The stridor disappeared at rest, but the seal-like barky cough has persisted.

A

Croup

How well did you know this?
1
Not at all
2
3
4
5
Perfectly
53
Q

A 3-year-old boy is brought to the emergency department by his parents in the late evening. He has developed a sudden onset of a seal-like barky cough, accompanied by clear nasal discharge. His parents became alarmed when he developed stridor, which persists throughout the trip to the hospital. On examination, he has a seal-like barky cough and inspiratory stridor when at rest, which worsens with agitation. Persistent sternal indrawing is also evident at rest.

A

Croup

How well did you know this?
1
Not at all
2
3
4
5
Perfectly
54
Q

Define CF

A

Cystic fibrosis (CF) is a severely life-shortening genetic disease resulting from abnormalities in the cystic fibrosis transmembrane conductance regulator (CFTR), a chloride channel found in cells lining the lungs, intestines, pancreatic ducts, sweat glands, and reproductive organs.

There are over 1500 known disease-causing mutations that interrupt various stages of CFTR synthesis and function.

The most common clinical manifestations are pancreatic dysfunction, resulting in calorie malabsorption; and lung disease, resulting from a cycle of mucus retention, infection, and inflammation.

How well did you know this?
1
Not at all
2
3
4
5
Perfectly
55
Q

Epidemiology of CF

A

There are about 10,400 cases in the UK, 30,000 cases in the US, and 70,000 worldwide.

The incidence among white people is about 1/3000; the incidence is lower among people of African, Hispanic, and Asian descent. It is most common among people of European descent.

One study suggests that the incidence may be decreasing since the institution of newborn screening.

Lung disease is the most common cause of morbidity and mortality.

How well did you know this?
1
Not at all
2
3
4
5
Perfectly
56
Q

RFs for CF

A

FHx
Known carrier parent
Ethnicity (white)

Approximately 1 in 28 white people is a carrier of a CFTR mutation

How well did you know this?
1
Not at all
2
3
4
5
Perfectly
57
Q

Aetiology of CF

A

CF is a genetic disease caused by mutations in the cystic fibrosis transmembrane conductance regulator (CFTR), an anion channel found in the apical membrane of epithelial cells. Patients may be either homozygous or heterozygous with respect to CFTR mutations. Carriers of one CFTR mutation and one normal CFTR allele do not demonstrate disease in most cases.

Pathophysiology:

Mutations in CFTR result in abnormal salt transport by epithelial cells, resulting in thick, sticky secretions.

In the pancreas, this leads to blockage of exocrine ducts, early activation of pancreatic enzymes, and eventual autodestruction of the exocrine pancreas. Therefore, most patients require supplemental pancreatic enzymes.
In the intestine, bulky stools can lead to intestinal blockage.
In the respiratory system, the absence of CFTR function results in mucus retention, chronic infection, and inflammation that eventuate in the destruction of lung tissue

How well did you know this?
1
Not at all
2
3
4
5
Perfectly
58
Q

Sx of CF

A

COMMON

RFs
\+ve Newborn screen
Failure to pass meconium 
- May have meconium ileus 
Failure to thrive 
Voracious appetite 
Wet-sounding cough 
Recurrent lower airway infection
Chronic sinusitis 
Genital abnormalities in males -> lack vas deferens 
Malabsorptive stool with steatorrhoea
Digital clubbing
GORD
UNCOMMON
Haemoptysis
Wheeze
Increased AP diameter chest 
Hx pancreatitis/appendicitis
How well did you know this?
1
Not at all
2
3
4
5
Perfectly
59
Q

Ix for CF

A

Sweat test - positive (sweat chloride >60 mmol/L [≥60 mEq/L]) [NB 30-60 = must do more Ix]

Immunoreactive trypsinogen test - +ve but must be followed up with an additional test

Genetic testing - presence of 2 disease-causing mutations

How well did you know this?
1
Not at all
2
3
4
5
Perfectly
60
Q

Rx for CF

A

GENERAL ONGOING Rx:

Chest physio, PEEP, Oscillatory devices: airway clearance

Salbutamol bronchodilator

Mucolytic: IE dornase alfa + Hypertonic Saline

IF PSEUDOMONAS COLONISED : Tobramycin

PROPHYLAXIS: Azithromycin 250mg TD

IBUPROFEN SLOWS LUNG DECLINE IN >6YOs

CFTR modulators:

Ivacaftor: F508del mutation.

Lumacaftor/ivacaftor OR Tezacaftor/ivacaftor : F508del mutation.

Ivacaftor increases CFTR activity
Lumacaftor moves CFTR to lumen [apical membrane]

LUNG TRANSPLANTATION

_____________________

If meconium ileus at birth:
Lactulose -> surgery + NG decompression in the meantime

_____________________

GI DISEASE
Pancreatic enzyme replacement (pancreatin)
GORD Rx
Vitamin replacement

ursodeoxycholic acid if LIVER involvement

How well did you know this?
1
Not at all
2
3
4
5
Perfectly
61
Q

Prognosis of CF

A

This is a genetic disease for which there is no cure. However, the outlook for patients with this condition has greatly improved. In the past 50 years, the mean age of survival has risen from infancy/school age to almost 40 years old.

How well did you know this?
1
Not at all
2
3
4
5
Perfectly
62
Q

Complications of CF

A
Distal intestinal OB
Pneumo/haemothorax
Intussusception
Chronic resp acidosis
Delayed puberty
Acute resp failure
Cor pulmonale
DM metlitus
Osteoporosis
Depression/anxiety
Chronic sinusitis
Failure to thrive
Nasal polyps 
Rectal prolapse
How well did you know this?
1
Not at all
2
3
4
5
Perfectly
63
Q

A 1-year-old child presents with failure to thrive. By history, the child was born at the 50th percentile for weight, but has crossed multiple percentile lines despite having a ravenous appetite. The child has more bowel movements per day than other children of the same age, and the stools often look shiny and have an unusually foul smell. In addition, the child has been treated with multiple courses of antibiotics for a persistent, wet cough. On measurement, the child is small for age, with weight and length below the third percentile

A

CF

How well did you know this?
1
Not at all
2
3
4
5
Perfectly
64
Q

Patients with pancreatic-sufficient CF may present with chronic sinusitis with involvement of all the sinuses, recurrent pancreatitis, and/or appendicitis. Older males may present with infertility due to congenital bilateral absence of the vas deferens. The pathological finding of inspissated, haematoxylin-staining material within the crypts of the appendix is pathognomonic.

A

CF

How well did you know this?
1
Not at all
2
3
4
5
Perfectly
65
Q

Define developmental dysplasia of the hip

A

The term developmental dysplasia of the hip (DDH) represents a spectrum of conditions affecting the proximal femur and acetabulum, ranging from acetabular immaturity to hip subluxation and frank hip dislocation. In true DDH, the femoral head has a persistently abnormal anatomical relationship with the pelvic acetabulum, which leads to abnormal bony development that can ultimately result in premature arthritis and significant disability. This is distinct from transient dysplasia, which represents acetabular immaturity in which the anatomical relationship stabilises and normalises over a period of weeks to months.

There is a high rate of spontaneous resolution of hips identified as abnormal by both clinical examination and ultrasound in the neonatal period. Frank dislocations and persistently abnormal examinations should be referred to a paediatric orthopaedist.

How well did you know this?
1
Not at all
2
3
4
5
Perfectly
66
Q

Epidemiology of developmental dysplasia of the hip

A

A recent UK review of neonatal screening for DDH suggested that the incidence of DDH is generally given as 1-2 per 1000 live births.

The reported prevalence of DDH has increased significantly since the advent of clinical and sonographic screening, suggesting the possibility of overdiagnosis.

DDH is more common in girls, with a 4-fold increased risk compared with boys.

W>B

The condition is also more common in infants who are breech at or near-term delivery.

How well did you know this?
1
Not at all
2
3
4
5
Perfectly
67
Q

Aetiology of developmental dysplasia of the hip

A

Although the aetiology of DDH has not been rigorously evaluated, it is generally believed that genetic, hormonal, and/or mechanical factors play a role. Observational studies have shown that infants with a family history of DDH have an increased relative risk of the condition; however, most cases occur in children with no family history. It is believed that those with DDH have increased ligamentous laxity, and that this is most pronounced at the time of delivery. Intrauterine confinement and positioning may play a role given that infants born breech, first-born infants, and large infants have been shown to have a higher relative risk of the condition.

How well did you know this?
1
Not at all
2
3
4
5
Perfectly
68
Q

RFs of developmental dysplasia of the hip

A

STRONG
Female
FHx
Breech

WEAK
Postural deformity eg congenital muscular torticollis
Restricted intraauterine space - oligohydramnios, macrosomia, multiple gestation

How well did you know this?
1
Not at all
2
3
4
5
Perfectly
69
Q

Sx of developmental dysplasia of the hip

A

ORTOLANI TEST
The hip is flexed to 90° and abducted, with the examiner’s fingers are placed laterally over the greater trochanter or hip joint. The examiner then uses anterior pressure over the trochanter in an attempt to identify a dislocated hip that is relocatable.
Should refer to a paediatric orthopaedist if frank instability is appreciated.

BARLOW TEST
The hip is flexed to 90° and adducted, the examiner’s hand is placed on the knee, and posterior pressure is placed through the hip in an attempt to identify dislocatable hips.

Should refer to a paediatric orthopaedist if frank instability is appreciated.

Limited hip abduction

UNCOMMON
Abnormal positioning of leg or delayed crawling
Toe walking

How well did you know this?
1
Not at all
2
3
4
5
Perfectly
70
Q

Ix of developmental dysplasia of the hip

A

<6 months:
US hips - subluxation on provocative testing; abnormal relationship between femoral head and acetabulum

> 6 months:
Beyond 6 months of age, hip radiographs are typically preferred over ultrasound.
- abnormal relationship between femoral head and acetabulum (assessed by acetabular index, Shenton’s line, ossification of femoral head)

How well did you know this?
1
Not at all
2
3
4
5
Perfectly
71
Q

Rx of developmental dysplasia of the hip

A

Hip dysplasia/subluxation

1) - MILD DYSPLASIA OBSERVE WITH MONTHLY SCANS AS MANY IMPROVE
- SUBLUX for 3w IN A NEONATE = warrants Rx as dislocated hip
2) If dysplasia persists or worsens, consideration is given to use of a Pavlik harness to enhance optimum hip development.

DISLOCATION:

1) CLOSED reduction with SPICA casting
2) OPEN reduction with SPICA casting

Open reduction is where the fracture fragments are exposed surgically by dissecting the tissues

Closed reduction is the manipulation of the bone fragments without surgical exposure of the fragments.

> 6YO
- will need Salvage osteotomies

How well did you know this?
1
Not at all
2
3
4
5
Perfectly
72
Q

Prognosis of developmental dysplasia of the hip

A

Prognosis is dependent upon the age at presentation, the extent of treatment needed, and the occurrence of complications. The success of closed or open reduction of a hip dislocation depends upon the remodelling potential of a child’s femoral head and acetabulum to achieve an adequately spherical shape with adequate coverage. This remodelling potential is more reliable in younger children, and within the first 12 to 18 months after reduction is achieved. The remodelling potential remains but is likely reduced during the subsequent years of growth.

For patients presenting as infants and using the Pavlik harness, success rates approach 90% and complication rates are considered low. Rates of avascular necrosis range between 1% to 3% for the majority of hips treated. Femoral nerve palsy has an incidence of 2.5% and typically is transient, resolving once the harness is removed

How well did you know this?
1
Not at all
2
3
4
5
Perfectly
73
Q

Complications of developmental dysplasia of the hip

A
Rx related avascular necrosis
Residual dysplasia
Degenerative joint disease 
Pavlik Harness disease
Limb length inequality 
Knee valgus
Back pain 
Rx related nerve palsy
How well did you know this?
1
Not at all
2
3
4
5
Perfectly
74
Q

Define congenital adrenal hyperplasia

A

AKA no cortisol OR aldosterone

Congenital adrenal hyperplasia (CAH) is a family of inherited enzyme deficiencies that impair normal corticosteroid synthesis by the adrenal cortex. The most common enzyme deficiency is 21-hydroxylase deficiency, which accounts for over 90% of cases. CAH due to 21-hydroxylase deficiency can be classified as either classical (simple virilising or salt-wasting types) or non-classical.

CAH is suspected in females who are virilised at birth, who become virilised postnatally, or who have precocious puberty or adrenarche.

Males with virilisation in childhood and infants of either sex with a salt-wasting crisis in the first 4 weeks of life are likely to be affected with CAH.

Serum concentrations of delta-androstenedione and progesterone are increased in males and females with 21-hydroxylase-deficient CAH.

Serum concentrations of testosterone and adrenal androgen precursors are increased in affected females and prepubertal males.

How well did you know this?
1
Not at all
2
3
4
5
Perfectly
75
Q

What is the epidemiology of congenital adrenal hyperplasia?

A

Screening studies indicate a worldwide incidence of classical 21-hydroxylase-deficient CAH as 1 in 13,000 to 1 in 54,000 live births.

The prevalence frequency of non-classical 21-hydroxylase-deficient CAH is considerably higher, at 1 in 1000, in white populations, with an even higher frequency among selected ethnic groups - most notably, Ashkenazi Jews.

How well did you know this?
1
Not at all
2
3
4
5
Perfectly
76
Q

What is the aetiology of congenital adrenal hyperplasia?

A

CAH is an autosomal recessive disorder, and the gene encoding 21-hydroxylase enzyme, CYP21A2, is mapped to the short arm of chromosome 6 (6p21.3). To date, more than 100 mutations have been described, including point mutations, small deletions, small insertions, and complex rearrangements of the gene.

The production of cortisol occurs in the zona fasciculata of the adrenal cortex through 5 enzymatic steps.

How well did you know this?
1
Not at all
2
3
4
5
Perfectly
77
Q

RFs for congenital adrenal hyperplasia

A

Genetic predisposition

How well did you know this?
1
Not at all
2
3
4
5
Perfectly
78
Q

Sx of congenital adrenal hyperplasia

A

COMMON

WL
Failure to thrive
Vomiting
Hypotension
Ambiguous genitalia
- Prader Score: degree of virilisation of the external genitalia of females as proposed by Prader.
Hyperpigmentation
Poor feeding
Irregular menses 
Infertility
Male pattern baldness in females
Short stature
Precocious puberty
PCOS
Hirstuism 

UNCOMMON
Severe cystic acne

How well did you know this?
1
Not at all
2
3
4
5
Perfectly
79
Q

Ix for congenital adrenal hyperplasia

A

serum 17-hydroxyprogesterone - HIGH if 21OH classical

serum 11-deoxycortisol - HIGH if 11-beta-hydroxylase deficiency

serum chemistry - hyponatraemic, hyperkalaemic, metabolic acidosis; azotaemia

microfilter paper radioimmunoassay for 17-hydroxyprogesterone - H if 21-OH def

Genetic analysis - mutation and autosomal recessive inheritance pattern

ACTH stimulation testing

Renin high in salt wasting

Pelvic USS not usually indicated but does show enlarged glands with classical CAH

How well did you know this?
1
Not at all
2
3
4
5
Perfectly
80
Q

Rx for congenital adrenal hyperplasia

A

Mother of potentially affected foetus: antenatal Dexamethasone

DURING ANY ACUTE STRESS EG SURGERY / ILLNESS ECT
MUST GIVE ADDITIONAL glucocorticoid

NORMAL DAY-DAY
Hydrocortison 10-15mg/square meter body

SALT WASTING FORM
May need fludrocortisone + NaCl

IF GROWTH AFFECTED:
Somatotropin

Genital surgery

How well did you know this?
1
Not at all
2
3
4
5
Perfectly
81
Q

Prognosis of congenital adrenal hyperplasia

A

Improved care has resulted in a good prognosis and normal life expectancy. The main concerns in children are the preservation of fertility and healthy sexual function, and maintenance of general wellbeing. This includes bone health and the assessment and management of cardiovascular disease risk.

Changes in childhood play behaviour correlate with reduced female gender satisfaction and reduced heterosexual interest in adulthood. The rates of bisexual and homosexual orientation were increased in women with all forms of 21-hydroxylase-deficient CAH. They were found to correlate with the degree of antenatal androgenisation. Affected adult females are more likely to have gender dysphoria, and experience less heterosexual interest and reduced satisfaction with the assignment to the female sex. Males affected with CAH do not show a general alteration in childhood play behaviour, core gender identity, and sexual orientation.

How well did you know this?
1
Not at all
2
3
4
5
Perfectly
82
Q

Complications of congenital adrenal hyperplasia

A

Adrenal crisis
Testicular adrenal rests
Short stature

How well did you know this?
1
Not at all
2
3
4
5
Perfectly
83
Q

A 7-day-old newborn boy is brought into hospital with vomiting and poor feeding. The infant is dehydrated and tachycardic. Electrolytes reveal hyponatraemia and hyperkalaemia. The patient demonstrates signs of shock. On genital exam, a phallic structure is noted with hyperpigmentation of the scrotum, but no testes are palpated.

A

Congenital adrenal hyperplasia

Under-virilisation of newborn males can be seen in 3-beta-hydroxysteroid dehydrogenase deficiency and 17-alpha-hydroxylase deficiency.

How well did you know this?
1
Not at all
2
3
4
5
Perfectly
84
Q

A 16-year-old, previously healthy female presents with acne, hirsutism, and irregular menses. Her pubertal history reveals breast development at 9 years of age and pubic hair development at 7 years of age, and she reported 1 episode of vaginal spotting at approximately 11.5 years of age. A family history indicates some female relatives with symptoms of infertility, irregular menses, polycystic ovary syndrome, or alopecia. She is significantly shorter than her target height.

A

Congenital adrenal hyperplasia

How well did you know this?
1
Not at all
2
3
4
5
Perfectly
85
Q

Define febrile seizure

A

The American Academy of Pediatrics announced a standard definition of febrile seizures as a seizure occurring in febrile children between the ages of 6 and 60 months who do not have an intracranial infection, metabolic disturbance, or history of afebrile seizures.

The first occurrence is usually before 3 years of age, but is infrequent in children younger than 6 months. Febrile seizures have a peak incidence at 18 months of age and are most common between 6 months and 5 years. Most febrile seizures are simple with approximately 20% to 30% being complex.

Febrile seizures are usually self-limiting; an increased risk of developing epilepsy is low except in a small number of cases.

Most febrile seizures resolve spontaneously and quickly, and do not require acute or long-term anticonvulsant treatment.

How well did you know this?
1
Not at all
2
3
4
5
Perfectly
86
Q

Epidemiology of febrile seizure

A

2-5% cumulative incidence

1.6M:1F

How well did you know this?
1
Not at all
2
3
4
5
Perfectly
87
Q

Aetiology of febrile seizure

A

Viral infections triggering fever are most common, with bacteraemia as an infrequent cause.

(HHV)-6 infection

Influenza A virus is the most common cause in Asian countries

Febrile seizures are dependent upon a threshold temperature and this seems to vary from one individual to another.

The American Academy of Pediatrics considers fever to be any increase of the body’s external temperature over 38ºC

How well did you know this?
1
Not at all
2
3
4
5
Perfectly
88
Q

RFs for febrile seizure

A
STRONG
Fever
Young age
FHx febrile seizure
Viral/bacterial infection outside the central nervous system 
WEAK
Male
Vaccinations
Antenatal exposure to nicotine
Iron deficiency
Complications of pregnancy, labour + delivery
How well did you know this?
1
Not at all
2
3
4
5
Perfectly
89
Q

Sx of febrile seizure

A
RFs
Febrile illness >38.3
Seizure:
- LOC
- Generalised tonic-clonic 
- Consciousness usually recovered quickly within 30mins
- No sequelae
- Less commonly, some seizures are prolonged, focal or multiple, and recovery of consciousness is delayed.

NORMAL post-ictal exam

How well did you know this?
1
Not at all
2
3
4
5
Perfectly
90
Q

Ix for febrile seizure

A

Diagnosis is clinical, however alternative causes must be ruled out:

Lumbar puncture: normal cells, protein, glucose
Viral studies: may be +ve
Blood culture: may be +ve
FBC
Serum glucose: usually N
How well did you know this?
1
Not at all
2
3
4
5
Perfectly
91
Q

Rx for febrile seizure

A

ANTIPYRETIC

  • paracetamol 10-15mg/kg 4-6hrly
  • ibuprofen 5-10 mg/kg orally every 6-8 hours

Anticonvulsant:
diazepam, midazolam, lorazepam, fosphenytoin

How well did you know this?
1
Not at all
2
3
4
5
Perfectly
92
Q

Prognosis of febrile seizure

A

Prognosis is generally favourable.

Febrile seizures recur in approximately 30% of children during subsequent febrile illnesses.

Most recurrences occur within 2 years. The risk of non-febrile seizures and epilepsy developing after simple febrile seizures is 5% or less. However, after complex febrile seizures, the risk of developing epilepsy is 10% to 20%.

How well did you know this?
1
Not at all
2
3
4
5
Perfectly
93
Q

Complications of febrile seizure

A
Todd's paralysis (temporary hemiparesis 
Non-febrile seizure
Epilepsy 
Focal epilepsy
Mesial Temporal sclerosis
Behaviour and cognitive disorders
How well did you know this?
1
Not at all
2
3
4
5
Perfectly
94
Q

A previously healthy and developmentally normal 18-month-old boy presents to the emergency department by ambulance after his parents witnessed a seizure. The parents report the boy had a febrile illness with mild upper respiratory symptoms and they treated him with paracetamol and ibuprofen at home. The child then began to have frequent jerking movements of all limbs. The rectal temperature was 39.5°C (103.1°F). The parents called the emergency services, and a paracetamol suppository was administered during transport to the emergency department. The jerking stopped after approximately 5 minutes. Afterwards, the child was sleepy but responsive to verbal stimulation. Examination revealed a diffuse erythematous maculopapular rash and a normal mental and neurological status.

A

Febrile seizure

How well did you know this?
1
Not at all
2
3
4
5
Perfectly
95
Q

A 10-month-old girl is brought to the emergency department with a history of recurrent right arm and leg jerking followed by prolonged sleepiness. The parents report a 2-day history of fever with chest congestion and irritability. The child is admitted to hospital for neurological evaluation.

A

Febrile seizure

How well did you know this?
1
Not at all
2
3
4
5
Perfectly
96
Q

Febrile seizures before 6 months of age in a child with a relatively low fever are atypical, and require a full investigation to exclude acute bacterial meningitis or other CNS pathology. Another atypical presentation is a child with a prolonged focal (complex) febrile seizure who has Todd’s paralysis (transient hemiparesis) on recovery of consciousness. In one prospective series of 95 patients with febrile seizure, 2% had Todd’s paralysis. [7] Febrile status epilepticus is a prolonged or recurring seizure with fever and without recovery of consciousness between episodes. Classically, the duration is 30 minutes or more; some include seizures of shorter duration (10 minutes or more). Tonic-clonic generalised status is most common. In a large prospective, controlled study, [8] patients with febrile seizure status were more likely to have neurological abnormalities, a history of neonatal seizures, and a family history of epilepsy. A prospective multi-centre study of children with febrile status epilepticus (FEBSTAT study) found a median age of 1.3 years, and seizures that were most often focal, partial and long, lasting a median of 68 minutes. [9] Febrile status epilepticus was frequently the first febrile seizure, and status was unrecognised in the emergency department.

A

Febrile seizure

How well did you know this?
1
Not at all
2
3
4
5
Perfectly
97
Q

Define viral gastroenteritis in children

A

Viral gastroenteritis is an infection of the GI tract by a virus, usually rotavirus. It is usually self-limiting, but if untreated may result in morbidity and mortality secondary to dehydration, electrolyte imbalance, and metabolic acidosis.

Associated with substantial morbidity in developed countries. Continues to be a cause of significant mortality in developing countries.

Presents with vomiting and diarrhoea (with or without fever, nausea, or abdominal pain).

Dehydration, which may be associated with electrolyte imbalance and metabolic acidosis, is the most frequent and dangerous complication.

Oral rehydration therapy is the mainstay of treatment.

Self-limiting disease with excellent prognosis if correctly treated.

How well did you know this?
1
Not at all
2
3
4
5
Perfectly
98
Q

What is the aetiology of viral gastroenteritis in children

A

Viruses account for approximately 70% to 87% of episodes of acute gastroenteritis in children, with Rotavirus being the most common identifiable cause.

Other viral pathogens include caliciviruses, astroviruses, and adenoviruses.

Other viruses such as coronaviruses, parvoviruses, and picobirnaviruses have also been linked to human gastroenteritis.

How well did you know this?
1
Not at all
2
3
4
5
Perfectly
99
Q

What is the epidemiology of viral gastroenteritis in children

A

Worldwide, 3 billion to 5 billion cases of acute gastroenteritis occur each year in children <5 years, resulting in 1.5 million to 2.5 million deaths.

The average child <5 years of age experiences 2.2 diarrhoeal episodes per year in developed countries, and this rate in developing countries is significantly higher.

Viral gastroenteritis is spread mainly by the faecal-oral route; this may occur through contaminated hands, utensils, food, and drink. Gastroenteritis is associated with poverty and poor environmental or personal hygiene.

Spread among family members is common, and outbreaks may occur in day-care centres and hospitals. Several epidemiological studies have suggested that rotavirus infection can be transmitted by respiratory droplets.

In temperate regions, rotavirus gastroenteritis has a marked seasonality, peaking during the cold months.

How well did you know this?
1
Not at all
2
3
4
5
Perfectly
100
Q

List RFs for viral gastroenteritis in children

A
Age <5
Poor personal hygiene
Exposure 
Day-care/nursery
Winter months 
Poverty
Lack of immunisation 
Lack of breast feeding 
Immunodeficiency
How well did you know this?
1
Not at all
2
3
4
5
Perfectly
101
Q

Sx of viral gastroenteritis in children

A
COMMON
RFs 
Vomiting
Non-bloody diarrhoea
Hyperactive bowel sounds
Low grade fever
Abdominal pain
Evidence of dehydration 
Decreased bodyweight
Non-distended abdomen 
Abdominal tenderness

UNCOMMON
Mucus in stool

How well did you know this?
1
Not at all
2
3
4
5
Perfectly
102
Q

Ix for viral gastroenteritis in children

A

CLINICAL DIAGNOSIS

Measurements of serum electrolytes, urea, and creatinine are usually not necessary to assess children with acute gastroenteritis, as the results do not change the therapeutic strategy.

CONSIDER
U+E - N unless severe deydration
FBC - if worried re sepsis
Stool culture - Negative
Enzyme immunoassay for detection of viral antigen - CAN detect offending pathogen but is pointless rly
How well did you know this?
1
Not at all
2
3
4
5
Perfectly
103
Q

Rx for viral gastroenteritis in children

A

Oral rehydration solutions
Ondansetron - if vomiting prevents ORT
IV fluids

How well did you know this?
1
Not at all
2
3
4
5
Perfectly
104
Q

Prognosis of viral gastroenteritis in children

A

With proper treatment, the prognosis is excellent. Viral gastroenteritis is self-limited; the typical episode lasts 3 to 7 days. Prolonged diarrhoea may indicate secondary disaccharidase deficiency.

Worldwide, acute gastroenteritis accounts for 1.5 to 2.5 million deaths annually. Most deaths occur in developing countries.

How well did you know this?
1
Not at all
2
3
4
5
Perfectly
105
Q

Complications of viral gastroenteritis in children

A
Volume depletion
U+E imbalance
Metabolic acidosis - Often complicates acute diarrhoea, as a result of faecal losses of bicarbonate and impaired renal excretion of hydrogen ions.
Hypoglycaemia
Viral antigenaemia
Secondary bacteraemia 
Afebrile seizure
Shock
How well did you know this?
1
Not at all
2
3
4
5
Perfectly
106
Q

A 6-month-old boy presents with a history of low-grade fever of 37.7°C (100°F) and non-bilious vomiting (6 episodes per day) for 2 days. This is followed by diarrhoea (15 stools per day) 24 hours later. Stools are watery and do not contain blood or mucus. His mother is unsure of the urine output but the infant has lost 0.7 kg (1.5 lb) of weight in the past few days. His mother also reports that the infant is more irritable and has decreased activity level since the onset of the illness. The infant has never been breast-fed and has not had rotavirus vaccination. He attends a day-care centre. His 2-year-old brother who attends the same day-care centre has just got over an ‘upset tummy’.

A

Viral gasteroenteritis

How well did you know this?
1
Not at all
2
3
4
5
Perfectly
107
Q

Define hirschsprung’s disease

A

A congenital condition characterised by partial or complete colonic functional obstruction associated with the absence of ganglion cells.

Because of the aganglionosis, the lumen is tonically contracted, causing a functional obstruction. The aganglionic portion of the colon is always located distally, but the length of the segment varies. This determines the varied manifestations of the disease.

The vast majority of patients present in the newborn period up to 1 year of age. Diagnosis later in life occurs rarely.

Presents with vomiting, abdominal distension, and/or enterocolitis.

May be associated with Down’s syndrome and multiple endocrine neoplasia type IIA.

Definitive diagnosis is with a rectal biopsy.

Initial treatment is bowel irrigation, followed by a definitive surgical treatment, either transanally alone or with laparoscopic assistance. Rarely, colostomy or ileostomy is required to manage severe enterocolitis, with definitive pull-through delayed.

Irrigations often do not work for patients with total colonic Hirschsprung’s disease. Total colectomy with ileo-procto anastomosis and a protective ileostomy is recommended at the time of diagnosis. The ileostomy is closed after the child is toilet trained for urine and can accept rectal irrigations.

How well did you know this?
1
Not at all
2
3
4
5
Perfectly
108
Q

Aetiology of hirschsprung’s disease

A

There have been new developments in determining associated genetic defects. Hirschsprung’s disease is thought to be due to a deletion in the long arm of chromosome 10. The location of this mutation is between 10q11.2 and 10q21.22.

This deletion seems to overlap the region of the RET (rearranged during transfection) proto-oncogene. Multiple endocrine neoplasia type IIA is also associated with a deletion of this proto-oncogene.

The absence of ganglion cells and the presence of hypertrophic nerves, as well as an increase in the enzyme acetylcholinesterase, are the keys to pathological diagnosis of the dysfunctional bowel segment.

How well did you know this?
1
Not at all
2
3
4
5
Perfectly
109
Q

RFs of hirschsprung’s disease

A

STRONG
Downs syndrome

WEAK
MEN2A
Male
Gene mutations 10/21

How well did you know this?
1
Not at all
2
3
4
5
Perfectly
110
Q

Epidemiology of hirschsprung’s disease

A

1 in 5k births
White >
M>F

Approximately 5% to 32% of all individuals affected with the disease have an associated congenital anomaly.

How well did you know this?
1
Not at all
2
3
4
5
Perfectly
111
Q

Sx of hirschsprung’s disease

A
COMMON
Vomiting
Explosive passage of liquid and foul stools
Abdo distension
Delayed passageof meconium
Fever
Failure to thrive

UNCOMMON
Septic shock

How well did you know this?
1
Not at all
2
3
4
5
Perfectly
112
Q

Ix of hirschsprung’s disease

A

Plain Xray - air-fluid levels present; dilated colon

Contrast enema - contracted distal bowel and dilated proximal bowel, with demonstration of the location of the transition zone in between

Rectal biopsy - absence of ganglion cells and presence of thickened (i.e., >40 microns), non-myelinated nerves; increased activity of acetylcholinesterase; absence of nicotinamide adenine dinucleotide phosphate diaphorase-containing neurons; increase in the amount of acetylcholinesterase-containing nerve bundles

How well did you know this?
1
Not at all
2
3
4
5
Perfectly
113
Q

Rx of hirschsprung’s disease

A

Bowel irrigation
IV fluids/ABx
Decompression by colostomy or ileostomy
Definitive surgery

Total colonic aganglionosis -> cannot have bowel irrigation: only ileostomy

How well did you know this?
1
Not at all
2
3
4
5
Perfectly
114
Q

Complications of hirschsprung’s disease

A
Ischaemic enterocolitis
Post-op enteroC
Post op constipation/diarrhoea
Duhamel rectal pouch
Faecal incontinence post surgery 
Pseudo-incontinence
Fistula/pelvic abscess
How well did you know this?
1
Not at all
2
3
4
5
Perfectly
115
Q

Prognosis of hirschsprung’s disease

A

The outcome for typical (rectosigmoid) disease is good. Enterocolitis can occur postoperatively but usually disappears after the first year of life. Soiling is usually treatable with the correct medical management.

True faecal incontinence is avoidable with proper technique preserving the sphincters and anal canal.

Patients who have had pull-through and suffer from incapacity to empty the colon may have a retained piece of colon with transition zone, or a twisted pull-through.

How well did you know this?
1
Not at all
2
3
4
5
Perfectly
116
Q

A 4-day-old baby presents with bilious vomiting and significant abdominal distension.

A

Hirschsprungs

How well did you know this?
1
Not at all
2
3
4
5
Perfectly
117
Q

A 1-month-old baby boy presents with feeding intolerance, abdominal distension, and copious diarrhoea.

A

Hirschsprungs

How well did you know this?
1
Not at all
2
3
4
5
Perfectly
118
Q

Diarrhoea with vomiting, fever, and abdominal distension (enterocolitis) are common presenting symptoms. Older children may have persistent abdominal distension and/or failure to thrive as a manifestation of chronic moderate enterocolitis. When the abdomen is distended, the infant can become quite ill from sepsis, hypovolaemia, and endotoxin-related shock due to bacterial translocation.

A

Hirschsprungs

How well did you know this?
1
Not at all
2
3
4
5
Perfectly
119
Q

Define Henoch-Schonlein purpura

A

Henoch-Schonlein purpura (HSP) is the most common vasculitis of childhood and affects the small vessels. HSP is characterised by the classic tetrad of rash, abdominal pain, arthritis/arthralgia, and glomerulonephritis.

Most common vasculitis of childhood.

Rash of palpable purpura is present in all cases.

Most cases resolve with symptomatic treatment.

Complications can occur, with renal failure being the most common cause of death.

How well did you know this?
1
Not at all
2
3
4
5
Perfectly
120
Q

Epidemiology of Henoch-Schonlein purpura

A
Normally 3-15yo
50% of cases = before 3YO
22.1 cases per 100,000 population
Asian>W>B
HSP can follow a URTI, and as a result it mainly occurs in the autumn, winter, and spring.
How well did you know this?
1
Not at all
2
3
4
5
Perfectly
121
Q

Aetiology of Henoch-Schonlein purpura

A

The underlying cause of HSP remains unknown. It is an immune-mediated vasculitis, with a variety of infectious and chemical triggers having been proposed as a cause. Many cases of HSP occur after a URTI, especially streptococcal infections.

HSP is a small-vessel leukocytoclastic vasculitis characterised by the tissue deposition of IgA-containing immune complexes within affected organs. Skin biopsies of lesions show neutrophils and monocytes as the predominant cell types. Fluorescence microscopy shows IgA, C3, and fibrin deposition within the small vessels.

The pathogenesis of this disease is similar to that of IgA nephropathy.

How well did you know this?
1
Not at all
2
3
4
5
Perfectly
122
Q

RFs for Henoch-Schonlein purpura

A

M
3-15
Hx URTI
Some medicines: (e.g., penicillin, cefaclor, minocycline, hydralazine, and phenytoin)

How well did you know this?
1
Not at all
2
3
4
5
Perfectly
123
Q

Sx of Henoch-Schonlein purpura

A

COMMON
RFs
Arthralgias
Abdominal pain
Rash - erythematous, non-blanching, usually lower extremities
Sx renal disease: can vary from mild (i.e., asymptomatic haematuria and/or proteinuria) to severe (i.e., rapidly progressive nephritis, nephrotic syndrome, and renal failure). It can produce microscopic haematuria, proteinuria, and RBC casts.

UNCOMMON 
Headaches
Seizures
Pulmonary haemorrhage
Scrotal pain/swelling
How well did you know this?
1
Not at all
2
3
4
5
Perfectly
124
Q

Ix of Henoch-Schonlein purpura

A

Urinlaysis: assess renal involvement - may show RBCs, proteinuria, or casts

coagulation studies - N

How well did you know this?
1
Not at all
2
3
4
5
Perfectly
125
Q

Rx for Henoch-Schonlein purpura

A

No nephrotic syndrome:
Analgesics - ibu/para
Prednisolone if severe oedema / pain

IF NEPHROTIC RANGE PROTEIN
- Corticosteroid + immunosuppressant (azathioprine/ciclophosphamide)

May need plasmapheresis/renal dialysis/transplantation if very severe

How well did you know this?
1
Not at all
2
3
4
5
Perfectly
126
Q

Prognosis of Henoch-Schonlein purpura

A

The outcome for patients with HSP is usually excellent. Complete recovery occurs in 94% of children and 89% of adults.

One third of patients may have a recurrence within 4 months, but the subsequent episode is generally milder; recurrences are more common in patients with nephritis.

How well did you know this?
1
Not at all
2
3
4
5
Perfectly
127
Q

Complications of Henoch-Schonlein purpura

A
Pulmonary haemorrhage 
GI haemorrhage/intussusception
Renal impairment 
ESRD
CNS complications
Uveitis
How well did you know this?
1
Not at all
2
3
4
5
Perfectly
128
Q

A 5-year-old boy is brought in by his mother. He presents with a 4-day history of a rash on his lower extremities, mild abdominal cramping, and diffuse joint pain. His mother reports that he was recently treated for a URTI.

A

Henoch-Schonlein purpura

How well did you know this?
1
Not at all
2
3
4
5
Perfectly
129
Q

While the rash occurs in all patients, the other classic symptoms do not always manifest. Scrotal pain and swelling may occur in about 13% of boys with HSP.

Other organ systems may be involved, including the CNS (patients may present with headaches or seizures) and pulmonary system (patients may present with pulmonary haemorrhage).

A

HSP

How well did you know this?
1
Not at all
2
3
4
5
Perfectly
130
Q

Define Kawasaki disease

A

Kawasaki disease (KD) is an acute, febrile, self-limiting, systemic vasculitis of unknown origin that almost exclusively affects young children. In an immunogenetically pre-disposed host, one or more infectious agents may play a role in triggering the clinical manifestations of the disease. Clinically, it is characterised by fever, polymorphic rash, conjunctivitis, mucosal erythema with strawberry tongue, induration of the hands and feet, and unilateral cervical lymphadenopathy. Morbidity and mortality depend on coronary aneurysms that develop in 20% to 25% of untreated patients. KD is the leading cause of acquired heart disease in children under 5 years of age in the US and other developed countries.

Acute febrile illness lasting 5 or more days.

Typical signs include fever, polymorphic rash, injected eyes, and mucosal erythema with strawberry tongue.

Swelling and erythema of the hands and feet occur in the acute stage, followed by desquamation in the second week.

Unilateral non-purulent cervical lymphadenopathy is present in about 40% of cases.

Coronary aneurysms develop in 20% to 25% of untreated patients.

Standard treatment includes intravenous immunoglobulin and/or aspirin. In resistant cases, corticosteroids or a tumour necrosis factor (TNF)-alpha inhibitor may be necessary.

How well did you know this?
1
Not at all
2
3
4
5
Perfectly
131
Q

Epidemiology of Kawasaki disease

A
13-24month usually
Rare in 1st 6month
80% occur before 5
Asian > black > white origin
M>F
How well did you know this?
1
Not at all
2
3
4
5
Perfectly
132
Q

Aetiology of Kawasaki disease

A

The cause of KD remains unknown. However, the following observations suggest that this disease is triggered by an unknown infectious agent.

Clinical picture: KD overlaps with infectious diseases such as scarlet fever and adenoviral infection.

Seasonal occurrence: in the US and other geographic areas, the peak occurrence of KD is in the winter/spring, similar to that seen in numerous viral diseases.

Epidemics with clear epicentre: temporal clusters have been reported in the US, Japan, and worldwide. Moreover, in Japan, outbreaks have been observed to start in one area and spread throughout the country over a period of 3 months.

Age at onset: peak incidence is in the toddler age group; 80% of the cases are in infants under 5 years old, and the rarity of cases under 3 months of age suggests protective transplacental antibodies

How well did you know this?
1
Not at all
2
3
4
5
Perfectly
133
Q

RFs for Kawasaki disease

A

Asian ancestry
3month - 4yo
Male

How well did you know this?
1
Not at all
2
3
4
5
Perfectly
134
Q

Sx of Kawasaki disease

A
COMMON
Polymorphous rash - diffuse, maculopapular erythematous rash. Occasionally scarlatiniform- or erythema multiforme-type rash with target lesions on the arm and trunk. Groin erythema or desquamation and fine pustules over extensor surfaces of extremities can occur.
Conjunctival injection - aka RED eyes
Mucositis - dry, erythematous, fissured lips that bleed easily, erythema of the oral and pharyngeal mucosa, and strawberry tongue with prominent papillae and erythema 
Skin changes on extremities 
Unilateral enlarged cervical LNs
Coronary artery aneurysms 
Fever + EXTREME irritability 
UNCOMMON
Pericarditis with effusion
Congestive Heart Failure 
Joint pain/oedema
Neurological manifestations: aseptic meningitis/facial nerve palsy

VERY RARE
GI manifestations
Urological manifestations
Other derm manifestations

How well did you know this?
1
Not at all
2
3
4
5
Perfectly
135
Q

Ix of Kawasaki disease

A

FBC - Normochromic Anaemia, leukocytosis, thrombocytosis (plts most important diagnostically)
ESR - Raised
CRP - raised
Echo - coronary artery dilatations or aneurysms

CONSIDER
Urinalysis - sterile pyruria 
LFTs - raised LFTS, low alb
ECG - rule out MI
CxR - may see cardiomegaly
LP - if Sx meningitis (may have aseptic men)
How well did you know this?
1
Not at all
2
3
4
5
Perfectly
136
Q

Rx of Kawasaki disease

A
1st: IVIG
\+ High dose aspirin 25mg dose QTD
2nd:
Prednisolone/methylpred + aspirin
2nd: Infliximab 5mg/kg single dose + aspirin
3rd: Ciclosporin + aspirin

RISK FOR MI
Low risk - CV assessment
High risk - Aspirin + LMWH + adjunct: clopidogrel + BB

How well did you know this?
1
Not at all
2
3
4
5
Perfectly
137
Q

Prognosis of Kawasaki disease

A

Kawasaki disease (KD) is an acute, self-limiting illness. The immediate outcome has improved dramatically, with a decrease in the frequency of coronary artery aneurysms to less than 3% following the introduction of intravenous immunoglobulin (IVIG) therapy. Overall, the mortality rate is less than 0.5%.

However, in untreated patients it is associated with significant morbidity and mortality. Delayed diagnosis, particularly of incomplete KD, and KD in very young children, poses a challenge as these patients have a high risk of aneurysms. The long-term prognosis of children with giant aneurysms remains a concern due to their associated risk of ischaemia or thrombosis. Management of such patients includes thromboprophylaxis and careful identification of evolving stenoses. Selected patients may need invasive revascularisation procedures.

How well did you know this?
1
Not at all
2
3
4
5
Perfectly
138
Q

Complications of Kawasaki disease

A
Myocarditis
Pericarditis
Coronary aneurysms
Valvulitis
Arthritis
Mild hepatic dys
Gallbladder disease
Pneumonitis
Aseptic meningitis
Coronary thrombosis
ACS
Systemic artery aneurysms
Extremity gangrene
Bowel ischaemia and necrosis
How well did you know this?
1
Not at all
2
3
4
5
Perfectly
139
Q

A previously healthy 1-year-old girl was admitted to a children’s hospital with a 7-day history of spiking fever up to 39.5°C (103°F). Three days after the onset of fever she developed left-sided neck swelling and nappy rash, and became progressively fussy and irritable. She was seen in an emergency department, diagnosed with cervical adenitis, and sent home on oral antibiotics. The mother noted continued irritability, high fever, and decreased oral intake. On subsequent admission she was extremely irritable, with a temperature of 38.9°C (102°F), heart rate of 140 beats per minute, respiratory rate of 40 breaths per minute, and blood pressure 110/54 mmHg. There were no signs of nuchal rigidity. Both palpebral and bulbar conjunctivae were deep red and injected, lips were dry and crusted, the oropharynx hyperaemic with some areas of ulcerated mucosa, and the tongue papillae were enlarged and red (strawberry appearance). Examination of the neck revealed a mildly tender left unilateral mass, measuring 4 cm. The skin showed a generalised polymorphous, erythematous, macular, blanching rash, in addition to severely red and desquamated perineal region. Her extremities, especially palms and soles, were swollen, red, and mildly tender.

A

Kawaski Disease

How well did you know this?
1
Not at all
2
3
4
5
Perfectly
140
Q

Some cases do not fulfil well-accepted criteria and are called incomplete/atypical Kawasaki disease (KD). This presentation is more common among children <1 year of age, who are at higher risk for the development of coronary artery aneurysms (CAAs) if untreated. In these cases of insufficient clinical criteria, presence of coronary abnormalities or CAAs must be shown on echocardiogram. A patient can present with prolonged fever (>5 days) and 2 or 3 of the classic criteria such as generalised polymorphous skin rash and red injected eyes. Infantile periarteritis nodosa is part of the spectrum of KD. The coronary artery aneurysmal lesions are clinically and pathologically indistinguishable from those seen in KD. Acute febrile mucocutaneous lymph node syndrome was initially described before KD was recognised. It is now part of the spectrum of KD.

A

Kawaski Disease

How well did you know this?
1
Not at all
2
3
4
5
Perfectly
141
Q

Define intussusception

A

Intussusception is a common cause of intestinal obstruction in young children and is defined as the prolapse of one part of intestine into the lumen of the adjoining distal part.

This condition most often occurs in the ileocaecal region. The lead point of the intussusception is most often an enlarged lymph node (Peyer’s patch) in the terminal ileum. Occasionally, the lead point is an anatomic abnormality of the intestine (a ‘pathological lead point’). Conditions resulting in pathological lead points include luminal polyps, malignant tumours (including lymphoma), and benign mass lesions such as lipomata, Meckel’s diverticulum, Henoch-Schonlein purpura, and enteric duplication cysts.

Intussusception is clinically important. It results in venous obstruction and bowel-wall oedema that can progress to bowel necrosis, perforation, and, rarely, death.

Most commonly occurs in infants aged between 3 and 12 months, with a peak at the age of approximately 9 months.

Presentation often includes colicky abdominal pain, flexing of the legs, fever, lethargy, and vomiting, with blood in the stool in some cases.

When there is clinical suspicion, imaging has a valuable role. Plain abdominal x-ray may reveal an intestinal obstruction and paucity of wind in the right lower quadrant. The presence or absence of free gas will influence subsequent management; abdominal ultrasound undertaken by an experienced radiologist will usually establish the diagnosis; contrast enema (most often air but may be liquid contrast) is the most specific and sensitive test for diagnosis. As well as being used therapeutically, it can also be used diagnostically where doubt remains.

The pathological location is typically ileocaecal.

Treatment involves reduction by contrast enema. Air is likely to be more effective and safer than liquid; in cases where this is unsuccessful or where peritonitis exits, surgery is required. Open reduction is then performed in uncomplicated cases, and intestinal resection for cases complicated by bowel necrosis and perforation.

How well did you know this?
1
Not at all
2
3
4
5
Perfectly
142
Q

Epidemiology of intussusception

A

18-56 per 100k
5-7m normally
75% <12

How well did you know this?
1
Not at all
2
3
4
5
Perfectly
143
Q

Aetiology of intussusception

A

The aetiology of most cases of intussusception is unclear but is likely to be related to hyperplasia of Peyer’s patches and lymphoid tissue in the intestinal wall resulting from antecedent viral infection.

These enlarged lymph nodes have been suggested as the lead point in idiopathic intussusception. This review will focus on idiopathic intussusception in infants.

Pathological lead points are anatomical abnormalities of the intestine, such as luminal polyps, malignant tumours (including lymphoma), and benign mass lesions (e.g., lipomata, Meckel’s diverticulum, Henoch-Schonlein purpura, and enteric duplication cysts).

Intussusception is the telescoping of one portion of the intestine (the intussusceptum) into the lumen of the intestine immediately distal to it (the intussuscipiens). The mesentery is dragged alongside the proximal bowel wall into the distal lumen resulting in obstruction of venous return. Oedema, mucosal bleeding, and increased pressure result. If arterial flow becomes compromised, ischaemia, necrosis, and perforation can occur.

How well did you know this?
1
Not at all
2
3
4
5
Perfectly
144
Q

RFs of intussusception

A

STRONG
Male
6-12m

WEAK
Antecedent viral illness
1st gen rotavirus vaccination

How well did you know this?
1
Not at all
2
3
4
5
Perfectly
145
Q

Sx of intussusception

A

COMMON
RFs
Male
6-12m
Vomiting (may be bilious/non bilious)
Lethargy / irritability IN BETWEEN WAVES of pain ( can be the only Sx)
Blood per rectum -> “redcurrent jelly stool”
Pallor
Palpable abdominal mass (usually RUQ/EPIgastrum)

UNCOMMON
Hypovolaemic shock
Poor feeding
Abdo distention

How well did you know this?
1
Not at all
2
3
4
5
Perfectly
146
Q

Ix of intussusception

A

Abdo plain Xray (always initial investigation if perforation or obstruction is suspected) - may appear normal; visible abdominal mass; abnormal wind pattern; air-fluid levels; dilated bowel loops; empty right lower quadrant; ‘target sign’; free intra-abdominal air (may be indicative of intestinal perforation as a complication of intussusception)

USS (higher sensitivitt/spec) - tissue mass; ‘target sign’; doughnut sign; multiple concentric ring sign; crescent-in-doughnut sign; pseudokidney sign; sandwich sign; abnormal Doppler flow

Can do diagnostic enema - meniscus sign; coiled spring sign

How well did you know this?
1
Not at all
2
3
4
5
Perfectly
147
Q

Rx of intussusception

A

STABLE
Fluid resus and contrast enema reduction
BS ABx - ie gentamycin/vanc/piptaz

UNSTABLE - shock/perforation/peritonitis/bowel wall necrosis
Fluid resus + urgent surgical reduction

How well did you know this?
1
Not at all
2
3
4
5
Perfectly
148
Q

Prognosis of intussusception

A

The recurrence rate of intussusception after contrast enema (air or contrast reagent) reduction is approximately 10% and does not significantly differ based on the type of contrast reduction performed.

Surgical reduction has been associated with a recurrence rate of 2% to 5%.

Mortality in intussusception is related to delay in presentation, septic shock, and inadequate fluid resuscitation.

How well did you know this?
1
Not at all
2
3
4
5
Perfectly
149
Q

Complications of intussusception

A

Intestinal ischaemia, necrosis and perforation

How well did you know this?
1
Not at all
2
3
4
5
Perfectly
150
Q

A 9-month-old boy presents to the emergency department with a 24-hour history of colicky abdominal pain, anorexia, fever, and progressive lethargy. Episodes of pain last 1 to 2 minutes marked by crying and drawing his knees to his chest, alternating with 20-minute pain-free periods where he behaves normally. The infant has vomited yellow material several times, which became green just prior to coming to hospital. Also, just prior to presentation, he had a bowel movement that appeared red, similar to redcurrant jelly. An abdominal examination reveals a distended, tender abdomen with pain out of proportion to examination, and a palpable right-sided abdominal mass. This could only be felt when he was settled between waves of pain, and was easiest to feel when he was lying on his side on his mother’s lap.

A

intussusception

How well did you know this?
1
Not at all
2
3
4
5
Perfectly
151
Q

The classic triad of intussusception includes colicky abdominal pain, redcurrant jelly stool, and a palpable abdominal mass. This triad is present in 20% to 40% of patients. In 60% of patients, at least 2 of these features are present at the time of presentation. [3] Lethargy has also been identified as a common presenting symptom of intussusception. [5] Symptoms that have been identified as independently predictive of intussusception include rectal bleeding, history of vomiting, and male sex. [6] Ileo-ileal intussusception can occur as a postoperative event, typically following abdominal surgery, especially resection of Wilm’s tumour or neuroblastoma, pull-through for Hirschsprung’s disease, and correction of malrotation.

A

intussusception

How well did you know this?
1
Not at all
2
3
4
5
Perfectly
152
Q

Define laryngomalacia

A

Laryngomalacia (LM) is a congenital abnormality that predisposes to dynamic supraglottic collapse during the inspiratory phase of respiration, resulting in intermittent upper airway obstruction and stridor. LM is the most common source of stridor in infants and the commonest congenital laryngeal anomaly.

Congenital abnormality of the larynx cartilage that predisposes to dynamic supraglottic collapse during the inspiratory phase of respiration, resulting in intermittent upper airway obstruction and stridor.

Most common laryngeal anomaly and most frequent congenital cause of stridor in infants.

Natural history is presentation in early infancy with maximal symptoms at 6 to 8 months before gradual improvement and spontaneous resolution within 12 to 24 months.

Presents with inspiratory stridor. Some patients have upper airway obstruction with associated feeding difficulties. Frequently associated with GORD.

Diagnosis is made by the clinical features and with flexible laryngeal endoscopy. Direct rigid laryngoscopy under anaesthesia may also be required. Possibility of additional airway lesions should be considered.

Treatment depends on disease severity; observation alone (with treatment of associated GORD) is appropriate for most cases. Endoscopic supraglottoplasty may be required for more severe disease. Tracheostomy and pressure-assisted ventilation are other possible therapies.

How well did you know this?
1
Not at all
2
3
4
5
Perfectly
153
Q

Epidemiology of laryngomalacia

A

60-70% of congenital laryneal anomalies

M:F 2:1
Usually presents in first few weeks of life

How well did you know this?
1
Not at all
2
3
4
5
Perfectly
154
Q

Aetiology of laryngomalacia

A

The aetiology of laryngomalacia (LM) is still not fully understood, although several theories have been proposed.

  • Immaturity of cartilage
  • Neuromuscular incoordination
  • Abnormalities in supraglottic anatomy
How well did you know this?
1
Not at all
2
3
4
5
Perfectly
155
Q

RFs for laryngomalacia

A
STONG
GORD
Neurological abnormalities
Laryngeal anatomical abnormalities
Male
Genetic syndrome disorder
How well did you know this?
1
Not at all
2
3
4
5
Perfectly
156
Q

Sx of laryngomalacia

A

Onset within two weeks of birth
Stridor
Resp distress - Clinical features include nasal flaring; suprasternal, intercostal, or subcostal recession; abdominal respiration; and tracheal tug. Sternal recession and/or pectus excavatum may be present.
Resolution of symptoms by age 2

NORMAL CRY - This serves to differentiate from other laryngeal conditions where abnormal or absent cry is a feature (vocal cord palsy, laryngeal web).

Feeding difficulties

UNCOMMON
WL
Hypotonia
Dysmorphic features

How well did you know this?
1
Not at all
2
3
4
5
Perfectly
157
Q

Ix for laryngomalacia

A

Flexible laryngoscopy - dynamic collapse of the supraglottic tissues on inspiration; visible narrowing and obstruction of the supraglottic airway; anatomical anomalies; evidence of GORD

How well did you know this?
1
Not at all
2
3
4
5
Perfectly
158
Q

Rx of laryngomalacia

A

Observation (typically resolves by 2yo)
GORD Rx - ranitidine/omeprazole

BIPAP can be used

SEVERE disease:
SURGICAL Rx - supraglottoplasty

How well did you know this?
1
Not at all
2
3
4
5
Perfectly
159
Q

Complications of laryngomalacia

A
GORD
Life threatening airway OB
Failure to thrive
Aspiration
Tracheostomy-related comps
Pulmonary HTN
How well did you know this?
1
Not at all
2
3
4
5
Perfectly
160
Q

Prognosis of laryngomalacia

A

For most infants with laryngomalacia (LM) the prognosis is excellent. The natural course typically involves gradual worsening after onset, with subsequent spontaneous resolution. LM is most severe when the child is 6 to 8 months of age, before gradually improving spontaneously by 12 to 24 months of age.

How well did you know this?
1
Not at all
2
3
4
5
Perfectly
161
Q

A 3-month-old boy is referred by his paediatrician with intermittent inspiratory stridor and progressive feeding difficulties resulting in failure to thrive. The stridor has been present since he was 1 week old. His parents report that his noisy breathing is present more frequently and is particularly prominent when he is lying down, feeding, or crying. He has difficulty feeding and chokes frequently but has a normal cry. His mother reports that he feeds slowly and often has to stop several times during the feed to ‘gasp for breath’. He has recently been diagnosed with GORD.

A

laryngomalacia

How well did you know this?
1
Not at all
2
3
4
5
Perfectly
162
Q

A 6-week-old boy presents to the emergency department with inspiratory stridor, present since a few days after birth. The stridor is loud with associated mild suprasternal indrawing (tracheal tug). Otherwise, the infant is resting comfortably and is thriving (he is on the 75th centile for weight since birth).

A

laryngomalacia

How well did you know this?
1
Not at all
2
3
4
5
Perfectly
163
Q

Laryngomalacia (LM) is often detected in patients with concurrent neurological conditions or syndromes. Children with reduced tone of the larynx due to underlying neurological pathology often present with concurrent LM. These patients can be more difficult to treat due to their underlying hypotonia. A late-onset variant has also been described that gives feeding problems in toddlers, sleep apnoea in children, and intolerance of exercise in teenagers.

A

laryngomalacia

How well did you know this?
1
Not at all
2
3
4
5
Perfectly
164
Q

Define meconium aspiration syndrome

A

Meconium aspiration syndrome (MAS) is defined as respiratory distress in the newborn due to the presence of meconium in the trachea. It occurs exclusively in the immediate neonatal period. It also covers respiratory distress in an infant, born through meconium-stained amniotic fluid, that cannot be explained otherwise.

Defined as respiratory distress in the newborn due to the presence of meconium in the trachea.

Infants born through meconium-stained amniotic fluid are at risk of developing meconium aspiration syndrome, particularly in the presence of maternal and fetal risk factors.

Soon after birth, infants present with respiratory distress (tachypnoea, chest retractions, and hypoxia). Some may be asymptomatic and apparently vigorous at birth, and develop severe respiratory distress hours later. Signs of post-maturity, a strong risk factor, include green/yellow-coloured skin; long, stained nails; and dry, scaling skin.

Diagnosis is confirmed by chest x-ray.

Management is largely supportive. Infants should be monitored closely to ensure adequate oxygenation and ventilation.

In the presence of persistent pulmonary hypertension, inhaled nitric oxide and extracorporeal membrane oxygenation therapy may be required.

How well did you know this?
1
Not at all
2
3
4
5
Perfectly
165
Q

Epidemiology of meconium aspiration syndrome

A

MAS is a serious complication associated with meconium-stained amniotic fluid (MSAF). In general, MSAF is found in 7% to 22% of term deliveries, with higher rates (22% to 44%) in post-term deliveries (>42 weeks).

> MAS with gestational age
Induction of labour reduces chances

How well did you know this?
1
Not at all
2
3
4
5
Perfectly
166
Q

Aetiology of meconium aspiration syndrome

A

Meconium is the first stool of the newborn. It is a collection of secretions from the fetal intestine and is a sterile, thick, black-green, viscous material containing pancreatic and biliary secretions, swallowed fetal cells, hair, and vernix. If aspirated into the lungs it causes local inflammation of the airways and release of several cytokines. Presence of meconium in the amniotic fluid and aspiration of meconium-stained amniotic fluid (MSAF) are the main causes of MAS. Passage of meconium may be a physiological maturation phenomenon during fetal development, and MSAF is uncommon between 20 weeks’ and 34 weeks’ gestation.

MSAF may have pathological causes: fetal distress and intrapartum hypoxia secondary to placental insufficiency, cord compression, cord around the neck, oligohydramnios, maternal hypertension, pre-eclampsia, maternal diabetes, maternal drug abuse, or smoking.

These factors can lead to gasping and aspiration of meconium by the fetus and the newborn. Certain infections, such as group B streptococcal infections and listeriosis, may be associated with MAS. However, one study found no evidence of maternal or neonatal listeriosis in MAS cases and concluded that MSAF is not a useful indicator for listeriosis in mother or baby.

Maternal chorioamnionitis may also contribute to stress and fetal passage of meconium. During delivery, about 5% of neonates with meconium passage aspirate the meconium, triggering lung injury and respiratory distress.

How well did you know this?
1
Not at all
2
3
4
5
Perfectly
167
Q

RFs for meconium aspiration syndrome

A
STRONG
>42w
Maternal: HTN, pre-eclampsia, eclampsia, smoking, substance abuse
Fetal distress
Oligohydramnios
Thick meconium
Apgar score <7
Chorioamnionitis
CS

WEAK
Black/east asian
Male

How well did you know this?
1
Not at all
2
3
4
5
Perfectly
168
Q

Sx of meconium aspiration syndrome

A
RFs
Sx of postmaturity: green/yellow-coloured skin and umbilical cord; long, stained nails; dry and peeling skin; post-mature appearance; and loss of subcutaneous tissue
Tachypnoea
Cyanosis
Systolic murmur
Chest wall asymmetry (tension pneumo)
Dec AE
Barrel shaped chest
Grunting
Chest retractions
Rales
Rhonchi
Tachycardia 
Hypotension
How well did you know this?
1
Not at all
2
3
4
5
Perfectly
169
Q

Ix of meconium aspiration syndrome

A

CXR - irregular pattern, patchy infiltrations, atelectasis, hyperexpanded lung fields, consolidation; may show pleural effusion, pneumothorax, or pneumomediastinum; cardiomegaly may be seen

FBC - rule out infection
CRP - rule out infection
Blood culture - Normal

How well did you know this?
1
Not at all
2
3
4
5
Perfectly
170
Q

Rx of meconium aspiration syndrome

A
O2
ABx - gentamicin
CPAP ventilation
Vasopressor if low BP
Surfactant may be given through ET 

Severe resp distress: Morphine sulfate + intubation

Infants who are refractory to mechanical ventilation with high oxygen inspiration and surfactant treatment invariably have concomitant persistent pulmonary hypertension (PPHN). iNO should be given with a starting concentration of 20 ppm, along with conventional or high-frequency ventilation

ECMO - last line

How well did you know this?
1
Not at all
2
3
4
5
Perfectly
171
Q

Prognosis of meconium aspiration syndrome

A

The course of MAS may vary from mild to moderate disease to a severe form leading to death, unless prompt appropriate intervention is provided. Data from a large population of term babies with the diagnosis of MAS corroborate previous observations in smaller population studies.

In most infants (80%), the course of illness is short, lasting 3 to 4 days, after which they are discharged home.

Nine percent of patients are transferred to level III+ intensive care units. Most of these infants spend 1 to 2 weeks under respiratory support and are discharged home, 5% require oxygen support at discharge (at 28 days of life), and 4.9% developed seizures

Long-term follow-up studies suggest that MAS infants may have several neurological developmental disabilities, particularly if they have experienced in utero hypoxia.

How well did you know this?
1
Not at all
2
3
4
5
Perfectly
172
Q

Complications of meconium aspiration syndrome

A
Pneumothorax
Persistent pulmonary HTN
Pneumomediastinum
Chronic lung disease
Cerebral palsy
Seizure
Reactive airway disease
Infections
How well did you know this?
1
Not at all
2
3
4
5
Perfectly
173
Q

A term, male newborn weighing 2968 g (6 lb, 9 oz) has been delivered at a tertiary hospital at 39 weeks’ gestation by emergency caesarean section because of fetal tachycardia and meconium-stained amniotic fluid (MSAF). The mother is a 17-year-old primigravida who had group B beta-streptococcal-positive culture at 35 weeks’ gestation. At delivery, the infant is limp and blue with poor respiratory effort. He has an Apgar score of 4 at 1 minute. The infant’s skin and umbilical cord are stained green, and he continues to demonstrate tachypnoea, with a respiratory rate of 70/minute, mild retractions, and grunting.

A

Meconium aspiration syndrome

How well did you know this?
1
Not at all
2
3
4
5
Perfectly
174
Q

A female infant is delivered post-term (at 42 weeks, 6 days) by emergency caesarean section because of fetal distress and thick MSAF. The birth weight is 3000 g (6 lb 10 oz). She presents with moderate retractions and grunting with an Apgar score of 3 at 1 minute.

A

Meconium aspiration syndrome

How well did you know this?
1
Not at all
2
3
4
5
Perfectly
175
Q

Atypical presentations include an initial normal presentation, with deterioration later if meconium-stained amniotic fluid and fetal distress are not recognised at birth. In some term births in which the history of meconium aspiration is not obvious, cyanotic heart disease may be incorrectly diagnosed.

Although MAS is common in term and post-mature babies, it can also be seen in preterm infants, though less commonly. In preterm infants, MAS may be misdiagnosed as respiratory distress syndrome resulting from surfactant deficiency.

A

Meconium aspiration syndrome

How well did you know this?
1
Not at all
2
3
4
5
Perfectly
176
Q

Define Meckel’s diverticulum

A

The most common congenital malformation of the small bowel, Meckel’s diverticulum (MD) is present in 1% to 3% of the population.

It is a true diverticulum that results from the failure of the vitelline duct to obliterate during the fifth week of fetal development. Patients are often asymptomatic. However, this embryological remnant may cause bleeding, obstruction, inflammation, or perforation.

Most common congenital abnormality of the small bowel.

Majority of patients remain asymptomatic for their entire life.

Majority of symptomatic patients present before the age of 2 years.

GI bleeding is a common presenting symptom in children and adults. The most useful diagnostic tool in cases with bleeding is the technetium-99m pertechnetate scan (Meckel’s scan).

Obstructive symptoms are another common presenting symptom and may be caused by several mechanisms, including intussusception.

Meckel’s diverticulitis occurs in 20% of patients who become symptomatic and is clinically indistinguishable from appendicitis.

How well did you know this?
1
Not at all
2
3
4
5
Perfectly
177
Q

Aetiology of Meckel’s diverticulum

A

Meckel’s diverticulum (MD) is a congenital abnormality of unknown cause. It results from an incomplete obliteration of the omphalomesenteric (vitelline) duct. In the 3-week-old embryo, the yolk sac communicates with the gut through a wide vitelline duct, which receives its blood supply from paired vitelline arteries. During week 8, the duct is normally obliterated when the placenta replaces the yolk sac as the source of fetal nutrition. The left vitelline artery usually involutes and the right one forms the superior mesenteric artery.

How well did you know this?
1
Not at all
2
3
4
5
Perfectly
178
Q

Epidemiology of Meckel’s diverticulum

A

Most common congenital malformation of the small bowel

1% to 3% of the population.

Most asymptomatic

Males 2-3x more likely to have symptomatic complications

How well did you know this?
1
Not at all
2
3
4
5
Perfectly
179
Q

RFs for Meckel’s diverticulum

A

Age <2

How well did you know this?
1
Not at all
2
3
4
5
Perfectly
180
Q

Sx of Meckel’s diverticulum

A
Age <2
Haematochezia 
Intractable constipation 
N+V (SBO)
Abdominal cramps
RLQ abdominal pain

UNCOMMON
Palpable abdominal mass
Hypotension/tachycardia (GI bleed)

How well did you know this?
1
Not at all
2
3
4
5
Perfectly
181
Q

Ix for Meckel’s diverticulum

A

FBC - leukocytosis / low Hb
Technetium 99 - ectopic focus
Plain abdo X-ray - dilation/pneumoperitoneum/intussusception
CT abdo/pelvis - blind-ending fluid-filled and/or gas-filled structure in continuity with distal ileum

US abdomen / contrast enema - IF intussusception is suspected

How well did you know this?
1
Not at all
2
3
4
5
Perfectly
182
Q

Rx of Meckel’s diverticulum

A

ASYMPTOMATIC
- no Rx / could excise

SYMPTOMATIC
Bleeding - excision + ?blood transfusion

Perforation - give ABx: cefotaxime + clindamycin/metronidazole

How well did you know this?
1
Not at all
2
3
4
5
Perfectly
183
Q

Prognosis of Meckel’s diverticulum

A

If symptomatic patients are treated in a timely fashion, the prognosis for Meckel’s diverticulum (MD) is excellent.

The risk of developing a postoperative complication ranges from 2% to 7%.

The most common post-surgical complication is adhesive bowel obstruction.

How well did you know this?
1
Not at all
2
3
4
5
Perfectly
184
Q

Complications of Meckel’s diverticulum

A

PostOP:

  • wound infection
  • intestinal leakage
  • BO
How well did you know this?
1
Not at all
2
3
4
5
Perfectly
185
Q

A 20-month-old boy is admitted to hospital with a history of having passed a bloody stool 8 hours before presentation. He has previously been well. On examination, he is pale and distressed but has no abdominal mass or tenderness. A contrast enema proves negative for intussusception.

A

Meckel’s diverticulum

How well did you know this?
1
Not at all
2
3
4
5
Perfectly
186
Q

A 68-year-old man presents to the accident and emergency department with a 24-hour history of colicky central abdominal pain associated with anorexia and intractable constipation (obstipation). The pain is associated with nausea and vomiting. He has had no previous abdominal surgery. On examination, he is dehydrated with no clinical signs of sepsis. His abdomen is distended and diffusely tender with no rebound tenderness or guarding. He has no hernias. Laboratory studies reveal an elevated white blood cell count. The abdominal plain x-rays show grossly dilated small bowel loops with paucity of gas in the colon. However, after 24 hours of treatment for small bowel obstruction, his abdominal pain worsens.

A

Meckel’s diverticulum

How well did you know this?
1
Not at all
2
3
4
5
Perfectly
187
Q

Diverticulitis occurs in 20% to 30% of patients with Meckel’s diverticulum (MD) who become symptomatic and is clinically indistinguishable from appendicitis, with periumbilical pain that radiates to the RLQ.

Diverticular obstruction can lead to distal inflammation, necrosis, and perforation, resulting in an abscess, peritonitis, or, rarely, haemoperitoneum.

A

Meckel’s diverticulum

How well did you know this?
1
Not at all
2
3
4
5
Perfectly
188
Q

Define Neonatal jaundice

A

Neonatal jaundice is the yellowing discoloration of the skin and sclera of a neonate, which is caused by increased levels of bilirubin in the blood. A neonate refers to an infant in the first 28 days of life

Usually noted clinically when serum bilirubin is >85.5 micromol/L (5 mg/dL). Occurs in 50% to 70% of term neonates. Most cases physiological.

Jaundice in the first 24 hours of life is considered pathological.

Treatment for severe hyperbilirubinaemia includes phototherapy and/or exchange transfusion.

The major complication of unconjugated hyperbilirubinaemia is kernicterus.

How well did you know this?
1
Not at all
2
3
4
5
Perfectly
189
Q

Epidemiology of Neonatal jaundice

A

Most common condition in newborns requiring medical attention

50-70% of term babies
80% of preterm

Usually appears 2-4 days old -> resolves 1-2 weeks later without the need for Rx

Hyperbilirubinaemia <8% births

How well did you know this?
1
Not at all
2
3
4
5
Perfectly
190
Q

RFs for Neonatal jaundice

A
STRONG
East asian
American indian
Maternal diabetes
LBW
<37weeks
Decreased caloric intake/WL
Breast feeding 

WEAK
Oxytocin in labour
Delayed cord clamping

How well did you know this?
1
Not at all
2
3
4
5
Perfectly
191
Q

Aetiology for Neonatal jaundice

A

Physiological jaundice can be a result of:

Increased bilirubin load secondary to increased red blood cell (RBC) volume, decreased RBC life span, or increased enterohepatic circulation

Decreased uptake by the liver because of decreased ligandins or binding of ligandins to other anions

Decreased conjugation in the liver because of decreased uridine diphosphoglucuronyl transferase (UDPGT) activity. UGT1A1 gene polymorphisms of Gly71Arg and TATA promoter, which decrease UDPGT enzymatic activity, have been noted to be significant risk factors associated with neonatal hyperbilirubinaemia

Decreased excretion into bile.

Pathological jaundice with unconjugated hyperbilirubinaemia can be a result of:

Haemolytic anaemias: these result in increased destruction of RBCs, with resultant increased haem, which is converted to excess unconjugated bilirubin; the immature liver is unable to handle the excess load. They can be the result of blood group incompatibility (rhesus, ABO), RBC enzyme defects (glucose-6-phosphate dehydrogenase deficiency; pyruvate kinase deficiency), RBC membrane defects (e.g., hereditary spherocytosis, infantile pyknocytosis), thalassaemia, drug-induced (by vitamin K, sulphonamides, nitrofurantoin, anti-malarials, penicillin), or sepsis

Extravasation of blood: sequestration of blood in cavities result in increased bilirubin load. Examples include cephalhaematoma; intracranial, pulmonary, or gastrointestinal haemorrhage; large haemangiomas; excessive ecchymoses; or petechiae

Polycythaemia: increased number of RBCs leads to increased production of bilirubin

Increased enterohepatic circulation: delayed gastrointestinal transit increases bilirubin levels. Examples include intestinal atresia/stenosis, pyloric stenosis, Hirschsprung’s disease, meconium ileus/plug syndrome

Defective conjugation: congenital deficiencies of UDPGT enzyme include Crigler-Najjar syndrome; UDPGT enzyme inhibition can be the result of drugs (e.g., novobiocin), or Lucey-Driscoll syndrome

Metabolic conditions (galactosaemia, hypothyroidism, tyrosinosis, hypermethioninaemia, maternal diabetes)

Breastfeeding (including failure to establish lactation)

Decreased binding of bilirubin to albumin: increased availability of the free (unconjugated) bilirubin to cross the blood-brain barrier. This can be caused by drugs (sulphonamides, penicillin, gentamicin), acidosis, asphyxia, hypothermia, increased osmolality, or hypoglycaemia.
Pathological jaundice with conjugated hyperbilirubinaemia (direct bilirubin is >34.2 micromol/L (2.0 mg/dL)) can be a result of:

Hepatocellular disease:

Metabolic or genetic defects. Examples include alpha1-antitrypsin deficiency, cystic fibrosis, Zellweger’s syndrome, Dubin-Johnson syndrome (absence of multidrug resistance-associated protein 2 from the canalicular membrane of hepatocytes), Rotor’s syndrome (organic-anion-transporting polypeptide [OATP]1B1 and OATP1B3 are absent at the sinusoidal membrane of hepatocytes), and galactosaemia

Infections. Examples include rubella, cytomegalovirus, herpes, syphilis, hepatitis A and B, toxoplasmosis, and urinary tract infection with Escherichia coli

Total parenteral nutrition [10]

Neonatal haemochromatosis

Idiopathic neonatal hepatitis

Shock.

Intrahepatic biliary disease due to Alagille syndrome (arteriohepatic dysplasia), or inspissated bile syndrome

Extrahepatic biliary disease due to biliary atresia, choledochal cyst, bile duct stenosis, cholelithiasis.

How well did you know this?
1
Not at all
2
3
4
5
Perfectly
192
Q

Sx of Neonatal jaundice

A

Jaundice (first appearing on face - spreads caudally)
Male
FHx anaemia/splenectomy/jaundice
Maternal exposure to sulphonamides/antimalarials

BILIRUBIN ENCEPHALOPATHY Sx:
Hypertonia
High pitched cry
Retrocollis (cervical dystonia (CD) that produces patterned, repetitive muscle contractions that result in neck extension)
Opisthotonus (spasm of the muscles causing backward arching of the head, neck, and spine)

How well did you know this?
1
Not at all
2
3
4
5
Perfectly
193
Q

Ix of Neonatal jaundice

A

Transcutaneous bilirubinometer - screening test: elevated
Serum bilirubin - Increased

Coombs - to diagnose ABO/RH iso-immunisation
FBC - potential sepsis
Reticulocytes - pot sepsis
Peripheral blood smear - haemolysis causes

How well did you know this?
1
Not at all
2
3
4
5
Perfectly
194
Q

Rx of Neonatal jaundice

A

Physiological = no Rx
Jaundice is physiological if it occurs in the second postnatal day and resolves in 7-10 days and transcutaneous measurement is normal.

Acute bilirubin encephalopathy

  • Immediate exchange transfusion
  • Phototherapy
  • Hydration
  • IVIG

Treat underlying cause

How well did you know this?
1
Not at all
2
3
4
5
Perfectly
195
Q

Prognosis of Neonatal jaundice

A

Most neonates with neonatal unconjugated hyperbilirubinaemia do well after phototherapy and/or exchange transfusion. Kernicterus should be preventable if recommendations for hyperbilirubinaemia management are carried out in a timely manner.

How well did you know this?
1
Not at all
2
3
4
5
Perfectly
196
Q

Complications of Neonatal jaundice

A

Neurological damage
Acute bilirubin encephalopathy
Exchange transfusion complications (electrolyte disturbances, bleeding, infection, cardiac arrhythmias, thrombosis with embolisation, necrotising enterocolitis, and graft-versus-host disease)
Phototherapy complications (insensible water loss, loose stools, skin rash, and potential retinal damage)
Kernicterus (chronic bilirubin encephalopathy) - diagnosed pathologically by gross yellow staining and necroses of neurons in the basal ganglia, hippocampal area, and cerebellum. If the neonate survives, the clinical features include chorio-athetoid cerebral palsy, paralysis of upward gaze, sensorineural hearing loss, dental dysplasia, and intellectual deficits (less often in the mental retardation range).

How well did you know this?
1
Not at all
2
3
4
5
Perfectly
197
Q

A baby boy of approximately 36 weeks gestational age is born to a primigravida mother. Pregnancy and delivery are uncomplicated, with Apgar scores of 9 at 1 and 5 minutes. Mother’s and baby’s blood groups are both O+. Mother chooses to exclusively breastfeed the baby. At 24 hours of life, the baby is noted to be jaundiced and the total serum bilirubin is noted to be 119.7 micromol/L (7 mg/dL). He is discharged home later the same day with an appointment for follow-up with the paediatrician at 1 week of age. However, 48 hours later, the baby is brought to the emergency department. History from the mother reveals that the baby has progressively become more jaundiced, is not breastfeeding well and is lethargic. Examination also reveals evidence of moderate volume depletion and significant jaundice (including the soles). The neurological examination is normal and total serum bilirubin is 342.1 micromol/L (20 mg/dL).

A

Neonatal jaundice

How well did you know this?
1
Not at all
2
3
4
5
Perfectly
198
Q

A term baby is born to a mother who had a previous baby with a history of jaundice in the newborn period, not requiring hospitalisation. Pregnancy and delivery are uncomplicated, with Apgar scores of 8 and 9 at 1 and 5 minutes, respectively. Mother’s and baby’s blood groups are O+ and B+, respectively. At 12 hours of life, the baby is noted to be jaundiced and the total serum bilirubin is 85.5 micromol/L (5 mg/dL). Tests reveal direct Coombs’ test to be positive and presence of microspherocytes on the peripheral smear.

A

Neonatal jaundice

How well did you know this?
1
Not at all
2
3
4
5
Perfectly
199
Q

The neonate may present with clinical signs of bilirubin encephalopathy. These include irritability with a high-pitched cry, possibly fever and increased muscle tone (usually involving the extensor group of muscles), and characteristically intermittent backwards arching of the neck (retrocollis) and trunk (opisthotonus). Decreased tone and abnormal Moro reflex are possible manifestations.

A

Neonatal jaundice

How well did you know this?
1
Not at all
2
3
4
5
Perfectly
200
Q

Define constipation

A

Constipation in childhood is the infrequent passage of stools leading to 1 or more of the following: painful defecation; overflow faecal incontinence; rectal faecal impaction; or active defecation avoidance behaviour.

Childhood constipation is typically characterised by infrequent bowel evacuations, large stools, and difficult or painful defecation.

Symptoms may result from low fibre, poor nutrient, and/or insufficient water intake, which leads to excessive hardening of the stool. This often starts as an acute problem but can progress to faecal impaction and chronic constipation.

Particularly in young children, any cause of painful defecation may provoke active withholding. Withholding may worsen the constipation and lead to a vicious cycle.

Duration of constipation and amount of stool burden may depend on capacity of the child’s rectum, the degree of megarectum, and other factors including rectal sensory function.

After faecal disimpaction, maintenance stool softeners are essential and are often required for many months or years to reduce relapse risk.

How well did you know this?
1
Not at all
2
3
4
5
Perfectly
201
Q

Aetiology of constipation

A

In most children, no aetiological factors can be found. Childhood constipation results from a combination of painful defecation (from any cause) and a rectum of sufficient capacity to allow stools to be withheld.

The likely sequence of events probably follows initial hard stools, often due to low fibre, poor nutrient, and/or insufficient water intake or, less commonly, due to delayed colonic motility. The excessive colonic time for stool transport leads to high levels of colonic reabsorption of water and further hardening of the stool. This often starts as an acute problem, which can then progress to:

Pain on passing the stool, which leads to involuntary and later learned sphincter contraction to avoid discomfort
Increasing delays between episodes of defecation, leading to enlargement of residual stool volume in the rectum
Further increase in the size of the loaded rectum and more effective withholding tactics
Increase in pain and fear as delayed stool is incrementally larger and thus even more painful
Eventually, overflow faecal incontinence occurs when the stool stimulates rectal contractions and the internal sphincter temporarily relaxes, with passage of softer stool going around retained hardened fragments.

How well did you know this?
1
Not at all
2
3
4
5
Perfectly
202
Q

RFs of constipation

A
STRONG
Low fibre
Poor nutrient
Genetic predisposition
Infection
Stress
Obesity 
Low birth weight
Psychiatric Hx

WEAK
Immune dysregulation
Low fluid intake
Trauma

How well did you know this?
1
Not at all
2
3
4
5
Perfectly
203
Q

Epidemiology of constipation

A

3% of consultations in an average paediatric clinic

Prevalence - 0.7% to 26.9%

How well did you know this?
1
Not at all
2
3
4
5
Perfectly
204
Q

Sx constipation

A
COMMON
Difficult/painful defecation
Long interval between stools
Faecal incontinence
Small volume, soft incontinent tool
Faecal incontinence
Palpable faecal mass per abdomen
Otherwise healthy child
Abdominal pain
UNCOMMON
Anal tissue 
Associated bladder problems
Abnormal anal appearance 
Abnormal neuro examination
How well did you know this?
1
Not at all
2
3
4
5
Perfectly
205
Q

Ix constipation

A

1st investigations:
NONE - history and examination findings are often sufficient to diagnose the condition.

CONSIDER
X-ray - if stool non-palpable
USS - may see rectal mass
Rectal mucosal biopsy - usually only done if abdominal distension and no passing of meconium: excludes Hirschsprung’s

How well did you know this?
1
Not at all
2
3
4
5
Perfectly
206
Q

Rx constipation

A

NO IMPACTION <1yo
Dietary modification
Osmotic laxative: Lactulose 1ml/kg OD/BD

NO IMPACTION >1yo
Could add Senna <1 tablet daily

With impaction <3yo
Usually need a laxative
- lactulose / glycerol rectal / polyethene glycol

With impaction >4yo
Could do all of above
AND phosphate enema

IE overall:
Osmotic laxative + stimulant laxative + faecal softener

Faecal softener: liquid paraffin / docusate sodium

How well did you know this?
1
Not at all
2
3
4
5
Perfectly
207
Q

Prognosis of constipation

A

1/3rd continues through puberty

Long-term prognosis depends on the degree of megarectum and the presence of other illnesses, particularly psychological disorders such as autism, or major psychosocial problems.

How well did you know this?
1
Not at all
2
3
4
5
Perfectly
208
Q

Complications of constipation

A
Anal fissure
Incontinence
Abdominal pain
Dehydration
Megarectum
Aspiration pneumonia
How well did you know this?
1
Not at all
2
3
4
5
Perfectly
209
Q

A 5-month-old baby boy presents with difficulty and delay in passing hard stools. His mother reports that he strains for several hours and may even miss a day, before passing stool with screaming and occasional spots of fresh blood on the stool or nappy. He has recently been weaned from breastfeeding to cows’ milk formula, which he had been reluctant to drink initially. The child is thriving and now feeding normally. There was no neonatal delay in defecation and no history of excessive vomiting or abdominal distension.

A

Constipation

How well did you know this?
1
Not at all
2
3
4
5
Perfectly
210
Q

A 14-year-old girl, concerned about body image, altered her diet and decreased her oral intake hoping to lose weight. Additionally, she avoided toilets at school due to their lack of cleanliness. She presented to her paediatrician with the complaint of abdominal pain, distension, bloating, and difficult, painful defecation.

A

Constipation

How well did you know this?
1
Not at all
2
3
4
5
Perfectly
211
Q

In 90% to 95% of children with constipation, the problem is functional. [1] However, organic causes should be considered in making this diagnosis. Chronic constipation may present after any cause of painful defecation such as an anal fissure, perianal streptococcal infection, food intolerance (particularly cows’ milk allergy), lichen sclerosis et atrophicus, and penetrative child sexual abuse. Refusal to defecate may also be a presentation of complex psychological problems, such as communication disorders that fall within the autistic spectrum or children with attention deficit disorder. [2] [3] Faecal incontinence in later childhood may occur after poor sensory or behavioural response to faecal loading of the rectum secondary to longstanding megarectum. Rarer causes of constipation include those presenting in the first weeks of life, such as Hirschsprung’s disease or anorectal anomalies. In teenage years, abnormal diets as well as eating disorders may lead to constipation.

A

Constipation

How well did you know this?
1
Not at all
2
3
4
5
Perfectly
212
Q

Define faecal incontinence in children

A

The involuntary passage of stool from the rectum through the anus resulting in episodes of stooling at inappropriate times, occurring at an age after which toilet training is typically complete (in most, 4 years of age or older).

Involuntary defecation at inappropriate times, occurring after the typical age of completed toilet training (generally 4 years).

In most cases, caused by overflow soiling associated with chronic constipation and severe faecal impaction in an otherwise healthy child without underlying anatomical abnormalities.

May have a significant negative impact on the quality of life for the child and the family.

A thorough history and physical examination are the crux of diagnosis. Laboratory and radiological studies may be used on a case-by-case basis to determine the underlying aetiology.

The goal of therapy is for patient to stool 1 to 3 times per day in the toilet with no episodes of soiling.

Treatment involves complete colon disimpaction, long-term treatment with laxatives and stool softeners, and bowel re-education. Any underlying causes should also be addressed.

How well did you know this?
1
Not at all
2
3
4
5
Perfectly
213
Q

Aetiology of faecal incontinence in children

A

Functional faecal incontinence associated with chronic constipation and severe faecal impaction

The child has no underlying anatomical abnormality, but the severe faecal impaction leads to dilation of the rectum.
Chronic dilation leads to decreased rectal sensation, so the child has less urge to defecate.
Softer stool forms above the hard impaction, and the softer stool leaks around the hard stool, known as ‘overflow’.
Functional non-retentive faecal incontinence

The exact aetiology is unknown, but possible contributing factors include psychological, behavioural, or emotional disturbances, genetics, and decreased motor function and sensation in the gastrointestinal tract.
Aetiology is likely to be multi-factorial. [10]
Organic faecal incontinence

Anatomical abnormalities of the gastrointestinal tract or neurological denervation of the gastrointestinal tract leads to inability to control bowel movements. [11]
The child has either an underlying anatomical abnormality (e.g., anorectal malformations, spinal and neurological abnormalities, or post-surgical abnormality) that causes the lack of bowel control, or a medical condition (e.g., hypothyroidism, coeliac disease, or cystic fibrosis) that may result in faecal impaction with overflow incontinence.
Medication overuse

Rarely, faecal incontinence may be due to overuse of medication (i.e., laxatives or orlistat) used mainly by adolescents and adults.

How well did you know this?
1
Not at all
2
3
4
5
Perfectly
214
Q

Epidemiology of faecal incontinence in children

A

The overall prevalence of faecal incontinence was found to be 3%, with more boys (3.7%) than girls (2.4%) being affected.

Anorectal malformations occur in 1 in 5000 live births, and 25% of these children experience faecal incontinence that requires long-term treatment.

How well did you know this?
1
Not at all
2
3
4
5
Perfectly
215
Q

RFs of faecal incontinence in children

A
STRONG
Chronic constipation
Male sex
Age 5-6yo
Diet lacking in fibre
Inadequate fluid intake
Delayed or inadequate toilet training
Anorectal Malformations
Hirschprung's (50% post surgery)
Spinal abnormalities ie bifida

WEAK
Psychological or behavioural problems due to stressful family events

How well did you know this?
1
Not at all
2
3
4
5
Perfectly
216
Q

Sx of faecal incontinence in children

A
COMMON
Hx surgery 
Faecal soiling or diarrhoea
Constipation
Perianal skin irritation
Behaviour problems
Abdo cramping
Posturing
Enuresis
Anorectal malformation 
Anormal abdo examination 

UNCOMMON
Abnormal rectal examination
Spinal deformities
Weakness/absent reflexes

How well did you know this?
1
Not at all
2
3
4
5
Perfectly
217
Q

Ix faecal incontinence in children

A

1st:
Abdo Xray - severe stool retention in faecal incontinence associated with chronic constipation; large, distended bowel loops (bottleneck appearance) may be noted in Hirschsprung’s disease

OTHER
Consider barium enema if suspecting hirschsprungs (no poo in >72hrs)

If hypo thyroid Sx - TSH/T4

Sweat chloride if suspect CF

Manometry studies - abnormal sphincter

How well did you know this?
1
Not at all
2
3
4
5
Perfectly
218
Q

Rx of faecal incontinence in children

A

With faecal impaction:

  1. Oral laxative - liq paraffin, magnesium citrate or polyethylene glycol
  2. Enema - sodium phosphate or liquid paraffin
  3. BOTH enema and laxative

With organic cause unresponsive to Rx - surgery

Bowel retraining - get child to strain for 5-10 mins per day 3x

Loperamide

How well did you know this?
1
Not at all
2
3
4
5
Perfectly
219
Q

Prognosis of faecal incontinence in children

A

Functional non-retentive
In one study, only 29% of the patients with functional non-retentive faecal incontinence were successfully treated after 2 years of intensive therapy. After 18 years of age, 15% continued to have faecal incontinence despite treatment.

Anorectal malformations
Patients with vestibular fistula, perineal fistula, rectal atresia, recto-urethral bulbar fistula, or imperforate anus without fistula will probably have voluntary bowel movements by 3 years of age.

How well did you know this?
1
Not at all
2
3
4
5
Perfectly
220
Q

Complications of faecal incontinence in children

A

Poor QoL
Irritant dermatitis of peri-anal area / buttocks / perineum
Worsening soiling
Enuresis

How well did you know this?
1
Not at all
2
3
4
5
Perfectly
221
Q

Concerned parents of an embarrassed 7-year-old boy present with a complaint of frequent faecal soiling of his underwear for the past 6 months. The boy also experiences frequent bedwetting and has a large, painful bowel movement in the toilet only twice a week. The boy is otherwise well and active, and admits to not wanting to use the toilet at school or to stop playing video games to use the toilet at home.

A

Faecal incontinence in children

Functional

How well did you know this?
1
Not at all
2
3
4
5
Perfectly
222
Q

Define enuresis

A

Enuresis is defined as normal micturition that occurs at an inappropriate or socially unacceptable time or place. As recommended by International Children’s Continence Society, in this monograph ‘enuresis’ is reserved for micturition during sleep, or bedwetting. Daytime wetting is called ‘incontinence’.

Primarily nocturnal symptoms in children older than 5 years of age.

Differentials include diabetes, medications, emotional problems, UTI, spina bifida, seizure disorder, and neurogenic bladder.

Treatment is commonly involves behavioural changes, alarm therapy, or desmopressin.

Emotional support and encouragement is vital to management.

How well did you know this?
1
Not at all
2
3
4
5
Perfectly
223
Q

Epidemiology of enuresis

A

Prevalence at 5yo = 5-10%

A report from the UK states that between 2% and 3% of 12- to 14-year-olds and 1% to 2% of people aged 15 years and older wet the bed twice a week on average.

In otherwise healthy adults aged 18 to 64 years, studies show a 0.5% prevalence of enuresis.

How well did you know this?
1
Not at all
2
3
4
5
Perfectly
224
Q

Aetiology of enuresis

A

Disorders of sleep arousal

Decreased functional bladder capacity

Nocturnal polyuria
- Poorly concentrated urine as assessed by urinalysis can also be indicative of nocturnal polyuria.

Nocturnal overactive bladder
- Children who do not respond to interventions with alarm therapy or desmopressin may suffer from overactive bladder. Often daytime symptoms are masked by moderating fluid intake to minimise daytime urination.

How well did you know this?
1
Not at all
2
3
4
5
Perfectly
225
Q

RFs of enuresis

A

STRONG
Genetic predisposition
Upper airway ob/sleep disordered breathing

WEAK
Constipation
ADHD
Psychological disorders

How well did you know this?
1
Not at all
2
3
4
5
Perfectly
226
Q

Sx of enuresis

A
RFs
Increased fluid intake at night
Urinary frequency - is variable and can be too frequent (>8 voids per day) or too infrequent (<3 voids per day).
Constipation
Caffeine

UNCOMMON
Urinary urgency

How well did you know this?
1
Not at all
2
3
4
5
Perfectly
227
Q

Ix of enuresis

A

Urinalysis - N
To exclude infection, signs of renal disease, poor urine concentrating ability, or glycosuria indicative of diabetes mellitus. In monosymptomatic nocturnal enuresis, U/A should be normal.

Urinary tract US - N

How well did you know this?
1
Not at all
2
3
4
5
Perfectly
228
Q

Rx of enuresis

A

<7yo - reassurance
- Rx only begins after this

Bladder training
Alarm therapy
Desmopressin - 0.2 to 0.4 mg orally once daily at bedtime, dose may be increased up to 0.6 mg/day

SOME PTs
Detrusor relaxing drugs: oxybutynin 5mg orally
Tolterodine 1mg orally

How well did you know this?
1
Not at all
2
3
4
5
Perfectly
229
Q

Complications of enuresis

A

Psychological disorders

How well did you know this?
1
Not at all
2
3
4
5
Perfectly
230
Q

Prognosis of enuresis

A

While short-term recurrence is the rule, especially with pharmacological interventions, long-term resolution is almost inevitable as only 0.5% of adults have nocturnal enuresis

How well did you know this?
1
Not at all
2
3
4
5
Perfectly
231
Q

A 7-year-old boy is brought to the clinic by his parents, who complain that he continues to wet the bed at night, 2 to 3 times a week. Further investigation determines that both the mother and the father had nocturnal enuresis but report that they just grew out of it. The child and family are clearly distressed about this and are willing to do whatever it takes to improve the situation.

A

Enuresis

How well did you know this?
1
Not at all
2
3
4
5
Perfectly
232
Q

The most common form of enuresis is monosymptomatic nocturnal enuresis, which is usually referred to as bedwetting. Other types of enuresis are associated with baseline voiding dysfunction. All other conditions in which voiding dysfunction manifests itself with the loss of urine are more appropriately defined as incontinence. Monosymptomatic enuresis occurring only at night is dealt with separately from incontinence, both diagnostically and therapeutically.

Parents of young children who have enuresis rarely bring them to a physician, as such behaviour is socially appropriate. The age at which this becomes unacceptable varies from culture to culture. The DSM-5 and the medical community generally recognise 5 years old as the cut-off. [2] [3] Additionally, the consensus of the International Children’s Continence Society is that the number of acceptable wet nights is between 1 and 3 per month; more than this and the child and/or parents are typically concerned enough to bring it to the attention of their physician. [4] Most of these children will present having never been consistently dry. Secondary enuresis can be associated with a stressful life event or a new medical condition, and warrants further investigation.

A

Enuresis

How well did you know this?
1
Not at all
2
3
4
5
Perfectly
233
Q

Define Rh incompatibility

A

Rhesus (Rh) incompatibility is caused by destruction of fetal red blood cells (RBCs) from transplacental passage of maternally derived immunoglobulin G (IgG) antibodies. IgG antibodies are produced by the maternal immune system, usually against the RhD antigen. These antibodies can freely cross the placenta, binding to and destroying RBCs. More than 50 known RBC antibodies potentially cause Rh incompatibility. The consequence is progressive fetal anaemia, which, untreated, may ultimately lead to hydrops fetalis (collection of fluid in serous compartments) and death.

Maternal antibodies produced in response to paternally derived D antigens on fetal red blood cells are the leading cause of severe haemolytic disease of the fetus and newborn (erythroblastosis fetalis).

Effective immunoprophylaxis of rhesus D-negative at-risk mothers is key to primary prevention.

Intrauterine fetal transfusion is a life-saving treatment for severely affected fetuses.

Survival rates are more than 90%.

How well did you know this?
1
Not at all
2
3
4
5
Perfectly
234
Q

Epidemiology of Rh incompatibility

A

About 15% of the white population has a RhD-negative blood type.

Seven percent of black people have this blood type.

In the UK, about 16% of the white population is RhD negative. In 2005, it was estimated that about 65,000 RhD-positive babies were born in the UK to women who were RhD-negative (accounting for 10% of all births).

How well did you know this?
1
Not at all
2
3
4
5
Perfectly
235
Q

Aetiology of Rh incompatibility

A

Rh incompatibility is caused by destruction of fetal red blood cells (RBCs) from transplacental passage of maternally derived immunoglobulin G antibodies. Passage of fetal cells into the maternal circulation and fetomaternal haemorrhage (FMH) is a frequent occurrence, detectable in 65% of pregnancies either antenatally or in the early postnatal period.

Placental trauma of varying degrees may lead to sensitising FMH. FMH increases throughout pregnancy (3% first trimester, 43% second trimester, and 64% third trimester).

FMH has been found in 1% to 6% of external cephalic versions. Small amounts of FMH (>0.1 mL) are potentially immunising and occur in 2% of patients undergoing amniocentesis. The incidence of FMH at the time of chorionic villus sampling is about 14%. The incidence is higher with transplacental cordocentesis.

How well did you know this?
1
Not at all
2
3
4
5
Perfectly
236
Q

RFs of Rh incompatibility

A
STRONG 
RhD neg mother
FMH
Invasive foetal procedures
Placental trauma
Abortion
Multiparity
Omission of Rh immunoprophylaxis

WEAK
ECV
Molar pregnancy
Ectopic pregnancy

How well did you know this?
1
Not at all
2
3
4
5
Perfectly
237
Q

Sx of Rh incompatibility

A

RFs

How well did you know this?
1
Not at all
2
3
4
5
Perfectly
238
Q

Ix of Rh incompatibility

A

1st
Maternal blood - Rh-neg
Maternal serum Rh Ab screen - +ve screen

Consider
Paternal blood typing
Fetal US - signs of fetal anaemia - may show subcutaneous oedema, ascites, pleural effusion, or pericardial effusion
Doppler of MCA - ≥1.5 MoM - fetal anaemia
Kleihauer-Betke test/flow cytometry- measures fetal blood in maternal circulation

How well did you know this?
1
Not at all
2
3
4
5
Perfectly
239
Q

Rx of Rh incompatibility

A

Anti-D immunoglobulin

  • After FMH
  • Amniocentesis / chorionic villus
  • > 40 weeks gestation

Intravascular intrauterine blood transfusions possible

Neonates with erythroblastosis are immediately evaluated by a paediatrician to determine the need for exchange transfusion, phototherapy, or intravenous immunoglobulin.

How well did you know this?
1
Not at all
2
3
4
5
Perfectly
240
Q

Prognosis of Rh incompatibility

A

RhD alloimmunisation is a disease that will usually progress during the pregnancy and in future pregnancies. Therefore, follow-up of these patients is required through antibody titres and fetal middle cerebral artery Doppler ultrasound during subsequent pregnancies.

How well did you know this?
1
Not at all
2
3
4
5
Perfectly
241
Q

Complications of Rh incompatibility

A

Hyperbilirubinaemia and kernicterus - Encephalopathy, athetoid cerebral palsy, and/or sensorineural deafness may result from deposition of bilirubin into the basal gangli

Transfusion related fetal Brady
Transfusion related neurodevelopmental disorders
Fetal / neonatal hydrops - Hydrops fetalis is defined as abnormal accumulation of fluid in 2 or more fetal compartments.
Neonatal anaemia

How well did you know this?
1
Not at all
2
3
4
5
Perfectly
242
Q

A 32-year-old woman presents at 25 weeks’ gestation in her third pregnancy with a positive antibody screen. She is known to be Rh-negative with a Rh-positive sexual partner. Two previous children were born overseas: the first child was carried to term and is healthy. The second child, also born at term, underwent phototherapy in the immediate neonatal period due to jaundice. The patient did not have anti-D prophylaxis given antenatally or postnatally in the previous pregnancies. Physical examination is normal.

A

Rh incompatibility

How well did you know this?
1
Not at all
2
3
4
5
Perfectly
243
Q

A 38-year-old primigravida woman presents for routine antenatal care. Her blood type is known to be Rh-negative with a negative indirect Coombs test, and her sexual partner is Rh-positive. She has been counselled regarding the need for Rh immunoprophylaxis at 28 weeks of pregnancy and postnatally if her newborn is found to be Rh-positive.

A

Rh incompatibility

How well did you know this?
1
Not at all
2
3
4
5
Perfectly
244
Q

Manifestations of severe erythroblastosis fetalis include ultrasound evidence of significant effusions in serous cavities, organomegaly, polyhydramnios, and extensive skin oedema (anasarca). Anti-RhD antibody titres in severe disease are usually high (>1:32 dilutions). Anti-Kell antibodies may be associated with profound fetal anaemia and hydrops in the presence of low antibody titres due to suppression of erythropoiesis. Evidence suggesting severe fetal anaemia includes high peak systolic velocities on Doppler ultrasound of the middle cerebral artery, low biophysical profile scores, and a sinusoidal fetal heart rate pattern. Although these manifestations of severe fetal disease are usually not detected in a first affected pregnancy, significant fetomaternal haemorrhage from any cause may lead to a secondary immune response and hydrops fetalis, even in a primiparous patient.

A

Rh incompatibility

How well did you know this?
1
Not at all
2
3
4
5
Perfectly
245
Q

Define intestinal malrotation

A

Intestinal malrotation is a term used to describe an entire spectrum of rotational and fixation disturbances that can occur during embryonic development.

Abnormal caecal attachments to the right upper peritoneal cavity (i.e., Ladd’s bands) can cross the second portion of the duodenum creating an extraluminal low-grade obstruction of the duodenum, which may present with signs of partial obstruction. However, the most significant pathological concerns in malrotation are a lack of intestinal fixation to the retro-peritoneum and a narrow midgut mesenteric base that predisposes to a twisting of the small bowel in the form of midgut volvulus. This condition creates an abrupt obstruction of the duodenum, resulting in bilious vomiting.

If the twisting at the base also obstructs flow in the superior mesenteric artery, the entire small intestine and proximal colon may become acutely ischaemic and subsequently necrotic within a few hours.

Patients with malrotation are at risk for volvulus and should be identified whenever possible to allow for proper therapy.

An entire spectrum of rotational and fixation disturbances that can occur during embryonic development.

The anatomical variant that poses the highest risk of volvulus is a narrow midgut mesenteric base accompanied by lack of retro-peritoneal midgut fixation. This variant cannot be reliably determined from any radiological studies.

Malrotation predisposes patients to a risk of midgut volvulus.

Emergency surgical consultation is appropriate before obtaining any diagnostic studies if midgut volvulus is suspected. Upper GI contrast studies should be ordered in patients who are stable.

Treatment is surgical correction, the procedure of choice being the Ladd procedure.

How well did you know this?
1
Not at all
2
3
4
5
Perfectly
246
Q

Epidemiology of intestinal malrotation

A

0.2% of upper GI contrast studies

Autopsy suggests 0.5%

How well did you know this?
1
Not at all
2
3
4
5
Perfectly
247
Q

Aetiology of intestinal malrotation

A

Congenital

How well did you know this?
1
Not at all
2
3
4
5
Perfectly
248
Q

RFs of intestinal malrotation

A

Embryological abnormality

How well did you know this?
1
Not at all
2
3
4
5
Perfectly
249
Q

Sx of intestinal malrotation

A
COMMON
Billous green vomiting (>90% of midgut volvulus)
Abdominal pain (inconsolable state)
Infant <1y
Normal abdo exam - a distended abdomen is more commonly a bowel obstruction further downstream. Vomiting and abdominal pain with a flat abdomen should alert physicians to the presence of volvulus
Distention
Tenderness
Tachycardia + HTN
Tachycardia + 
hypotension

UNCOMMON
WL
Dark blood in nappy
Rebound tenderness and guarding

How well did you know this?
1
Not at all
2
3
4
5
Perfectly
250
Q

Ix for intestinal malrotation

A

1st Ix

Upper GI contrast - right-sided duodenum (malrotation); duodenum courses inferior or medial to normal (malrotation); bird-beak cut-off of duodenum (volvulus); corkscrew of duodenum (volvulus); a web in the duodenum (duodenal atresia)

CT Abdo with oral+IV contrast - no oral contrast beyond duodenum (volvulus); no contrast in the distal superior mesenteric artery (volvulus with ischaemia); twirling of the superior mesenteric artery and vein (volvulus); transposition of superior mesenteric artery and vein (malrotation); a transition point in bowel calibre, right-sided duodenum; duodenum courses anterior or to right of superior mesenteric artery

Plain abdo x-ray - distended stomach and proximal duodenum with paucity of bowel gas seen distally (volvulus or duodenal web); normal x-ray (malrotation without volvulus or volvulus without complete obstruction)

FBC - may have elevated WCC/polycythaemia

ABG - metabolic acidosis with respiratory alkalosis resulting in profound decrease in partial pressure of carbon dioxide

How well did you know this?
1
Not at all
2
3
4
5
Perfectly
251
Q

Rx of intestinal malrotation

A

OBSTRUCTION WITH ISCHAEMIA
Emergency surgery - open laparotomy + Ladd procedure
+ Cefoxitin IV before surgery

Obstruction without ischaemia
Urgent surgery - ladd procedure open laparotomy

IF intermittent/partial
Can do open or laparoscopic

How well did you know this?
1
Not at all
2
3
4
5
Perfectly
252
Q

Prognosis of intestinal malrotation

A

Mortality with midgut volvulus is approximately 10% and usually depends on degree of intestinal necrosis.

Survivors of volvulus are at risk for recurrent volvulus of around 10%.

How well did you know this?
1
Not at all
2
3
4
5
Perfectly
253
Q

Complications of intestinal malrotation

A

Short-gut syndrome post op
Volvulus following Ladd procedure
Ladd procedure related adhesive small bowel

How well did you know this?
1
Not at all
2
3
4
5
Perfectly
254
Q

A 7-day-old female infant is brought to the emergency department with acute-onset bilious vomiting of 6 hours’ duration. The patient has previously tolerated breastfeeding without emesis and has stooled normally. Physical exam is benign without significant tenderness or distention.

A

Intestinal malrotation

How well did you know this?
1
Not at all
2
3
4
5
Perfectly
255
Q

A 1-month-old male infant is brought to the emergency department with acute onset of inconsolable discomfort and bilious vomiting of 6 hours’ duration. Physical exam reveals a tight, distended abdomen with some dark blood in the nappy. He is tachycardic and tachypnoeic.

A

Intestinal malrotation

How well did you know this?
1
Not at all
2
3
4
5
Perfectly
256
Q

Define child abuse

A

Child abuse (including neglect) is any form of maltreatment of a child, either by inflicting harm or by failing to act to prevent harm. Children may be abused in a family, institutional, or community setting, by those close to them, such as a parent or caregiver or, more rarely, by a stranger. They may be abused by adults or by other children. There are 4 categories of child abuse: physical abuse, emotional abuse (also called psychological maltreatment), sexual abuse, and neglect. This monograph primarily addresses physical child abuse. For information on sexual child abuse, please see our detailed content on sexual abuse.

How well did you know this?
1
Not at all
2
3
4
5
Perfectly
257
Q

Epidemiology of child abuse

A

Child abuse is a world-wide phenomenon and can affect children of all ages; however, the highest incidence occurs to infants and toddlers

In industrialised countries it is estimated that 4% to 16% of children are physically abused, around 10% are neglected or emotionally abused, 5% of boys and 5% to 10% of girls are exposed to penetrative sexual abuse, and 30% are exposed to any form of sexual abuse. Around 80% of child abuse is perpetrated by carers or parents.

How well did you know this?
1
Not at all
2
3
4
5
Perfectly
258
Q

Aetiology/RFs for child abuse

A

STRONG
Domestic violence
Substance abuse/ mental health disorder in parents
Excessive crying/frequent tantrums in infancy
Immature parents/coping skills
Parent / carer abused as child

WEAK
Poor socio-economic status
Demanding parenting role

How well did you know this?
1
Not at all
2
3
4
5
Perfectly
259
Q

Sx of child abuse

A

COMMON
RFs
Inconsistent/changing Hx
Inconsistent injuries in isolation/combination IE - unexplained bruising in a pre-mobile child and other suspicious injuries that do not fit with the developmental age of the child (e.g., a child not yet independently mobile may be unlikely to fall against a particular object).
Bruising
Subdural haemorrhages in infant/young toddler
Long bone fractures in premobile child - high specific for abuse
Rib fractures in absence of major trauma
Immersion scalds
Family known to social services
Poor bonding
Faltering growth
Dental neglect
Petechaie with bruising
Extensive multilayered retinal haemorrhages extending to peripheries

UNCOMMON
Small bowel perforation in child <3
Torn Frenum 
Apnoeas 
Cigarette burns
Frequent accidental poisonings
Contact burns
Dental injuries
Caustic burns
How well did you know this?
1
Not at all
2
3
4
5
Perfectly
260
Q

Ix for child abuse

A
FBC - check platelets/anaemia
Clotting profile - exclusion
Dilated fundoscopy 
Skeletal surgery
CT brain - haemorrhages
LFTs - raised with direct injury 
Serum calcium - N
Serum phosphate - N
Alk-phos - raised with fracture
PTH - N
Vit D - N
VWF assay
How well did you know this?
1
Not at all
2
3
4
5
Perfectly
261
Q

Rx of child abuse

A

Management of injuries
Reporting to authorities
- Doctors may be asked to provide written reports for use in multidisciplinary meetings, police investigations, and civil or criminal courts, and may be required to appear as witnesses (of fact or as experts) in court.
Social service intervention
Mental health services - CBT/play therapy

How well did you know this?
1
Not at all
2
3
4
5
Perfectly
262
Q

Prognosis of child abuse

A

The mortality from abusive head trauma (AHT) is high. Around 20% to 30% of AHT child victims die of their injuries, and around two-thirds survive only to have significant long-term disability, including neurological, behavioural, and cognitive sequelae.

Children who have been abused manifest a variety of negative outcomes as adults, including high-risk behaviours such as binge drinking, drug use, and criminal behaviour.

Child abuse of any form (emotional, physical, or sexual abuse, neglect) is also associated with increased mental health problems in adulthood. This may include depression, post-traumatic stress disorder, substance abuse, or anti-social personality disorder.

How well did you know this?
1
Not at all
2
3
4
5
Perfectly
263
Q

Complications of child abuse

A
Kyphosis 
Psyhological scarring
Difficulty with relationships
PTSD
Developmental delay 
Brain damage
Failure to thrive
Death
Blindness
Disfigurement
How well did you know this?
1
Not at all
2
3
4
5
Perfectly
264
Q

A 6-year-old boy is noticed by his teacher to have a black eye. When asked how he sustained the injury, the child states that his stepfather punched him. He is referred to social services and brought to the paediatrician for further assessment. On examination he has a bruise to the outer aspect of his left orbit. On careful inspection, there are 2 linear bruises to the left cheek extending towards the ear, a bruise to the anterior and posterior aspect of the right pinna, and 4 small bruises to the outer aspect of the upper arm. On close questioning, the boy states that his stepfather grabbed him by the arm and slapped him on the face after he had disturbed him when he was watching television. The boy also states that there are lots of arguments in the home, that his stepfather is frightening and has hit his mother, and that if he is naughty, his stepfather will grab his ear and twist it until it hurts.

A

Child abuse

How well did you know this?
1
Not at all
2
3
4
5
Perfectly
265
Q

A 5-month-old baby is brought to the accident and emergency department by ambulance. His mother states that he has not been feeding and has been irritable for the past 24 hours. She states that he vomited any food he was given and subsequently went “blue” and appeared to stop breathing, prompting her to call an ambulance. In the A&E department, the baby appears lethargic but responsive. Examination is unremarkable with the exception of a small bruise to the right cheek and a torn frenum (or frenulum). The mother is anxious about the baby. She is 19 years old and had a previous baby who died of SIDS. She had this baby with her second partner who has recently left her. The baby is admitted for observation and investigation. A modified Glasgow coma scale of 8 is recorded and the baby has a generalised seizure while on the ward. FBC, platelets, a clotting screen, a septic screen, and the biochemical profile are normal. Bi-lateral extensive retinal haemorrhages are noted on indirect ophthalmology. The skeletal survey shows healing posterior rib fractures on the left side to ribs 7 to 10. A CT brain scan shows multiple small subdural haemorrhages over the convexity and in the interhemispheric fissure, with diffuse hypoxic ischaemic injury. A diagnosis of non-accidental head injury seems most likely and a referral is made to police and social services.

A

Child abuse

How well did you know this?
1
Not at all
2
3
4
5
Perfectly
266
Q

Define osteomyelitis

A

Osteomyelitis is an inflammatory condition of bone caused by an infecting organism, most commonly Staphylococcus aureus . It usually involves a single bone but may rarely affect multiple sites.

Broadly, bone infection is either haematogenous (originating from bacteraemia) or contiguous focus (originating from a focus of infection adjacent to the area of osteomyelitis). Despite these different causes all forms of acute osteomyelitis may evolve and become chronic, sharing a final common pathophysiology, with a compromised soft-tissue envelope surrounding dead, infected, and reactive new bone.

Suspect osteomyelitis in those with a history of open fracture, recent orthopaedic surgery, or a discharging sinus; in the immunocompromised patient; or in the unwell child.

Plain radiographs provide a good initial imaging modality for screening acute and chronic osteomyelitis.

Magnetic resonance imaging and computed tomography (CT) may be used to determine diagnosis and treatment decisions. In some cases positron emission tomography/CT or single-photon emission computed tomography/CT may have a role.

Diagnosis must be confirmed using deep microbiological samples via radiological guided biopsy or open surgery.

In chronic osteomyelitis, surgery to remove the dead bone is the primary treatment modality. Antibiotics alone cannot achieve a cure.

Giving empirical antibiotics without microbiological sampling should be avoided, with the exception of patients who are septic and unwell. Even then, blood cultures should be taken before antibiotics are given.

Only cultures from deep sites are reliable. There is no value in surface or sinus swabs.

How well did you know this?
1
Not at all
2
3
4
5
Perfectly
267
Q

Epidemiology of osteomyelitis

A

A study in the US showed that the overall age- and sex-adjusted annual incidence of osteomyelitis was 21.8 cases per 100,000 person-years between 1969 and 2009. The annual incidence was higher for men than for women and increased with age.

How well did you know this?
1
Not at all
2
3
4
5
Perfectly
268
Q

Aetiology of osteomyelitis

A

Osteomyelitis may be caused from haematogenous spread, direct inoculation of micro-organisms into bone, or from a contiguous focus of infection. A trivial skin infection may be the source of bacteraemia, or it may have emerged as the result of a more serious infection such as acute or subacute bacterial endocarditis. Intravenous drug misuse has been linked to haematogenous osteomyelitis involving the long bones or vertebrae.

Common micro-organisms in acute haematogenous osteomyelitis

Infants:

S aureus
Group B streptococci
Aerobic gram-negative bacilli
Children up to 4 years:

S aureus
Streptococcus pyogenes
Haemophilus influenzae (in those not immunised)
Kingella kingae (increased incidence in children under 4 years)
Older children and adults:

S aureus
Older adults:

Gram-negative bacilli
Patients with intravascular devices:

S aureus
Candida species
Patients who misuse intravenous drugs:

S aureus
Pseudomonas aeruginosa
Patients with sickle cell disease; patients from developing countries:

S aureus
Salmonella species

How well did you know this?
1
Not at all
2
3
4
5
Perfectly
269
Q

RFs for osteomyelitis

A
STRONG
Penetrating injury
Surgical contamination
IV drug use
DM
Periodontitis
How well did you know this?
1
Not at all
2
3
4
5
Perfectly
270
Q

Sx of osteomyelitis

A
Non-specific pain at site of infection
Malaise and fatigue
Local inflammation, erythema, swelling
Low grade fever
Wound drainage 
Scars/previous flaps 
Reduced range of movement
UNCOMMON
UTI symptoms
Torticollis 
Limb deformity
Tenderness to percussion
How well did you know this?
1
Not at all
2
3
4
5
Perfectly
271
Q

Ix for osteomyelitis

A

WBC - High
ESR - usually raised
CRP - usually raised
Plain X-ray - lytic lesion/osteopenia

Consider
USS
MRI bone
CT scan
Radionucleotide scans
Histology - TB/actinomycosis require histology
How well did you know this?
1
Not at all
2
3
4
5
Perfectly
272
Q

Rx of osteomyelitis

A

Acute:
High dose ABx - take blood cultures

Chronic:
May need debridement

How well did you know this?
1
Not at all
2
3
4
5
Perfectly
273
Q

Prognosis of osteomyelitis

A

Management of osteomyelitis in dedicated units, with close multidisciplinary working, is associated with high success rates and high patient-satisfaction scores. The prognosis from early intervention is usually better, particularly in the context of implant-related infection. Delay in treating these patients is likely to worsen long-term outcomes. The functional outcome for osteomyelitis of the lower extremity depends on the status of the foot and knee.

How well did you know this?
1
Not at all
2
3
4
5
Perfectly
274
Q

Complications of osteomyelitis

A
Drug reactions
Flap failure
Amputation
Growth disturbance
Joint stiffness
Infection recurrence
Fracture
Neurological impairment secondary to bony collapse
IV catheter related infections
How well did you know this?
1
Not at all
2
3
4
5
Perfectly
275
Q

A 40-year-old man who suffered an open tibial fracture in a motor vehicle accident 6 months ago presents with swelling and pain in his lower leg.

A

Osteomyelitis

How well did you know this?
1
Not at all
2
3
4
5
Perfectly
276
Q

A 5-year-old boy fell off his bicycle 2 weeks ago and has stopped walking and complains of non-specific pain in his leg. His mother reports that he apparently has had flu, with fever and chills.

A

Osteomyelitis

How well did you know this?
1
Not at all
2
3
4
5
Perfectly
277
Q

Symptoms of osteomyelitis vary with the duration of the disease and may have a gradual onset over several days. A patient with acute osteomyelitis typically presents as acutely unwell, with bone pain in the region affected along with tenderness, warmth, and swelling. Pain may occur with or without movement. This classic presentation does not occur in all cases. Some sites, such as the vertebrae or pelvis, may present a diagnostic challenge with a systemically unwell patient displaying signs of sepsis but without clear localising signs.

Chronic osteomyelitis generally has a longer duration of symptoms. The pain may be less severe, with minimal fever and fewer constitutional symptoms. There is often a history of a discharging sinus or signs of old healed sinuses, soft-tissue abscesses, or scars from previous surgery or injury. Patients may have lived with discharging sinuses for years, having previously been told that there is nothing that can be done for their problem.

Chronic osteomyelitis may produce long-term ill health with weight loss, malaise, fatigue, or depressed mood. Acute systemic upset is less common but pyrexia, sweating attacks, and anorexia are associated with flare-ups of the disease.

A

Osteomyelitis

How well did you know this?
1
Not at all
2
3
4
5
Perfectly
278
Q

Define phimosis/paraphimosis

A

Paraphimosis is a condition caused when the foreskin of the uncircumcised penis is retracted and left behind the glans penis, leading to vascular engorgement and oedema of the distal glans. Medical emergency.

Caused when the foreskin of an uncircumcised penis is retracted and left behind the glans, leading to vascular engorgement and oedema of the distal glans.

In its acute form, it is a medical emergency requiring rapid evaluation and possible surgical decompression.

Can occur at any age but is most common in children and older people.

Circumcision is preventive and is curative if the process develops.

Most cases are iatrogenic in aetiology

  • Phimosis = unretractable foreskin
How well did you know this?
1
Not at all
2
3
4
5
Perfectly
279
Q

Epidemiology of phimosis/paraphimosis

A

The incidence of paraphimosis is unknown. Only isolated case reports exist in the literature. Due to the fact that most cases are iatrogenically or inadvertently induced, precise reporting of the condition may be compromised. Additionally, some patients may not report the condition or may allow it to evolve into the chronic form

How well did you know this?
1
Not at all
2
3
4
5
Perfectly
280
Q

Aetiology of phimosis/paraphimosis

A

The most common aetiology follows retraction of the foreskin of an uncircumcised male penis by a health professional during penile examination, catheterisation, or cystoscopy, who then neglects to return the foreskin back over the glans penis.

The coexistence of poor hygiene and recurrent bouts of bacterial infection (balanitis) makes this condition more likely. Additional causes of paraphimosis include sexual intercourse in an uncircumcised male with phimosis (narrow opening of the foreskin), varying sexual practices that cause constriction of the foreskin, penile piercings, parasitic infestations, lichen sclerosis, inadequate circumcision, or haemangiomas of the penis.

How well did you know this?
1
Not at all
2
3
4
5
Perfectly
281
Q

RFs for phimosis/paraphimosis

A
STRONG
Lack of circumcision
Urinary catheterisation
Dependence on carer
Tight foreskin
Phimosis
Poor hygiene
Bacterial infection
Parasitic infection
Lichen sclerosis
DM
Penile piercing
Haemoangiomas

WEAK
Peripheral vascular disease
Inadequate circumcision

How well did you know this?
1
Not at all
2
3
4
5
Perfectly
282
Q

Sx of phimosis/paraphimosis

A
COMMON 
Penile pain
Band of retracted foreskin tissue beneath the glans
Swollen glans penis
Indwelling catheter 

UNCOMMON
Black tissue on glans
Non-pliable glans penis
Inability to urinate freely

How well did you know this?
1
Not at all
2
3
4
5
Perfectly
283
Q

Ix for phimosis/paraphimosis

A

Clinical

How well did you know this?
1
Not at all
2
3
4
5
Perfectly
284
Q

Rx of phimosis/paraphimosis

A

Ischaemia and necrosis:
Urgent surgery

No Ischaemia/Necrosis
Manual manipulation
Surgical reduction

How well did you know this?
1
Not at all
2
3
4
5
Perfectly
285
Q

Prognosis of phimosis/paraphimosis

A

Following prompt reduction of paraphimosis, most patients will have a full recovery. Foreskin swelling will subside over subsequent days. Depending on the aetiology, appropriate education of the patient and/or carers is recommended. Some patients may not warrant circumcision. Patients who have evidence of phimosis should be counselled about the appropriateness of circumcision. In addition, those who have a repeated episode of paraphimosis or have undergone a dorsal slit procedure warrant follow-up circumcision.

How well did you know this?
1
Not at all
2
3
4
5
Perfectly
286
Q

Complications paraphymosis

A

Necrosis of glans+foreskin

How well did you know this?
1
Not at all
2
3
4
5
Perfectly
287
Q

A 72-year-old uncircumcised man is brought to the emergency department with concerns about penile swelling. He has a history of dementia and is in a facility for long-term care. He is having problems with incontinence and a decision was made to manage this with long-term Foley catheter drainage. The catheter was changed 2 days prior to presentation in the emergency department. On the day of presentation, the foreskin is noted to be retracted completely, with engorgement and oedema of the glans and pain on palpation.

A

Paraphymosis

How well did you know this?
1
Not at all
2
3
4
5
Perfectly
288
Q

A 6-month-old uncircumcised boy is brought to the paediatrician’s clinic with a 24-hour history of penile swelling. The parents have noted that the foreskin has been difficult to retract. During his bath the night before, the mother had finally been able to completely retract the foreskin. In the clinic, the foreskin remains retracted below the glans penis, which is oedematous and engorged. The infant appears to be in pain when the penis is manipulated

A

paraphymosis

How well did you know this?
1
Not at all
2
3
4
5
Perfectly
289
Q

Due to embarrassment, some patients may not present until necrosis of the glans penis has occurred. A chronic form may also occur, and in these patients a fibrotic ring and chronic engorgement and oedema of the glans penis are noted.

A

paraphymosis

How well did you know this?
1
Not at all
2
3
4
5
Perfectly
290
Q

Define pyloric stenosis

A

In infantile hypertrophic pyloric stenosis (HPS), hypertrophy of the pyloric sphincter results in narrowing of the pyloric canal. It is the most common cause of gastric outlet obstruction in the 2- to 12-week-old age group. Pyloric stenosis leads to progressive and projectile vomiting.

Recurrent projectile non-bilious vomiting, typically in a 3- to-6-week-old infant (usually male), but may occur on older infants.

Features may include a history of feeding intolerance with multiple formula changes.

Failure to thrive/weight loss may progress to increasing volume depletion. An olive-shaped mass may be palpable in the right upper abdomen.

Ultrasound shows pyloric channel length >17 mm and pyloric muscle thickness >4 mm.

Treatment is with intravenous fluid and electrolyte replacement, followed by pyloromyotomy (open or laparascopic).

Complications of surgery include wound infection, gastric or duodenal mucosal perforation, or incomplete myotomy.

How well did you know this?
1
Not at all
2
3
4
5
Perfectly
291
Q

Epidemiology of pyloric stenosis

A

2-4/1000 births

M:F 4:1

How well did you know this?
1
Not at all
2
3
4
5
Perfectly
292
Q

Aetiology of pyloric stenosis

A

Hyperacidity as a result of antral distention with feeding and hypertrophy of the pylorus from repeated contraction is believed to be a cause.

Additionally, poor pyloric muscle neuronal innervation is believed to play a role.

Nitric oxide synthase deficiency is also implicated as a biochemical cause, by decreasing smooth muscle relaxation.

Exposure to oral erythromycin, especially during the first 2 weeks of life, is associated with significant odds of developing pyloric stenosis.

How well did you know this?
1
Not at all
2
3
4
5
Perfectly
293
Q

RFs for pyloric stenosis

A

STRONG
1st born male infant
FHx

WEAK
Prematurity
Erythromycin in 1st 2 weeks life
Maternal exposure to macrolides

How well did you know this?
1
Not at all
2
3
4
5
Perfectly
294
Q

Sx of pyloric stenosis

A
COMMON
RFs
Non-bilious projectile vomiting 
3-6 weeks old
Upper abdominal mass
Multiple formula changes
Tachycardia (volume depletion)
Decreased wet nappies
Dry MM
Flat/depressed fontanelles
Constipation
Poor weight gain
Irritability 

UNCOMMON
Peristaltic waves

How well did you know this?
1
Not at all
2
3
4
5
Perfectly
295
Q

Ix for pyloric stenosis

A

U+E - in severe cases, hypochloraemic alkalosis and mild hypokalaemia

USS abdomen - pyloric muscle thickness >4 mm, pyloric canal length >17 mm

How well did you know this?
1
Not at all
2
3
4
5
Perfectly
296
Q

Rx of pyloric stenosis

A

IV fluid resus - Intravenous fluid replacement should be provided at 1.5 times maintenance rate with 5% dextrose plus 0.45% saline.

Pyloromyotomy - after electrolyte balance corrected

How well did you know this?
1
Not at all
2
3
4
5
Perfectly
297
Q

Prognosis of pyloric stenosis

A

The success of surgical treatment is near 100% and complication rates are negligible.

How well did you know this?
1
Not at all
2
3
4
5
Perfectly
298
Q

Complications of pyloric stenosis

A

Post op emesis
Surgical wound infection
Surgical mucosal perforation
Incomplete myotomy

How well did you know this?
1
Not at all
2
3
4
5
Perfectly
299
Q

Define sepsis in children

A

Sepsis is a clinical syndrome resulting from a dysregulated immune response to infection. It is characterised by derangements in multiple pathobiological processes, which may lead to widespread tissue injury. It encompasses a clinical spectrum of severity, including severe sepsis, septic shock, and multi-organ failure. Sepsis is a leading cause of morbidity and mortality in children worldwide.

Initial clinical presentation may be non-specific (especially in younger age groups).

Given the time-critical nature of severe sepsis and septic shock, when sepsis is suspected on clinical grounds it is usually best to initiate sepsis investigations and treatment, including administering antibiotics and fluid resuscitation. These should continue until sepsis has been excluded.

Progression to organ failure and shock is often very rapid, so early recognition and treatment is crucial.

Empirical broad-spectrum antibiotic therapy (based on the most probable pathogens) should be administered as soon as possible, and always within the first hour following recognition.

Other treatments are primarily supportive, and should be delivered according to internationally recognised consensus-based guidelines.

How well did you know this?
1
Not at all
2
3
4
5
Perfectly
300
Q

Epidemiology of sepsis in children

A

Case fatality 8.9%

Roughly 7.7% of paediatric ICU admissions

How well did you know this?
1
Not at all
2
3
4
5
Perfectly
301
Q

Aetiology of sepsis in children

A

Early onset neonatal - think GBS, S.aureus, H influenza and coat neg staphylococci

Late onset neonatal - coagulation neg staphylococci

Infants + young children - S.pneumonia, N meningitidis, S aureus, H influenza

Fungal (e.g., Candida species, Aspergillus species) and viral (e.g., influenza, respiratory syncytial virus, human metapneumovirus, varicella, and herpes simplex virus) pathogens account for up to 5.3% and 2.9% of severe sepsis in children

How well did you know this?
1
Not at all
2
3
4
5
Perfectly
302
Q

RFs for sepsis in children

A
STRONG 
Immunodeficiency
Comorbidities
Younger age
Perinatal RFs for infection - maternal fever, prolonged rupture of membranes (>18 hours), maternal carriage of group B streptococci, and chorioamnionitis; fetal factors include fetal distress and poor 5-minute Apgar score (i.e., ≤6).
Healthcare associated factors: parenteral nutrition, mechanical ventilation, chronic central venous access, and use of vascular catheters (arterial, central venous, or peripheral).
Recent surgery 
Breached skin integrity

WEAK
Male

How well did you know this?
1
Not at all
2
3
4
5
Perfectly
303
Q

Sx of sepsis in children

A
COMMON
RFs
Fever/low temp
Tachypnoea
Tachycardia
Bradycardia (neonates/infants)
Altered mental state
Decreased perfusion 
Change in feeding
Dec UO
Mottling of skin, ashen appearance, cyanosis
Low O2 sat
UNCOMMON
Vasoplegia
Non-blanching purpuric rash
Hypotension
Specific focal Sx
How well did you know this?
1
Not at all
2
3
4
5
Perfectly
304
Q

Ix for sepsis in children

A
FBC - inc WCC/thrombocytopenia
Serum glucose
BLOOD CULTURE - +ve
Urinalysis - UTI focus
Urine culture - UTI focus
Blood gases - base deficit
Serum lactate - >2 usually
U+E - deranged
Serum creatinine - elevated
LFTs - abnormal
Coagulation studies - abnormal
CRP - elevated
CXR - may reveal focus in older children 

CONSIDER
LP - if suspect meningitis
HSV PCR blood/CSF

How well did you know this?
1
Not at all
2
3
4
5
Perfectly
305
Q

Rx of sepsis in children

A

SEPSIS 6:
1. Administer oxygen - keep >94%
2. Take blood cultures - think source control
3. IV BS-ABx
4. IV fluid if lactate >2mmol - 500STAT
NB lactate >4 = call critical care
5. Serial lactate - recheck after 10ml challenges
6. Measure UO + complete fluid balance chart

  • Maintain airway
  • Consider vasoactive-inotropic support, IE adrenaline/NA
    0. 1-1microgram/kg/min

May need blood transfusion
Corticosteroid

EMPIRAL ABX:
Apicillin + Cefotaxime/Gentamicin

OR

BenPen + Gentamicin

CONSIDER
Aciclovir if severe sepsis (may be HSV1)
Nystatin - prevents candidiasis

IF neonatal - also give prostaglandins (Rx until duct-dependent cardiac lesion can be ruled out)

How well did you know this?
1
Not at all
2
3
4
5
Perfectly
306
Q

Prognosis of sepsis in children

A

Without treatment, severe sepsis carries a mortality rate in excess of 80%.

With treatment, overall mortality is approximately 10% in children up to the age of 19 years.

In children with cancer, overall mortality from sepsis is 17%. The rate increases to 30% in children who have undergone haematopoietic stem cell transplants.

How well did you know this?
1
Not at all
2
3
4
5
Perfectly
307
Q

Complications of sepsis in children

A
AKI
Myocardial dysfunction
DIC
Hypoglycaemia 
Hyperglycaemia
NEC
MOF
Persistent pulmonary hypertension of the newborn
Hypocalcaemia
Abdominal compartment syndrome
Thyroid insufficiency
Neurological sequelae
How well did you know this?
1
Not at all
2
3
4
5
Perfectly
308
Q

A 5-week-old, full-term male infant presents with progressive post-feeding emesis for the past 2 weeks. Initially he was diagnosed as having formula intolerance; formula type was changed several times without relief. Subsequently, he was thought to have gastro-oesophageal reflux. The parents continue to report non-bilious post-feeding emesis, which has become progressively forceful and projectile.

A

Pyloric stenosis

How well did you know this?
1
Not at all
2
3
4
5
Perfectly
309
Q

A previously well 1-year-old girl presents to the accident and emergency department with a history of lethargy and fever for 24 hours. She recently had symptoms suggestive of a viral upper respiratory tract infection. Her parents report that for a few hours prior to presentation she had become drowsy and difficult to rouse. They also report that they had noticed a rash developing on her trunk and limbs shortly before presentation. On initial assessment the following features are identified: reduced level of consciousness (response to painful stimulus only); tachycardia (heart rate 190 beats per minute); prolonged capillary refill time (>5 seconds peripherally); cold peripheries (core-toe temperature gap >10°C [>18°F]); fever (core temperature 39°C [102°F]); tachypnoea (respiratory rate 40 beats per minute) and grunting on expiration; and a widespread, non-blanching, purpuric rash on the trunk and limbs.

A

Sepsis

How well did you know this?
1
Not at all
2
3
4
5
Perfectly
310
Q

A 2-week-old preterm male neonate develops transient apnoeas and bradycardic episodes while in the neonatal intensive care unit. He had been born at 30 weeks’ gestation after spontaneous onset of preterm labour. He had required intubation and mechanical ventilation for 48 hours following birth for neonatal respiratory distress syndrome. Standard dosing of surfactant was administered during this time. He required respiratory support with continuous positive airway pressure for 1 week after his extubation, and was cycling on and off high-flow oxygen therapy at the time of this event. He had established full enteral feeding after a period of parenteral feeding via a percutaneous central venous catheter (long-line). The long-line was still in situ at the time of this event, and was planned for removal that day. In addition to the apnoeas and bradycardias, it was noted that he had temperature instability and increased capillary refill time (>3 seconds); both of these features were a change from the previous observation trends.

A

Sepsis

How well did you know this?
1
Not at all
2
3
4
5
Perfectly
311
Q

The typical presentation of sepsis varies according to the age of the child. Whereas older children often present with a focus of infection, infants and neonates usually present with non-specific symptoms and signs. For example, the early signs of sepsis in preterm infants are often apnoeas and bradycardias. In the neonatal population, including preterm infants, any change from the patient’s normal pattern of observations should raise the suspicion of sepsis.

Septic shock commonly presents as ‘cold shock’ with profound peripheral vasoconstriction and impaired myocardial contractility. However, another mode of presentation is ‘warm shock’ characterised by systemic vasoplegia (dilated peripheral vasculature and ‘flash’ capillary refill) with a high cardiac output and bounding pulses. Studies suggest that this mode of presentation is more common in hospital-acquired sepsis.

A

Sepsis

How well did you know this?
1
Not at all
2
3
4
5
Perfectly
312
Q

SIRS classification

A

Systemic inflammatory response syndrome (SIRS):

Generalised inflammatory response defined by the presence of 2 or more of the following criteria (abnormal temperature or white cell count must be one of the criteria):
Abnormal core temperature (<36°C or >38.5°C [<97°F or >101°F])
Abnormal heart rate (>2 standard deviations above normal for age, or <10th percentile for age if child is <1 year of age)
Raised respiratory rate (>2 standard deviations above normal for age, or mechanical ventilation for acute lung disease)
Abnormal white cell count in circulating blood (above or below normal range for age, or >10% immature white cells).

How well did you know this?
1
Not at all
2
3
4
5
Perfectly
313
Q

Define slipped capital femoral epiphysis

A

Slipped capital femoral epiphysis (SCFE) is the most common hip disorder in the adolescent age group. It occurs when weakness in the proximal femoral growth plate allows displacement of the capital femoral epiphysis. SCFE is a misnomer; it is the metaphysis that displaces anteriorly and superiorly, leading to the slipped state.

May present with an acute/insidious onset of pain and limp.

The disorder is typically seen in the adolescent age group.

Associated systemic disease is a common feature.

Obligatory external rotation on hip flexion is a key examination finding.

Recommended surgical treatment is in situ pinning; prophylactic fixation of the contralateral hip may be necessary when concomitant metabolic disease is present.

How well did you know this?
1
Not at all
2
3
4
5
Perfectly
314
Q

Epidemiology of slipped capital femoral epiphysis

A

Roughly 1 in 10,000
M>F
Average age of onset was 11.6 years for girls and 12.6 years in boys.

How well did you know this?
1
Not at all
2
3
4
5
Perfectly
315
Q

Aetiology of slipped capital femoral epiphysis

A

The aetiology of SCFE is unknown in most cases. Obesity is recognised as the most strongly associated risk factor. Obesity increases the shear stress across the physis, weakens it, and causes the characteristic displacement in SCFE. Hormonal involvement associated with the adolescent growth spurt may also provide insight into the aetiology of SCFE. The physis weakens at puberty, possibly due to the effect of circulating gonadotrophins. The weakening effect of testosterone on the physis offers a causative hypothesis for the high incidence in males. The lower incidence in female adolescents may be due to oestrogen increasing the strength of the physis and narrowing its width. SCFE is associated with endocrine disorders, but the vast majority of children with SCFE are obese. Radiotherapy is also thought to be a risk factor for the condition. There is no evidence of a genetic predisposition.

How well did you know this?
1
Not at all
2
3
4
5
Perfectly
316
Q

RFs for slipped capital femoral epiphysis

A

STRONG
Puberty
Obesity
Endocrine disorders

WEAK
Male sex 
Ancestry 
Geographic region
Prior radiotherapy
How well did you know this?
1
Not at all
2
3
4
5
Perfectly
317
Q

Sx of slipped capital femoral epiphysis

A

COMMON
Weight >90th centile
RFs
Gait with affected leg externally rotated

Groin/Knee referred pain
Bilateral hip pain - bilateral in 60%
Tendelenburgs gait
Restricted range of motion

UNCOMMON
Weight <50th centile
Symptoms of hypothyroidism/panhypopituitarism
Renal failure - renal osteodystrophy
Recent trauma
How well did you know this?
1
Not at all
2
3
4
5
Perfectly
318
Q

Ix slipped capital femoral epiphysis

A

Bilateral anteroposterior xray - Klein’s line does not intersect the femoral head

Frog-leg lateral x-rays - Klein’s line does not intersect the femoral head; Bloomberg’s sign positive

How well did you know this?
1
Not at all
2
3
4
5
Perfectly
319
Q

Rx of slipped capital femoral epiphysis

A

Surgical repair

Prophylactic fixation of contralateral hip

How well did you know this?
1
Not at all
2
3
4
5
Perfectly
320
Q

Prognosis of slipped capital femoral epiphysis

A

Prognosis is related to the initial severity of the slip, success of surgery, avoidance of serious complications, underlying disorders, and bilaterality. A study of stable SCFE cases treated with surgical dislocation of the hip noted that 87% of cases had some degree of labral damage and 85% of cases had some degree of cartilage damage.

How well did you know this?
1
Not at all
2
3
4
5
Perfectly
321
Q

Complications of slipped capital femoral epiphysis

A

Chondrolysis
Late deformity
Contralateral hip SCFE
Osteonecrosis

How well did you know this?
1
Not at all
2
3
4
5
Perfectly
322
Q

A 13-year-old boy presents with hip, groin, thigh, and medial knee pain. He is overweight and recently experienced an adolescent growth spurt. On physical examination, the affected leg is externally rotated and there is limited range of motion in the hip joint. He is unable to bear weight on the affected leg.

A

Slipped capital femoral epiphysis

How well did you know this?
1
Not at all
2
3
4
5
Perfectly
323
Q

Define sudden infant death syndrome (SIDS)

A

“The sudden death of an infant under 1 year of age, which remains unexplained after a thorough case investigation, including performance of a complete autopsy, examination of the death scene, and a review of the clinical history”.

Leading cause of infant death beyond the neonatal period.

Incidence roughly 1 in 2000 infants.

Peak incidence between 1 and 3 months of age, although events may occur up to 12 months of age.

Risk factors include exposure to tobacco smoke (antenatally and postnatally); prone and side sleeping; bed-sharing during sleep; a sleep environment that includes soft mattress and/or sleeping surface (including sofa or armchair), and placement of soft sleep bedding; over-bundling/over-heating; prematurity; exposure to alcohol and illicit drugs (antenatally and postnatally); and viral infection. Several risk factors may co-exist in any given patient.

Protective factors include use of a dummy during sleep, immunisation, and breastfeeding.

Careful evaluation of death by trained forensics teams is necessary to rule out other causes of death, including suffocation, asphyxia, entrapment, infection, ingestions, metabolic diseases, arrhythmia-associated cardiac channelopathies, and trauma (accidental or non-accidental).

How well did you know this?
1
Not at all
2
3
4
5
Perfectly
324
Q

What is the epidemiology of sudden infant death syndrome (SIDS)

A

SIDS remains the leading cause of post-neonatal infant death.

1 in 2000 infants

Peak incidence 1-3months

M>F

Black>white

How well did you know this?
1
Not at all
2
3
4
5
Perfectly
325
Q

Aetiology of sudden infant death syndrome (SIDS)

A

Aetiology is currently unknown. Indeed, the identification of a specific aetiology/acute process in an infant as a cause of death is sufficient to eliminate the diagnosis of SIDS.

The frequent autopsy finding of mild upper airway inflammation in suspected cases of SIDS raises the possibility that intercurrent respiratory tract infections have a role in SIDS, particularly in combination with over-bundling.

Associations with other modifiable risk factors such as sleeping position, sleeping surface, sleeping arrangements, smoke exposure, not breastfeeding, and non-use of a dummy have been well established, although direct causative roles have yet to be determined.

These 3 key risk categories/factors are:

A vulnerable infant (e.g., prematurity, low birth weight, disordered autonomic regulation)
A critical period during homeostatic control development (e.g., cardiorespiratory regulatory mechanism maturation)
An exogenous stressor (e.g., smoke exposure, sleep environment, over-heating).

How well did you know this?
1
Not at all
2
3
4
5
Perfectly
326
Q

RFs of sudden infant death syndrome (SIDS)

A
STRONG
Side/prone position at last sleep 
Bed sharing
Soft sleeping environment 
Maternal cigarette smoking (ante+post)
Increased number smokers in house
Formula feeding 
Non-use of dummy
Premature birth
WEAK
Maternal substance abuse
Single mother
Dec maternal age
Low level antenatal care
Low maternal education
Low socio-economic status 
Black/american indian
Lack of immunisations
How well did you know this?
1
Not at all
2
3
4
5
Perfectly
327
Q

Sx of sudden infant death syndrome (SIDS)

A

Major risk factors include prone position at last sleep, bed-sharing, soft sleeping surface/environment, maternal cigarette smoking, increasing number of smokers in house, smoking in same room as child, premature birth, formula feeding, non-use of pacifier, and maternal alcohol or drug use.

No factor appears to be sufficient, in itself, to initiate a SIDS event. Risk factors should be placed in the Triple-Risk hypothesis model and the aggregate of co-existing factors used to evaluate the risk of a SIDS event in any given patient.

How well did you know this?
1
Not at all
2
3
4
5
Perfectly
328
Q

Ix for sudden infant death syndrome (SIDS)

A

Everything is negative

Autopsy inconclusive

How well did you know this?
1
Not at all
2
3
4
5
Perfectly
329
Q

Rx of sudden infant death syndrome (SIDS)

A

Carer follow up + grief counselling

How well did you know this?
1
Not at all
2
3
4
5
Perfectly
330
Q

Prognosis of sudden infant death syndrome (SIDS)

A

After a SIDS event, counselling is given to the parents and other carers and surviving siblings. It may be necessary to continue counselling for a period of time, as not all maladaptive behaviours may manifest immediately after the event. Parents should be offered testing for metabolic and genetic conditions that may mimic SIDS.

For parents considering pregnancy again, reassurance can be given that the likelihood of another SIDS event is extremely low. After birth of subsequent children, healthcare providers can consider recommending more frequent follow-up, as much for parental reassurance as for monitoring the infants.

How well did you know this?
1
Not at all
2
3
4
5
Perfectly
331
Q

Define conversion and somatic symptom disorders

A

Conversion and somatic symptom disorders are psychiatric conditions that fall under the somatic symptoms and related disorders category of the DSM-5. Somatic symptom and related disorders are those with distressing physical symptoms that are not fully explained by other medical, neurological, or psychiatric disorders (also known as somatisation).

Conversion disorder is characterised by voluntary motor or sensory function deficits that suggest neurological or medical conditions but are rather associated with clinical findings that are not compatible with such conditions. Somatic symptom disorder is characterised by one or more somatic symptoms that are distressing or result in significant disruption of daily life. To meet DSM-5 criteria, these patients must have excessive thoughts, feelings, or behaviours related to the somatic symptoms or associated health concerns as manifested by at least one of the following: disproportionate and persistent thoughts about the seriousness of one’s symptoms; persistently high levels of anxiety about health or symptoms; excessive time or energy devoted to these symptoms or health concerns. Importantly, even if any one somatic symptom is not continuously present, the state of being symptomatic is persistent (typically more than 6 months).

Conversion and somatic symptom disorders are both categorised as somatic symptom and related disorders, previously termed somatoform disorders.

Somatic symptom and related disorders are psychiatric conditions where patients experience distressing physical symptoms that are not fully explained by other medical, neurological, or psychiatric disorders, as well as abnormal thoughts, feelings, and behaviours in response to these symptoms. They may result from psychological stress that is unconsciously (without awareness) expressed somatically, though the underlying cause is not fully understood.

Risk factors include being female, having a history of abuse or adverse childhood events, and having personality traits of alexithymia (difficulty expressing emotions) or neuroticism. Symptoms that persist with an external focus of control, and without awareness of the psychological and stress-related interplay, can lead to considerable functional impairment and distress.

Diagnosis is made by clinical interview, behavioural observation, physical examination suggestive of pseudoneurological causes, and tests to rule out medical or neurological causes. The diagnosis should not be made solely on the basis of medically unexplained symptoms; rather, it should be based on evidence from the clinical examination and the patient’s abnormal thoughts, feelings, and behaviours in response to the medically unexplained symptoms.

Good doctor-patient relationships and validation of the patient’s suffering are essential for effective management. Treatment includes cognitive behavioural therapy and avoiding unnecessary medicines, tests, and procedures. Diagnosis and treatment of associated comorbid psychiatric conditions benefit overall functioning and recovery.

Long-term management involves interrupting perpetuating factors, maintaining the same doctor, and providing strategies for self-efficacy, distress tolerance, coping, and modulating the interaction of anxiety, stress, and physical symptoms.

How well did you know this?
1
Not at all
2
3
4
5
Perfectly
332
Q

Aetiology of conversion and somatic symptom disorders

A

Chronic and/or acute intrapsychic (emotional/psychological) stress or conflict, in combination with either emotional processing deficits, avoidance tendencies, or social, cultural, or family taboos against emotional expression, seems to be important in the aetiology of somatisation. Societal attitudes in which people exhibiting psychological distress are less tolerated than those who are physically ill are also likely to reinforce somatisation.

Cognitively, disturbances in attention and control through dissociation, misattribution, and misinterpretation probably contribute to both conversion and somatic symptom disorder symptoms.

How well did you know this?
1
Not at all
2
3
4
5
Perfectly
333
Q

Epidemiology of conversion and somatic symptom disorders

A

75% female

Conversion:
5-22/100k
Early-mid adulthood

Somatic:
10% to 15% of primary care patients have multiple unexplained symptoms that are present for >2 years
The prevalence of somatic symptom disorder is not known but may be around 5% to 7%

How well did you know this?
1
Not at all
2
3
4
5
Perfectly
334
Q

RFs of conversion and somatic symptom disorders

A
Hx sexual/physical abuse
Hx unstable childhood
Hx trauma related disorders
Female
Alexithymia
Neuroticism
Previously poor doctor patient relationships
How well did you know this?
1
Not at all
2
3
4
5
Perfectly
335
Q

Sx of conversion and somatic symptom disorders

A
Recent stressors
Unusual neurodeficits
Give-way weakness
False sensory findings
Distractible symptoms
Inconsistent paralysis 
"seizures with awareness"
Bizarre movements
Gait disorders
Cognitive complaints
Speech disturbance
Swallowing disturbance

Psuedoclonus
Convergence spasm

How well did you know this?
1
Not at all
2
3
4
5
Perfectly
336
Q

Ix of conversion and somatic symptom disorders

A

Lab - N
EEG - N
Personality testing - mental illness trait, personality disorder trait, hypochondriasis, depression, hysteria, anxiety, somatisation, pain disorder, or health concern

How well did you know this?
1
Not at all
2
3
4
5
Perfectly
337
Q

Rx of conversion and somatic symptom disorders

A

Psychotherapy
CBT
Hypnosis
Biofeedback training
benzodiazepines 1-2mg lorazepam, 5-10mg diazepam
Antidepressant - sertraline 25mg, fluoxetine 10mg, citalopram 10mg
Biofeedback training

Atypical antipsychotic 
Aripiprazole 2mg
Quetiapine 25mg
lanzipine 2.5mg
Risperidone 0.5mg

ECT

How well did you know this?
1
Not at all
2
3
4
5
Perfectly
338
Q

Prognosis of conversion and somatic symptom disorders

A

CONVERSION
Between 50% and 90% of patients with conversion disorder exhibit short-term resolution of symptoms after reassurance, but up to 25% of these responders relapse or develop new conversion symptoms over time.
Favourable prognostic indicators for patients with conversion disorder include acute onset of symptoms, precipitation by a well-defined stressful event, good premorbid health, and absence of psychiatric or neurological comorbidities

SOMATIC
Reported remission rates range from very low (<10%) [116] to a 50% recovery within 1 year.
Patients with many somatic symptoms, anxiety or depression, and old age or marked impairment were more likely to have persistent symptoms.

How well did you know this?
1
Not at all
2
3
4
5
Perfectly
339
Q

Complications of conversion and somatic symptom disorders

A

Depression
Anxiety
Suicidal ideation
Substance abuse

How well did you know this?
1
Not at all
2
3
4
5
Perfectly
340
Q

A 21-year-old woman presents to the accident and emergency department with acute onset of left-sided body twitching after a minor accident in which she hit her head. Physical examination, laboratory investigations, and imaging studies are normal. Over the next several weeks, she begins experiencing episodes of full-body movements. In the week before the initial symptom onset, her boyfriend (for whom she also worked) broke up with her, and she had a conflict with her parents. She and her boyfriend have since reconciled. There is no reported history of abuse, but there is a family dynamic of high expectations of the patient. Neurological examination and EEG during a typical episode are normal.

A

Conversion and somatic symptom disorders

How well did you know this?
1
Not at all
2
3
4
5
Perfectly
341
Q

A 35-year-old married woman seeks treatment for pelvic pain. On review of systems, she reports several years of various symptoms, including GI problems (constipation, abdominal pain, nausea, and vomiting); headaches; vulvodynia; fatigue; all-over body pains; paraesthesias; and several sensitivities to environmental factors and medications. She has visited the accident and emergency department on several occasions and has been previously admitted to hospital for persistent GI symptoms. Extensive GI work-up, laboratory studies, and laparoscopy to rule out endometriosis have been unrevealing. She reports a stressful childhood, with an alcoholic father and sexual abuse by her grandfather.

A

Conversion and somatic symptom disorders

How well did you know this?
1
Not at all
2
3
4
5
Perfectly
342
Q

Male, middle-aged, and older patients are also seen with conversion and somatic symptom and related disorders, although this is less common. Importantly, these disorders can coexist with other medical and neurological conditions. Previous health-related traumatic experiences (e.g., myocardial infarction, cancer) in a patient with high illness anxiety is associated with development of somatisation.

A

Conversion and somatic symptom disorders

How well did you know this?
1
Not at all
2
3
4
5
Perfectly
343
Q

Define acute sinusitis

A

Acute sinusitis (also commonly known as acute rhinosinusitis) is a symptomatic inflammation of the mucosal lining of the nasal cavity and paranasal sinuses, where clinical symptoms have been present for 4 weeks or less. It can be caused by either a viral or a bacterial infection.

Majority of cases in adults and children are of viral aetiology.

Duration of symptoms more than 10 days often indicates bacterial cause.

Imaging is not required for diagnosis unless complications are suspected.

Condition is usually self-limiting; however, symptomatic therapy is recommended.

Antibiotics are only recommended in select patient groups (e.g., immunocompromised or with severe disease).

How well did you know this?
1
Not at all
2
3
4
5
Perfectly
344
Q

Epidemiology of acute sinusitis

A

16% of adults
0.5-13% of viral -> bacterial
13% of children by age 3
F>M

School-age children on average contract 6 to 8 upper respiratory tract infections per year, and of these, 5% to 10% will be complicated by sinusitis.

How well did you know this?
1
Not at all
2
3
4
5
Perfectly
345
Q

Aetiology of acute sinusitis

A

The most common cause of acute sinusitis is a viral infection.

he three most common bacteria are Streptococcus pneumoniae (20% to 43% of cases), Haemophilus influenzae (22% to 35% of cases), and Moraxella catarrhalis (2% to 10% of cases). M catarrhalis is less common in the adult population.

How well did you know this?
1
Not at all
2
3
4
5
Perfectly
346
Q

RFs for acute sinusitis

A

STRONG
Viral URTI
Allergic Rhinitis

WEAK
GORD
- Gastroesophageal reflux into the nasopharynx can cause symptoms that may be mistaken for sinusitis. In the paediatric population, it is associated with chronic sinusitis but does not contribute to acute sinusitis.

How well did you know this?
1
Not at all
2
3
4
5
Perfectly
347
Q

Sx of acute sinusitis

A
COMMON
<10 day Sx (viral)
>10 days <4w (bacterial)
Symptoms that worsen after an initial improvement
Purulent nasal discharge
Blocked nose
Swollen septum
Facial pain/pressure

UNCOMMON
Severe symptoms at onset
Dental pain (acute maxillary sinusitis)

Other
COMMON
Cough
Myalgia
Sore throat
Hyposmia
Oedematous turbinate

UNCOMMON
Fever

How well did you know this?
1
Not at all
2
3
4
5
Perfectly
348
Q

Rx for acute sinusitis

A

Suspected Viral:
Supportive
Antipyretic: paracetamol/ibuprofen
Decongestant eg pseudoephedrine

Intranasal corticosteroid possible:
mometasone nasal

Decongestant:
Ipatropium nasal

Saline nasal irrigation

Mucolytic:
Guaifenesin

BACTERIAL

  • amoxicillin
  • amoxicillin/clavulanate
  • clindamycin
How well did you know this?
1
Not at all
2
3
4
5
Perfectly
349
Q

Ix for acute sinusitis

A

Clinical diagnosis

IF refractory to Rx/immunocompromised then nasal endoscopy possible
Sinus culture

How well did you know this?
1
Not at all
2
3
4
5
Perfectly
350
Q

Prognosis of acute sinusitis

A

In general, acute sinusitis is a self-limiting disease and generally resolves within 1 month. However, the use of antibiotics in appropriately selected patients may limit the length and severity of symptoms.

How well did you know this?
1
Not at all
2
3
4
5
Perfectly
351
Q

Complications of acute sinusitis

A

Complications are more commonly seen in the paediatric population, and occur due to direct extension of the infection into neighbouring structures. Orbital spread of infection with orbital cellulitis or orbital abscess represent the most common complications. Intracranial spread of infection resulting in meningitis or abscess is much less common.

Cavernous sinus thrombosis

How well did you know this?
1
Not at all
2
3
4
5
Perfectly
352
Q

A 19-year-old woman presents with a 12-day history of purulent nasal drainage and nasal congestion, and reports a history of fever, myalgia, and facial pressure. She is otherwise healthy and works as a teacher. After 5 days of illness, the patient’s symptoms started to improve; however, they have worsened in the last few days, despite the use of over-the-counter medications. Physical examination shows oedematous mucosa of the inferior turbinate. There is also thick mucus in the nasal cavity. Nasal endoscopy demonstrates purulent drainage and a small polyp in the ostiomeatal complex. The adenoids are small and erythematous.

A

Acute sinusitis

How well did you know this?
1
Not at all
2
3
4
5
Perfectly
353
Q

A 33-year-old man with a medical history of paediatric-onset asthma, atopic dermatitis, and allergic rhinitis presents with a 7-day history of facial pressure, dental pain, nasal blockage, and hyposmia. The patient developed these symptoms after recently mowing his lawn. The symptoms have not improved despite use of an intranasal corticosteroid, an antihistamine, and intranasal saline washes. Physical examination shows a septum deviated to the left side, and a large concha bullosa on the right side. There are no polyps, but there are swollen turbinates and thin, clear mucus present.

A

Acute

How well did you know this?
1
Not at all
2
3
4
5
Perfectly
354
Q

Define otitis externa

A

Acute otitis externa (AOE) is defined as diffuse inflammation of the external ear canal, which may also involve the pinna or tympanic membrane.

It is a form of cellulitis that involves the skin and subdermis of the external auditory canal, with acute inflammation and variable oedema.

It is most commonly caused by bacterial infection. The diagnosis of AOE requires the presence of rapid onset (generally within 48 hours) of symptoms within the past 3 weeks, coupled with signs of ear canal inflammation.

How well did you know this?
1
Not at all
2
3
4
5
Perfectly
355
Q

Epidemiology of otitis externa

A

AOE has a lifetime incidence of 10%.

Peak 7-12 yo

How well did you know this?
1
Not at all
2
3
4
5
Perfectly
356
Q

Aetiology of otitis externa

A

Most commonly caused by bacterial infections. It is often polymicrobial, but the most common pathogens are Pseudomonas aeruginosa (20%-60% prevalence) and Staphylococcus aureus (10%-70% prevalence).

Other aetiologies are idiopathic, trauma (from scratching, aggressive cleaning), chemical irritants, allergy (most commonly to antibiotic ear drops such as neomycin), high-humidity conditions, swimming, or skin disease (allergic dermatitis, atopic dermatitis, psoriasis).

How well did you know this?
1
Not at all
2
3
4
5
Perfectly
357
Q

RFs for otitis externa

A
STRONG
External auditory canal obstruction 
High humidity
Warmer temperatures
Swimming 
Local trauma
Allergy
Skin diseas
DM
Immunocompromised
Prolonged use topical antibacterials

WEAK
Chemical irritants

How well did you know this?
1
Not at all
2
3
4
5
Perfectly
358
Q

Sx of otitis externa

A
COMMON
RFs
Ear pain 
Tragal tenderness
Ear canal swelling/erythema
Otorrhoea
Aural fullness
Itching
Dec hearing
Erythematous timpanic membrane 

UNCOMMON
Granulation tissue in ear canal - malignant otitis externa

How well did you know this?
1
Not at all
2
3
4
5
Perfectly
359
Q

Ix otitis externa

A

Pneumatic Otoscopy - Normal (unless also otitis media)
Typanometry - N (unless also otitis media)

Ear culture - if failed response

How well did you know this?
1
Not at all
2
3
4
5
Perfectly
360
Q

Rx of otitis externa

A

Antibacterial drops
ciprofloxacin, can include hydrocortisone too
or acetic acid

Refractory -> oral cipro

Fungal -> acetic acid or tolnaftate topical

Malignant/necrotising = needs debridement

How well did you know this?
1
Not at all
2
3
4
5
Perfectly
361
Q

Prognosis of otitis externa

A

Patients with uncomplicated diffuse otitis externa usually respond to treatment. Between 65% and 90% of patients have clinical resolution within 7 to 10 days, regardless of agent used.

The mortality rate of malignant otitis externa has decreased over the years from 50% to 0%-15%.

How well did you know this?
1
Not at all
2
3
4
5
Perfectly
362
Q

Complications of otitis externa

A

Contact dermatitis
Cranial nerve palsy (facial-> usually malignant/./nec otitis ext)
Osteomyelitis

How well did you know this?
1
Not at all
2
3
4
5
Perfectly
363
Q

A 35-year-old man presents with a 2-day history of rapid-onset severe ear pain and fullness. The patient complains of otorrhoea and mild decreased hearing. He reports that his symptoms started after swimming. No fever is reported. On physical examination the external ear canal is diffusely swollen and erythematous. He has tenderness of the tragus and pain with movement of the auricle. The tympanic membrane was partially visualised due to the swelling. The concha and the pinna look normal. Neck examination fails to reveal any lymphadenopathy.

A

otitis externa

How well did you know this?
1
Not at all
2
3
4
5
Perfectly
364
Q

Malignant or necrotising otitis externa is a form of otitis externa that is more common in older patients with uncontrolled diabetes or in patients with immunodeficiency. [1] [4] In malignant otitis externa, the infection and the inflammatory process involve not only the skin and soft tissue of the external auditory canal but the bone tissue of the temporal bone as well. [5] If left untreated, osteomyelitis of the petrous bone and/or skull base could result. [5] [6] It is most commonly caused by Pseudomonas species. [1] [5] Patients usually present with severe ear pain, otorrhoea, and fullness, and are not responding to the conventional treatment of AOE. Depending on the stage of presentation and the extent of invasion, patients may have facial weakness and other cranial nerve abnormalities. [1] On physical examination the external auditory canal is swollen, with evidence of granulation tissue in the floor of the canal. [1] The diagnosis is usually made by CT or MRI scans, which show presence of soft tissue and bone destruction. [5] Technetium-99 or gallium scans will show increased radioisotope uptake in the temporal bone and/or skull base

A

otitis externa

How well did you know this?
1
Not at all
2
3
4
5
Perfectly
365
Q

Otomycosis is fungal otitis externa. Acute fungal otitis externa is less common than acute bacterial otitis externa. [1] It is most commonly caused by Aspergillus species. [3] It presents in a similar way with ear pain, itching, aural fullness, and otorrhoea. Physical examination reveals swollen ear canal skin and discharge. The presence of black spores indicates Aspergillus niger as the causative organism. [1] [3] White filamentous hyphae can often be seen. The definitive diagnosis of otomycosis can be helped by microscopic examination and ear cultures. Otomycosis should be suspected in patients who fail treatment with antibacterial agents. [3] Secondary fungal infection of the external auditory canal is well known after prolonged treatment with topical antibacterial agents

A

otitis externa

How well did you know this?
1
Not at all
2
3
4
5
Perfectly
366
Q

Define acute otitis media

A

Acute otitis media (AOM) is an infection involving the middle ear space and is a common complication of viral respiratory illnesses.

May present with otalgia, irritability, decreased hearing, anorexia, vomiting, or fever, usually in the presence of an ongoing viral respiratory infection.

Physical examination will reveal a bulging, opacified tympanic membrane with decreased mobility. The membrane may be white, yellow, pink, or red.

Diagnosis is generally made with conventional otoscopy. Additional tests might include pneumatic otoscopy or tympanometry to confirm the presence of an effusion.

Treatment includes pain control with analgesics and might include antibiotics.

Complications include perforation of the tympanic membrane and, rarely, mastoiditis, seventh cranial nerve palsy, or sigmoid sinus thrombosis.

How well did you know this?
1
Not at all
2
3
4
5
Perfectly
367
Q

Epidemiology of acute otitis media

A

More than 80% of children experience at least one episode of AOM before the age of 2 years with a peak incidence between 6 and 18 months.

Children with anatomical anomalies (e.g., cleft palate, cleft uvula) or immunological deficiencies encounter more AOM than their peers. Environmental risk factors include childcare attendance, exposure to older siblings, exposure to tobacco smoke, absence of breastfeeding, bottle feeding in a supine position, and dummy use. A higher incidence among boys, children with a family history of AOM, and certain ethnic groups (Native Americans and Native Alaskans) suggests a genetic susceptibility.

How well did you know this?
1
Not at all
2
3
4
5
Perfectly
368
Q

RFs of acute otitis media

A
STRONG
Nursery 
Older siblings
Young age
FHx
Absence of breast feeding
Supine feedings
Low socio-econ
Craniofacial anomaly
Immunological deficiency

WEAK
Male
Dummy use
Env tobacco exposure

How well did you know this?
1
Not at all
2
3
4
5
Perfectly
369
Q

Aetiology of acute otitis media

A

Respiratory viruses account for most cases of otitis media and are self-limiting. Co-infections of the middle ear with a virus and a bacterium demonstrate the role that both play in the development of acute, suppurative otitis media or pus drum.

The most common bacteria responsible for AOM are Streptococcus pneumoniae (approximately 40%), non-typable Haemophilus influenzae (25% to 30%), and Moraxella catarrhalis (10% to 15%).

Unfortunately, an aetiological diagnosis in a clinic setting is not readily feasible.

How well did you know this?
1
Not at all
2
3
4
5
Perfectly
370
Q

Sx of acute otitis media

A
COMMON
Otalgia 
Preceding URTI
Bulging tympanic membrane
Myringitis
Irritability 
Sleep disturbance
Fever

UNCOMMON
Decreased appetite

How well did you know this?
1
Not at all
2
3
4
5
Perfectly
371
Q

Ix for acute otitis media

A

Clinical

Typanometry - not required but - flat (B) curve (low compliance)

How well did you know this?
1
Not at all
2
3
4
5
Perfectly
372
Q

Define non-allergic rhinitis

A

Non-allergic rhinitis (NAR) refers to a group of chronic rhinitis sub-types that are not caused by allergy or infection. At least eight sub-types have been proposed, including vasomotor rhinitis (VMR [also known as ‘autonomic rhinitis’, ‘non-allergic rhinopathy’, and ‘idiopathic non-allergic rhinitis’]), non-allergic rhinitis with eosinophilia syndrome (NARES), atrophic rhinitis, senile rhinitis, gustatory rhinitis, drug-induced rhinitis, hormonal rhinitis, and occupational rhinitis. VMR is the most common sub-type.

To establish a definitive diagnosis of NAR, all other chronic rhinitis syndromes should be properly considered and excluded.

Environmental tobacco smoke, perfumes and fragrances, as well as temperature and barometric changes may aggravate symptoms in NAR, but specific IgE responses by skin or serological testing are all negative. The presence of eosinophils in the nasal mucosa in NARES distinguishes it from other sub-types of NAR.

It is a chronic condition that should be distinguished from a common cold, which can manifest with symptoms of NAR but is self-limiting.

Traditional triggers such as cat or dog exposure should be absent.

Symptoms and examination findings can overlap between perennial allergic rhinitis and non-allergic rhinitis (NAR), with nasal turbinates swollen and beefy red, scant mucus, cobblestoning of posterior pharynx from chronic post-nasal drainage, and retraction of tympanic membranes indicating congestion.

A diagnosis of NAR requires negative specific IgE responses by skin or serological testing.

Differentiation between non-allergic rhinitis with eosinophilia syndrome and other sub-types of NAR is determined by the presence or absence of eosinophilia in the nasal passage.

Treatment is based on symptoms, and all patients should be counselled on avoidance of triggers. Symptom control in NAR requires a balance between the control of excess secretions and over-suppression. First-line treatments include intranasal corticosteroids, intranasal antihistamines, and intranasal ipratropium.

Structural problems or other complicating conditions should be ruled out with imaging if initial therapeutic trials fail to relieve symptoms. Possibilities include osteomeatal complex obstruction that occurs as a result of chronic inflammation or recurrent infections, severe nasal septal deviation and nasal polyposis, or, less commonly, tumour or foreign body.

How well did you know this?
1
Not at all
2
3
4
5
Perfectly
373
Q

Aetiology of non-allergic rhinitis

A

It has been proposed that, in response to local mucosal injury, a compensatory response might over-amplify axonal release of neuropeptides, which then initiate the vascular and glandular responses characteristic of NAR.

The most popular mechanism postulated for vasomotor rhinitis patients has been an autonomic imbalance between the sympathetic and parasympathetic nervous system, resulting in parasympathetic hyperactivity leading to nasal congestion and drainage.

Parasympathetic responses are largely responsible for increased blood flow leading to engorgement of venous sinusoids, plasma extravasation, and increased release of secretions of mucus-secreting cells.

Otherwise, as a diagnosis based on exclusion, the aetiological factors in NAR are also largely negative risk factors. It is more likely to be the diagnosis in those with no family history of allergies, onset of symptoms after age 35, no seasonal symptoms, and no symptom exacerbation by cats. It is positively associated with triggers such as perfumes/fragrances and pot pourris.

How well did you know this?
1
Not at all
2
3
4
5
Perfectly
374
Q

Epidemiology of non-allergic rhinitis

A

Prev = 17-52%
Prevalence in children is unknown but is probably under-diagnosed due to the difficulty of children recognising and communicating signs and symptoms.

How well did you know this?
1
Not at all
2
3
4
5
Perfectly
375
Q

RFs for non-allergic rhinitis

A

STRONG
Symptom onset >35y
No exacerbation with cats
Neg FHx of allergy

WEAK
No seasonal changes
Fragrance exposure
Smoke exposure
Exposure to hairspray 
Exposure to pot pourris
Change in temperature
Barometric changes
High oestrogen state
BBlocker use
Intranasal drug abuse
How well did you know this?
1
Not at all
2
3
4
5
Perfectly
376
Q

Sx of non-allergic rhinitis

A
COMMON
Presence of RFs
Nasal congestion
PN-drip
Rhinorrhoea
Sore throat
Ear plugging
Sinus headaches
Postural change triggers
Sneezing

UNCOMMON
Purulent discharge
Bilateral symptoms
Sinus pain

How well did you know this?
1
Not at all
2
3
4
5
Perfectly
377
Q

Ix for non-allergic rhinitis

A

Allergy skin testing - Negative - exclude AR
Serological specific IgE testing - Negative

Consider
Nasal provocation test - symptoms return

How well did you know this?
1
Not at all
2
3
4
5
Perfectly
378
Q

Rx of non-allergic rhinitis

A

Trigger avoidance
Intranasal antihistamine - Azelastine BD
Saline nasal irrigation
Intranasal corticosteroid

COMBINATION

Oral antihistamines
Intranasal ipatropium

Surgery possible but rarely done - very very final option:
CO2 turbinectomy and partial laser turbinectomy

How well did you know this?
1
Not at all
2
3
4
5
Perfectly
379
Q

Complications of non-allergic rhinitis

A

Decongestant SEs: insomnia, tachycardia, increased blood pressure, and urinary retention and can aggravate underlying cardiac conditions
headache
Recurrent sinusitis
SEs nasal corticosteroids: bleeding/ulceration
SEs of atihistamines: excessive dryness and can aggravate benign prostatic hypertrophy
Eustacian tube dysfunction

How well did you know this?
1
Not at all
2
3
4
5
Perfectly
380
Q

Complications of non-allergic rhinitis

A

Decongestant SEs: insomnia, tachycardia, increased blood pressure, and urinary retention and can aggravate underlying cardiac conditions
headache
Recurrent sinusitis
SEs nasal corticosteroids: bleeding/ulceration
SEs of atihistamines: excessive dryness and can aggravate benign prostatic hypertrophy
Eustacian tube dysfunction

How well did you know this?
1
Not at all
2
3
4
5
Perfectly
381
Q

Define tonsillitis

A

Acute tonsillitis is an acute infection of the parenchyma of the palatine tonsils. This definition does not include tonsillitis as part of infectious mononucleosis, although tonsillitis may occur in isolation or as part of a generalised pharyngitis. The clinical distinction between tonsillitis and pharyngitis is unclear in the literature, and the condition is often referred to simply as “acute sore throat”.

Can be difficult to distinguish clinically from viral pharyngitis.

Most patients do not seek medical help.

Most cases resolve spontaneously and do not require antibiotics. Antibiotics are used to treat group A beta-hemolytic streptococcal infection.

There is some evidence that tonsillectomy may be effective in selected children with recurrent severe acute tonsillitis.
_______________________

Centor criteria [18]
The Centor criteria give an indication of the likelihood of a sore throat being due to bacterial infection. The criteria are:

 Tonsillar exudate
 Tender anterior cervical adenopathy
 Fever over 38°C (100.5°F) by history
 Absence of cough.
If 3 or 4 of Centor criteria are met, the positive predictive value is 40% to 60%. The absence of 3 or 4 of the Centor criteria has a fairly high negative predictive value of 80%.
How well did you know this?
1
Not at all
2
3
4
5
Perfectly
382
Q

Epidemiology of tonsillitis

A

In UK general practice, recurrent sore throat has an annual incidence of 100 per 1000 population.

Acute tonsillitis is more common in children between the ages of 5 and 15 years.

The prevalence of bacterial tonsillitis, specifically group A beta-haemolytic streptococci (GABHS), is 15% to 30% of children with sore throat and 5% to 15% of adults with sore throat.

Acute tonsillitis is most commonly seen in winter and early spring in temperate climates, although it may occur at any time of the year.

How well did you know this?
1
Not at all
2
3
4
5
Perfectly
383
Q

Aetiology of tonsillitis

A

Tonsillitis is usually viral; it is most commonly caused by the rhinovirus, followed by the coronavirus, and the adenovirus. Less commonly it is caused by the influenza virus, the parainfluenza virus, enteroviruses, or herpes viruses. In tonsillitis associated with infectious mononucleosis, the most common infective agent is the Epstein-Barr virus.

Common bacterial pathogens include beta-haemolytic and other streptococci, with the most common being group A beta-haemolytic streptococci (GABHS). GABHS is responsible for 15% to 30% of all cases of acute tonsillitis in children aged between 5 and 15 years and for 5% to 10% of all tonsillitis in adults. Group C beta-haemolytic streptococci are the cause in about 5% of patients.

Rarer bacterial causes include Mycoplasma pneumoniae and Neisseria gonorrhoeae , which can be the cause in sexually active adolescents (particularly those engaging in oral-genital sex).

How well did you know this?
1
Not at all
2
3
4
5
Perfectly
384
Q

RFs of tonsillitis

A

WEAK
Age 5-15
Contact with infected poeple

How well did you know this?
1
Not at all
2
3
4
5
Perfectly
385
Q

Sx of tonsillitis

A
COMMON
RFs presence
Pain on swallowing 
Fever >38
Exudate: particularly when it is caused by group A beta-haemolytic streptococci
Sudden onset Sx
Tonsillar erythema/enlargement
Lymphadenopathy 
Associated with infective process:
Headache
Abdominal pain 
N+V
Presence cough/runny nose
How well did you know this?
1
Not at all
2
3
4
5
Perfectly
386
Q

Ix for tonsillitis

A

Throat culture - not nec routine
Rapid strep antigen test - identification of group A beta-haemolytic streptococci (GABHS) [lower sensitivity but fast]

WCC - rule out infectious mononucleosis - A raised WBC count with neutrophilia is suggestive of a bacterial infection, whereas a raised WBC count with lymphocytosis and atypical lympocytes is suggestive of infectious mononucleosis.

Heterophile antibodies - inf mono

Vaginal and cervical, or penile, and rectal cultures - if suspect N gon

Persistent/recurrent symptoms - check HIV

How well did you know this?
1
Not at all
2
3
4
5
Perfectly
387
Q

Rx for tonsillitis

A

Not due to strep - manage with analgesics para/ibu/aspirin/naproxen

If Group A strep:
Analgesia
+ ABx:
Ben-Pen, amoxicillin, azithromycin, clarithromycin

SEVERE swelling + group A
- Consider corticosteroids

Recurrent episodes:
Consider tonsillectomy

How well did you know this?
1
Not at all
2
3
4
5
Perfectly
388
Q

Prognosis of tonsillitis

A

Acute tonsillitis is an acute, self-limiting infective condition that normally resolves completely within 1 week with no sequelae.

In vulnerable people (e.g., infants, very old people, immunosuppressed or immunocompromised patients), tonsillitis may run a more severe course. Antibiotics and/or admission to hospital for a limited period of time may be advisable.

Very rarely (but more commonly in developing countries) acute tonsillitis may be associated with significant suppurative complications, such as neck abscess, and non-suppurative complications, such as rheumatic fever or acute glomerulonephritis.

How well did you know this?
1
Not at all
2
3
4
5
Perfectly
389
Q

Complications of tonsillitis

A

Scarlet fever: A diffuse erythematous rash, which is a manifestation of delayed-type skin reactivity to pyrogenic exotoxin (erythrogenic toxin, usually types A, B, or C) produced by Streptococcus species.

Characteristically the rash blanches with pressure and has multiple small papules. It generally starts on the head and neck, and is associated with circumoral pallor and a strawberry tongue. It subsequently spreads to the trunk, sparing the palms and soles, and is more marked over the skin folds.

Acute sinusitis
Acute Otitis media
Peritonsillar / neck abscess
Acute rheumatic fever
Acute post-streptococcal glomerulonephritis (manifesting with haematuria and oedema)
Streptococcal toxic shock syndrome

Paediatric Autoimmune Neuropsychiatric Disorders Associated with Streptococci (PANDAS) syndrome - not on DSM but has been described

How well did you know this?
1
Not at all
2
3
4
5
Perfectly
390
Q

Define epiglottitis

A

Epiglottitis is a cellulitis of the supraglottis with the potential to cause airway compromise, and should be treated as a surgical emergency until the airway is examined and secured.

Pertinent diagnostic criteria include the classic ‘tripod’ position of the patient, drooling, high fever, and a toxic appearance.

A cellulitis of the supraglottis that may cause airway compromise. An airway emergency, especially in children, and precautionary measures must be taken.

Epiglottitis is classically described in children aged 2 to 6 years of age; however, it may manifest at any age, including in newborns. It may now be more common in older children and adults due to the Haemophilus influenzae type B (Hib) vaccine.

The most common signs are rapid onset of high fever, sore throat, inability to control secretions, classic tripod positioning, difficulty breathing, and irritability. Adults may have a more indolent presentation and may not require airway intervention (only about 20% of adults require intubation whereas most children do).

Cooperation between the otolaryngologist, paediatrician, or emergency department physician and anaesthetist is crucial.

No action should be taken that could stimulate a child with suspected epiglottitis, including examination of the oral cavity, starting intravenous lines, blood draws, or even separation from a parent. Similar caution is required in fulminant acute epiglottitis in adults.

Diagnosis is made on clinical grounds, and laboratory or other interventions should not preclude or delay timely control of the airway in a suspected case of epiglottitis.

Once the airway has been secured and antibiotics have been initiated, the condition usually resolves rapidly.

Vaccination does not preclude the possibility of epiglottitis or even the possibility of epiglottitis from Haemophilus influenzae.

How well did you know this?
1
Not at all
2
3
4
5
Perfectly
391
Q

Epidemiology of epiglottitis

A

Most 3-5yo

4.9 per 100k

How well did you know this?
1
Not at all
2
3
4
5
Perfectly
392
Q

Aetiology of epiglottitis

A

Infection of the supraglottis: classically with Haemophilus influenzae , but also from other potential pathogens, including Streptococcus pneumoniae , Staphylococcus aureus , and MRSA.

Other rare bacterial pathogens such as Pasteurella multocida have been reported.

Less commonly, viral pathogens such as parainfluenza have been implicated as aetiological agents.

Another rare aetiology of epiglottitis is fungal infection. Several reports of candidial epiglottitis have been reported.

Traumatic: a mechanical insult to the epiglottis can occur, most commonly from caustic ingestion or thermal injury, which may result in marked oedema and inflammation of the epiglottis.

How well did you know this?
1
Not at all
2
3
4
5
Perfectly
393
Q

RFs of epiglottitis

A

STRONG
No vaccination with H influenza vaccine
Immunocompromise
Middle age

How well did you know this?
1
Not at all
2
3
4
5
Perfectly
394
Q

Sx of epiglottitis

A
COMMON
RFs 
No vaccination (Hib vacc)
Sore throat
Dysphagia
Drooling (failure to swallow)
Toxic appearance
Acute distress
Fever 
Tripod position
Difficulty breathing
OTHER COMMON
Decreased oral intake
Muffled voice
Stridor
Irritability
How well did you know this?
1
Not at all
2
3
4
5
Perfectly
395
Q

Ix for epiglottitis

A

Laryngoscopy - swelling of supraglottic structures
Lateral neck radiograph - markedly enlarged epiglottis, referred to as a ‘thumbprint sign’

Can do:
Cultures of blood/supraglottis (epiglottis) - positive cultures - NB often not possible as it’s more important to secure airway

How well did you know this?
1
Not at all
2
3
4
5
Perfectly
396
Q

Rx of epiglottitis

A
  1. SECURE AIRWAY + O2
  2. IV Abx - cefotaxime/ceftriaxone
  3. Dexamethasone orally
  4. Inhaled adrenaline
  5. Still require ABx when stable and extubated
How well did you know this?
1
Not at all
2
3
4
5
Perfectly
397
Q

Complications of epiglottitis

A

Mediastinitis
Cellulitis of neck
Pneumonia - post intubation

How well did you know this?
1
Not at all
2
3
4
5
Perfectly
398
Q

Prognosis of epiglottitis

A

Epiglottitis is an acute condition and, though very severe (especially so in children), if the patient is treated appropriately the prognosis is excellent for a quick and complete recovery. Other than in unvaccinated or immunocompromised individuals, there is no theoretical increased risk for future episodes of this disease.

There are numerous sequelae of not treating an infectious process that causes airway obstruction. These include: death, mediastinitis, neck space infection, necrotising fasciitis, pneumonia, aspiration, and asphyxiation.

How well did you know this?
1
Not at all
2
3
4
5
Perfectly
399
Q

Define acute pharyngitis

A

Acute pharyngitis is characterised by the rapid onset of sore throat and pharyngeal inflammation (with or without exudate). Absence of cough, nasal congestion, and nasal discharge distinguishes bacterial from viral aetiologies. It can be caused by a variety of viral and bacterial pathogens, including group A Streptococcus (GAS), as well as fungal pathogens ( Candida ). Bacterial pharyngitis is more common in winter (or early spring), while enteroviral infection is more common in the summer and autumn. Generally a self-limited condition with resolution within two weeks.

Hallmarked by acute onset of sore throat; the absence of cough, nasal congestion and discharge suggests a bacterial aetiology.

Rapid antigen detection tests allow immediate point-of-care assessment of group A Streptococcus (GAS) pharyngitis.

The goal of treatment of GAS is to prevent acute rheumatic fever, reduce the severity and duration of symptoms, and prevent transmission.

Acute pharyngitis is generally a self-limited condition with resolution within two weeks. Infected individuals are not, however, immune to reinfection with most aetiological pathogens.

The only situation in which antibiotic prophylaxis to prevent GAS infections is recommended is for individuals with a history of rheumatic fever.

How well did you know this?
1
Not at all
2
3
4
5
Perfectly
400
Q

Epidemiology of acute pharyngitis

A

Bacterial pharyngitis is more common in winter (or early spring), while enteroviral infection is more common in the summer and autumn. Pharyngitis is most common in school-aged children during the winter months. Seasonal colonisation with group A Streptococcus (GAS) reaches its peak (up to 20% of children) during this season.

GAS pharyngitis is the focal point of clinical interest in pharyngitis, as the main goal of therapy is prevention of rheumatic fever associated with this organism. GAS pharyngitis, however, represents less than one third of all cases of acute pharyngitis.

How well did you know this?
1
Not at all
2
3
4
5
Perfectly
401
Q

Aetiology of acute pharyngitis

A

Acute sore throat is most commonly due to viral organisms or group A Streptococcus (GAS). Common viral causes include the Epstein-Barr virus (mononucleosis), adenoviruses, enteroviruses, influenza A and B, and parainfluenza. GAS pharyngitis is spread from person to person, especially in the winter, via the respiratory droplet route; however, it represents less than one third of all cases of acute pharyngitis. Other streptococci that can cause pharyngitis include groups B, C, and G streptococci. Other bacteria found rarely in adolescents with pharyngitis include Mycoplasma species and Arcanobacterium haemolyticum .

HIV, chlamydia, or gonorrhoea may be implicated as a cause of acute pharyngitis in sexually active adolescents or sexually abused children.

In low- and middle-income countries, diphtheria and measles are also prominent causes of acute pharyngitis.

Tularaemia may be a cause if there is a history of eating meat from undomesticated animals.

Candida infection is common as a source of sore throat in immunocompromised individuals, including those undergoing chemotherapy or oropharyngeal irradiation for cancer.

How well did you know this?
1
Not at all
2
3
4
5
Perfectly
402
Q

RFs for acute pharyngitis

A
COMMON
Nasal colonisation with Group A Strep
GAS infected contact
Sexual activity/abuse
Ingestion of non-domestic meats (Tularaemic ulceroglandular pharyngitis is acquired from the ingestion of partially cooked wild animal meat.)
Immunocompromised
Inhaled corticosteroids
Lack of immunisation

WEAK
Irradiation

How well did you know this?
1
Not at all
2
3
4
5
Perfectly
403
Q

Sx of acute pharyngitis

A
COMMON
RFs
Child/adolescent
Winter/spring (Bacterial)
Summer/autumn (enteroviral)
Rhinorrhoea, nasal congestion and cough (viral)
Sore throat
Pharyngeal exudate (GAS)
Adenopathy [cervical]
Fever >37
Headache
N+V + Abdo pain 

MEASLES:
Conjunctivitis
Koplik spots
Macropapular rash

UNCOMMON
Scarlatiniform rash
Pharyngeal ulceration (in tularaemia)

How well did you know this?
1
Not at all
2
3
4
5
Perfectly
404
Q

Ix for acute pharyngitis

A

Rapid antigen test for Group A strep - can be +ve/neg

IF NEG

  • do throat swab culture
  • test growth of chalmidya/gon if sexually active

Consider monospot for EBV heterophile ABs

How well did you know this?
1
Not at all
2
3
4
5
Perfectly
405
Q

Rx of acute pharyngitis

A

Supportive

  • paracetamol
  • ibuprofen
  • lidocaine topical 2%

In instances when the rapid test for GAS proves negative, a reasonable approach is no antibiotics at all, with a next day follow-up if the throat culture is positive

Confirmed GAS

  • ben-pen
  • pheno-pen
  • amox
  • azithromycin

Recurrence:
Tonsillectomy

Infectious mononucleosis
- prednisolone orally

Candida
- antifungal - clotrimazole or fluconazole

Diptheria
- Diptheria antitoxin [equine]

How well did you know this?
1
Not at all
2
3
4
5
Perfectly
406
Q

Prognosis of acute pharyngitis

A

Antibiotic therapy of group A Streptococcus (GAS) pharyngitis results in a lessening of symptom intensity and duration, and prevents the long-term complication of rheumatic fever. Symptom resolution is within one to a few days. Infected individuals are not, however, immune to reinfection. Complications of viral pharyngitis are extremely uncommon. Symptoms usually go away within 7 to 10 days.

How well did you know this?
1
Not at all
2
3
4
5
Perfectly
407
Q

A 7-year-old girl presents with abrupt onset of fever, nausea, vomiting, and sore throat. The child denies cough, rhinorrhoea, or nasal congestion. On physical examination, oral temperature is 38.5°C (101°F), and there is an exudative pharyngitis with enlarged, tender anterior cervical lymph nodes. A rapid antigen test is positive for group A Streptococcus (GAS).

A

pharyngitis

How well did you know this?
1
Not at all
2
3
4
5
Perfectly
408
Q

A 7-year-old girl presents with acute sore throat accompanied by fever. On examination, oral temperature is 37°C (98.6°F), and there is an exudative pharyngitis without palpable cervical nodes. Both the rapid antigen test and throat culture are negative for GAS.

A

pharyngitis

How well did you know this?
1
Not at all
2
3
4
5
Perfectly
409
Q

Pharyngitis may be part of the presentation of viral upper respiratory infections that usually can be distinguished by the presence of rhinorrhoea, nasal congestion, and/or cough. Acute pharyngitis frequently occurs with acute infectious mononucleosis. It may rarely, in sexually active or sexually abused individuals, accompany acute HIV infection (with associated adenopathy, rash, fever, and splenomegaly), chlamydia, or gonorrhoea (no distinctive clinical features). Diphtheria and measles may present as pharyngitis and should always be a consideration when sore throat is encountered in low- and middle-income countries or in unvaccinated children. Tularaemia should be considered if symptoms are not responsive to penicillin treatment or if there is a history of eating meat from undomesticated animals. Candida infection is common as a source of sore throat in immunocompromised individuals, including those undergoing chemotherapy or oropharyngeal irradiation for cancer.

A

pharyngitis

How well did you know this?
1
Not at all
2
3
4
5
Perfectly
410
Q

A 4-year-old boy presents to the emergency department with complaints of dysphagia, fever, drooling, and muffled voice. Symptoms have progressively worsened over the course of the day. He is toxic-appearing, and leans forwards while sitting on his mother’s lap. He is drooling, and speaks with a muffled ‘hot potato’ voice. The parents deny trauma or evidence of foreign-body ingestion. They have no recollection of the child receiving a Haemophilus influenzae type B (Hib) vaccine.

A

Epiglottitis

How well did you know this?
1
Not at all
2
3
4
5
Perfectly
411
Q

The presentation of a patient with epiglottitis, especially in the post-Hib vaccine era, can be varied. [3] Vaccination is not 100% effective, so it is possible that patients may present with only some of the typical symptoms. Adults may have a more indolent presentation than children. [4]

A South Korean review documented a high incidence of epiglottic cysts in patients presenting with epiglottis (29%). In this series, these cysts predisposed patients to more severe airway obstruction and a higher risk of recurrence. [5]

A

Epiglottitis

How well did you know this?
1
Not at all
2
3
4
5
Perfectly
412
Q

A 6-year-old previously healthy boy presents with acute onset of fever of 39°C (102°F), severe throat pain that is exacerbated by swallowing, headache, and malaise. On examination his tonsils are symmetrically enlarged and red, with purulent exudate. He has multiple enlarged, painful anterior neck lymph nodes, but no other lymphadenopathy and no splenomegaly. He has no runny nose or cough, and no difficulty breathing.

A

Tonsillitis

How well did you know this?
1
Not at all
2
3
4
5
Perfectly
413
Q

Tonsillitis may occur in isolation or as part of a generalised pharyngitis. The clinical distinction between tonsillitis and pharyngitis is unclear in the literature, and the condition is often referred to simply as “acute sore throat”. Its severity may vary from a mild sore throat that responds well to simple analgesics to a severe sore throat that is associated with significant malaise and reduced oral intake, necessitating admission to hospital.

A

Tonsillitis

How well did you know this?
1
Not at all
2
3
4
5
Perfectly
414
Q

A 45-year-old male presents with chronic post-nasal drainage, cough, and nasal congestion that began approximately 2 years ago but has been getting progressively worse over the past few months. He snores, which results in sore throats in the morning. He experiences pressure over his forehead and behind his eyes, along with ear plugging and popping. His symptoms are aggravated by strong odours, such as perfumes/fragrances and tobacco smoke, and by weather changes. He is not bothered by cats, dogs, freshly cut grass, or dust. He never had these symptoms as a child and has no family history of allergies. These symptoms interfere with his work, as he has to make presentations to clients and is concerned that the constant clearing of his throat is diminishing his effectiveness.

A

Non-allergic rhinitis

How well did you know this?
1
Not at all
2
3
4
5
Perfectly
415
Q

Define laryngitis

A

Laryngitis refers to inflammation of the larynx. This can lead to oedema of the true vocal folds, resulting in hoarseness. Laryngitis can be acute or chronic, infectious or non-infectious. Accompanying signs of infectious laryngitis include odynophagia, cough, fever, and respiratory distress. The most common variant is acute viral laryngitis, which is self-limiting and usually related to an upper respiratory infection. Bacterial laryngitis can be life-threatening. Haemophilus influenzae is one of the most frequently isolated bacteria. Other causes include tuberculosis (TB), diphtheria, syphilis, and fungi. Non-infectious causes of laryngitis include reflux laryngitis, vocal strain and chronic irritant laryngitis.

Laryngitis is inflammation of the larynx, which can lead to oedema of the true vocal folds. Causes may be infectious or non-infectious (e.g., vocal strain, reflux laryngitis, chronic irritative laryngitis).

Generally clinically diagnosed.

Symptoms of acute disease, most commonly hoarseness, generally arise over a period of <7 days, are usually preceded by a viral upper respiratory infection (URI), and are ordinarily self-limiting. Patients may present with airway distress and high fever. Exudative tonsillopharyngitis with fever and anterior cervical lymphadenitis is highly suggestive of a bacterial origin.

The airway should be assessed first. Diligence and promptness are key, as they can be lifesaving.

Chronic laryngitis presents with hoarseness lasting >3 weeks. A thorough evaluation and specialist consultation should be obtained, because symptoms are similar to those of laryngeal malignancy.

Treatment for viral laryngitis consists of voice rest and hydration. For bacterial causes, antibiotics are used along with supportive measures. Vocal strain is managed with voice therapy and vocal hygiene.

In more pronounced cases, especially in children in whom the larynx is already small, oedema may lead to narrowing of the airway and airway compromise.

How well did you know this?
1
Not at all
2
3
4
5
Perfectly
416
Q

Epidemiology of laryngitis

A

Accurate figures with regard to acute laryngitis are difficult to collect, because it is generally unreported. Sore throat accounts for 1% to 2% of all patient visits to a primary care physician in the US. This accounts for approximately 7.3 million annual visits for children and 6.7 million for adults.

Viral agents tend to have annual periods of peak prevalence, such as rhinovirus infections in autumn and spring, and influenza virus infection epidemics generally from December to April.

TB laryngitis is historically a sequela of pulmonary TB, but recent cases without pulmonary involvement have been encountered. TB is the most common granulomatous disease of the larynx.

Fungal infections:

Generally caused by Candida albicans , Blastomyces dermatitis , Histoplasma capsulatum , and Cryptococcus neoformans .
Non-infectious causes of laryngitis include the following:

Irritant laryngitis (e.g., due to toxic exposure)
Allergic
Traumatic, especially due to heavy vocal use.

How well did you know this?
1
Not at all
2
3
4
5
Perfectly
417
Q

RFs of laryngitis

A
STRONG
Hx URTI
Incomplete/absent Haem Influenza (HIB) vaccine
Incomplete/absent diptheria vaccine
Contact with infected
Travel to endemic TB/diptheria
HIV/immunocompromise
Residence in nursing home
Inhaled corticosteroid/prolonged ABx use
Heavy vocal use
Tobacco use
How well did you know this?
1
Not at all
2
3
4
5
Perfectly
418
Q

Aetiology of laryngitis

A

Infectious laryngitis may be caused by viral, bacterial, or fungal infection.

Virus infection:

Generally the most common cause of infectious laryngitis
Rhinovirus is the most common virus that is aetiologically associated with URIs
Other causative viruses include parainfluenza virus, respiratory syncytial virus, influenza, and adenoviruses
Parainfluenza viruses type 1 and type 2, as well as influenza viruses, are the most common pathogens responsible for croup.
Bacterial infection:

Pathogens consist of Moraxella catarrhalis , Haemophilus influenzae , Streptococcus pneumoniae , Staphylococcus aureus , and Klebsiella pneumoniae
Epiglottitis is most frequently caused by Haemophilus influenzae type B
Diphtheria is caused by Corynebacterium diphtheriae . Occasional cases may be caused by Corynebacterium ulcerans
Although atypical forms of acid-fast bacilli can play a role, most TB infections are caused by Mycobacterium tuberculosis
Syphilis is a less common cause.

How well did you know this?
1
Not at all
2
3
4
5
Perfectly
419
Q

Sx of laryngitis

A
COMMON 
RFs presence
Hoarseness - most characteristic symptom of laryngitis.
Dysphagia 
Sore throat
Odynophagia
Cough
Hyperaemia of oropharynx
Hx heavy vocal cord use
GORD

UNCOMMON
Oropharyngeal white-grey exudates - DIPTHERIA

OTHER COMMON / Concomitant
Rhinitis
Fatigue/malaise
Fever
Enlarged tonsils
Enlarged/tender cer nodes
PNDrip
Dyspnoea

OTHER UNCOMMON / Concomitant
WL - TB
Tonsillopharyngeal exudate: bacterial
Acute resp distress - think acute epiglottitis, croup, or diphtheria
Drooling - epiglottitis
Stridor - acute epiglottitis, croup, or diphtheria.

How well did you know this?
1
Not at all
2
3
4
5
Perfectly
420
Q

Ix for laryngitis

A

1st
Laryngoscopy - mainstay of diagnosis - Performed if the patient presents initially to an otolaryngology specialist, but most primary care physicians are not experienced in the technique and diagnose most cases of viral laryngitis clinically

CONSIDER
Biopsy if suspect TB
Cultures - if bacterial
Nasal swab - bacterial
Rapid antigen - GAStrep
Other TB Ix if suspected
How well did you know this?
1
Not at all
2
3
4
5
Perfectly
421
Q

Rx of laryngitis

A

IF AIRWAY COMPROMISE
- Stabilise
NO SUSPICION DIPTHERIA = give corticosteroid
SUSPICION DIPTHERIA = isolate and give BENPEN + diptheria antitoxin

ACUTE VIRAL

  • Supportive
  • Guaifenesin / codeine

ACUTE BACTERIAL

  • Can give phenoxybenz
  • Guaifenesin / codeine

TB
- referral + TB therapy

VOCAL REST

FUNGAL -> refer otolaryngologist

How well did you know this?
1
Not at all
2
3
4
5
Perfectly
422
Q

Complications of laryngitis

A

Airway compromise
Formation of true vocal folds
Largyneal stenosis (TB)
Muscle tension dysphonia

How well did you know this?
1
Not at all
2
3
4
5
Perfectly
423
Q

Prognosis of laryngitis

A

Acute infectious laryngitis
This is often a self-limiting disease. With adequate voice rest and hydration the voice will return to normal within days. Continued extensive voice use can result in injury to the true vocal folds and formation of pathologies. It may also lead to the development of compensatory behaviour and can result in muscle tension dysphonia. Therefore, the patient needs to be counselled on the importance of voice rest and hydration.

Diphtheria
Patient age and immunisation status are important factors in terms of likely prognosis. Elderly and very young patients generally have a poorer prognosis, whereas past history of immunisation usually leads to a better prognosis. Any delay in administration of diphtheria antitoxin is more likely to result in associated toxic complications. Therefore, it is important to give diphtheria antitoxin as soon as possible.

Tuberculosis
Once appropriate treatment is started, laryngeal lesions should regress. If left untreated, progressing lesions can cause fibrosis, scarring, and, as a result, laryngeal stenosis. This may necessitate tracheotomy.

How well did you know this?
1
Not at all
2
3
4
5
Perfectly
424
Q

A 45-year-old man presents with hoarseness for 5 days, cough, and pain on swallowing. He has no fever but complains of increased mucus in his throat and occasional difficulty breathing. He has no prior history of hoarseness, surgery to the larynx, intubation, or vocal abuse. He has slight throat pain but denies reflux symptoms. On examination, there is no acute respiratory distress. His oral cavity is within normal limits, but the oropharynx shows hyperaemia. The tonsils are slightly enlarged and erythemic. Mirror examination of the larynx reveals diffuse oedema and erythema of the laryngeal structures, with increased mucus in the glottis. The airway is patent. There are no lesions involving the true vocal folds, and they are both mobile.

A

Laryngitis

How well did you know this?
1
Not at all
2
3
4
5
Perfectly
425
Q

A 45-year-old man has hoarseness for the past 3 weeks, accompanied by painful swallowing and cough. He has no fever and states that he has lost 2.5 kg over the last 3 months. He is known to be HIV positive. Indirect laryngoscopy reveals an exophytic lesion on the left true vocal fold, with oedema of both true vocal folds.

A

Laryngitis

How well did you know this?
1
Not at all
2
3
4
5
Perfectly
426
Q

Diphtheria is encountered rarely in developed nations but can still infect children and adults who are immunocompromised or have not received vaccinations. Initial symptoms include hoarseness and sore throat. There is progressive shortness of breath as the patient becomes generally ill. On examination, the patient is toxic. Oral examination reveals white-grey exudates on the tonsils and the soft palate, extending down to the base of the tongue. Other less common causes of infectious laryngitis include syphilis and fungal infection. In patients who are using corticosteroid inhalers, the onset of hoarseness should raise the suspicion of possible laryngeal candidiasis, which is the most common type of fungal laryngitis. Patients with laryngitis due to vocal trauma will have an accompanying history of increased voice use and high vocal demands.

A

Laryngitis

How well did you know this?
1
Not at all
2
3
4
5
Perfectly
427
Q

Define pertussis / whooping cough

A

Pertussis (also known as whooping cough) is an upper respiratory tract infection (URTI) characterised by a severe cough. Bordetella pertussis is the typical aetiological agent.

Patients can be infectious for several weeks if it is left untreated. B pertussis will spontaneously clear from the nasopharynx within 3 to 4 weeks from the onset of the cough in about 80% to 90% of patients if untreated; however, infants who have not been vaccinated or treated can remain culture-positive for more than 6 weeks. Other Bordetella species that may rarely cause pertussis or pertussis-like cough include B parapertussis , B bronchiseptica , or B holmesii ; these species are not vaccine-preventable. In China, pertussis is known as the ‘100-day cough’.

With the resurgence of pertussis in highly vaccinated populations, the disease continues to be a public health and medical concern.

Three identifiable stages typical in childhood cases: catarrhal, paroxysmal, and convalescent.

Initial symptoms may be similar to a cold, with rhinorrhoea and lacrimation, or a dry cough followed by episodes of severe coughing. Fever may be absent or low-grade.

Inspiratory whooping is a characteristic symptom in children but may be absent in infants, adolescents, and adults.

Culture of the bacterium Bordetella pertussis from nasal secretions can confirm the diagnosis, especially early in the course of the disease. A negative culture does not exclude the diagnosis. Other diagnostic tests include polymerase chain reaction (PCR) and serology.

Macrolide antibiotics are the preferred first-line agent for treatment and prophylaxis.

Universal childhood immunisation with the acellular pertussis vaccine is advised. Booster vaccination is recommended for pregnant women. In the US, booster vaccinations are also recommended for non-pregnant adults.

Cocooning is a prevention strategy to protect newborns and infants.

How well did you know this?
1
Not at all
2
3
4
5
Perfectly
428
Q

Epidemiology of pertussis / whooping cough

A

Despite being a vaccine-preventable disease, pertussis remains a fairly common condition in developed countries that have longstanding immunisation programmes with high vaccine uptake, and it is still considered a public health concern.

Hypotheses that may explain the resurgence of pertussis include: the theoretical evolving virulence of B pertussis , waning vaccine-conferred immunity, waning herd immunity or incomplete booster vaccinations, the introduction of acellular vaccines, and increased physician awareness of pertussis in adolescents and adults.

In England in 2016, there were 5945 confirmed cases of pertussis, a rate of 11 per 100,000, an increase from previous years with 4190 cases in 2015 and 3388 cases in 2014.

However, in 2012, there were 9367 confirmed cases of pertussis in England (a rate of 18 cases per 100,000). This was considerably higher than the cases reported in 2011 (1053; rate of 2 cases per 100,000).

In the UK, US, and other developed countries, an epidemic outbreak of pertussis typically occurs every 3 to 4 years.

How well did you know this?
1
Not at all
2
3
4
5
Perfectly
429
Q

Aetiology of pertussis / whooping cough

A

B pertussis is the typical aetiological agent.
Other Bordetella species that may rarely cause pertussis or pertussis-like cough include B parapertussis , B bronchiseptica , or B holmesii ; these species are not vaccine-preventable

How well did you know this?
1
Not at all
2
3
4
5
Perfectly
430
Q

RFs for pertussis / whooping cough

A

STRONG
Age < 6months - Infants under 6 months of age are not fully vaccinated and so are at increased risk. The series of primary vaccines for pertussis starts at 2 months and ends at 6 months.

Baby born to mother infected by >34weeks
No / incomplete immunisation
School teachers / healthcare workers - As an adult, immunity can decrease, especially if the last booster vaccine has not been administered in previous 5 years.
Close contact with infected person

How well did you know this?
1
Not at all
2
3
4
5
Perfectly
431
Q

Sx of pertussis / whooping cough

A
COMMON
RFs
Cough (begins 1-2w post infection)
Inspiratory whooping 
Rhinorrhoea - early stages
Post-tussive vomiting / gagging

OTHER COMMON
Sneezing
Absent/low grade fever
Decreased appetite

UNCOMMON
Apnoea (baby)

How well did you know this?
1
Not at all
2
3
4
5
Perfectly
432
Q

Ix for pertussis / whooping cough

A

Culture of a nasopharyngeal aspirate or swab from the posterior nasopharynx - Definitive diagnostic test, with a specificity of 100%. May be positive for B pertussis

PCR naso-pharyngeal aspirate - Sensitivity of 94% and a specificity of 97%

Serology - IF NO VACCINE - Because a vaccination can induce antibodies (i.e., IgM, IgA, and IgG antibodies), serological assays cannot differentiate infection from vaccine response

FBC - inc WCC

How well did you know this?
1
Not at all
2
3
4
5
Perfectly
433
Q

Rx of pertussis / whooping cough

A

INFANT <1month
Azithromycin / clarithromycin
Infants aged under 1 month who receive a macrolide should be monitored for hypertrophic pyloric stenosis (IHPS) DONT GIVE ERYTHROMYCIN

> 1month / non-preg
Azithromycin / clarithromycin / erythromycin base

If given after 3 weeks of onset of cough, treatment may have limited benefit.

How well did you know this?
1
Not at all
2
3
4
5
Perfectly
434
Q

Prognosis of pertussis / whooping cough

A

Infants <1 year in age account for the majority of deaths from pertussis in the US. The mortality rate of hospitalized infants is 1%.

In older children and adults, the prognosis is generally very good. It may take a few months for the cough to entirely resolve. Most healthy older children and adults will make a full recovery. Those with comorbid conditions, however, have a higher risk of morbidity and mortality. Because infection in newborns and infants can be severe, they need careful monitoring to avoid complications. The hospitalisation rate of adults in the US is about 3%.

The rate of developing pneumonia is up to 5% and rib fracture is up to 4%.

How well did you know this?
1
Not at all
2
3
4
5
Perfectly
435
Q

Complications of pertussis / whooping cough

A
Pneumonia
Seizure
Apnoea/brady
Rib fracture
Encephalopathy 
Otitis media
How well did you know this?
1
Not at all
2
3
4
5
Perfectly
436
Q

A 40-year-old high school teacher presents with cold symptoms lasting 3 weeks. She has low-grade fever, fatigue, and paroxysms of coughing. Her cold symptoms were initially mild but gradually increased in severity, resulting in her presentation to the emergency department. OTC cold medications have not provided relief.

A

pertussis / whooping cough

How well did you know this?
1
Not at all
2
3
4
5
Perfectly
437
Q

A 12-month-old female infant presents with spasmodic cough, cyanosis around her lips and fingers during coughing, and post-tussive vomiting. Her parents report that she has had a cold for approximately 3 weeks, and her appetite has decreased. The infant’s mother reports that she herself has been coughing for 6 weeks. The infant’s immunisation records are incomplete.

A

pertussis / whooping cough

How well did you know this?
1
Not at all
2
3
4
5
Perfectly
438
Q

Define vasovagal syncope / neurally mediated reflex syncope

A

Syncope is a syndrome characterised by a relatively sudden, temporary, and self-terminating loss of consciousness, associated with the inability to maintain postural tone, with rapid and spontaneous recovery. The causes vary widely from patient to patient, but they have a common underlying pathophysiology. The transient loss of consciousness with syncope is the result of a temporary inadequacy of cerebral nutrient flow, most often triggered by a fall in systemic arterial pressure below the minimum needed to sustain cerebral blood flow. Neurally mediated reflex syncope (NMRS) refers to a group of related conditions or scenarios in which symptomatic hypotension occurs as a result of neural reflex vasodilation and/or bradycardia. Vasovagal syncope (VS) refers to a particular type of NMRS also known as the common faint.

Neurally mediated reflex syncope (NMRS) refers to a group of related conditions or scenarios in which symptomatic hypotension occurs as a result of neural reflex vasodilation and/or bradycardia.

Vasovagal syncope (VS) refers to a particular type of NMRS also known as the common faint.

VS has many manifestations and is generally considered to encompass faints triggered by emotional upset, fear, and pain, as well as those occurring in less well-defined circumstances.

Patient education is the foundation of treatment for most NMRS syndromes, including VS.

Patients must be informed that, although reflex faints are almost never life-threatening, they tend to recur (often in clusters), and injury can result if preventive measures are not taken seriously.

Strategies for reducing syncope recurrences in the long term comprise 1) physical techniques to improve orthostatic tolerance, 2) pharmacological interventions to prevent depletion of intravascular volume and/or enhance arterial and venous tone, 3) cardiac pacing to avert bradycardia.

How well did you know this?
1
Not at all
2
3
4
5
Perfectly
439
Q

Epidemiology of vasovagal syncope / neurally mediated reflex syncope

A

Syncope is estimated to account for approximately 1% of accident and emergency department visits in Europe.

Prevalence of syncope varies in the population: 15% of children.

Vasovagal syncope is the single most common cause of syncope. It tends to peak in frequency in young adults and then again in older people. Unfortunately, in older patients the historical features may be less clear cut, making the diagnosis more difficult to establish. Most studies suggest that women (especially younger women) are more susceptible to vasovagal faints, although reporting bias may play a role, as young men may be less inclined to report such events.

How well did you know this?
1
Not at all
2
3
4
5
Perfectly
440
Q

Aetiology of vasovagal syncope / neurally mediated reflex syncope

A

Syncope has many possible causes, and these causes are commonly grouped in 3 main groups: neurally-mediated reflex syncopal syndromes, orthostatic, or cardiovascular.

In susceptible people, vasovagal syncope may be triggered by prolonged periods of upright posture; relative dehydration; excessively warm, closed-in environments; or extreme emotions. Common places for these events are churches, restaurants, and long queues.

How well did you know this?
1
Not at all
2
3
4
5
Perfectly
441
Q

RFs for vasovagal syncope / neurally mediated reflex syncope

A
STRONG
Hx syncope
Prolonged standing
Emotional stress
Dehydration
Preceding episode of nausea / vomiting 
Preceding episode of severe pain
How well did you know this?
1
Not at all
2
3
4
5
Perfectly
442
Q

Sx of vasovagal syncope / neurally mediated reflex syncope

A
STRONG
Nausea
Light headedness
Pallor 
Diaphoresis
Diminished vision/hearing
Physical injury
Fatigue after episode
Palpitations prior
Bradycardia at time
How well did you know this?
1
Not at all
2
3
4
5
Perfectly
443
Q

Ix for vasovagal syncope / neurally mediated reflex syncope

A

INITIAL

12 lead ECG - rule out cardiac causes
Serum Hb - rule out anaemia
BM/BG - rule out hypo
BHcG - rule out preg (can cause syncope)
Cardiac enzymes - rule out MI
D-Dimer - rule out PE
serum cortisol - rule out addisons
Urea/creatinine - check dehydration
OTHER
Tilt-table 
Standing BP
Valsalva manouvre 
Carotid sinus masage 
- all to rule out alternatives if unclear
How well did you know this?
1
Not at all
2
3
4
5
Perfectly
444
Q

Rx of vasovagal syncope / neurally mediated reflex syncope

A

Education re triggers
Rehydration

Can give fludrocortisone to aid re-expansion but RAREKY used

How well did you know this?
1
Not at all
2
3
4
5
Perfectly
445
Q

Prognosis of vasovagal syncope / neurally mediated reflex syncope

A

Recurrences are common but often occur in clusters. Most therapies have not been studied adequately to ascertain effectiveness for preventing subsequent events. However, data for physical manoeuvre intervention and for the drug midodrine tend to support benefit. In the long term, the mortality risk is very low, but injury is a concern due to recurrent falls, especially in older people

How well did you know this?
1
Not at all
2
3
4
5
Perfectly
446
Q

Complications of vasovagal syncope / neurally mediated reflex syncope

A

injury/fracture

Haemorrhage 2nd trauma

How well did you know this?
1
Not at all
2
3
4
5
Perfectly
447
Q

Define Legg-Calvé-Perthes’ disease

A

A self-limiting disease of the femoral head comprising of necrosis, collapse, repair, and re-modelling that presents in the first decade of life and is more commonly seen in boys. The cause is hypothesised to be single or multiple vascular events, followed by re-vascularisation. In later life, it can lead to a painful and poorly functioning hip. The disease was first described independently by Jacques Calvé, Arthur T. Legg, and Georg Perthes in 1910

Patients commonly have a characteristic phenotype of short stature, delayed bone age, and hyperactivity.

Although several aetiologies have been proposed, a multi-factorial involvement is most likely.

Typically unilateral, though bilateral involvement is present in 10% of cases.

Clinical features include a limping gait and hip pain frequently radiating into the thigh, knees, groin, or buttocks. Pain is worse with activities.

Treatment is age-dependent. Options include mobilisation and monitoring, non-surgical or surgical containment, or salvage procedures.

How well did you know this?
1
Not at all
2
3
4
5
Perfectly
448
Q

Epidemiology of Legg-Calvé-Perthes’ disease

A

Typically, Perthes’ disease presents in 4- to 8-year-olds with a mean age of 7 years, although it can affect children between 2 and 12 years of age. It is about 4 or 5 times more common in males than females. Perthes’ disease is bilateral in about 10% to 12% of cases.

The disease appears very uncommon in black people, though there are no true studies of incidence. Studies of incidence demonstrate that South Asians have 3 times the rate of East Asians, and that Caucasians have 9 times the rate of East Asians. More northerly latitudes, independent of race, appear to have greater disease incidence.

How well did you know this?
1
Not at all
2
3
4
5
Perfectly
449
Q

RFs for Legg-Calvé-Perthes’ disease

A
STRONG
Male
4-8yo
Socio-economic deprivation
Hypercoagulable states
WEAK
Urban population
Eskimos, Japanese, some central European races
Transient hip synovitis
Hip joint effusion
Passive smoking 
Skeletal dysplasias
Trisomy 21
Congenital defects - Genitourinary tract and inguinal anomalies have been demonstrated to have an association with Perthes' disease. 
Behaviour disorders - ADHD
Endocrinopathy
How well did you know this?
1
Not at all
2
3
4
5
Perfectly
450
Q

Aetiology of Legg-Calvé-Perthes’ disease

A

The cause of Legg-Calvé-Perthes’ (Perthes’) disease is hypothesised to be single or multiple vascular events, followed by re-vascularisation. Although several theories have been proposed over the years, it appears that Perthes’ is most likely to be multifactorial.

How well did you know this?
1
Not at all
2
3
4
5
Perfectly
451
Q

Sx of Legg-Calvé-Perthes’ disease

A

STRONG
RFS
Limp - painless limp that can be described as a gluteus medius lurch, though may be antalgic during acutely painful episodes
Limited range motion of hip - There is a flexion deformity of the hip in the acute setting. With progression of the disease adduction in flexion, internal rotation, and abduction in extension are limited due to impingement lesions. The later stage of the disease may be characterized by a global reduction in all ranges of motion with associated pain indicative of joint arthritis.

OTHER COMMON
Short stature
Muscle wasting
Hyperactivity
Trendelenburgs 
Synovitis 
UNCOMMON
Knee pain
Thigh pan
Groin buttock pain
Asymmetric limb length 
Minor trauma
How well did you know this?
1
Not at all
2
3
4
5
Perfectly
452
Q

Ix for Legg-Calvé-Perthes’ disease

A

Bilateral hip x-rays - femoral head collapse and fragmentation; subchondral

CONSIDER
FBC - N
ESR - N / reactively increased
CRP - N / reactively increased

Bone scintigraphy - cold spot in the affected hip early in the disease process

MRI possible

How well did you know this?
1
Not at all
2
3
4
5
Perfectly
453
Q

Rx of Legg-Calvé-Perthes’ disease

A

Supportive pain relief - paracetamol 75mg/kg 4-6, max / ibuprofen (10mg/kg 6-8hrly)

<5yo

  1. Mobilisation + monitoring
  2. Non-surgical containment
    - A dynamic radiographic screening arthrogram allows assessment of the optimal position in which to contain the hip. This position is then maintained in an abduction cast, splint, or brace, or a combination of all 3 where a spica and adductor tenotomy is followed by a period in an abduction brace.

5-7YO
Epiphyseal involvement >50%
- SURGICAL containment

7-12YO
SAME but also:
- SALVAGE procedure
Salvage options address either the acetabulum to recreate or deepen the socket or aim to improve congruence between the weight-bearing areas by altering the femoral head orientation with a femoral valgus osteotomy.

> 12 with arthritis
Replacement arthroplasty after skeletal maturity

How well did you know this?
1
Not at all
2
3
4
5
Perfectly
454
Q

Prognosis of Legg-Calvé-Perthes’ disease

A

The prognostic variables include femoral head sphericity, congruence with the acetabulum, age at presentation and the geometric extent of the femoral head involvement. Congruence is a more important predictor of future arthritis than femoral head sphericity.

In later life, Legg-Calvé-Perthes’ (Perthes’) disease leads to a painful and poorly functioning hip. More than 50% of patients with Perthes’ disease will develop signs of osteoarthritis between their 4th and 5th decades.

When girls are affected, involvement of the femoral head is more severe compared with that in boys of the same age. Females also tend to have a more severe involvement than males at a later age.

Predictors of poor outcome include recurrent synovitis, lateral subluxation, more than 50% femoral head involvement and a collapsed lateral pillar.
Predictors of good outcome include femoral head involvement classified as Catterall group I or II, Salter Thompson group A, Herring sub-types A or B, and children under 7 years of age.

How well did you know this?
1
Not at all
2
3
4
5
Perfectly
455
Q

Complications of Legg-Calvé-Perthes’ disease

A

Osteoarthritis LT
Limb length inequality LT
Stiffness + loss of rotation V

How well did you know this?
1
Not at all
2
3
4
5
Perfectly
456
Q

A 5-year-old Caucasian boy is brought in to the orthopaedic clinic by his mother with complaints of a limp favouring the right side with no associated pain. This painless limp had started insidiously 3 weeks earlier and was first noticed by the school physical education teacher. The mother notes that it has recently been getting worse. He is one of 3 siblings and lives with his single mother. The other siblings include an elder sister (from his mother’s earlier marriage) and a younger brother. His mother’s current partner is a heavy smoker. His mother recalls that 1 year earlier he came from school and complained of right knee pain. This was initially overlooked for a few days, but when it persisted he was taken to the general practitioner who reassured his mother but did not arrange follow-up. The symptoms had recurred the following month when he was taken to the emergency department and blood tests and x-rays were reportedly normal.

A

Legg-Calvé-Perthes’ disease

How well did you know this?
1
Not at all
2
3
4
5
Perfectly
457
Q

A 7-year-old girl presents with a painless limp, although she has been treated for acute pain in the past. She has a free range of hip motion. Plain anteroposterior and frog lateral radiographs reveal Legg-Calvé-Perthes’ disease of the right hip. She undergoes screening of the right hip under a general anaesthetic with an arthrogram to determine the best position of right femoral head containment in the hip joint. A surgical containment by way of a varus lateral opening wedge osteotomy of the proximal femur, fixed with an AO (Arbeitgemeinschaft fur Osteosynthesefragen) 3.5 mm pre-contoured plate and held with 3.5-mm AO screws, is performed. She is maintained non-weight-bearing post-procedure. A radiograph 4 weeks later reveals a good containment of the hip joint, satisfactory alignment at the osteotomy and some early callus formation medially. A further radiograph 3-months postoperatively shows a good union at the osteotomy and a well-contained hip joint. Weight-bearing and resumption of normal activities is then allowed progressively.

A

Legg-Calvé-Perthes’ disease

How well did you know this?
1
Not at all
2
3
4
5
Perfectly
458
Q

The typical age of presentation varies between 4 and 8 years but can range from 2 to 12 years with a few reports of involvement in adolescents. The usual phenotype of a child with Legg-Calvé-Perthes’ (Perthes’) disease is a person with short stature, delayed bone age and a rather hyperactive child. A limp is the most common presentation with a large proportion being unilateral afflictions. Bilateral involvement is seen in up to 10% to 12% of the cases, however, both hips are rarely at the same stage of the disease at any given time. Pain may vary in site, intensity and radiation patterns. It may be referred to the knee, thigh, groin, or buttocks. Perthes’ disease is a non-traumatic condition, although a history of minor trauma may be noted.

A

Legg-Calvé-Perthes’ disease

How well did you know this?
1
Not at all
2
3
4
5
Perfectly
459
Q

Define allergy in a child

A

Allergy = a hypersensitivity reaction initiated by immunological mechanisms. This can be IgE mediated [eg peanut] or non IgE mediated [coeliac disease]

Hypersensitivity = objectively reproducible symptoms in response to a defined stimulus at a dose which is tolerated by normal people

How well did you know this?
1
Not at all
2
3
4
5
Perfectly
460
Q

Epidemiology of allergy in a child

A

40% if children have AR, asthma or eczema and up to 6% develop a food allergy

Increasing in prevalence in many countries

Commonest chronic diseases of childhood, and causes of hospitalisation and school absence

Significant morbidity and can be fatal

20 children die from asthma each year
2 from food allergy

How well did you know this?
1
Not at all
2
3
4
5
Perfectly
461
Q

Aetiology/RFs of allergy in a child

A

Polymorphisms in genes lead to susceptibility

Abnormal immune response to harmless stimuli (usually protein)

Developing immune system is “sensitised” before the allergic response develops
N.B. sensitisation can be occult, EG egg sensitisation from trace amounts in maternal breast milk

Only a few stimuli typically account for most allergic disease:
Inhalant: dust mite, plant pollen, pet dander, moulds [asthma/AR]
Ingestant: nuts, seeds, legumes, cows milk, egg, seafood, fruits
Bites/stings, drugs, latex

Some proteins may be broken down by cooking: eg apples

Early phase:

  • release of histamine, LTs and PGs
  • urticaria, angiooedema, sneezing, bronchospasm

Late phase 4-6 hrs later
- nasal congestion, cough, bronchospasm

NB non-IgE mediated = delayed onset of symptoms by 4-6 hrs + more varied clinical picture

How well did you know this?
1
Not at all
2
3
4
5
Perfectly
462
Q

Sx of allergy in a child

A

Mouth breathing
OSA/snoring
Allergic salute
Pale/swollen inferior turbinates
Hyperinflated chest or Harrison sulci from chronic untreated asthma
Atopic eczema affecting limb flexures
Allergic conjunctivitis: may be prominent creases ie DennieMorgan folds, blue grey discolouration below the lower eyelids

Check growth with food allergy
- also with corticosteroid Rx

How well did you know this?
1
Not at all
2
3
4
5
Perfectly
463
Q

Immunotherapy Rx of allergy in a child

A

Allergen immunotherapy
AR, conjunctivitis, asthma, anaphylaxis, insect stings
- SCIT/SLIT
- Must have good adherence, only 1 allergen
- Months of progressive low dose exposure under specialist supervision

How well did you know this?
1
Not at all
2
3
4
5
Perfectly
464
Q

Food allergy definition

A

Pathological immune response mounted against a specific food allergen. Usually IgE mediated.
NB a non-immunological hypersensitivity reaction to specific food is called food intolerance.

How well did you know this?
1
Not at all
2
3
4
5
Perfectly
465
Q

Aetiology of food allergy in children

A

Infants most common is milk, egg and peanut
Children = peanut, tree nut, fish and shellfish

Food allergy can be secondary. If so, usually because of cross-reactivity of the allergen IE similar protein structures
EG apple + birch tree = oral allergy syndrome

How well did you know this?
1
Not at all
2
3
4
5
Perfectly
466
Q

Clinical features of food allergy/intolerance in children

A

IgE - urticaria, facial swelling, anaphylaxis, 10-15 mins after food
Non IgE - diarrhoea, vomiting, abdominal pain, sometimes failure to thrive, colic/eczema can be present, can present as blood in the stools from proctitis
Intolerance - diarrhoea, vomiting, bloating, cramps, steatorrhoea

How well did you know this?
1
Not at all
2
3
4
5
Perfectly
467
Q

Diagnosis of food allergy/intolerance in children

A

IgE mediated - skin prick tests or RAST, NB both can yield false positive IE individual sensitised but not allergic, but usually a greater response = greater likelihood of allergy

Non IgE - Usually relies on clinical Hx + examination, + endoscopy/biopsy. Diagnosis is supported by presence of eosinophilic infiltrates

Both IgE/non - gold standard is exclusion of food, followed by a double blind placebo challenge

How well did you know this?
1
Not at all
2
3
4
5
Perfectly
468
Q

Rx of food allergy/intolerance in children

A

Avoidance
Allergic attack education
Adrenaline / EpiPen

Food allergy to cows milk / eggs usually resolves in early childhood, but nuts/seafood usually persists

How well did you know this?
1
Not at all
2
3
4
5
Perfectly
469
Q

Rx of allergic rhinoconjunctivitis

A

Second gen non-sedative antihistamines (top/oral)
Topical corticosteroid nasal/eye preps
Chromoglycate (mast cell stabiliser) eye drops
LTRA eg montelukast
Nasal decongestants - DO NOT USE FOR MORE THAN 7 days else risk of rebound effect
Allergen IT
Systemic corticosteroids ARE NOT USED DUE TO RISK OF ADVERSE EFFECTS

How well did you know this?
1
Not at all
2
3
4
5
Perfectly
470
Q

Define urticaria / angiooedema

A

Rash resulting from exposure to allergen or viral infection

Angiooedema = involvement of deeper tissues: swelling of the lips and soft tissues around the eyes

Chronic = >6weeks - usually non-allergic

NB physical urticarias: cold, delayed pressure, heat contact, solar, vibratory

NB other causes: water (aquagenic), sweating (cholinergic), aspirin, NSAIDs, C1 esterase inhibitor deficiency [angiooedema w/o urticaria/pruritis]

How well did you know this?
1
Not at all
2
3
4
5
Perfectly
471
Q

Aetiology of cows milk intolerance

A
  • May be IgE mediated: immediate Sx on ingestion - urticaria->angio oedema-> anaphylaxis depending on severity
    DIAGNOSED BY: skin prick test
  • Non - IgE - loose stools, failure to thrive, skin prick negative, elimination reduces symptoms, reintroduction reproduces Sx
  • Non-allergic cows milk hypersensitivity - aka temporary lactose intolerance - prev well, develops D+V, vomiting settles but watery stools continue for several weeks, on stool sample - NO PATHOGENS but reducing substances (ie detection of glucose, fructose, lactose, galactose, and pentose, depicting malabsorption)
How well did you know this?
1
Not at all
2
3
4
5
Perfectly
472
Q

Define anal fissure

A

An anal fissure is a tear of the squamous epithelial mucosa of the anal canal, between the anocutaneous junction and the dentate line. They most commonly occur during passage of a firm stool. Anal fissures are common in infancy, and they represent the most common cause of bright rectal bleeding at any age.

How well did you know this?
1
Not at all
2
3
4
5
Perfectly
473
Q

Aetiology of anal fissure

A

The generally accepted proximate cause of the anal fissure is a mechanical tear resulting from the passage of hard stool. An unhealed fissure may become infected and develop into a chronic ulcer. A healed fissure may develop into a classic sentinel skin tag in the posterior midline.

Fissures have a predilection for the posterior midline (90%) but may also be located in the anterior midline or laterally. The explanation for this phenomenon is both anatomic and functional. The posterior commissure of the anoderm is less well perfused than other anodermal regions.

How well did you know this?
1
Not at all
2
3
4
5
Perfectly
474
Q

Epidemiology of anal fissure

A

Most fissures affecting the pediatric population manifest in children aged 6-24 months; however, the overall incidence of the problem is not well described.

How well did you know this?
1
Not at all
2
3
4
5
Perfectly
475
Q

RFs for anal fissure

A

Constipation

Low fibre diet

How well did you know this?
1
Not at all
2
3
4
5
Perfectly
476
Q

Sx of anal fissure

A

Cry on bowel movements
Streaks of blood on surface of faeces
Hard stool
Fresh blood on toilet paper/loo

It is important to remember that underlying systemic illness frequently manifests with anal lesions. Thus, pertinent negatives, such as fever, rash, oral or skin lesions, weight loss, diarrhea, and abdominal pain, should be excluded.

Visualisation of fissure on examination
The fissure appears as a minor laceration, usually in the midline, and is more often posterior than anterior. If the fissure is chronic, a small external skin tag (ie, sentinel tag) may be identified at the base of the laceration; this represents epithelialized granulomatous tissue secondary to chronic inflammation.

If a fissure is suspected, palpation of the abdomen is essential to check for palpable masses (stool) in the left lower quadrant.

If the examination is limited by pain and the diagnosis remains unclear, an examination under anesthesia should be pursued.

How well did you know this?
1
Not at all
2
3
4
5
Perfectly
477
Q

Rx of anal fissure

A

Will heal in 10-14 days
Surgery only required if doesn’t heal after 6-8weeks (eg, chemical or surgical sphincterotomy)

Conservative
Fibre diet, stool softeners (osmotic usually polyethylene glycol)

Chemical sphincterotomy - topical GTN 0.2%, [SE GTN headache]
Topical diltiazem 2% gel
Topical nifedipine with lidocaine
Botox injections reduce anal sphincter tone by inhibiting AcH release (normally not done in children)

Surgery:
Outpatient open lateral internal sphincterotomy

How well did you know this?
1
Not at all
2
3
4
5
Perfectly
478
Q

Ix for anal fissure

A

Clinical

How well did you know this?
1
Not at all
2
3
4
5
Perfectly
479
Q

Prognosis of anal fissure

A

Recurrence rates of open or closed lateral sphincterotomy have been reported to be 0-10%, with most of the recurrences occurring in adults and with chronic fissures. In contrast, anal dilatations have the highest rates of fistula recurrence (10-30%) and, for this reason, are not recommended in children.

How well did you know this?
1
Not at all
2
3
4
5
Perfectly
480
Q

Complications of anal fissure

A

Headache and diarrhea are the most common complications of administering topical nitrates and stool softeners, respectively. Significant hypotension with topical nitrate administration has not been reported in the literature. However, during the first office visit, children and their families should be questioned about a history of vascular headaches, and blood pressure should be taken before the initial application of topical nitrates. Incontinence has not been associated with these therapeutic regimens.
Short-term complications of operative therapy include urinary retention, hematoma formation, and incontinence. Long-term complications, such as difficulty controlling flatus, daytime soiling of underwear, and nighttime incontinence, are noted with both open and closed internal sphincterotomy.

How well did you know this?
1
Not at all
2
3
4
5
Perfectly
481
Q

Define asthma

A

Asthma is a chronic respiratory disorder characterised by variable airway inflammation, airway obstruction, and airway hyper-responsiveness. These features interact to determine the clinical symptom pattern of the individual. While the majority of asthmatic children have an intermittent symptom phenotype, the minority have persistent symptoms, reflecting the underlying chronic inflammation. In older children, as in adults, this may lead to permanent structural alterations of the airway wall (airway remodelling) and potentially a more severe asthma phenotype.

This topic covers the treatment of children up to 12 years of age. Children 12 years and older are treated the same as adults, except for emerging therapies such as bronchial thermoplasty. Please see our Asthma in adults topic for more information.

As paediatric asthma differs from adult asthma, child-specific asthma guidelines should be used and adult and adolescent guidelines should not be extrapolated to younger age groups.

The prevalence of childhood asthma appears to have plateaued in many affluent countries. However, asthma remains a significant cause of childhood morbidity and mortality, and is still a common problem managed in ambulatory and emergency care settings.

Most children with asthma have mild intermittent asthma and do not require daily therapy. Minimum doses and medications for maximal control should be used in those who require preventative therapies. Therapy should be individualised.

A number of important differential diagnoses, adherence and environmental issues should be considered when evaluating a child with suspected asthma, especially in very young children or when high doses of inhaled corticosteroids are required.

How well did you know this?
1
Not at all
2
3
4
5
Perfectly
482
Q

Epidemiology asthma

A

Paed asthma = most common chronic respiratory disease in the developed world

10% prev roughly

Prepubertal male + post pubertal females most common

How well did you know this?
1
Not at all
2
3
4
5
Perfectly
483
Q

Aetiology of asthma

A

Mixed genetic / environmental

Transient early wheezing (1 or more episodes of lower respiratory tract illness [LRTI] with wheezing in the first 3 years of life, but no wheezing at 6 years of age)
Late-onset wheezing (no history of LRTI with wheezing in the first 3 years, but wheezing at 6 years of age)
Persistent wheezing (1 or more episodes of LRTI with wheezing in the first 3 years of life, and wheezing at 6 years of age).

How well did you know this?
1
Not at all
2
3
4
5
Perfectly
484
Q

RFs for asthma

A
STRONG 
Allergic sensitisation/positive SPT
Atopic disease
Wheezing triggered by non-viral non-allergic environmental factors (change in weather, environmental tobacco smoke, exercise, and emotion)
Resp virus early in life
Serum eosinophilia (>4%)
Fix
Passive/active smoking
Abnormal lung function/hyperresponsiveness

WEAK
Female
Air pollution
Obesity

How well did you know this?
1
Not at all
2
3
4
5
Perfectly
485
Q

Sx of asthma

A
COMMON
Wheezing
SOB/dyspnoea
Features atopic disease
Tachypnoea
Recessions
Accessory muscle use 
Hx of response to Rx
OTHER COMMON
>3yo
Dry night time cough
Dyspnoea on exertion 
Expiratory wheezing

UNCOMMON
Chest wall deformity - Harrison sulci, hyperinflation

How well did you know this?
1
Not at all
2
3
4
5
Perfectly
486
Q

Ix for asthma

A

1st
Spirometry - dec FEV1/FVC ratio >0.8 [obstructive pattern]
>12% improvement post bronchodilator
CxR - hyperinflation/normal

CONSIDER
PEFR challenge - BEWARE (limited role in paediatric asthma due to its effort dependence and lack of sensitivity)
Airway challenge - Dec FEV1
Exercise challenge
FBC - Eosinophilia
Sweat test - NORMAL [CF]
FeNO - elevated
How well did you know this?
1
Not at all
2
3
4
5
Perfectly
487
Q

Rx of asthma

A

0-4yo

  1. SABA PRN + education
  2. Low dose ICS ie Fluticasone propionate 50 micrograms/day + SABA
  3. Add LTRA - Montelukast 4mg oral
  4. Med dose ICS 125ug fluti
  5. Add LTRA
  6. High dose ICS 250ug fluti
  7. Add LABA

IE NO LABAs in <4 until very last line Rx

5+
1. SABA
2. Low dose ICS
3. Add LTRA/theophylline/sodium chromoglycate
4. Med dose ICS + SABA 
OR Low dose ICS + LABA/LTRA/theophylline
5. High dose ICS + LABA/LTRA/theophylline
6. Immunomodulator - omalizumab
7. Oral CS / immunomodulator
How well did you know this?
1
Not at all
2
3
4
5
Perfectly
488
Q

Prognosis of asthma

A

Life expectancy equivalent

ICS
Regular use has negative effects on short-term linear growth, particularly at higher doses. A transient slowing in height velocity does occur initially, and may affect final achieved adult height.

How well did you know this?
1
Not at all
2
3
4
5
Perfectly
489
Q

Complications of asthma

A
Exacerbation - ST H
Remodelling - V M
Dental caries
RML syndrome
Oro candidiasis
Adrenal suppression
How well did you know this?
1
Not at all
2
3
4
5
Perfectly
490
Q

Define pre-school wheeze

A

A wheeze present in pre-school children. Thought to be viral-associated, resulting from small airway susceptibility to narrowing/obstruction during inflammation/aberrant immune responses to viral infections.
Thus = episodic

How well did you know this?
1
Not at all
2
3
4
5
Perfectly
491
Q

RFs of pre-school wheeze

A

Maternal smoking during/after preg
Prematurity
Male
Usually resolves by 5

How well did you know this?
1
Not at all
2
3
4
5
Perfectly
492
Q

Aetiology of pre-school wheeze

A
Bronchiolitis, pneumonia 
Transient early wheezing 
Non-atopic wheeze after LRTI
Atopic asthma
Cardiac failure 
Inhaled foreign body
How well did you know this?
1
Not at all
2
3
4
5
Perfectly
493
Q

Children may present with episodes of recurrent cough. These episodes may be triggered by viral infections, change in weather, or exercise. The cough is typically dry in nature and only occasionally associated with an audible wheeze. Closer questioning may reveal the predominant trigger and a feeling of chest tightness and difficulty breathing accompanying the cough. A beta-2 agonist MDI typically provides relief.

A

Asthma

How well did you know this?
1
Not at all
2
3
4
5
Perfectly
494
Q

Indications for hospital admission in asthma?

A

Persisting dyspnoea/tachy despite reliever therapy
Exhaustion
Marked reduction in PEFR
<92% saturation on air

CxR only indicated if there are unusual Sx / Infection Sx

How well did you know this?
1
Not at all
2
3
4
5
Perfectly
495
Q

Rx of an asthma attack?

A
SEVERE
O2 via face mask 
Try SABA 10 puffs / oral or IV pred
Neb ipatropium 
Repeat bronchodilators every 20-30mins

Life threatening
- AKA silent chest, altered consciousness, cyanosis, O2 <92%, PEFR <33%

Neb salb+ipa bromide
IV hydrocortisone
Discuss with senior clinician or PICU
Repeat bronchodilator every 20-30 min

How well did you know this?
1
Not at all
2
3
4
5
Perfectly
496
Q

Rx of an asthma attack?

A
SEVERE
O2 via face mask 
Try SABA 10 puffs / oral or IV pred
Neb ipatropium 
Repeat bronchodilators every 20-30mins

Life threatening
- AKA silent chest, altered consciousness, cyanosis, O2 <92%, PEFR <33%

Neb salb+ipa bromide
IV hydrocortisone
Discuss with senior clinician or PICU
Repeat bronchodilator every 20-30 min

How well did you know this?
1
Not at all
2
3
4
5
Perfectly
497
Q

Define autism spectrum disorder

A

Autism spectrum disorder (ASD) is characterised by persistent impairments in social communication, and restricted, repetitive, and stereotyped patterns of behaviours, interests, or activities.

Abnormal development is present during early childhood, but may only become manifest later. There may be a history of language delay (single-word or phrase speech delay) and 25% of children lose previously acquired acquired language skills (regression).

Approximately 20% to 30% of children develop epilepsy and 50% have intellectual disability; others have ability in the average or above average range.

How well did you know this?
1
Not at all
2
3
4
5
Perfectly
498
Q

Epidemiology of autism spectrum disorder

A

In the last 10 years, the detected prevalence of ASD has increased substantially.

More recent data suggest that at least 1% of children have ASD; some studies report a prevalence of around 2%.

Around 50% of children with ASD have an intellectual disability.

Data suggest that the rise in ASD diagnosis has been accompanied by a fall in the prevalence of other neurodevelopmental disorders and that more accurate clinical diagnosis underlies the increase in detected prevalence.

How well did you know this?
1
Not at all
2
3
4
5
Perfectly
499
Q

Aetiology of autism spectrum disorder

A

It is well established that there are strong genetic influences in the development of ASD, with a sibling recurrence risk in the region of 10%.

In a minority of cases (approximately 10%) ASD occurs in association with cytogenetically detectable chromosomal abnormalities and recognised genetic syndromes, such as fragile X syndrome, tuberous sclerosis, and Down’s syndrome.

How well did you know this?
1
Not at all
2
3
4
5
Perfectly
500
Q

RFs for autism spectrum disorder

A

STRONG
Male 4:1
FHx

WEAK
Genetic variants
Chromosomal abnormalities - NF1/Downs/tuberous sclerosis

How well did you know this?
1
Not at all
2
3
4
5
Perfectly
501
Q

Sx of autism spectrum disorder

A

COMMON
RFs
Language delay or regression - no single words by 2yo, or phrases by 33months, lack of babble in 1st yr, echolalia, stereotyped speech, NB some may develop language very early
Verbal + non-verbal communication impairment - dec pointing, lack of social interaction, dec facial expressions, unable to depict emotion
Social impairment - uninterested in play, lack of friendships, misunderstandings
Repetitive/rigid/stereotyped behaviour or interests - repetitive language or requests, toys, play, obsessions, distressed if this is altered.

OTHER COMMON
Placid or irritable as a baby
Feeding difficulties 
Unusual posturing
Motor stereotypes - repetitive snapping, clapping, spinning etc
Sensory interests 

UNCOMMON
Macrocephaly

Signs of fragile X syndrome, tuberous sclerosis, dysmorphism (may indicate chromosomal disorder) are weakly associated with ASD.

How well did you know this?
1
Not at all
2
3
4
5
Perfectly
502
Q

Ix for autism spectrum disorder

A

ASD screening questionnaire - eg autism screening questionnaire
Childhood autism rating scale
Childhood autism screening test
Consider audiology

How well did you know this?
1
Not at all
2
3
4
5
Perfectly
503
Q

Rx of autism spectrum disorder

A
Applied behaviour analysis
Picture exchange communication systems
ASD preschool programme 
Early start Denver model 
ChildsTalk

PHARMACOLOGICAL

Some psychotropics may aid those with very challenging behaviours, aggression, or irritability that does not respond to behavioural techniques

Risperidone
Aripiprazole
Fluoxetine
Methylphenidate
Atomoxetine
Melatonin - for sleep disturbance 
Buspirone
How well did you know this?
1
Not at all
2
3
4
5
Perfectly
504
Q

Complications of autism spectrum disorder

A
Developmental regression
Intellectual disability
Epilepsy
Anxiety
Depression
How well did you know this?
1
Not at all
2
3
4
5
Perfectly
505
Q

Prognosis of autism spectrum disorder

A

any adults with ASD require lifelong full-time care. About 15% of adults with ASD will live independent lives, whereas 15% to 20% will live alone with community support.

Verbal and overall cognitive capacity seem to be the most important predictors of ability to live independently as an adult.

How well did you know this?
1
Not at all
2
3
4
5
Perfectly
506
Q

A 3-year-old boy presents following concerns about language development. He started using single words at age 18 months but still doesn’t use 2 words together. He stopped using words he had previously learnt between 18 and 24 months, but has now regained most of these words. He also seems uninterested in engaging with other children. He occasionally engages with his parents but less than they think he should. He doesn’t tend to look at them much and he has difficulty maintaining eye contact with them. When he wants something he pulls them to where the object is and screams; he doesn’t point like other children. His parents have also noticed that he does not play in the same way as other children of his age; he tends to line toys up, or plays with certain aspects of them, such as the car doors. He doesn’t use the toys in the imaginative way that other children do. When his toys are moved he becomes very upset. He tends to become distressed when he thinks there is change around the house. In contrast, he is not concerned when either of his parents leaves the house. He tends to flap his hands at times and his parents report him staring at the ceiling lights for 10 to 20 minutes at a time. He is a fussy eater and hates being messy.

A

ASD

How well did you know this?
1
Not at all
2
3
4
5
Perfectly
507
Q

A 24-year-old filing clerk at the local library is referred for assessment as a result of increasing tearfulness. He has recently graduated in computer science at university and was offered this job as he knows the librarians well, having spent most weekends at the library throughout his life. He reports never having made friends throughout school or university and describes feeling lonely as a consequence. At assessment he presents as reasonably well-kempt but dressed in a somewhat old-fashioned and quirky manner, and uses eye contact only fleetingly. His speech is monotone with rapid explosive bursts making it difficult to understand what he is saying. Throughout the assessment he fails to elaborate on anything without prompting. He only becomes animated when talking about artificial intelligence in computer technology, not recognising that his assessor does not understand and is not particularly interested in this topic.

A

ASD

508
Q

Children with ASD have a wide range of clinical presentations (see diagnostic approach). Patterns of difficulties and behaviours vary with age but language delay is a common parental concern. A detailed neurodevelopmental and current functioning history is required. Babies may be either unusually placid or irritable; feeding difficulties are common. During the toddler years, impairments in speech, communication, play, and social functioning become more obvious. There may be regression of previously learnt language skills; regression of motor skills alone should make clinicians think of other possible diagnoses, and children should be appropriately investigated. Feeding difficulties are common, with particular rigidities around certain foodstuffs and the environment when eating. There may also be sensory-related difficulties such as a negative, and sometimes idiosyncratic, reaction to certain textures, sounds, and other sensory stimuli. By contrast, some children are interested in particular sensory environments or activities. Children may show motor mannerisms, such as hand flapping or spinning. Repetitive behaviours can be very challenging to manage. Most children with ASD also have coexisting conditions, such as difficulties with sleep, or anxiety. [6]

People with ASD can be diagnosed from early childhood into late adulthood. Many young people and adults have mental health diagnoses, such as anxiety disorder, or depression. Mental health conditions can be a presenting feature in young or older adults.

For adults with ASD, a diagnosis may or may not be sought, depending on the extent to which an individual’s difficulties are affecting their daily life. When identified in adulthood, ASD is often identified as the cause of difficulties at home, in social relationships or those with a partner, or in the workplace.

Some adults seek a diagnosis for the first time following a diagnosis of ASD in one of their children, or in another family member.

A

ASD

509
Q

Children with ASD have a wide range of clinical presentations (see diagnostic approach). Patterns of difficulties and behaviours vary with age but language delay is a common parental concern. A detailed neurodevelopmental and current functioning history is required. Babies may be either unusually placid or irritable; feeding difficulties are common. During the toddler years, impairments in speech, communication, play, and social functioning become more obvious. There may be regression of previously learnt language skills; regression of motor skills alone should make clinicians think of other possible diagnoses, and children should be appropriately investigated. Feeding difficulties are common, with particular rigidities around certain foodstuffs and the environment when eating. There may also be sensory-related difficulties such as a negative, and sometimes idiosyncratic, reaction to certain textures, sounds, and other sensory stimuli. By contrast, some children are interested in particular sensory environments or activities. Children may show motor mannerisms, such as hand flapping or spinning. Repetitive behaviours can be very challenging to manage. Most children with ASD also have coexisting conditions, such as difficulties with sleep, or anxiety. [6]

People with ASD can be diagnosed from early childhood into late adulthood. Many young people and adults have mental health diagnoses, such as anxiety disorder, or depression. Mental health conditions can be a presenting feature in young or older adults.

For adults with ASD, a diagnosis may or may not be sought, depending on the extent to which an individual’s difficulties are affecting their daily life. When identified in adulthood, ASD is often identified as the cause of difficulties at home, in social relationships or those with a partner, or in the workplace.

Some adults seek a diagnosis for the first time following a diagnosis of ASD in one of their children, or in another family member.

A

ASD

510
Q

Define breath-holding attack

A

Breath-holding spells (BHS) are the occurrence of episodic apnea in children, possibly associated with loss of consciousness, and changes in postural tone.

511
Q

Epidemiology of breath-holding attack

A
5% of population
M=F
6-18 months mainly 
Disappear after 5yo
Unusual before 6months
Fix
512
Q

Aetiology / RFs for breath-holding attack

A

FHx

4 types

  1. Simple - holding of breath in and expiration
  2. Cyanotic - anger/frustration, blue in colour, loss of tone, LOC, dec CO, no post-ictal phase/incontinence, child is fine between spells
  3. Pallid breath-holding - child turns pale, LOC with little crying, some relationship to adult syncope
  4. Complicated breath-holding - generally begins as either a cyanotic or pallid spell that then is associated with seizure like activity. Normal EEG.
513
Q

Sx of breath-holding attack

A

4 types

  1. Simple - holding of breath in and expiration
  2. Cyanotic - anger/frustration, blue in colour, loss of tone, LOC, dec CO, no post-ictal phase/incontinence, child is fine between spells
  3. Pallid breath-holding - child turns pale, LOC with little crying, some relationship to adult syncope
  4. Complicated breath-holding - generally begins as either a cyanotic or pallid spell that then is associated with seizure like activity. Normal EEG.

Lack of incontinence, last of post-ictal phase, ECG rules out arrhythmia

514
Q

Ix for breath-holding attack

A

Clinical
ECG - rule out arrythmia
Some evidence that anaemic = more prone to BHA/S

515
Q

Rx of breath-holding attack

A

Reassurance
No definitive Rx

Some evidence to suggest iron therapy if anaemic may help

516
Q

Define GORD

A

The involuntary passage of gastric contents into the oesophagus

Very common in infancy

Inappropriate relaxation of the oesophageal sphincter as a result of functional immaturity.

Resolves by 12months

517
Q

Aetiology / RFs GORD

A

Neurodevelopmental disorders
Preterm
Surgery for oesophageal atresia
Diaphragmatic hernia

518
Q

Sx of GORD

A

Recurrent regurgitation or vomiting
Normal weight
Otherwise well

NB more dangerous with neurodevelopmental disorders

519
Q

Ix for GORD

A

Clinical

Can do
24hr oesophageal ph monitoring
24 hr impedance monitoring
Endoscopy to identify oesophagi’s / exclude other causes of vomiting

520
Q

Rx of GORD

A

Uncomplicated = reassure parents

Thickening agents for milk - nestargel / carobel
Position 30deg post feeds

More significant - h2 antagonists - ranitidine, PPI - omeprazole

No response - consider cows. milk protein allergy

Surgical management only if stricture / very severe + no response. Nissen fundiplication.

521
Q

Complications / prognosis GORD

A

Excellent prognosis

522
Q

Define Osgood-Schlatter’s disease

A

Osgood-Schlatter’s disease (OSD) is an overuse syndrome of the paediatric population that typically affects young athletes during their adolescent growth spurt. It presents with pain, tenderness to palpation, and swelling directly over the tibial tubercle. It is typically a self-limiting condition that resolves after a period of activity modification, with ultimate resolution occurring when patients reach skeletal maturity.

Overuse syndrome of the paediatric population that results in traction apophysitis of the tibial tubercle.

Typically occurs during an adolescent growth spurt in young athletes who participate in sports that involve repeated knee flexion and forced extension. Males are affected more often than females.

Diagnosis is clinical; patients typically present with pain, swelling, warmth, and localised tenderness to palpation over the tibial tubercle.

Conservative treatment is successful in most patients and consists of activity modification, ice, stretching, and strengthening exercises.

Plain x-rays of the knee are used to rule out other lesions of the proximal tibia if pain is unilateral and/or severe and persistent.

Up to 10% of patients may experience pain as adults due to formation of a separate ossicle at the tibial tubercle.

Rarely, when this condition fails to respond to non-surgical management, excision of the affected part of the tibial tubercle predictably relieves symptoms.

523
Q

Epidemiology of Osgood-Schlatter’s disease

A
21% athletic adolescents 
4.5% non-athletic
8-12yo for girls
12-15yo boys
M>F
Bilateral in 20-30%
524
Q

Aetiology of Osgood-Schlatter’s disease

A

Traumatic theory: micro-avulsion at the insertion of the patellar tendon (apophysis) caused by repeated traction injury.

Circulation theory: disturbance in the circulation of the apophysis due to rapid bone growth during adolescence, leading to the inability of the apophysis to withstand traction forces from the patellar tendon.

525
Q

RFs for Osgood-Schlatter’s disease

A

STRONG
Adolescent males
Athletic participation
Hx OSD in contralateral knee

WEAK
High position of patella (patella Alta)
More proximal/broader tibial attachment of patella tendon
Increased external tibial torsion

526
Q

Sx of Osgood-Schlatter’s disease

A
COMMON
RFs
Pain at tibial tubercle 
Worsened by extension
Localised tenderness

OTHER COMMON
Activity limitation
Localised warmth
Prominence of tibial tubercle (late disease)
Pain at tubercle with resisted knee extension

527
Q

Ix for Osgood-Schlatter’s disease

A

Plain X-ray - may be normal; often demonstrate enlarged tibial tubercle, sometimes with fragmentation of the apophysis - Should be ordered initially if symptoms are unilateral, severe, or persistent, and if there is any history of trauma.

US - pretibial swelling
MRI

528
Q

Rx of Osgood-Schlatter’s disease

A
Acute
Rest
NSAIDs - ibuprofen 400mg orally max 2400mg/day
Physio to strengthen quads 
Bracing and immobilisation

Severe
Surgery - excision of the ossicle and/or osteoplasty of the tibial tubercle, with excellent long-term outcomes

529
Q

Prognosis of Osgood-Schlatter’s disease

A

There is excellent prognosis in over 90% of cases with non-operative management.

530
Q

Complications of Osgood-Schlatter’s disease

A

Recurvatum (hyper-extension) knee deformity -> high riding patella -> early osteoarthritis

Persistence of bony deformity

531
Q

A 13-year-old male basketball player presents with several months of insidious onset of unilateral anterior knee pain, worse during practice and games and alleviated by rest, ice, and anti-inflammatory medicines. Physical examination demonstrates prominence of the tibial tubercle, with mild swelling and tenderness to palpation over the tubercle.

A

Osgood-Schlatter’s disease

532
Q

Define prematurity

A

A premature infant is an infant born before 37 completed weeks of gestation. Term infants are defined as those born between 37 and 41 weeks of gestation.

A premature infant is an infant born before 37 weeks’ gestation.

In addition to immediate post-birth resuscitation, efforts to reduce excessive oxygen exposure, hyperventilation, hypothermia, and hypoglycaemia must be made. Consultation with a neonatologist as soon as possible is recommended to provide expertise to reduce potential morbidity.

Gestational age determination done within hours of birth is important for designating further care of the premature newborn. The risk of adverse outcomes varies inversely with gestational age.

The New Ballard Score uses measurements of neuromuscular and physical maturity to complement the maternal history and ultrasonographic findings to estimate gestational age, although a first-trimester ultrasound remains the most accurate assessment of gestational age.

Extremely premature infants (gestational age <28 weeks) exhibit the greatest morbidity and mortality, although complications may occur with any degree of prematurity.

533
Q

Epidemiology of prematurity

A

5-9% of births in Europe

The risk for preterm birth is higher in black (16% to 18% of live births) than white (5% to 9% of live births) women, with a lower rate in East Asian and Hispanic women. The majority (60% to 70%) of preterm infants are born near term (34 to 36 weeks’ gestation), 20% are moderately premature (32 to 33 weeks), 15% are severely premature (28 to 31 weeks), and 5% are extremely premature (<28 weeks).

534
Q

Aetiology of prematurity

A

Modifiable factors include:

A short inter-pregnancy time interval: a 2-fold risk increase in preterm birth if there is <6 months between pregnancies. [1]
A maternal pregnancy BMI <19 or >35. [1] [3]
Psychological or social stress and maternal depression increase the risk of preterm birth 1- to 2-fold. [1]
Tobacco use during pregnancy increases the risk of preterm birth 1- to 2-fold. Cocaine and heroin use are also associated with preterm delivery. [1]
Intra-uterine infection may account for as much as 40% of preterm births. Atypical micro-organisms ( Mycoplasma and Ureaplasma ) have been implicated. [1]
Severe bacterial vaginosis is associated with a 1- to 3-fold increase in the rate of preterm birth. [1]
Poorly controlled maternal illness such as diabetes mellitus (or gestational diabetes) is associated with an increased likelihood of congenital anomalies that may lead to preterm delivery. Poorly controlled maternal hypertension may also lead to preterm delivery. [1]
Non-modifiable causative factors include:

Race: black women have a higher risk of preterm delivery compared with white women. Black women are 3 to 4 times more likely to have a very early preterm birth than other racial and ethnic groups. [1]
Previous preterm birth: results in a 2- to 3-fold increased risk of preterm delivery for future pregnancies. [1]
Chronic medical diseases such as diabetes, asthma, thyroid disease, hypertension. Maternal surgery in the second or third trimester can result in preterm labour. [1]
Maternal causes: pre-eclampsia/pregnancy-induced hypertension, antepartum haemorrhage, oligohydramnios or polyhydramnios, cervical or uterine abnormality. [1]
Fetal causes: multiple gestation, fetal growth restriction. [1]
Preterm premature rupture of membranes (PPROM) is associated with preterm birth. Concurrent intra-uterine infection frequently precipitates labour. [1]
Development of preterm spontaneous labour is a major cause of preterm births, due to the limited ability to stop labour once it is progressive. [1]
Between 30% and 35% of preterm births occur as a result of fetal or maternal conditions, 40% to 45% are due to spontaneous labour with intact membranes, and 25% to 30% are due to PPROM.

535
Q

RFs for prematurity

A

STRONG

Spontaneous preterm labour (most commonly intrauterine inflammation due to infection)
Intrauterine infection
PPROM 
Pre-eclampsia
Pregnancy induced HTN
Abruption/antepartum haemorrhage
Poly/oligohydramnios
Severe bacterial vaginosis
Multiple gestation 
Previous preterm birth
Fetal abnormality
Cervical incompetence 
Uterine abnormality 
Gestational DM
Maternal surgery during pregnancy 
Chronic maternal illness
Maternal pregnancy BMI <19 >35
WEAK
Short interpregancy interval 
Drug use
Stress/depression
Non-white
536
Q

Sx of prematurity

A
COMMON
RFs
SFH - if fibroids, malposition, maternal body habitus, multiple gestation, oligohydramnios or polyhydramnios, fetal growth restriction (FGR), or the presence of a small-for-gestational-age (SGA) infant,
Infant physical maturity score
Infant neuromuscular maturity score 
Infant birth weight
537
Q

Ix for prematurity

A

Fetal biometry during antenatal period - assessment of crown-rump length, fetal femur length, head and abdominal circumference, biparietal diameter.

538
Q

Rx of prematurity

A

<31w
Resus, PPV, CPAP, transfer to NICU
40% O2
Temperature maintenance
Umbilical vascular access
Dextrose 10%
Empirical ABx - Ampicillin + gent or amp + cefotaxime
Caffeine - treats recurrent apnoea (discontinued with no apneoa after 5-7 days)
Surfactant
Inotropes if hypotensive - dopamine/dobutamine (if fails - hydrocortisone)
Prostaglandins if congenital heart disease to maintain ductal patency

32-36
NG feed - Slow advancement of enteral feeding is therefore required. Breast milk and human milk-based fortifiers are recommended. Reduces NEC

539
Q

Prognosis of prematurity

A

Morbidity and mortality for all complications of prematurity are inversely proportional to gestational age. Although the overall incidence of premature birth is <15%, it accounts for close to 70% of infant mortality. Morbidity of prematurity is most pronounced in extremely premature infants.

Most infants require hospitalisation until 35 to 36 weeks’ post-menstrual age to reach appropriate milestones prior to discharge. These include maintenance of normothermia outside of an isolette, adequate oral intake, demonstration of adequate growth/weight gain, and absence of apnoea or bradycardia. Parents must demonstrate comfort in caring for their infant and addressing specific issues.

Extreme prematurity: gestational age <28 weeks
This group suffers the greatest mortality and complications of prematurity, which include bronchopulmonary dysplasia (BPD), white matter injury (WMI) and neurodevelopmental impairment (NDI), necrotising enterocolitis (NEC), intraventricular haemorrhage (IVH), patent ductus arteriosus (PDA), sepsis, pneumonia, retinopathy of prematurity (ROP), and behavioural/motor/cognitive problems such as ADHD, poor motor skills, and lower IQ scores.

Hospital courses are very prolonged, and parents often require significant support during periods of critical illness.

Severe prematurity: gestational age 28 to 31 weeks
Mortality is significantly lower than that seen in extremely premature infants. Overall morbidity is also less significant, although severe IVH, WMI and NDI, and behavioural/motor/cognitive problems occur. Sepsis, NEC, PDA, and ROP are also present in some infants. Long-term outcome is related to the extent of the pathology in each infant.

Moderate prematurity: gestational age 32 to 33 weeks
This group of infants is less often affected by respiratory distress syndrome (RDS), IVH, or ROP. However, problems of sepsis, PDA, and NDI may be present and should be managed as appropriate. Nutritional deficiency and failure to thrive should be managed with adequate caloric intake and follow-up of growth.

Late-preterm: gestational age 34 to 36 weeks
These infants rarely suffer the severe debilitating complications seen with prematurity (e.g., PDA, RDS, IVH, ROP, periventricular leukomalacia, NEC). However, although often treated the same as ‘term’ infants, they may experience significant difficulties with feeding and hospital re-admission for dehydration, jaundice, and sepsis. Long-term cognitive and behavioural deficits are comparable with term infants.

540
Q

Complications of prematurity

A
RDS
Sepsis
Anaemia of prem
Failure to thrive
Jaundice -  due to inadequate gastrointestinal losses of bilirubin, compounded by the slow advancement of enteral nutrition
Interventricular haemorrhage 
NEC
Patent ductus arterioles
Retinopathy of prematurity 
Re-hospitalisation
Cerebral palsy / neurodevelopmental delay 
ADHD
Bronchopulmonary dysplasia/chronic lung disease
Short stature
Death
Abuse/neglect
541
Q

A 19-year-old woman presents to the emergency department in active labour. She states she is 27 weeks pregnant and that her waters broke a few days ago. She is rushed to obstetrics for immediate examination. Fetal monitoring reveals severe prolonged heart rate decelerations and an emergent caesarean section is performed for fetal indications. The infant is born apnoeic, floppy, and bradycardic. There is no vernix present and there is sparse lanugo.

A

Premature birth

542
Q

A 22-year-old multiparous woman who has continued to smoke ‘several’ cigarettes per day during pregnancy presents to her obstetrician in premature labour and with premature rupture of membranes. The fetus is currently at 34 weeks’ gestation. On examination she is dilated to 7 cm, has contractions every 3 minutes, and the obstetrician confirms that delivery is imminent. The infant is delivered vaginally and appears well immediately after birth. Apgar scores are 9 at both 1 and 5 minutes of age.

A

Premature birth

543
Q

Define conjunctivitis

A

Conjunctivitis is the inflammation of the lining of the eyelids and eyeball caused by bacteria, viruses, allergic or immunological reactions, mechanical irritation, or medicines.

Symptoms include an irritated red eye with a watery or purulent discharge.

Allergic conjunctivitis is usually bilateral with watery discharge and itching.

Treatment for allergic conjunctivitis includes topical mast cell stabilisers and antihistamines; bacterial conjunctivitis treatment includes topical antibiotics; viral conjunctivitis requires symptomatic treatment.

Bacterial and viral conjunctivitis is highly contagious; measures to prevent spread of infection should be considered.

544
Q

Epidemiology of conjunctivitis

A

1% GP consultations

The incidence rate appears to be higher in children <1 year old (80 cases per 1000 patient-years) than in children >4 years of age (12 cases per 1000 patient-years).

Seasonal conjunctivitis represents more than 90% of cases of allergic conjunctivitis.

Bacterial conjunctivitis is more common in children, and viral conjunctivitis is more common in adults. Of infectious conjunctivitis, 42% to 80% is bacterial, 3% is chlamydial, and 13% to 70% is viral.

Viral conjunctivitis represents up to 50% of all acute conjunctivitis in primary care.

545
Q

Aetiology of conjunctivitis

A

The most common bacterial pathogens in infective conjunctivitis include Pneumococcus , Staphylococcus aureus , Moraxella catarrhalis , and Haemophilus influenzae.
N.gonorrhoea (hyper acute purulent)/chlamidya(persistent)

Viral conjunctivitis can be caused by adenovirus, herpes simplex, Epstein-Barr, varicella zoster, molluscum contagiosum, coxsackie, and enteroviruses.

 Allergic conjunctivitis
 Atopic keratoconjunctivitis
 Seasonal allergic conjunctivitis
 Vernal conjunctivitis
 Giant papillary conjunctivitis

Bacterial conjunctivitis
Gonococcal (hyperacute) bacterial conjunctivitis
Non-gonococcal bacterial conjunctivitis
Chlamydial conjunctivitis

Viral conjunctivitis
Adenoviral conjunctivitis
Herpetic conjunctivitis
Other viruses

Less common forms of conjunctivitis:

Mechanical/irritative/toxic conjunctivitis
Contact lens-related conjunctivitis
Toxic/chemical conjunctivitis
Medication-related conjunctivitis

Immune-mediated conjunctivitis
 Ocular cicatricial pemphigoid
 Graft-versus-host disease
 Stevens-Johnson syndrome
 Neoplastic
546
Q

RFs for conjunctivitis

A
STRONG
Exposure to infected person
Infection in one eye
Environmental irritants
Allergen exposure
Camps swimming pools, schools
Asian or mediterranean young male (allergic)
Atopy
Contact lense use
Ocular prosthesis [giant papillary conjunctivitis]
Mechanical irritation
WEAK
Asthma
Hayfever
Topical eye medicines
Anti-histamines (make eyes dry)
Sebacious gland carcinoma
547
Q

Sx of conjunctivitis

A
COMMON
RFs
Watery discharge - mainly viral 
Ropy mucoid discharge - allergic
Purulent discharge - bacterial
Itching predominant - allergic
Eyelids stuck in morning - viral/bacterial
Tender, pre-auricular lymphadenopathy - viral
OTHER COMMON
Conjunctival follicles - viral
Superficial punctate keratopathy - viral
Unilateral - bacterial 
Cornial sub epithelial infiltrates
Corneal pannus
548
Q

Ix for conjunctivitis

A

Rapid adenovirus assay
Cell culture - bacterial/viral
Gram stain - G-ve = N.gonn usually
PCR viral/bacterial DNA

Ocular PH for chemical splash - continue eye irrigation until pH 7 if abnormal

549
Q

Rx of conjunctivitis

A

ALLERGIC
Artificial tears - eg carmellose drops
Mast cell stabilisers eg sodium chromoglycate + antihistamine (topical or oral)
Topical NSAID if severe:
eg diclofenac ophthalmic: (0.1%) 1 drop into the affected eye(s) four times daily
Topical corticosteroid

BACTERIAL
Topical BS ABx

MILD = azithromycin ophthalmic: (1%) 1 drop into the affected eye(s) twice for one day, then once daily for 4 days
MOD-SEV = ciprofloxacin ophthalmic: (0.3%) 1 drop into the affected eye(s) four times daily
levofloxacin ophthalmic: (0.5%) 1 drop into the affected eye(s) four times daily

Chlamidya - azithromycin/doxy

VIRAL
Antihistamines/tears/topical corticosteroids
If confirmed adenovirus:
Topical ganciclovir

550
Q

Complications of conjunctivitis

A
Dry eyes
Keratitis
Subepithelial corneal infiltrates 
Lacrimal drainage problems
Symblepharon
551
Q

Prognosis of conjunctivitis

A

Allergic conjunctivitis
Most patients respond to treatment but may experience seasonal exacerbations.

Bacterial conjunctivitis
Generally self-limiting over 5 to 10 days; no significant long-term complications. Contact lens wearers and immunocompromised patients are at most risk of complications.

Viral conjunctivitis
Most cases are self-limiting; approximately 30% to 50% of patients with the form of adenovirus that causes epidemic keratoconjunctivitis may develop subepithelial infiltrates, which may lead to persistent visual loss and light sensitivity.

Non-infectious conjunctivitis
Most patients recover well after discontinuation of the cause.

552
Q

A 6-year-old girl with no significant past medical history presents 4 days after developing a red, irritated left eye. Her mother states that she has been wiping thick whitish-yellow discharge from her eye, and the eye is matted shut in the morning. She denies any exposure to an infected person, upper respiratory tract symptoms, or contact lens use. She also denies any significant pain or light sensitivity. On examination, the patient’s pupils are equal and reactive. She does not have a tender pre-auricular lymph node. Penlight examination does not reveal any corneal opacity, but thick, whitish discharge is seen.

A

conjunctivitis

553
Q

A 14-year-old boy with no significant past medical history presents 3 days after developing a red, irritated right eye that spread to the left eye today. He has watery discharge from both eyes and they are stuck shut in the morning. He reports recent upper respiratory tract symptoms and that several children at his day camp recently had pink eye. He denies significant pain or light sensitivity and does not wear contact lenses. On examination, his pupils are equal and reactive and he has a right-sided, tender pre-auricular lymph node. Penlight examination does not reveal any corneal opacity.

A

conjunctivitis

554
Q

Hyperacute presentation over 24 to 48 hours with copious whitish-yellow discharge in a sexually active person is consistent with Neisseria gonorrhoeae , unless shown otherwise. Allergic conjunctivitis is typically bilateral with itching and a ropy or watery discharge.

A

conjunctivitis

555
Q

Define retinoblastoma

A

The most common malignant intraocular tumour in children. It can be unilateral or bilateral, familial or spontaneous, and in 30% to 40% of cases is accompanied by a germinal mutation in the RB1 gene. It often causes an exudative retinal detachment with vitreous seeding.

Most common intraocular malignancy in children.

90% of all retinoblastoma cases are diagnosed by 3 years of age.

Disease can be unilateral or bilateral.

Most common presenting sign is leukocoria (white papillary reflex), which can often be seen in photographs.

In 30% to 40% of cases, the disease is associated with a germline mutation in the RB1 gene, which carries an associated increased risk of secondary non-ocular tumours.

10-year survival is 99% in resource-rich countries. Patients in resource-poor countries typically present with extraocular extension or metastatic disease with a dismal prognosis.

Treatment typically involves chemotherapy (systemic or intra-arterial) plus focal therapy with laser or cryotherapy. In advanced cases, enucleation may be required.

556
Q

Epidemiology of retinoblastoma

A

The incidence of retinoblastoma is 1 in 12,000 to 30,000 live births.

In the UK, between 50 and 60 children are newly diagnosed each year, and retinoblastoma accounts for only about 3 out of every 100 cancers occurring in children under the age of 15 years.

For many years, the reported median age at diagnosis has been 18 months, with the median age of diagnosis of bilateral cases occurring at 12 months and of unilateral cases at 24 months.

557
Q

Aetiology of retinoblastoma

A

Genetics

A mutation in both alleles of the RB1 gene is widely believed to be a prerequisite for the presence of disease. Other mutations are likely to be necessary for progression to clinical retinoblastoma. [10] One study showed that gene expression profiles may be dynamic, varying in a temporal and regionally (apical and basal tumour regions) dependent fashion. [11] However, only 10% of patients have a previously established family history of the disease. The majority of cases (even in the 25% to 30% of patients with the bilateral germinal form of the disease) have no antecedent family history and occur as a result of spontaneous mutations either early in embryogenesis or de novo in one set of parental germline cells. [12]

Viral exposure

One small clinical study performed in Mexico indicated that the presence of HPV sequences in retinoblastoma tumour tissue may play a role in the development of sporadic retinoblastoma. [13]

Advanced paternal age

There is evidence that mutations of RB1 are more common during spermatogenesis than oogenesis. [14] However, clinical studies examining the relative risk of patients with sporadic germinal disease with fathers older than 50 years have demonstrated mixed results.

558
Q

RFs for retinoblastoma

A

STRONG
Mutation in RB1 gene

WEAK
HPV exposure
Advanced paternal age

559
Q

Sx of retinoblastoma

A

COMMON
Age <3
Leukocoria (white pupillary reflex)
Strabismus

UNCOMMON
Fix
Pseudoorbital cellulitis
13q syndrome
Visual disturbances 
Ocular pain
Pinealoma
560
Q

Ix for retinoblastoma

A

Fundoscopy and examination under anaesthesia
Ophthalmic A- and B-scan ultrasound
MRI head/ orbit - pinealoma
BM aspiration+LP if suspect metastatic disease

561
Q

Rx of retinoblastoma

A

Enucleation (removal of eye)
Post-op chemo - carboplatin/vincristine (systemic/intraarterial)
Laser ablation
External beam therapy - Although external beam radiation was used frequently in the past, it is now reserved for cases that do not respond satisfactorily to chemotherapy/focal therapy, because it results in an increased risk of secondary cancers in this population.

562
Q

Prognosis of retinoblastoma

A

Patient survival is 99%, but there is obviously no ocular salvage

Highest ocular salvage = systemic chemo/focal therapy

563
Q

Complications of retinoblastoma

A

Intrarretinal haemorrhage / vascular occlusion post chemo
Cataract formation post beam radiation
Fibrosis of extra ocular muscles with carboplatin
Optic nerve atrophy with carboplatin
Secondary malignancy

564
Q

Define strabismus

A

Strabismus refers to a misalignment of the eyes. If strabismus develops in adults, it can cause diplopia (double vision) and visual confusion (seeing different objects in the same location), and it is an important cause of amblyopia in children. Whereas normally both eyes fixate (look at) the object of interest, in strabismus one eye fixates and the other (non-fixating eye) is deviated.

Misalignment of the visual axes of the eyes; may be latent or manifest and, if manifest, it may be constant or intermittent.

Common cause of diplopia (double vision) and visual confusion (seeing different objects in the same place) in adults.

Important cause for amblyopia (decreased vision in an anatomically normal eye caused by suppression) in children.

May be aesthetically obvious to the patient and others, resulting in psychosocial problems.

Evaluation involves a detailed medical history, including a complete ocular history, followed by thorough neurological and ophthalmic examinations.

Treatment is directed at restoring and maintaining ocular alignment, eliminating diplopia or visual confusion, enabling binocular vision, and restoring normal appearance.

If strabismus is secondary to an underlying cause (e.g., abducens nerve [cranial nerve VI] palsy causing esotropia), treatment of this condition is necessary.

565
Q

Epidemiology of strabismus

A

Prevalence of 1% of children aged <11 year

The prevalence of strabismus is higher in those with intellectual disabilities (44.1%) compared with the general population.

The prevalence in white people is 2% to 4%

566
Q

Risk factors for strabismus

A

STRONG
FHx
Prematurity

WEAK
Refractive error

567
Q

Aetiology of strabismus

A

The aetiology of most infantile and paediatric forms of strabismus is poorly understood. Comitant (concomitant) strabismus seems to be a complex genetic trait, with the involvement of more than one gene. Twin studies have revealed a concordance rate of 73% to 82% among monozygotic twins and 35% to 47% among dizygotic twins.

The rate for dizygotic twins is higher than the concordance for siblings, indicating that environmental factors are likely to be involved. No modifiable risk factors have been identified for most forms of comitant strabismus.

Strabismus is more common in children with global CNS problems, such as cerebral palsy or developmental delay.

568
Q

Sx of strabismus

A
Diplopia with both eyes open.
Eye misalignment 
Amblyopia (dec acuity in one eye) 
Abnormal eye movements
Visual confusion
Asthenopia (eye strain)

UNCOMMON
Intermittent closure of one eye
Cranial nerve palsy - Acquired paralytic strabismus is related to malfunction of one or more of the 3 cranial nerves (oculomotor, trochlear, abducens) providing motor supply to the extraocular muscles. This can be determined through a detailed examination of the cranial nerves.

Oculomotor nerve (cranial nerve III) palsy is characterised by exotropia, hypotropia, ptosis, and possible mydriasis (pupil dilation). Abducens nerve (cranial nerve VI) palsy is characterised by esotropia, and decreased abduction of the involved eye. Trochlear nerve (cranial nerve IV) palsy is characterised by hypertropia, elevation in adduction, and possible excyclotorsion of the involved eye.

569
Q

Ix for strabismus

A

Cover/uncover test - see if other eye will fixate
Alternate prism cover test (APCT)
Hirschburg test - asymmetrical reflexion of light
Krimsky test - uses prisms to determine the angle of misalignment

570
Q

Rx of strabismus

A
Treatment of refractive errors
Amblyopia Rx with patch/atropine
Extraocular muscle surgery 
Chemodenervation with botox
Overminus prescription
571
Q

Prognosis of strabismus

A

Infantile esotropia
The goal of treatment is to align the eyes. Usually, small-angle esotropia (up to 8 prism diopters) will result in stable alignment and allow rudimentary stereopsis. Elevation in adduction, latent nystagmus, and amblyopia occur in 30% to 50% of patients and may require further treatment. Although some studies suggest that aligning the eyes within the first 6 months of life results in better binocular function, this is not universally accepted.

572
Q

Complications of strabismus

A

Amblyopia
Decreased binocular vision
Psychosocial problems

573
Q

A 4-month-old healthy girl with normal antenatal and birth history is brought in by her parents, who note that both her eyes are looking toward the nose. Examination shows a large-angle esotropia with freely alternating fixation (i.e., each eye fixates objects, with no preference for either eye). No significant refractive error is present, and the remainder of the eye examination is normal. The infant is diagnosed with infantile esotropia.

A

strabismus

574
Q

A 34-year-old man complains of double vision after recovering from facial trauma that included a fracture of the left orbital floor. He describes the double vision as manifesting with one image on top of the other. The patient complains that he is unable to function unless he closes one eye. Examination shows hypotropia of the left eye, which becomes larger when the patient tries to look up. The remainder of the ocular examination is normal. A CT scan of the orbits demonstrates orbital tissue herniating through a fracture in the floor of the left orbit, and the patient is diagnosed with restrictive incomitant left hypotropia.

A

strabismus

575
Q

Define retinopathy of prematurity

A

Retinopathy of prematurity (ROP) is a serious vasoproliferative disorder that affects extremely premature infants. Retinopathy of prematurity often regresses or heals but can lead to severe visual impairment or blindness. Significant retinopathy of prematurity can lead to lifelong disabilities for the smallest survivors of neonatal ICUs (NICUs). It remains a serious problem despite striking advances in neonatology.

576
Q

Epidemiology of retinopathy of prematurity

A

50-70% of those <1250g

577
Q

Aetiology of retinopathy of prematurity

A

Retinal vasculature begins to develop around 16 weeks’ gestation. It grows circumferentially and becomes fully mature at term. Premature birth results in the cessation of normal retinal vascular maturation. Exposure of newborn premature infants to hyperoxia downregulates retinal vascular endothelial growth factor (VEGF). Blood vessels constrict and can become obliterated, resulting in delays of normal retinal vascular development. This hyperoxia-vasocessation is known as stage I of retinopathy of prematurity.

Early on, oxygen and nutrients can be delivered to the retina by means of diffusion from the underlying choroid capillary bed. The retina continues to grow in thickness and eventually outgrows its vascular supply. Over time, retinal hypoxia occurs and results in an overgrowth of vessels; this hypoxia-vasoproliferation is stage II of retinopathy of prematurity. See the image below.

578
Q

RFs for retinopathy of prematurity

A
LBW
Young gestational age
Severity of illness:
Sepsis
RDS
BPD
Prolonged exposure to supplemental O2
IVH
Patent ductus arteriosis
579
Q

Sx of retinopathy of prematurity

A

Opthalmologist diagnosis - screening should take place in those <1500-2000g of GA of <31w

Follow-up examinations are based on initial examination findings. Most infants are screened every 2 weeks.

580
Q

Ix for retinopathy of prematurity

A

Opthalmologist diagnosis - screening should take place in those <1500-2000g of GA of <31w

Follow-up examinations are based on initial examination findings. Most infants are screened every 2 weeks.

581
Q

Rx of retinopathy of prematurity

A

No standard medical therapies are available at this time.

SURGICAL
Ablative therapy currently consists of cryotherapy or laser surgery to destroy the avascular areas of the retina. The average gestational age (GA) at which surgery is necessary is usually 37-40 weeks.
If the retinopathy of prematurity continues to progress, more than one treatment may be required.
= CRYOTHERAPY OR LASER THERAPY
= LASER PREFERABLE

582
Q

Prognosis/Complications of retinopathy of prematurity

A
Severe visual impairment
Blindness
Myopia
Amblyopia
Strabismus

Late complications include myopia, amblyopia, strabismus, nystagmus, cataracts, retinal breaks, and retinal detachment.

Patients who did not progress beyond stage I or stage II have a good prognosis.
Patients with posterior zone I disease or stage III, IV, or V have a guarded prognosis for their vision.

583
Q

Define myopia

A

Near-sightedness, also known as short-sightedness and myopia, is an eye disorder where light focuses in front of, instead of on, the retina.

This causes distant objects to be blurry while close objects appear normal.

Other symptoms may include headaches and eye strain.

Severe near-sightedness increases the risk of retinal detachment, cataracts, and glaucoma.

584
Q

Aetiology of myopia

A

Combination of environmental and genetic factors.

The underlying mechanism involves the length of the eyeball growing too long or less commonly the lens being too strong. It is a type of refractive error.

585
Q

Epidemiology of myopia

A

22% world population
M=F
Uncorrected near-sightedness is one of the most common causes of vision impairment globally along with cataracts, macular degeneration, and vitamin A deficiency.[

586
Q

RFs for myopia

A

Focussing on near objects
Time spent indoors
Fix
High socioeconomic class

587
Q

Sx of myopia

A

Blurred vision
Straining
Headaches

588
Q

Ix for myopia

A

Eye examination

589
Q

Rx of myopia

A

Near-sightedness can be corrected with eyeglasses, contact lenses, or surgery.

590
Q

Complications of myopia

A

Severe near-sightedness increases the risk of retinal detachment, cataracts, and glaucoma.

591
Q

Define hypermetropia

A

Far-sightedness, also known as hyperopia, is a condition of the eye in which light is focused behind, instead of on, the retina.

This results in close objects appearing blurry, while far objects may appear normal.

As the condition worsens, objects at all distances may be blurry.

592
Q

Epidemiology of hypermetropia

A

Primarily affects young children
8% at 6yo
1% at 15yo
More common again after 40yo - effects 50%

593
Q

Aetiology of hypermetropia

A

The cause is an imperfection of the eyes. Often it occurs when the eyeball is too short, or the lens or cornea is misshapen.

Low converging power of eye lens because of weak action of ciliary muscles
Abnormal shape of the cornea
Far-sightedness is often present from birth, but children have a very flexible eye lens, which helps to compensate

594
Q

RFs for hypermetropia

A

FHx
Diabetes
Medications -
Tumours around the eye

595
Q

Sx of hypermetropia

A

Blurred vision with close objects
Preserved vision with far away objects

Headache
Eye strain

Accommodative dysfunction (difficulty with depth perception), amblyopia, binocular dysfunction (difficulty seeing with both eyes) and strabismus

596
Q

Ix for hypermetropia

A

A diagnosis of far-sightedness is made by utilizing either a retinoscope or an automated refractor-objective refraction; or trial lenses in a trial frame or a phoropter to obtain a subjective examination.

Slit lamp - examine the cornea/conjunctiva/anterior chamber/iris

597
Q

Rx of hypermetropia

A

Glasses
Contacts
Surgery:
Photorefractive keratectomy (PRK)
Removal of a minimal amount of the corneal surface
Laser assisted in situ keratomileusis (LASIK)
Laser eye surgery to reshape the cornea, so that glasses or contact lenses are no longer needed.
Refractive lens exchange (RLE)
A variation of cataract surgery where the natural crystalline lens is replaced with an artificial intraocular lens; the difference is the existence of abnormal ocular anatomy which causes a high refractive error.
Laser epithelial keratomileusis (LASEK)
Resembles PRK, but uses alcohol to loosen the corneal surface.

598
Q

Complications/prognosis of hypermetropia

A

Strabismus

Amblyopia

599
Q

Define lactose intolerance / lactase deficiency

A

Lactose intolerance refers to the development of gastrointestinal and/or systemic symptoms secondary to malabsorption of lactose caused by hypolactasia. It is characterised by reduced lactase (lactase-phlorizin hydrolase enzyme) concentration in the mucosal brush border (microvilli) of the small intestine, usually at the age of weaning.

Malabsorption of lactose, however, does not always manifest as gastrointestinal symptoms; development of symptoms and symptom severity depend on the amount and rate of lactose reaching the colon, as well as the type and amount of colonic flora.

Lactose intolerance is characterised by reduced lactase concentration in the mucosal brush border of the small intestine (also known as hypolactasia).

It exists in four distinct forms: primary, secondary, congenital, and developmental.

Symptoms can be gastrointestinal and/or systemic.

Dietary elimination and challenge are generally diagnostic.

Treatment includes reduction or elimination of dietary lactose.

If elimination of lactose is necessary, alternative calcium sources are recommended.

600
Q

Epidemiology of lactose intolerance / lactase deficiency

A

Primary thought to be >70% of world population (many asymptomatic)
Congenital = EXTREMELY rare - only 40 conformed cases

Primary more common in African American/American Indian/hispanic/latino

The age-related decline in lactase expression is usually complete during childhood

Onset is typically subtle and progressive in primary disease, and most people first experience intolerance symptoms in late adolescence and adulthood.

Secondary disease is more common in children, especially in developing countries where infections are the common cause

601
Q

Aetiology of lactose intolerance / lactase deficiency

A

In all mammals, lactase concentrations are at their highest shortly after birth and generally decline rapidly after the usual age of weaning.

Primary - due to polymorphisms

Secondary (acquired) - Results from loss of lactase activity in people with lactase persistence due to gastrointestinal illnesses that damage the brush border of the small intestine (e.g., viral gastroenteritis, giardiasis, coeliac sprue).
Endoplasmic reticulum stress (accumulation of misfolded proteins and alterations in calcium homeostasis in the endoplasmic reticulum leading to cell death) is one of the possible explanations of why loss of lactase persists after gut infections such as rotavirus.

Developmental hypolactasia

Related to suboptimal levels of lactase in the apical microvilli of the brush border surface of the small intestine in premature infants (as the mucosal structure has still not fully developed).

The combined increase in osmolality of the faecal water, accelerated intestinal transit, and generated hydrogen and methane accounts for the wide range of gastrointestinal symptoms (e.g., abdominal pain, distension, diarrhoea, flatulence). Other factors related to the severity of symptoms include the rate of gastric emptying, fat content of the food in which the sugar is ingested, the individual sensitivity to intestinal distension produced by the osmotic load of unhydrolysed lactose in the small bowel, and the response of the colon to the carbohydrate load.

602
Q

RFs of lactose intolerance / lactase deficiency

A
Black/native American/asian/hispanic
Adolescence and early adulthood
Fix lactase deficiency 
Enteritis/gastro-enteritis
Comorbid pre-disposing disease - HIV enteropathy, sprue (coeliac or tropical), Whipple's disease, carcinoid syndrome, cystic fibrosis, diabetic gastropathy, kwashiorkor, Zollinger-Ellison syndrome, chemotherapy treatment, colchicine use (for familial Mediterranean fever), and radiation enteritis can all cause secondary disease.
Hx IBS diagnosis
603
Q

Sx of lactose intolerance / lactase deficiency

A
RFs
Sx after ingestion of dairy - can be minutes - hours - >12hrs
Diarrhoea
Distension
Lethargy 
Mouth ulceration
Headache
Poor short term memory/concentration
Eczema 
Failure to thrive
OTHER COMMON
Abdominal pain, cramping, peri-umbilical
Stomach rumbles
Flatulence
Palipitations 
Uncommon
N+V
Constipation
Pallor 
WL
Short stature
604
Q

Ix for lactose intolerance / lactase deficiency

A

Trial of dietary lactose elimination
FBC

CONSIDER
Lactose hydrogen breath test if inconclusive 
Stool culture - N
Faecal pH - reduced
Fecal reducing substances - +ve
605
Q

Rx of lactose intolerance / lactase deficiency

A

Diet mod
Some dairy products are lower in lactose: hard cheeses, live-culture yoghurts, live-culture curds, live-culture cheeses, and milk-cereal mixtures.

Dietician consultation
Lactase Rx dairy products
Oral lactase supplementation
Calcium
Ergocalciferol
606
Q

Complications of lactose intolerance / lactase deficiency

A

Osteoporosis
Rickets
Malnutrition
Growth failure

607
Q

Prognosis lactose intolerance / lactase deficiency

A

Prognosis for patients with lactase deficiency with a clinical syndrome of lactose intolerance is excellent with lactose reduction. Persistent symptoms in a small number of patients are either due to ongoing inadvertent lactose exposure (especially from hidden lactose), irritable bowel syndrome, or poorly controlled underlying disorders of secondary lactase deficiency. Treatment is lifelong, and adherence may be a problem, especially in children.

608
Q

A 25-year-old woman presents with a 10-year history of intermittent diarrhoea, bloating, abdominal pain, and flatulence. Recently her symptoms have worsened. She thinks the change in symptoms may be related to her increased intake of milk over the last few months. Previously, she consumed moderate amounts of dairy produce, but little milk. She has a past history of eczema and asthma (takes salbutamol inhalers as needed) and was diagnosed with irritable bowel syndrome (IBS) by her family physician several years ago. Family history reveals that her only sister has been diagnosed with IBS and lactose intolerance. Abdominal examination reveals a slightly distended abdomen. Results of faecal testing for occult blood are negative.

A

Lactose intolerance

609
Q

Define coeliacs

A

Coeliac disease is a systemic autoimmune disease triggered by dietary gluten peptides found in wheat, rye, barley, and related grains. Immune activation in the small intestine leads to villous atrophy, hypertrophy of the intestinal crypts, and increased numbers of lymphocytes in the epithelium and lamina propria. Locally these changes lead to gastrointestinal symptoms and malabsorption. Systemic manifestations are diverse, potentially affecting almost every organ system.

Coeliac disease is common, affecting up to 1% of the general population, and may present at any age.

Presentation is varied and ranges from diarrhoea and failure to thrive, to iron-deficiency anaemia or osteoporosis.

Diagnosis is suggested by positive immunoglobulin A tissue transglutaminase serology, but must be confirmed by duodenal biopsy and histology.

The only current therapy is a strict, lifelong gluten-free diet.

Complications of untreated coeliac disease include gastrointestinal symptoms, malabsorption, increased risk of malignancy, and higher overall mortality than in the general population.

610
Q

Epidemiology of coeliacs

A

1% prevalence
W>M
Common age diagnosis = 40 BUT can be diagnosed t any age
20% of patients = “silent sufferers”

1% of coeliacs = “refractory” and untreatable by a gluten free diet

611
Q

Aetiology of coeliacs

A

Almost all carry HLADQ2/DQ8
- genetic PREDISPOSITION
IE most people who carry Dq2/8 do not get coeliacs

Factors that have been hypothesised to play a role include: the timing of initial gluten exposure; gastrointestinal infection leading to gluten antigen mimicry; or direct damage to the intestinal-epithelial barrier leading to abnormal exposure of the mucosa to gluten peptides. Reovirus infection has also been shown to promote inflammatory immunity and a decrease in oral tolerance to gluten.

612
Q

RFs for coeliacs

A
STRONG
FHx
IgA deficiency 
T1DM
AI/thyroid
WEAK
Down's syndrome 
Sjorgren's syndrome 
IBD
PBC
613
Q

Sx of coeliacs

A
COMMON
IgA deficiency
Diarrhoea
Bloating 
Abdo pain/discomfort
Anaemia (ID)
Dermatitis herpetiformis 
OTHER
Osteopenia
Fatigue
WL
Failure to thrive 
OTHER UNCOMMON
Apthous stomatitis 
Easy bruising
Ataxia
Peripheral neuropathy (deficiencies of B12, E, or D; folate or pyridoxine)
614
Q

Ix for coeliacs

A

FBC + smear - check for ID anaemia
Anti TTG/endomysial/gliadin
If dermatitis herpetiformis - skin biopsy
Gluten challenge
Biopsy - presence of intra-epithelial lymphocytes, villous atrophy, and crypt hyperplasia
Small bowel endoscopy - atrophy and scalloping of mucosal folds; nodularity and mosaic pattern of mucosa

615
Q

Rx for coeliacs

A

Gluten free diet
Calcium, vit D, iron supplementation
Dietician

Coeliac crisis:
Coeliac crisis is rare and presents with hypovolaemia, severe watery diarrhoea, acidosis, hypocalcaemia, and hypoalbuminaemia. Patients are often emaciated and have nutritional deficiencies caused by long-standing, untreated coeliac disease.
- Give corticosteroids orally

616
Q

Complications of coeliacs

A
Osteoporosis
Dermatitis herpetiformis
Malignancy - including intestinal and extra-intestinal lymphoma and carcinomas of the upper digestive tract.
Idiopathic pancreatitis
Pneumococcal infection
617
Q

Prognosis of coeliacs

A

Most, up to 90% in some studies, will have complete and lasting resolution of symptoms on a gluten-free diet alone. For the 10% with persistent symptoms, most of these will be attributed to ongoing gluten exposure, lactose intolerance, and irritable bowel syndrome. Less than 1% can be expected to develop refractory coeliac disease.

618
Q

A 46-year-old woman presents with fatigue and is found to have iron deficiency with anaemia. She has experienced intermittent episodes of mild diarrhoea for many years, previously diagnosed as irritable bowel syndrome and lactose intolerance. She has no current significant gastrointestinal symptoms such as diarrhoea, bloating, or abdominal pain. Examination reveals two oral aphthous ulcers and pallor. Abdominal examination is normal and results of faecal testing for occult blood are negative.

A

coeliacs

619
Q

A 9-year-old boy presents with vomiting for 5 days. His sister, who has coeliac disease, has had similar symptoms. His growth has been normal and he has not experienced any other possible symptoms of coeliac disease, except for intermittent constipation. Immunoglobulin A-tissue transglutaminase titre is 5 times the upper limit of normal.

A

coeliacs

620
Q

Atypical presentations include an asymptomatic patient, elevated liver enzymes, vitamin D deficiency, osteopenia or osteoporosis, constipation, aphthous stomatitis, nausea or vomiting, heartburn or gastro-oesophageal reflux disease, hyposplenia or asplenia, myalgias, arthralgias, peripheral neuropathy, alopecia, headaches, infertility, and adverse pregnancy outcomes.

A

coeliacs

621
Q

Define ASD

A

An atrial septal defect (ASD) is an opening in the atrial septum, excluding a patent foramen ovale.

There are 4 types of ASD: ostium secundum, ostium primum, sinus venosus, and unroofed coronary sinus.

There are 4 types of atrial septal defect (ASD): ostium secundum, ostium primum, sinus venosus, and unroofed coronary sinus. Secundum defects are the most common.

Most patients are asymptomatic. Untreated defects can produce right atrial enlargement, cardiac arrhythmias, and heart failure over time.

Secundum, primum, and coronary sinus defects with small shunts (ratio of pulmonary flow to systemic flow, Qp:Qs, <1.5) do not require treatment. Corrective closure is required if the shunt is larger (Qp:Qs ratio ≥1.5), there is right atrial enlargement, or the patient has a sinus venosus defect.

Corrective closure, if required, is usually performed at 2 to 4 years of age but can be performed at a younger age in symptomatic patients.

Percutaneous device closure is the preferred treatment for secundum defects, and surgical closure is reserved for larger secundum defects, technically challenging cases, and other defects.

If right-to-left shunting (Eisenmenger’s syndrome) occurs, the ASD is operable if the shunt is reversible with pulmonary vasodilators. If the shunt is irreversible, the treatment is largely supportive.

622
Q

Epidemiology of ASD

A

ASDs occur in 1 of 1500 live births. Secundum-type ASDs constitute 6% to 10% of congenital heart defects

M:F 1:2

Secundum defects account for 75% of ASDs. The other types of ASDs occur far less frequently: ostium primum (15% of ASDs), sinus venosus (5% of ASDs), and unroofed coronary sinus defects (1% of ASDs). ASDs can also occur in conjunction with more significant congenital heart defects.

623
Q

Aetiology of ASD

A

Most cases of ASDs occur sporadically with no family history of congenital heart defects. Girls are more likely than boys to develop ASDs. Maternal alcohol consumption increases the risk of developing a range of congenital heart conditions, including ASDs. Some causative mutations have been identified. An association between ASDs and upper extremity anomalies was first reported in 1960. This particular association has been linked to mutations of TBX5. Rare cases of familial ASDs have been identified, caused by mutations in the transcription factors NKX2.5 and GATA4, as well as the structural protein MYH6.

Inter-atrial blood flow is maintained throughout cardiac embryogenesis while two different septal structures form. The first septum to develop is septum primum. Programmed cell death in the anterosuperior aspect of septum primum results in an ostium secundum. Septum secundum then develops to the right of septum primum as an infolding of the right atrial wall and ultimately forms the limbus of the fossa ovalis, while septum primum functions as the valve of the fossa ovalis. [2] An abnormality during the development of these structures can result in an ASD.

Ostium secundum-type ASDs develop due to a lack of development of septum secundum. Ostium primum-type ASDs develop due to lack of closure of ostium primum by the endocardial cushions. Sinus venosus-type ASDs are thought to occur because of resorption of the wall between the superior vena cava and pulmonary veins, which also explains why this type of ASD is associated with anomalous drainage of the right upper pulmonary vein. [1] Occasionally, sinus venosus ASDs can develop between the inferior vena cava and the right atrium.

The direction of blood flow through the defect is related to the relative compliances of the two ventricles. In infancy, the right ventricle is relatively thick and non-compliant; therefore, the amount of left-to-right shunt is minimal. However, as the pulmonary vascular resistance decreases, the right ventricle becomes more compliant, increasing the left-to-right shunt. With age, the left-to-right shunt is exacerbated by increasing stiffness of the left ventricle associated with ageing and systemic hypertension.

The right atrium, right ventricle, and pulmonary arteries may enlarge due to the increased volume load. The pulmonary and tricuspid valves may become incompetent due to enlargement of the valves’ annuli. Despite the increased volume load, the pulmonary artery pressures are usually only slightly increased. However, the chronic volume overload can cause engorgement of the pulmonary arteries, capillaries, and veins. Some patients develop medial hypertrophy of the pulmonary arteries and pulmonary vascular obstructive disease. Patients become cyanotic if the shunt reverses.

624
Q

RFs for ASD

A

STRONG
Female
Maternal alcohol consumption

WEAK
FHx

625
Q

Sx of ASD

A

COMMON
Ejection systolic murmur - Best heard at the left upper sternal border. The murmur often radiates to the back.

Produced by excess volume of blood crossing the pulmonary valve.

Fixed splitting of second heart sound - Second heart sound does not become single with expiration.

OTHER UNCOMMON 
Mid diastolic murmur 
Congestive cardiac failure - Rare presentation that usually does not occur until after the fifth decade of life.
Failure to thrive
Sx of atrial arrhythmias
Cyanosis
Finger clubbing
626
Q

Ix for ASD

A

Echocardiogram - visualisation of the defect; determination of flow pattern and shunt volume

ECG - normal; or tall p waves, large R waves in V1, crochetage pattern in inferior limb leads

CxR - normal or enlarged heart with increased pulmonary markings

CONSIDER
Chest CT/MRI
Cardiac catheterisation to determine pulmonary vascular resistance

627
Q

Rx of ASD

A

LEFT TO RIGHT SHUNT

  • OBSERVE
  • Corrective closure - If the ratio of pulmonary to systemic blood flow, Qp:Qs, is or remains ≥1.5, or there is evidence of right atrial enlargement, the defect requires closure to prevent heart failure, atrial arrhythmias, and pulmonary vascular obstructive disease
  • Prophylactic ABx 6months - amoxicillin to prevent endocarditis
RIGHT TO LEFT SHUNT
Reversible
- Corrective closure + prophylactic ABx
Irreversible (eisenmengers)
- Bosentan/sildenafil/epoprostol
- Phlebotomy for hyper viscosity (they increase RBC production to compensate for hypoxaemia)
\+ HEART LUNG TRANSPLANT FOR EISENMENGERS
628
Q

Complications of ASD

A
Device erosion
Pericaridal effusion/tamponade
Paradoxical embolisation (usually adults with undiagnosed -> whereby an embolus causes a TIA or stroke)
Congestive HF
Infective endocarditis
AFibrillation
629
Q

Prognosis of ASD

A

The prognosis of ASDs without pulmonary vascular obstructive disease is excellent. The risk of death after surgical closure of an uncomplicated ASD is <1%. Long-term survival for patients aged <24 years at surgery is similar to that of age-matched controls. Patients who have surgery at age >24 years have poorer survival than age-matched controls.

Five- to 10-year follow-up after device closure is excellent, but long-term follow-up data are not available for device closure. The incidence of device erosion is 0.1% in the US, with most erosions occurring within 72 hours after placement.

Shunt reversal with inoperable Eisenmenger’s physiology is associated with a poor prognosis.

630
Q

Definition AVSD (complete)

A

Atrioventricular septal defects (AVSDs) are anatomic defects that arise from faulty development of the embryonic endocardial cushions. This spectrum ranges from a primum atrial septal defect and cleft mitral valve, known as a partial atrioventricular septal defect (partial AVSD), to defects of both the primum atrial septum and inlet ventricular septum and the presence of a common atrioventricular valve, referred to as complete atrioventricular septal defect (complete AVSD, CAVSD). The terms atrioventricular canal defect and endocardial cushion defect are used in reference to this group of defects; however, atrioventricular septal defect is now the preferred terminology. These defects, particularly the complete form, typically present in the fetal or neonatal period and are an important source of cardiac morbidity and mortality in this age group.

631
Q

Epidemiology of AVSD (complete)

A

2-9% of congenital heart disease
M=F
Seen more frequently in stillborns
Roughly 1 in 1000 of Down syndrome babies

632
Q

Aetiology of AVSD (complete)

A

Most frequent genetic abnormality associated = trisomy 21

Can occur with trisomy 13 + 18

Can occur with heterodoxy syndromes

Can rarely occur with digeorge

633
Q

RFs for AVSD (complete)

A

Advanced maternal age

634
Q

Sx of AVSD (complete)

A
Down's syndrome Sx
Failure to thrive
Tachypnoea
Harrison grooves - horizontal depression along lower border of chest at diaphragm insertion site due to chronic tachypnea
Prominent systolic heave 

The first heart sound is single and often accentuated. The second heart sound is narrowly split, with an accentuated pulmonary component. A crescendo-decrescendo murmur may be audible at the upper left sternal border due to increased blood flow through a normal pulmonary valve. A mid diastolic rumble may be audible at the lower left sternal border and apex due to the increased flow across the common atrioventricular valve. A holosystolic murmur is often appreciated at the apex due to atrioventricular valve insufficiency. Because the VSD in complete atrioventricular septal defect is large and unrestrictive, it is not associated with a murmur.

635
Q

Ix for AVSD (complete)

A

No specific lab tests
ECG - sinus rhythm
May be prolonged PR (atrial enlargement), RVH, possible LVH,

Chest radiography shows enlargement of the cardiac silhouette. Enlargement of the right atrium and right ventricle is most apparent. Evaluation of the left ventricle may be difficult because it is often displaced by the enlarged right ventricle. The main pulmonary artery segments are prominent, as well as the overall pulmonary vascular markings. In the setting of pulmonary vascular disease, the distal pulmonary vessels may have a lucent, pruned appearance.
Echocardiography reveals defects of the atrial and ventricular septae.
The subcostal 4-chamber and long axial oblique (modified left oblique) views reveal many important aspects of complete atrioventricular septal defect, including the size of the atrial and ventricular defects, the nature of the atrioventricular valve attachments, the distribution of atrioventricular valve tissue, and the left ventricular (LV) outflow tract (LVOT).

636
Q

Rx of AVSD (complete)

A

Diuretics, digoxin, ACE inhibitors -> used to alleviate tachypnoea and failure to thrive

Surgical
Single stage complete repair referred

637
Q

Prognosis/complications of AVSD (complete)

A

Patients with complete atrioventricular septal defect typically develop tachypnea and failure to thrive in the first few months of life. Tachypnea hampers normal feeding. In addition, respiratory tract infections, such as those due to respiratory syncytial virus (RSV), are poorly tolerated.

Without operation survival is poor.
Death due to HF or pneumonia
Childhood death from progressive pulmonary vascular obstructive diseases
Intimal fibrosis

Risk factors for surgical and late mortality and morbidity are identified. Preoperative risk factors for mortality include small size, unbalanced ventricular size, New York Heart Association class IV, and severe atrioventricular valve insufficiency.

Published 10y survival = 80-90%

Postoperative complications include arrhythmias, low cardiac output, pulmonary hypertension, atrioventricular valve stenosis, and mitral insufficiency.

638
Q

Define coarctation of the aorta

A

Coarctation of the aorta is defined as a narrowing in the aorta, most commonly at the site of insertion of the ductus arteriosus, just distal to the left subclavian artery. Less commonly, there may be diffuse arch hypoplasia with a long segment of narrowing, proximal to the left subclavian artery, or the obstruction may be in the abdominal aorta. It is often associated with a diffuse arteriopathy and bicuspid aortic valve. It typically presents with upper extremity systolic hypertension or murmur. If the narrowing is severe, it may present in the newborn period once the ductus arteriosus closes as low cardiac output and shock (critical coarctation).

Characterised by blood pressure differential between upper and lower extremities (upper >lower).

Physical examination may show diminished, absent, or delayed lower extremity pulses compared with upper extremity pulses.

Diagnosis is made by demonstration of aortic arch narrowing, typically by echocardiography.

Treatment may be surgical repair or percutaneous balloon dilation or stent placement.

Long-term complications include persistent hypertension or aneurysm formation at site of surgery or balloon.

639
Q

Epidemiology of coarctation of the aorta

A

6-8% of congenital heart defects
F>M
Sporadic cases mainly
Often seen with bicuspid aortic valve or VSD

There are also syndromic associations including Shone’s complex, Turner’s syndrome, DiGeorge syndrome, and hypoplastic left heart syndrome.

640
Q

Aetiology of coarctation of the aorta

A

The aetiology of congenital coarctation is incompletely understood, but 2 primary theories explain the narrowed segment:

The narrowed segment is under-developed during fetal life due to reduced blood flow across the developing arch. This is particularly thought to be responsible when associated with additional left-sided lesions such as hypoplastic left heart syndrome.

Ductal tissue extends into the thoracic aorta, and, when the ductus arteriosus constricts and closes postnatally, the thoracic aorta is constricted.

Much less commonly, aortic coarctation can be acquired. Takayasu arteritis, a large vessel vasculitis, can result in aortic coarctation. In these cases, the narrowed segment is often in the descending thoracic or abdominal aorta.

Severe narrowing of the aortic arch that presents in the neonatal period with low output cardiac failure and shock once the ductus arteriosus closes is termed critical coarctation. These infants require a prostaglandin E1 infusion to maintain a patent ductus arteriosus and surgical repair after medical stabilisation.

641
Q

RFs for coarctation of the aorta

A
Male
Young (<10)
Turners
DiGeorge 
Hypoplastic left heartsyndrome 
Stones complex
PHACE syndrome

WEAK
FHx

642
Q

Sx of coarctation of the aorta

A

COMMON
HTN at young age
Differential upper + lower extremity blood pressure
Diminished lower extremity pulses

OTHER COMMON
Ejection systolic murmur - (left sternal border and back)
Male predominance
Claudication
Headache (ICH/berry aneurysm association)
Systolic ejection click (associated bicuspid valve)

643
Q

Ix for coarctation of the aorta

A

1ST
ECG - may be normal; show RVH or LVH
CxR - age and severity dependent; may be normal, have cardiomegaly, or show posterior rib notching
Echo - discrete narrowing in the thoracic aorta; pressure gradient across narrowing

ADJUNCT/CONSIDER
CT angiography
MRI angiography
Cardiac catheterisation - abnormal gradient across narrowing; therapeutic intervention possible

644
Q

Rx of coarctation of the aorta

A

CRITICAL COARCTATION

  • MAINTAIN ductal patency by giving prostaglandin E1 - alprostadil
  • Surgical repair

<1yo
Surgical repair differs by case.
May be end-end anastamosis
Arch reconstruction

Percutaneous repair is generally not recommended for children <1 year old, as recurrence rates are high. Additionally, stent placement is limited both by patient size and by the rapid growth seen during infancy.

> 1yo
Percutaneous repair - potentially balloon angioplasty/stent insertion

645
Q

Complications of coarctation of the aorta

A

CAD
Systemic HTN
Recoarctation after repair
Aortic aneurysm

646
Q

Prognosis of coarctation of the aorta

A

The overall prognosis for patients with isolated coarctation is excellent. Patients may require long-term treatment for hypertension, even in the absence of residual coarctation. Anti-hypertensives, including ACE-inhibitors, beta blockers and angiotensin II receptor blockade, have been utilised.

Patients in whom the coarctation is associated with additional intra-cardiac defects typically are at higher risk of long-term problems related to the additional defects.

Recoarctation may be seen in both surgically and transcatheter-treated patients; therefore, ongoing lifetime outpatient follow-up with cardiology is necessary.

As patients age, careful follow-up with a cardiologist is imperative, as patients with aortic coarctation have a higher incidence of coronary artery disease.

647
Q

A 4-year-old boy presents to his paediatrician for a well-child visit. His mother reports him to be doing well and has no concerns. On examination, he is noted to have a right upper extremity blood pressure of 140/70 mmHg. His cardiovascular examination shows a quiet precordium, a normal point of maximal impact, normal S1, and normally split S2. A 2/6 long systolic murmur is heard over his back, and 2+ radial pulses and trace femoral pulses are felt.

A

Aortic coarctation

648
Q

A newborn infant is noted to have respiratory distress 2 days after birth. On examination she is mottled and has weak upper extremity pulses with no palpable femoral pulses. Her arterial blood gas shows a profound metabolic acidosis.

A

Aortic coarctation

649
Q

Rarely, an adult presents with hypertension and severe headache due to rupture of a berry aneurysm in the circle of Willis, which is thought to be associated with aortic coarctation in 10% of patients.

A

Aortic coarctation

650
Q

Define patent ductus arteriosus

A

Patent ductus arteriosus (PDA) describes the persistence of a fetal structure, known as the ductus arteriosus, after birth. This vascular structure, which connects the main pulmonary artery to the aorta, allows blood to bypass the lungs in utero. In term infants it functionally closes, usually in the first 48 hours of life.

A ductus arteriosus is a vascular fetal structure that usually closes in the first 48 hours after birth.

Persistence of the ductus arteriosus can result in heart failure, increased pulmonary pressures, and endarteritis.

The incidence and sequelae of a patent ductus arteriosus (PDA) are more significant in premature infants than infants born at full-term.

Clinical history and presentation can vary significantly depending on age of the child and the size of the ductus. Patients may be entirely asymptomatic or have signs and symptoms of heart failure and haemodynamic instability.

Treatment options vary depending on the age of the patient and the size of the ductus. Practice may vary significantly between institutions.

651
Q

Epidemiology of patent ductus arteriosus

A

Preterm > term
1-2 per 1000 term births (NB clinically silent PDA)
F>M
High altitude > low altitude

652
Q

Aetiology of patent ductus arteriosus

A

In the fetus, the patency of the duct is maintained by a low-oxygen environment and circulating prostaglandins produced by the placenta (PGE2 and PG12).

After birth, oxygen saturation increases and contributes to the constriction of the duct through potassium-mediated channels.

Post-natally, the level of circulating prostaglandins falls as the placenta is removed from the circulation. In addition, pulmonary blood flow increases resulting in increased metabolism of prostaglandins. There may be several other unknown factors that contribute to ductal closure.

Haemodynamic constriction is later followed by complete structural closure mediated by histological changes that obliterate the ductal lumen.

While the process of ductal closure is fairly well understood, the factors that result in persistence of the ductus are less clear. In preterm infants, there may be a decreased response to oxygen and a continued sensitivity to prostaglandins when compared to term infants.

653
Q

RFs for patent ductus arteriosus

A

STRONG
Prematurity
Maternal rubella
Female

WEAK
RDS
High altitude 
Fix
Black
654
Q

Sx of patent ductus arteriosus

A
COMMON
RFs
Infancy presentation 
Tachypnoea / SOB
Failure to thrive 
Exercise intolerance
Widened pulse pressure

Gibson murmur or machinery murmur, is best heard in the left infraclavicular area and peaks in late systole and continues through into diastole.

Apnoea
Low diastolic BP
Irritability
Diaphoresis (sweating)
Inc freq URTI
Murmur heard only during systole
Hyperdynamic precordium
Systolic thrill
3rd HS at apex
Mid-diastolic rumble heard at apex
Bounding peripheral pulses

UNCOMMON
Pulmonary rales

655
Q

Ix for patent ductus arteriosus

A

CXR - cardiomegaly, increased pulmonary markings
ECG - deep Q waves and tall R waves in leads II, III, Avf, V5, and V6. Sometimes widened P waves
Echocardiogram - GOLD standard. Two-dimensional and/or colour Doppler evidence of a PDA. Diastolic forward flow in the pulmonary artery. Left ventricular and/or left atrial enlargement. Diastolic flow reversal in the distal aortic arch

CONSIDER
Cardiac catherisation - not usually performed. Presence of shunt and any co-existing cardiac abnormalities; pulmonary HTN.

656
Q

Rx of patent ductus arteriosus

A

Premature LBW infants are treated prophylactically - Indometacin 0.1mg/kg IV 3 day

Premature infants <32w
Indometacin
OR Ibuprofen - inhibit prostaglandins (thought to be more effective)

Term infants + children (small-mod ducts)
Percutaneous catheter closure (usually >6months old, if under then wait as long as possible with furosemide Rx)

Large ducts:
Surgical ligation

Adults:
Percutaneous catheter device closure

657
Q

Prognosis of patent ductus arteriosus

A

Premature infants with a clinically significant PDA are at risk of increased mortality and morbidity from chronic lung disease, necrotising enterocolitis, and pulmonary haemorrhage.

Full-term infants and children:

Spontaneous closure of a patent ductus after 3 months of age is relatively rare. If a significant shunt is left untreated, it can result in the development of pulmonary obstructive disease that can become manifest as early as 15 months of life.
Overall prognosis of these patients is very good and typically after closure, patients are well and the procedure is considered curative.

658
Q

Complications of patent ductus arteriosus

A
RDS
NEC in prem
Pulmonary haemorrhage in prem
PHTN
CHF
Endarteritis 
Aneurysm of duct
659
Q

A 1.5 month-old infant girl is brought to her paediatrician for poor feeding. Since she was last seen at 2 weeks she has had poor weight gain. She sweats with feeds and seems to tire out easily. There is no significant family history. On physical examination she is noted to be tachypnoeic and uninterested in her bottle after a few minutes of feeding. She has increased work of breathing. On cardiac examination, she has a grade 4 continuous murmur that is heard in the left infraclavicular region and back. She also has an early diastolic rumble best heard at the apex. Her liver is 3 cm below her costal margin. Her pulses are bounding. Her CXR reveals an enlarged heart with a prominent main pulmonary artery segment and increased pulmonary markings.

A

patent ductus arteriosus

660
Q

A 28 week premature boy is treated with appropriate doses of surfactant. However, on his second day of life he has worsening symptoms of respiratory distress syndrome with increasing ventilatory requirements. He has also started demonstrating apnoeic episodes. He is noted to have a widened pulse pressure (30 mmHg) on his arterial line and he is starting to have some bloody stools. On physical examination, he is noted to have bounding pulses and a prominent precordial impulse. On auscultation a grade 3 systolic ejection murmur can be heard in the left infraclavicular area. His abdomen also appears distended. On CXR, his lung fields are almost completely opacified.

A

patent ductus arteriosus

661
Q

Clinical presentation depends on the age of the patient and the size of the shunt. Patients can be completely asymptomatic, have significant heart failure, or, if they present much later in life, may have signs and symptoms of pulmonary hypertension. Premature infants with a haemodynamically significant PDA usually develop clinical signs in the first week of life. Full-term infants with a PDA may present in early infancy with a murmur, or signs of heart failure if they have a large shunt across the ductus. Conversely, they may not present until late childhood with mild exercise intolerance if the shunt is smaller. Haemodynamically insignificant shunts will not be associated with symptoms but may be picked up by the presence of a murmur on examination. Atypically, if a haemodynamically significant PDA is missed in childhood, it may present in adulthood with heart failure, atrial arrhythmias, endarteritis, or, most seriously, with irreversible pulmonary vascular disease characterised by desaturation and evidence of right heart failure.

A

patent ductus arteriosus

662
Q

Define pulmonary stenosis

A

Pulmonary stenosis (PS) obstructs the blood flow from the right ventricle (RV) into the pulmonary bed, resulting in a pressure gradient greater than 10 mmHg across the pulmonary valve during systole. In 80% to 90% of all cases PS is found at the level of the valve, but it can also occur below the level of the valve or distally in the pulmonary arteries.

PS is commonly associated with other forms of congenital heart disease.

The increased subpulmonary ventricular pressure may lead to hypertrophy of the RV proportional to the degree of stenosis. Clinical symptoms range from none in mild PS to profound cyanosis and the potential for sudden death in critical PS.

Mostly congenital.

Symptoms ranging from none to profound cyanosis and the potential for sudden death.

Systolic ejection murmur, loudest over left upper sternal border.

Cyanotic patients treated with oxygen and prostaglandin E1 prior to diagnostic testing.

Diagnosis confirmed and severity classified by echocardiography.

Mild pulmonary stenosis is a benign condition requiring sequential cardiac follow-up but no therapy; prophylaxis for infective endocarditis not recommended.

Percutaneous balloon pulmonary valvuloplasty (PBPV) indicated in moderate to severe/critical lesions.

Surgical valvotomy reserved for treatment failure and contraindication to PBPV.

663
Q

Epidemiology of pulmonary stenosis

A

PS in association with other lesions is recognised in up to 30% of patients with congenital heart disease, and isolated PS has been estimated to occur in between 8% and 14% of all patients with congenital heart disease.

In people with Noonan’s syndrome, the isolated PS is frequently caused by a dysplastic pulmonary valve and has an incidence of up to 27%.

Over the last 20 years, there has been a significant increase in the rate of PS diagnosed, potentially due to increased access to Doppler echocardiography.

Compared with white people, black people have an increased rate of peripheral PS (5.35 versus 2.45 per 10,000 live births) and a statistical trend towards a higher incidence of valvular PS (4.48 versus 3.46 per 10,000 live births).

664
Q

Aetiology of pulmonary stenosis

A

The majority of cases are congenital. Possible embryological explanations for PS vary from a malformation of the bulbus cordis to fetal endocarditis.

A genetic abnormality is another possible aetiology, as PS is frequently associated with genetic syndromes such as Noonan’s, LEOPARD, Williams’, or Alagille’s syndromes.

Acquired PS is a rare condition associated with carcinoid syndrome, infectious endocarditis, myocardial tumours, and external compression. Carcinoid heart disease is caused by plaque formation on the endocardial surfaces of the right atrium and of the right valves, resulting in thickened and immobile valve leaflets.

665
Q

RFs for pulmonary stenosis

A
STRONG
Noonan syndrome
LEOPARD syndrome
Alagille syndrome
Williams syndrome
Congenital rubella syndrome
WEAK
Black
Carcinoid syndrome 
Infectious endocarditis 
Myocardial tumours 
External compression
666
Q

Sx of pulmonary stenosis

A

COMMON
Pathological ejection systolic murmur with or without systolic click

OTHER UNCOMMON
Rheumatic fever
Dyspnoea - With/without exertion in severe or critical PS
Chest pain - With/without exertion in severe or critical PS
Syncope 
Features of associated syndromes 
Failure to thrive
Cyanosis
Sx of RHF
RV heave
Systolic thrill
667
Q

Ix for pulmonary stenosis

A

1ST

ECG - Routinely performed in all patients with a pathological murmur with/without respiratory distress - mild PS: normal in 30% to 65% or mild right axis deviation; moderate PS: right axis deviation (abnormal for age) and RV conduction delay (abnormal for age); severe/critical PS: extreme right axis deviation, R wave large for age, right atrial enlargement with tall and peaked P wave in lead II and in precordial leads V1 to V3

CXR - Routinely performed in all patients with a pathological murmur with/without respiratory distress. Usually normal; may show prominent main pulmonary artery shadow; marked cardiomegaly, right atrial and ventricular enlargement, and decreased pulmonary vascularity may be seen in severe disease

Echo + doppler - abnormal morphology of valve; increased transvalvular gradient across pulmonary valve during systole (>10 mmHg).

Hb + Hct - increased in cyanosis caused by right-to-left shunt, leading to erythrocytosis
Pulse ox - LOW
ABG - low PaO2

668
Q

Rx of pulmonary stenosis

A

Acute

Mild = observe

Mod-severe
1. Percutaneous balloon pulmonary valvuloplasty
2. Surgical valvuloplasty
+ endocarditis prophylaxis

IF NEONATES + CRITICAL + RESP DISTRESS/CYANOSIS
Add Alpostadil + sup o2
- Cyanotic neonates who are unresponsive to oxygen can be treated with alprostadil (prostaglandin E1). This dilates arterioles and maintains patency of ductus arteriosus increasing blood flow to the lungs. Maximal effect seen in 30 minutes.

669
Q

Prognosis of pulmonary stenosis

A

Mild = normal survival (a static lesion, highest risk of progression is during childhood)

Mod/severe/critical PS
Percutaneous valvuloplasty produces excellent short- and long-term results in reducing the pulmonary valve obstruction.
The study identified the following independent risk factors for suboptimal outcome: higher initial pulmonary valvular gradient, high early residual gradient, dysplastic valve, and younger age at intervention.

670
Q

Complications of pulmonary stenosis

A
Vascular laceration during surgery 
Tearing of pulmonary valve annulus due to valvuloplasty
Pulmonary valve insufficiency 
RHF
SCD
Cardiac perforation
Tricuspid insufficiency 
Death post-valvuloplasty
671
Q

A full-term 3.3 kg newborn non-dysmorphic girl is found to have a systolic ejection murmur shortly after birth. She is clinically asymptomatic and fully saturated while breathing room air

A

pulmonary stenosis

672
Q

A 7-year-old boy with a normal karyotype and dysmorphic features such as webbing of neck, short stature, and pectus carinatum is incidentally found to have a prominent main pulmonary artery and cardiomegaly on chest x-ray for respiratory complaints.

A

pulmonary stenosis

673
Q

PS may also present as critical pulmonary stenosis. Critical pulmonary stenosis is associated with severe right ventricular outflow obstruction and right-to-left shunting of blood at the atrial level, resulting in cyanosis. The systolic murmur may be variable, is usually prominent, but may be surprisingly soft if the cardiac output is limited and there is limited blood flow across the stenotic valve. PS may also rarely be associated with other systemic diseases including intracardiac tumours, neurofibromatosis, and extrinsic compression lesions (neoplasms).

A

pulmonary stenosis

674
Q

Define tetralogy of fallot

A

Tetralogy of Fallot (TOF) is a congenital cardiac malformation. The key morphological abnormality is anterior and cephalad deviation of the muscular outlet of the ventricular septum, which causes the 4 classic findings: (1) a mal-alignment ventricular septal defect (VSD), (2) aorta over-riding the VSD, (3) right ventricular outflow tract obstruction, (4) secondary concentric right ventricular hypertrophy.

Historically, children with TOF presented with cyanosis that was progressive and life-limiting; untreated children with TOF would typically squat down, which would lead to increased pulmonary blood flow.

Ventricular septal defect with over-riding aorta and right ventricular (RV) outflow tract obstruction and resulting RV hypertrophy. The common embryological cause for this constellation of findings is anterior and cephalad deviation of the muscular outlet of the ventricular septum.

Usually presents in the neonatal period with a murmur, cyanosis, or both.

Diagnosed by echocardiography.

Treatment is by surgical repair. This usually consists of complete intracardiac repair typically during the neonatal or infant period. Occasionally, an aortopulmonary shunt is used palliatively before complete repair.

The most common long-term complications of complete repair are progressive pulmonary regurgitation and RV failure, atrial arrhythmias, and ventricular arrhythmias.

Acyanotic defects include atrial septal defects, isolated ventricular septal defects, and coarctation of the aorta. Cyanotic defects include TOF, total anomalous pulmonary venous return, transposition of great vessels, tricuspid atresia, persistent truncus arteriosus, and hypoplastic left-heart syndrome.

675
Q

Epidemiology of tetralogy of fallot

A

CHD is 3-4 per 1000
9 per 1000 congenital heart defects are TOF
M>F
No ethnic differences

676
Q

RFs for tetralogy of fallot

A
WEAK
Trisomy 21,18 or 13
Chromosome 22q11 deletions - DiGeorge 
Jagged1 gene mutations - Alagille's syndrome 
Mutation in NKX2.5 gene
FHx congenital HD

Increasing evidence suggests environmental factors, such as maternal diabetes and phenylketonuria and maternal ingestion of retinoic acids and trimethadione, may play a significant causative role in certain cases of congenital heart disease.

677
Q

Aetiology of tetralogy of fallot

A

Little is known about the exact aetiology of TOF. There is most likely to be interplay between genetic and environmental factors, but this has not been fully defined.

Patients with trisomy 21, 18, or 13 have a higher incidence of TOF than infants without trisomy.

22q11 (DiGeorge’s and associated syndromes)

Increasing evidence suggests environmental factors may play a significant role in some cases of CHD. Maternal diabetes, maternal phenylketonuria, and maternal ingestion of retinoic acids or trimethadione have all been associated with an increased risk of CHD.

678
Q

Sx of tetralogy of fallot

A

Hypercyanotic episodes - spells may present as episodic, increasing cyanosis in a baby with TOF.
Rapid, deep crying
Harsh ESM
Tachypnoea

UNCOMMON
Shock

679
Q

Ix for tetralogy of fallot

A

Pulse ox - low O2 sats
Echo - infundibular pulmonary stenosis, over-riding aorta, non-restrictive ventricular septal defect, concentric RVH
ECG - RVH with a rightwards axis, R in V1 and S in V6 above age-appropriate normals

680
Q

Rx of tetralogy of fallot

A

Rx of HYPERCYANOTIC SPELLS
Manouvres - Infant should be held in the parent’s arms and positioned with the knees to the chest.
BBlocker - propranolol 0.15-0.25mg/kg IV bolus, 2-8mg/kg/day oral
LAST ICU option = Phenylephrine to increase systemic vascular resistance above pulmonary - blood flows through the pulmonary circulation, rather than across the ventricular septal defect and into the systemic circulation.

Non-remitting severe cyanosis:
If pulmonary blood flow is severely restricted may need to perform shunt or ECMO

IF neonate with profound cyanosis -> give alprostadil

ONGOING
Complete surgical repair
Monitoring with potential pulmonary valve replacement
IE prophylaxis

681
Q

Prognosis of tetralogy of fallot

A

PREVIOUS life expectancy was 12yo

Current surgical outcomes are excellent with recent cohorts following complete repair showing survival rates of 100% at 1 month, 93% at 1 year, and 93% at 5 years. Freedom from reoperation rates were 100% at 1 month, 89% at 1 year, and 58% at 5 years.

Once the patient has undergone complete surgical repair, the prognosis and any problems that occur may be related to the patient’s individual anatomy and type of surgical repair, as well as any associated conditions. One study of survivors of the first year after surgical repair showed actuarial survival rates of 97% at 10 years, 94% at 20 years, 89% at 30 years, and 85% at 36 years.

Common long-term complications are related to arrhythmias, progressive pulmonary outflow obstruction, and progressive pulmonary regurgitation resulting in right ventricular failure.

Reasons for re-operation are based on progressive pulmonary obstruction and pulmonary regurgitation

682
Q

Complications of tetralogy of fallot

A
Cyanotic spells
Paradoxical emboli 
Progressive pulmonary regurg and RV failure
Ventircular arrhythmias
Atrial Arrhythmias
SCD
Congestive HF
683
Q

List cyanotic + acyanotic congenital heart defects

A

Acyanotic defects include atrial septal defects, isolated ventricular septal defects, and coarctation of the aorta.

Cyanotic defects include TOF, total anomalous pulmonary venous return, transposition of great vessels, tricuspid atresia, persistent truncus arteriosus, and hypoplastic left-heart syndrome.

684
Q

A 1-day-old infant in the general care nursery born at full term by uncomplicated spontaneous vaginal delivery is noted to have cyanosis of the oral mucosa. The baby otherwise appears comfortable. On examination, respiratory rate is 40 and pulse oximetry is 80%. A right ventricular lift is palpated, S1 is normal, S2 is single, and a harsh 3/6 systolic ejection murmur is heard at the left upper sternal border.

A

Tetralogy of Fallot

685
Q

A 1-day-old infant in the general care nursery born at full term by uncomplicated spontaneous vaginal delivery is noted to have a murmur on examination. The baby otherwise appears well. On examination, respiratory rate is 40 and pulse oximetry is 96%. Precordium is normoactive. With auscultation, S1 is normal, S2 is single, and a 2/6 systolic ejection murmur is heard at the left upper sternal border.

A

Tetralogy of Fallot

686
Q

Cyanosis occurs if severe right ventricular outflow tract obstruction forces blood returning to the right side of the heart to be shunted right to left across the ventricular septal defect (VSD) and out to the systemic circulation, by-passing the lungs. Hypercyanotic spells may present as episodic, increasing cyanosis in a baby with TOF. The baby is typically crying and breathing deeply and rapidly, but may not be in significant respiratory distress. The typical murmur of TOF may disappear during the spell. This presentation is potentially life-threatening and requires rapid intervention. Alternatively, a baby may be severely cyanotic at birth.

A

Tetralogy of Fallot

687
Q

Define VSD

A

Ventricular septal defects (VSDs) are congenital or acquired defects in the inter-ventricular septum that allow shunting of blood between the left and right ventricles. Eisenmenger’s syndrome is shunt reversal (blood flowing from the right to the left ventricle) leading to the distribution of de-oxygenated blood to the systemic arterial circulation.

A defect in the interventricular septum that allows shunting of blood between the left and right ventricles.

Usually congenital, but rarely acquired after myocardial infarction or trauma.

May be associated with other congenital defects such as tetralogy of Fallot.

Significant left-to-right shunting results in pulmonary hypertension, which, if left untreated, can progress to shunt reversal with cyanosis and the Eisenmenger’s syndrome.

Small shunts may close spontaneously in childhood and can be managed by observation.

Large shunts require surgical closure.

688
Q

Epidemiology of VSD

A

VSDs are among the most common congenital heart defects in infants and children, and an isolated VSD is seen in up to 3.5 infants per 1000 live births.

Most of these close spontaneously in childhood.

Type 2 (peri-membranous) defects are the most common VSDs, accounting for 70% of VSDs. Type 4 (muscular) defects account for 20%, while type 1 (supracristal or doubly committed juxta-arterial) defects and type 3 (inlet type) defects account for 5% each.

Down’s syndrome is commonly associated with atrioventricular septal defects, which are seen in between one third and one half of all patients. These may be atrial septal, ventricular septal, or both. Overall, atrioventricular septal defects are thought to be present in about 0.25 to 0.43 per 1000 live births.

Acquired VSDs, due to MI or trauma, are rare.

689
Q

Aetiology of VSD

A

Congenital

VSDs are usually developmental congenital conditions. No definite cause is known. In families with a strong history of congenital cardiovascular malformations, the incidence of VSDs is increased. Certain genetic abnormalities have a high incidence of associated VSD, the most common being Down’s syndrome, which is associated with congenital cardiovascular malformations in about 50% of cases.

Acquired

In rare cases, a VSD can occur as a result of acute MI. This usually occurs 2 to 5 days after the infarction and is marked by the onset of acute left-heart failure, chest pain, low cardiac output and shock.

690
Q

RFs for VSD

A

STRONG
FHx
Downs (TRI21)

WEAK
Maternal alcohol use in pregnancy

691
Q

Sx of VSD

A

Pansystolic murmur in the left parasternal region
- This murmur is generally holosystolic and does not increase with inspiration (unlike tricuspid regurgitation). The murmur is generally easily heard; a smaller defect generally results in a louder murmur.
Failure to thrive - Congenital heart disease should always be excluded in infants and children with unexplained failure to thrive.
SOB

OTHER COMMON
Recurrent pulmonary infections
Loud P2

UNCOMMON
Cyanosis (shunt reversal AKA Eisenmenger's syndrome)
Clubbing
Recent MI
Recent trauma
692
Q

Ix for VSD

A

1st
Echo - VSD visualised and diagnosed on 2D views; left-to-right shunt confirmed by the presence of a high-velocity jet across the VSD on colour Doppler
CXR - ranges from normal to cardiomegaly and increased pulmonary vascular markings to peripheral pruning of vascular markings

ECG - not diagnostic but - The ECG may be normal in small, non-restrictive defects.
In moderate-sized defects, there may be evidence of left ventricular and also sometimes left atrial enlargement.
In cases of larger shunts, there may be evidence of bi-ventricular hypertrophy.
In a few patients, there may be predominant right ventricular hypertrophy.

CONSIDER
Cardiac MRI
Cardiac catheterisation - measurement of pulmonary artery pressures; demonstration of the location and degree of shunting

693
Q

Rx of VSD

A

SMALL
In cases of small restrictive VSDs with Qp to Qs shunt ratio of <1.5, observation and follow-up is all that is generally indicated.
+ Endocarditis prophylaxis - amoxicillin/clindamycin

MEDIUM/LARGE
Corrective closure - Indicated when Qp to Qs shunt ratio is >1.5, PA systolic pressure >50 mmHg, increased LV and left atrial size, or deteriorating LV systolic function.

The usual procedure is open surgery in which a patch (bovine pericardium or synthetic material) is used to close the VSD.

+ Endocarditis prophylaxis - amoxicillin/clindamycin

LEFT-RIGHT SHUNT
Medicines include diuretics, and in some cases angiotensin-converting enzyme (ACE) inhibitors and digoxin.
+ corrective closure + Endoprophylaxis

EISENMENGER SYNDROME
MUST GIVE PULMONARY VASODILATORS:
- Bosentan/sildenafil/epoprostol
NB bosentan = endothelin1 antagonist

Heart-lung transplantation

694
Q

Prognosis of VSD

A

Patient outlook depends on the size of the defect.

Asymptomatic restrictive ventricular septal defect

This type, found in infants, closes spontaneously in about 50% of cases. Others are likely to remain asymptomatic.
Moderate-sized restrictive defect

May be asymptomatic on presentation. If left untreated over the long-term, many will develop pulmonary hypertension and later, symptoms of congestive heart failure. If treated with closure, these patients overall have an excellent outcome.
Large-sized non-restrictive defects

These tend to produce early pulmonary hypertension and early heart failure symptoms requiring early closure. If not closed, patients may develop severe pulmonary hypertension with reversal of blood shunting, resulting in cyanosis and Eisenmenger’s syndrome, which is then inoperable and associated with a poor prognosis.

695
Q

Complications of VSD

A

Aortic regurgitation
Complication of surgery = heart block
Post-operative dysrhytmias
Infective endocarditis

696
Q

An infant is noted at birth to have a cardiac murmur. Physical examination reveals a systolic murmur at the left sternal border. There is no clinical evidence of heart failure.

A

VSD

697
Q

An infant presents with symptoms of shortness of breath on exertion, and failure to thrive. Physical examination reveals a systolic murmur at the left sternal border and signs of congestive heart failure.

A

VSD

698
Q

Eisenmenger’s syndrome presents with central cyanosis, with or without finger clubbing, and patients may have evidence of heart failure and a history of recurrent pulmonary infections. Hyper-viscosity associated with Eisenmenger’s syndrome manifests with headache, fatigue, and sometimes mental-status changes. Usually, the patient will have a documented history of a VSD that was not corrected. A VSD secondary to MI presents 3 to 5 days after the initial MI with symptoms of heart failure or, in severe cases, cardiogenic shock. Traumatic VSDs can present at a variety of time frames from immediate to delayed; the presence and severity of symptoms depend on the size of the VSD.

A

VSD

699
Q

Define transposition of the great arteries

A

Transposition of the great arteries is a serious but rare heart defect present at birth (congenital), in which the two main arteries leaving the heart are reversed (transposed). The condition is also called dextro-transposition of the great arteries. A rarer type of this condition is called levo-transposition of the great arteries

Transposition of the great arteries changes the way blood circulates through the body, leaving a shortage of oxygen in blood flowing from the heart to the rest of the body.

Transposition of the great arteries is usually detected either prenatally or within the first hours to weeks of life.

700
Q

Aetiology of transposition of the great arteries

A

Transposition of the great arteries occurs during fetal growth when your baby’s heart is developing. Why this defect occurs is unknown in most cases.

Although some factors, such as genetics, rubella or other viral illnesses during pregnancy, maternal age over 40, or maternal diabetes, may increase the risk of this condition, in most cases the cause is unknown.

701
Q

RFs for transposition of the great arteries

A

A history of German measles (rubella) or another viral illness in the mother during pregnancy
Drinking alcohol during pregnancy
Smoking during pregnancy
A mother who has poorly controlled diabetes
Genetics, rubella or other viral illnesses during pregnancy, maternal age over 40, or maternal diabetes, may increase the risk of this condition

702
Q

Sx of transposition of the great arteries

A

Cyanosis
SOB
Lack of appetite
Poor weight gain

703
Q

Ix for transposition of the great arteries

A

Echo - visualisation
Cardiac catheterisation - only done if echo inconclusive. Cardiac catheterization may be done urgently to perform a temporary treatment for transposition of the great arteries (balloon atrial septostomy).
CXR - cardiomegaly

704
Q

Rx of transposition of the great arteries

A

Alprostadil - maintains PDA
Atrial septostomy - emergency

Surgery
Arterial switch operation

705
Q

Prognosis of transposition of the great arteries

A

Narrowing of coronary arteries
Arrhythmias
Heart muscle weakness or stiffness leading to heart failure
Narrowed connections where the great vessels are connected
Regurgitation

706
Q

Complications of transposition of the great arteries

A

Hypoxia
HF
Damage to lungs from hypoxia

707
Q

Define meningitis

A

Inflammation of the leptomeninges surrounding the brain and spinal cord. Meningitis commonly refers to inflammation of the arachnoid matter and CSF.

Infection of the meninges usually follow bacteraemia
• Inflammation and endothelial damage à cerebral oedema
• ­­ICP
• Dec Cerebral blood flow -> CEREBRAL CORTICAL INFARCTION
• Fibrin deposits block CSF resorption by the arachnoid villi -> hydrocephalus

708
Q

Epidemiology of meningitis

A

Peak incidence >2months

709
Q

Aetiology of meningitis

A

Viral most common
Bacterial = medical emergence
Non infectious causes: malignancy and A/I disease
Failure to respond to Rx: consider mycoplasma or B.burgdoferi or fungi

710
Q

RFs for meningitis

A

Immunodeficiency (inherited/acquired) Structural abnormalities of the skull or meninges Malignancy and autoimmune conditions Exposure to meningococcus/ HiB
Recent infection
Travel
Penetrating head trauma/ recent neurosurgical procedure

711
Q

Sx of meningitis

A
Older children:
Fever 
Headache 
Photophobia 
N&amp;V
Kernig’s sign + Neck stiffness Confusion 
Lethargy 
Irritability
<1yo:
Fever/ hypothermia 
Lethargy 
Respiratory distress 
Jaundice
Poor feeding
D&amp;V
Seizures
Bulging fontanelle Irritability and restlessness

Raised ICP sign:
May cause CUSHINGs triad:
Bradycardia, hypotension, abnormal breathing patter

Late Sx of raised ICP:
Papilloedema, bulging fontanelle in infants, opisthotonus (hyperextension of head and back)

NB Febrile purpura = meningiococcal sepsis until proven otherwise

Kernigs sign = flexion at hip causes flexion of neck. Knee cannot be fully extended

Brudzinskis sign = passive flexion of neck causes flexion of both legs/thighs

712
Q

Ix for meningitis

A

FBC + U+E + Glucose/creatinine LFTs
Blood cultures
Clotting (if purpura)
Blood gas (lactate/acidosis)

Septic screen = MCS, blood, throat swab, urine, stool, CSF

LP - if no contraindications
Rapid antigen testing: blood/CSF/Urine

Consider CT/MRI
CSF PCR

713
Q

Rx of meningitis

A

If in primary care:
GIVE IM BENZYLPENICILLIN

Rapid ABCD
Establish IV access + bolus fluids 20-40ml/kg if in shock

Empirical ABx = vancomycin 15mg/kg + ceftriaxone 50mg/kg

Glucose if hypoglycaemic
Correct acidosis / Rx seizures

Supportive

  • Fluid resus if needed
  • Regular PEWS scoring
  • Daily euro-exam
  • Daily head circumference if <18month old

Beyond neonatal period dexamethasone administered as well to minimise neurological sequelae

Household contacts of patients who have had group C meningococcal meningitis should be vaccinated

714
Q

Complications of meningitis

A

Neurological: seizures, ­ ICP, cerebral oedema, ischemia, cerebral abscess, hydrocephalus
Disseminated illness: Arthritis, vasculitis, pericarditis

715
Q

Prognosis of meningitis

A

5-10% mortality

About 15% of children who survive bacterial meningitis develop severe sequelae such as sensorineural hearing loss, motor problems, seizures, and mental retardation

20% to 30% of children have more subtle adverse outcomes such as cognitive, academic, and behavioral problems.

Up to one third of adults who have had bacterial meningitis have cognitive impairment.

716
Q

Define encephalitis

A

Inflammation of the brain parenchyma which manifests with altered neurological function (altered mental status/behaviour/ motor and sensory deficits/ seizure)

717
Q

Epidemiology of encephalitis

A

<28days old - increased risk of CMV, HSV + Rubella

Summer - enteroviruses
Winter - post infectious encephalitis
Blood transfusion - CMV EBV

718
Q

Aetiology of encephalitis

A
  1. Direct invasion of the cerebrum by a neurotoxic virus (e.g. HSV)
  2. Delayed brain swelling following a disordered neuroimmunological
    response to an antigen, usually a virus (post-infectious encephalopathy)
    e.g. following chickenpox
  3. A slow virus infection, e.g. HIV or subacute sclerosing panencephalitis
    (SSPE) following measles

Enteroviruses, respiratory viruses and herpes virus (UK)
Mycoplasma, Borrelia burgdoferi, Bartonella henselae, rickettsial infections and the arboviruses (worldwide)

719
Q

RFs for encephalitis

A
Travel
Contact
Immunisations
Immunodeficiency 
Recent viral infections 
Animal/insect bites
720
Q

Sx of encephalitis

A

Fever
Seizure
Decreased conscious level
Clinically similar to meningitis sometimes

DIAGNOSTIC CRITERIA

  1. Altered mental status >24hrs with no alternative cause diagnosed
  2. > 2 of the following:
    - Fever >38 with 72hrs of presentation
    - New onset seizures
    - New onset focal neurology
    - CSF WBC >5
    - Abnormal neuroimaging/EEG
721
Q

Ix for encephalitis

A

Bloods: FBC, U+Es, BUN, ALT, glucose, coagulation studies
LP: pleiocytosis (lymphocyte predominant), RBC = HSV infection
Imaging: EEGm+ CT/MRI

GOLD STANDARD = PCR of CSF

Exclude other causes:
• Blood culture
• CSF gram stain + culture (acid fast stain for ?TB)
• Urine and serum toxicology
• Metabolic studies: serum ammonia, lactate and Ph
• Antibody titres
• Brain biopsy

722
Q

Rx of encephalitis

A

Manage by ABC initially

Empirical acyclovir
- If HSV confirmed then continue Rx for a further 3/52

Supportive:
Stabilise cardiorespiratory system
Treat ongoing seizures
Monitor: Cardiorespiratory status, neurological status (GCS), fluid balance and electrolytes (SIADH may ensue)

EBV/VZV - consider methylprednisolone sodium succinate

723
Q

Complications of encephalitis

A

Death
Autonomic/hypothalamic dysfunction
Ischaemic stroke
Encephalitis lethargica (Von Economo’s disease)
Neurological sequelae
Seizures
Cerebral haemorrhage, thrombosis, vasculitis, hydrocephalus, chronic fatigue syndrome

724
Q

Prognosis of encephalitis

A

Prognosis is often bleak. Mortality and morbidity vary depending on the underlying aetiology, the immune status of the host, the extent and location of anatomic lesions, the development of complications, and the time to initiate treatment.

Roughly 10% mortality

Age >65 years old, immunocompromised (HIV or immunosuppressive medication-induced), mechanical ventilation, coma, acute thrombocytopenia, elevated CSF polymorphonuclear count, cerebral oedema, and status epilepticus are associated with poor outcomes.

In children, long-term morbidity occurs in up to two-thirds of patients. This includes fatigue, cognitive impairment, attention and deficit disorders, dysphasia, motor impairment, ataxia, epilepsy, and personality changes.

Post-encephalitic epilepsy occurs in 10% by 5 years and 20% by 20 years.

725
Q

A 56-year-old man presents to the emergency department with headache, fever, blurred vision, and somnolence followed shortly by unresponsiveness to verbal commands. For the last 2 weeks he had been feeling ill and had decreased appetite and myalgias. Three days prior to presentation he experienced intermittent confusion, severe headache, and fever. Examination was limited by a generalised tonic-clonic seizure, for which he received lorazepam.

A

Encephalitis

726
Q

A 19-year-old man presents to the emergency department with a witnessed generalised tonic-clonic seizure episode. One month previously he had an upper respiratory tract infection. Over the last 2 weeks he developed headaches, blurred vision, generalised weakness, and progressive difficulty in walking. Examination revealed pain on eye movement as well as limb and gait ataxia.

A

Encephalitis

727
Q

A 1-month-old girl presents to her primary care physician with a high fever, feeding difficulties, and irritability for the past 24 hours. Examination reveals altered mental status and a bulging fontanel.

A

Bacterial meningitis

728
Q

An 18-year-old male student presents with severe headache and fever that he has had for 3 days. Examination reveals fever, photophobia, and neck stiffness.

A

Bacterial meningitis

729
Q

Atypical clinical manifestations tend to occur in very young, older, or immunocompromised patients. In infants the signs and symptoms can be nonspecific and may include fever, hypothermia, irritability, lethargy, poor feeding, seizures, apnea, or a bulging fontanel. In older adults, often the only presenting sign of meningitis is confusion or an altered mental status.

A

Bacterial meningitis

730
Q

Define conduct disorder

A

Conduct disorder (CD) is a mental disorder diagnosed in childhood or adolescence that presents itself through a repetitive and persistent pattern of behavior in which the basic rights of others or major age-appropriate norms are violated. These behaviors are often referred to as “antisocial behaviors.”

It is often seen as the precursor to antisocial personality disorder, which is per definition not diagnosed until the individual is 18 years old.

731
Q

Epidemiology of conduct disorder

A

51.1 million people globally
1-10% of children
Incarcerated youths: 23-87%

732
Q

RFs of conduct disorder

A

Child abuse
Prenatal alcohol abuse
Maternal smoking in pregnancy

Associated with:
ADHD
Substance abuse
Learning disabilities and language impairment
Oppositional defiance disorder
“a pattern of angry/irritable mood, argumentative/defiant behavior, or vindictiveness” in children and adolescents. Unlike children with conduct disorder (CD), children with oppositional defiant disorder are not aggressive towards people or animals, do not destroy property, and do not show a pattern of theft or deceit.

PROTECTIVE FACTORS
Female
High IQ
Positive social orientations
Good coping skills
Supportive family/community relationships
733
Q

Sx of conduct disorder

A

Low level of fear
Lashing out at others
Aggression
Antisocial behaviour

Childhood onset type (symptoms before 10yo):
Often associated
ADHD
Academic problems
Increased family dysfunction
Increased aggression/violence

Adolescent onset type:
Less impairment that childhood-onset
Rebellion against authority figures
Rejection of conventional values

734
Q

Ix for conduct disorder

A

Diagnosed according to DSM-V:
It is diagnosed based on a prolonged pattern of antisocial behaviour such as serious violation of laws and social norms and rules in people younger than the age of 18.

FOUR CATEGORIES PRESENT:
aggression to people and animals, destruction of property, deceitfulness or theft, and serious violation of rules.

735
Q

Rx of conduct disorder

A

The most effective treatment for an individual with conduct disorder is one that seeks to integrate individual, school, and family settings. Additionally, treatment should also seek to address familial conflict such as marital discord or maternal depression.

736
Q

Prognosis/complications of conduct disorder

A

25-40% of youths with conduct disorder will develop antisocial personality disorder.

737
Q

Define global developmental delay

A

Global developmental delay is an umbrella term used when children are significantly delayed in their cognitive and physical development. It can be diagnosed when a child is delayed in one or more milestones, categorised into gross motor, fine motor and visual, hearing-language-speech, and social

738
Q

Aetiology of (global) developmental delay

A
Learning disabilities 
Downs syndrome
Fragile X
Childhood meningitis/encephalities
Hypothyroidism
Fetal alcohol syndrome 
ADHD
Absence seizures
Tuberous sclerosis 

Inborn errors of metabolism:

Medium-chain acyl-CoA dehydrogenase deficiency
Homocysteinuria
Congenital hypothyroidism
Isovaleric acidaemia
Glutaricaciduria type 1
Maple syrup urine disease
[all tested for on Guthrie]
739
Q

Sx of global developmental delay

A

Delayed milestones:
Limit ages: represents 2 SDs above median
o Primitivereflexes
§ Persistence beyond 6 months o Grossmotor
§ Head control – 4 months
§ Sits unsupported – 9 months
§ Stands independently – 12 months § Walks independently – 18 months

o Visionandfinemotor
§ Fixes and follows – 3 months
§ Reaches for objects – 6 months
§ Transfers objects between hands – 9 months § Pincer grip – 12 months

o Hearing,languageandspeech
§ Polysyllabic babble – 7 months
§ Consonant babble – 10 months
§ Saying 6 words with meaning – 18 months § Joins words – 2 years
§ 3 word sentances – 2.5 years

o Socialbehaviour
§ Smiles – 8 weeks
§ Fear of strangers – 10 months § Feeds self /spoon – 18 months § Symbolic play – 2/2.5 years
§ Interactive play – 3-3.5 years

740
Q

Ix for global developmental delay

A

Developmental screening + history can diagnose 1/3rd of cases

Other tests:
Chromosomal microarray + karyotyping (looking for trisomy, microdeletions, and duplications, most sensitive test)
Specific gene disorders eg Rett
Targeted MRI brain - more likely to contribute to a diagnosis if the child has abnormal physical signs such as microcephaly, macrocephaly, a change in head circumference, focal neurological signs, or epilepsy.

Neonatal Guthrie testing:

Medium-chain acyl-CoA dehydrogenase deficiency
Homocysteinuria
Congenital hypothyroidism
Isovaleric acidaemia
Glutaricaciduria type 1
Maple syrup urine disease
741
Q

Rx of global developmental delay

A

The specific management of children with global developmental delay will depend on their individual needs and underlying diagnosis. Early intervention is essential to support the child to reach their full potential. Specialists involved in the management of GDD in children include:

Speech therapists
Hearing specialists
Developmental paediatricians
Neurologists
Providers of Early Intervention Services (depending on location)
As well as involving professionals, parents can support the development of their child by playing with them, reading with them, showing them how to do tasks, and supporting them to participate in activities of daily living such as washing, dressing, and eating

742
Q

What is tested for on Guthrie

A
Medium-chain acyl-CoA dehydrogenase deficiency
Homocysteinuria
Congenital hypothyroidism
Isovaleric acidaemia
Glutaricaciduria type 1
Maple syrup urine disease
743
Q

What is tested for on Guthrie

A
Medium-chain acyl-CoA dehydrogenase deficiency
Homocysteinuria
Congenital hypothyroidism
Isovaleric acidaemia
Glutaricaciduria type 1
Maple syrup urine disease
744
Q

Define vertically transmitted infection

A

A vertically transmitted infection is an infection caused by pathogens (such as bacteria and viruses) that uses mother-to-child transmission, that is, transmission directly from the mother to an embryo, fetus, or baby during pregnancy or childbirth. It can occur when the mother gets an infection as an intercurrent disease in pregnancy. Nutritional deficiencies may exacerbate the risks of perinatal infection.

745
Q

RFs for vertically transmitted infection

A

Transplacental transmission or during childbirth

Contact with ill
Amniocentesis
Trauma
Childbirth

746
Q

Aetiology of vertically transmitted infection

A
T – Toxoplasmosis / Toxoplasma gondii
O – Other infections (see below)
R – Rubella
C – Cytomegalovirus
H – Herpes simplex virus-2 or neonatal herpes simplex
"Other infections" include:
Parvovirus B19
Coxsackievirus
Chickenpox (caused by varicella zoster virus)
Chlamydia[1]
HIV[2][3]
Further information: HIV and pregnancy
Human T-lymphotropic virus[4]
Syphilis[5]
Zika fever, caused by Zika virus, can cause microcephaly and other brain defects in the child.[6]

SOME PROPOSE

C – Chickenpox and shingles
H – Hepatitis, C, (D), E
E – Enteroviruses
A – AIDS (HIV infection)
P – Parvovirus B19 (produces Hydrops fetalis secondary to aplastic anemia)
T – Toxoplasmosis
O – Other (Group B Streptococcus, Listeria, Candida, Lyme disease)
R – Rubella
C – Cytomegalovirus
H – Herpes simplex
E – Everything else sexually transmitted (gonorrhea, Chlamydia infection, Ureaplasma urealyticum, human papillomavirus)
S – Syphilis
747
Q

Sx of vertically transmitted infection

A
MOTHER
Flu-like Sx
Asymptomatic
Fever
Petechial rash
SGA baby
Hepatosplenomegaly 
Jaundice
Neonatal effects:
Hearing impairment
Eye problems
Mental retardation
Autism
Death
748
Q

Ix for vertically transmitted infection

A

Blood, urine and CSF rested for evidence of aforementioned pathogens
Cultured or specific IgM testing

749
Q

Rx of vertically transmitted infection

A

Toxoplasmosis + syphilis = Rx with ABx

Rubella + VZV can be prevented by vaccinating the mother

Malaria prophylaxis

If mother has +ve Pap test at cervical screening, delivery by CS can be indicated

750
Q

Prognosis/complications of vertically transmitted infection

A

Each type of vertically transmitted infection has a different prognosis. The stage of the pregnancy at the time of infection also can change the effect on the newborn.

751
Q

Prognosis/complications of vertically transmitted infection

A

Each type of vertically transmitted infection has a different prognosis. The stage of the pregnancy at the time of infection also can change the effect on the newborn.

752
Q

Define learning disability

A

Intellectual impairment may be either generalised (cognitive impairment) or specific to one area (learning difficulty). Children with cognitive impairment have below-average IQ, at <70.

The degree of cognitive impairment depends on the underlying disorder and its severity. An IQ of 50 to 70 is classed as mild cognitive impairment. Children with a specific learning difficulty, on the other hand, have difficulties with particular mental tasks but a normal IQ. Examples include difficulties with speech (specific language impairment), reading and writing (dyslexia), and use of numbers (dyscalculia). Although the terms ‘learning difficulty’ and ‘cognitive impairment’ have specific definitions, they are sometimes used interchangeably by clinicians.

753
Q

Epidemiology of learning disability

A

1% children have cognitive impairment

Downs syndrome and fatal alcohol syndrome = most common

10% have learning disability
Dyslexia is the most common specific learning disability (affecting about 5% to 12% of children).

754
Q

Aetiology of learning disability

A

Can be congenital or acquired:

Developmental causes include a range of genetic conditions (such as Down’s syndrome or autism spectrum disorders), intra-uterine infections, fetal alcohol syndrome, teratogenic drugs, premature birth, and congenital hypothyroidism.

Acquired causes include central nervous system (CNS) infections (bacterial meningitis, encephalitis), CNS tumours, hypoxia, traumatic brain injury, and psychosocial deprivation.
Intrauterine infections - TORCH

Perinatal events:
Extreme prematurity
Perinatal hypoxia

Congenital hypothroidism
Phenylketonuria
Deprivation

Learning difficulties can be specific without generalised cognitive impairment:
ADHD
Dyslexia 
Dyscalculia 
Specific language impair
Central auditory processing disorder
Dyspraxia
Genetic:
Downs
Fragile X
Prader Willi
Angelmans 
Williams
Retts
Turners
Tuberous sclerosis
DiGeorge
Autism
755
Q

Sx of learning disability

A

Isolated / combination of delayed milestones

Oblique palpebral fissures, median epicanthic fold, low nasal bridge, low-set ears, central iris Brushfield spots, short curved fifth finger, and a single palmar crease are characteristic of Down’s syndrome.

Macrocephaly, long face, high arched palate, prominent jaw, epicanthic folds, macro-orchidism, and large ears are characteristic of fragile X syndrome. Strabismus may also be present.

Short stature, hypotonia, small hands and feet, and incomplete sexual development, often with cryptorchidism, are characteristic of Prader-Willi syndrome.

Widely spaced teeth, long philtrum, wide mouth, and full lips are seen in William’s syndrome.

Cleft palate, micrognathia, low-set ears, and facial dysmorphism are seen in DiGeorge syndrome.

Short stature, low-set ears, webbed neck, low hairline, and cubitus valgus are characteristic of Turner’s syndrome.

Skull deformity is seen in craniosynostosis.

Skin examination: signs of tuberous sclerosis, such as facial fibroangiomata or periungual lesions, may be seen.

Neurological examination: common subtle findings in children with specific learning disabilities include synkinesias (involuntary movements of one body part when moving another, such as mirror movements of the left hand when moving the right), difficulty with voluntary saccades, dystonic posturing with stress gait, and mild hyperreflexia. Other neurological signs are usually, but not always, in keeping with the developmental history. Spasticity, or variable tone, is likely to be due to cerebral palsy. Hypotonia is most likely to be associated with a genetic disorder (e.g., Down’s syndrome). Focal signs suggest intracerebral pathology.

Measurement of head circumference: microcephaly suggests a developmental disorder, which may be either genetic or acquired. Macrocephaly suggests a developmental disorder (such as fragile X syndrome or tuberous sclerosis), a metabolic disorder (such as glutaric aciduria), certain leukodystrophies (such as Alexander disease and Canavan disease), or possibly hydrocephalus, which is most commonly associated with structural brain abnormalities or a history of meningitis or intraventricular haemorrhage.

Ophthalmoscopy may aid in the diagnosis of congenital infections of the CNS (e.g., toxoplasma or cytomegalovirus [CMV]) and in some genetic disorders associated with cognitive delays (e.g., retinal or peripapillary astrocytic hamartomas in tuberous sclerosis).

756
Q

Ix for learning disability

A

Neurodevelopmental history + milestones

The ages and stages questionnaire

Congenital hypothyroidism normally screened for (2-4days)
Guthrie for phenylketonuria
IU infection tasing
CSF cytology if suspect meningitis/neuro infection

Genetic testing for syndrome

Skull XRAY if craniosynostosis

Abnormality in neuro exam = MRI preferred

757
Q

Rx of learning disability

A

Treat causes
Enable EIS
Referral to learning disability specialist CAMHs

758
Q

Rx of learning disability

A

Treat causes
Enable EIS
Referral to learning disability specialist CAMHs

759
Q

Definition of T1DM

A

Type 1 diabetes mellitus is a metabolic disorder characterised by hyperglycaemia due to absolute insulin deficiency. The condition develops due to destruction of pancreatic beta cells, mostly by immune-mediated mechanisms. In some patients there may be no evidence of autoimmune destruction of pancreatic beta cells; this is called idiopathic type 1 diabetes.

Type 1 diabetes mellitus is a metabolic disorder characterised by hyperglycaemia due to absolute insulin deficiency.

Patients most often present with a few days or weeks of polyuria, polydipsia, weight loss, and weakness.

Some patients may present with diabetic ketoacidosis.

Intensive glycaemic control has been shown to decrease the incidence of microvascular and macrovascular complications.

Microvascular complications include retinopathy, nephropathy, and neuropathy.

Macrovascular complications include coronary artery, cerebrovascular, and peripheral vascular disease.

760
Q

Epidemiology of T1DM

A

5-10% of all diabetics

130k diagnosed each year

WW incidence increase by 3% per yr

761
Q

Aetiology of T1DM

A

HLA DR/DQ polymorphisms

Associations with congenital rubella + human enteroviruses

Supplementation with vitamin D may be protective

Up to 90% of patients will have autoantibodies to at least one of 3 antigens: glutamic acid decarboxylase; insulin; and a tyrosine-phosphatase-like molecule, islet auto-antigen-2 (IA-2).

HLADQ2 associated with coeliac + DM

762
Q

RFs for T1DM

A

STRONG
HLA risk profile for DM = high in Finland

WEAK
Genetic predisposition
Infectious agents
Dietary factors (Further research is required to determine whether cow’s milk, early introduction of cereals, or maternal vitamin D ingestion increase type 1 diabetes risk.)

763
Q

Sx of T1DM

A

COMMON
RFs
Polyuria
Polydipsia

OTHER COMMON
Young age (typically 5-15yo)
WL
Blurred vision
N+V
Abdominal pain - DKA
Tachypnoea - DKA
Lethargy - DKA

UNCOMMON
Coma - DKA

764
Q

Ix for T1DM

A

Random plasma glucose - >11.1 (Confirms diagnosis in presence of symptoms of polyuria, polydipsia, and unexplained weight loss.)

Fasting plasma glucose - > 6.9mmol/L (>8hrs no calories)

2 hour plasma glucose after OGTT 75g load - >11.1

Plasma or urine ketones - confirms type 1 in the presence of hyperglycaemia

HBA1c - ≥48 mmol/mol (≥6.5%)

Fasting C peptide - low or undetectable (byproduct - levels reflect insulin production)

A/I markers - These include autoantibodies to glutamic acid decarboxylase, insulin, islet cells, islet antigens (IA2 and IA2-beta), and the zinc transporter ZnT8. +ve

765
Q

Rx of T1DM

A

Basal-bolus insulin:
Insulin glargine SC OD
Insulin determir SC BD

AND

Insulin aspart/lispro injected pre-meal

OR

Pump

INITIAL CHILD DAILY DOSE 0.5-1 unit per day, may increase in puberty

Adjunct = pre-meal insulin correction dose

IF UNCONTROLLABLE POST PRANDIAL HYPERGLYCAEMIA -> add amylin analogue: Pramlintide 15-60micrograms

Fixed-dose insulin is used when patients are already doing well on a fixed-dose regimen; or cannot manage 3 to 4 insulin injections daily; or have trouble mixing insulin.

insulin aspart protamine/insulin aspart: (70/30) injected subcutaneously twice daily

insulin lispro protamine/insulin lispro: (50/50, 75/25) injected subcutaneously twice daily

766
Q

Prognosis of T1DM

A

Untreated = fatal due to DKA

767
Q

Complications/prognosis of T1DM

A

Poorly controlled type 1 diabetes is a risk factor for chronic complications such as blindness, renal failure, foot amputations, and heart attacks. Intensive glycaemic control has been shown to decrease the incidence of microvascular and macrovascular disease in type 1 diabetes, and the decreased incidence of macrovascular disease has been shown to persist for up to 30 years.

Aim HBa1c <7% or 53mmol/mol

Overall, cardiovascular disease is the major cause of death and a major cause of morbidity for patients with diabetes. One analysis of patients with type 1 diabetes diagnosed before the age of 15 years found that the leading cause of death before the age of 30 years was acute complications of diabetes. After the age of 30 years cardiovascular disease was predominant, although death attributable to acute complications was still important in this age group.

Diabetic kidney disease - most common cause of ESRD

768
Q

A 12-year-old white girl is brought to the emergency department by her parents due to 12 hours of rapidly worsening nausea, vomiting, abdominal pain, and lethargy. Over the last week she has felt excessively thirsty and has been urinating a lot. Physical examination reveals a lean, dehydrated girl with deep rapid respirations, tachycardia, and no response to verbal commands.

A

T1DM

769
Q

The rate of beta-cell destruction varies in type 1 diabetes. In some patients, there may be a slow destruction leading to gradual onset of symptoms that is clinically indistinguishable from type 2 diabetes. When the initial presentation of type 1 diabetes occurs in adulthood, some refer to it as latent autoimmune diabetes in adults (LADA). It is useful to distinguish LADA from type 2 diabetes, because patients with LADA usually require insulin therapy. Features that suggest the presence of LADA rather than type 2 diabetes include 2 or more of the following: age of onset less than 50 years, acute symptoms, BMI less than 25 kg/m^2, and personal or family history of autoimmune disease.

A

T1DM

770
Q

Define DKA

What is the triad seen?

A

Diabetic ketoacidosis (DKA) is an acute metabolic complication of diabetes that is potentially fatal and requires prompt medical attention for successful treatment. It is characterised by absolute insulin deficiency and is the most common acute hyperglycaemic complication of type 1 diabetes mellitus.

Hyperglycaemia
Ketosis
Acidosis

Diabetic ketoacidosis (DKA) is characterised by a biochemical triad of hyperglycaemia, ketonaemia, and acidaemia, with rapid symptom onset.

Common symptoms and signs include polyuria, polydipsia, polyphagia, weakness, weight loss, tachycardia, dry mucous membranes, poor skin turgor, hypotension, and, in severe cases, shock.

Successful treatment includes correction of volume depletion, hyperglycaemia, electrolyte imbalances, and comorbid precipitating events, with frequent monitoring.

Complications of treatment include hypoglycaemia, hypokalaemia, hypoxaemia, and rarely pulmonary oedema.

Cerebral oedema, a rare but potentially rapidly fatal complication, occurs mainly in children. It may be prevented by avoiding overly rapid fluid and electrolyte replacement.

771
Q

Epidemiology of DKA

A

4% mortality

Up to 16-30% of new onset diabetes presents with DKA

772
Q

Aetiology of DKA

A

In DKA, there is a reduction in the net effective concentration of circulating insulin along with an elevation of counter-regulatory hormones (glucagon, catecholamines, cortisol, and growth hormone). These alterations lead to the extreme manifestations of metabolic derangements that can occur in diabetes. The two most common precipitating events are inadequate insulin therapy and infection. Underlying medical conditions such as MI or stroke that provoke the release of counter-regulatory hormones are also likely to result in DKA in patients with diabetes. Drugs that affect carbohydrate metabolism, such as corticosteroids, thiazides, pentamidine, sympathomimetic agents (e.g., dobutamine and terbutaline), second-generation antipsychotic agents, and immune checkpoint inhibitors may contribute to the development of DKA.
The use of sodium-glucose co-transporter 2 (SGLT-2) inhibitors has also been implicated in the development of DKA in patients with both type 1 and type 2 diabetes.

773
Q

RFs for DKA

A

STRONG
Inadequate or inappropriate insulin therapy
Infection
MI

WEAK
Pancreatitis 
Stroke
Acromegaly 
Hyperthyroidism
DRUGS:
corticosteroids, thiazides, pentamidine, sympathomimetics, second-generation antipsychotics, cocaine, immune checkpoint inhibitors, or SGLT2 inhibitors
(drugs affecting carbohydrate metabolism)

Cushings
Hispanicblack ancestry
Bariatric surgery

774
Q

Sx of DKA

A

COMMON

Sx of hyperglycaemia:
Polyuria
Polyphagia
Polydipsia
WL
Weakness
N+V (correlate with the degree of acidosis and may be confused with acute abdominal crisis)
Abdominal pain (correlate with the degree of acidosis and may be confused with acute abdominal crisis)
Dry mucous membranes
Poor skin turgor 
Sunken eyes
Tachycardia
Hypotension
Kussmaul
Acetone breath
Altered mental status 

UNCOMMON
Hypothermia

775
Q

Ix for DKA

A

Plasma glucose - High (usually >13.9)
ABG - acidosis central to diagnosis, dec bicarbonate
Capillary or serum ketones - beta-hydroxybutyrate elevated ≥3.8 mmol/L (≥0.04 mg/dL) in adults or ≥3.0 mmol/L (≥0.031 mg/dL) in children
Urine dip - glycosuria/ketonuria, may be leukocytes/nitrites if concomitant infection
Serum urea - Elevated
Creatinine - elevated
Sodium - low
Potassium - usually high
Chloride - usually low
Calcium + mag - usually low
Phosphate - normal or elevated
Anion gap - raised
Serum creatine phosphokinase - elevated if rhabdomyolysis (differentiate)
Serum lactate - elevated
LFT - N
Amylase - elevated
Lipase - N (differentiate from acute pancreatitis)

Serum osmolality - variable
FBC - elevated WCC can occur with concomitant infection

776
Q

Rx of DKA

A

General Rx:

IV fluids
Consider ICU if haemodynamic instability or cardiogenic shock, altered mental status, respiratory insufficiency, and severe acidosis. Intubation and mechanical ventilation are commonly required, with constant monitoring of respiratory parameters.

Correct:
Potassium
Bicarbonate
Phosphate

ADD INSULIN once serum potassium 3.3mmol/L

777
Q

Prognosis of DKA

A

The mortality rate is 5% in experienced centres. Death is rarely caused by the metabolic complications of hyperglycaemia or ketoacidosis but rather relates to the underlying illness. The prognosis is substantially worsened at the extremes of age and in the presence of coma and hypotension.

778
Q

Complications of DKA

A
Hypoglycaemia
Hypokalaemia (cramps, arrythias, tremor, flaccid paralysis etc..)
VTE
Cerebral oedema/brain injury 
ARDS
779
Q

A 20-year-old man is brought to the accident and emergency department with abdominal pain, nausea, and vomiting with increasing polyuria, polydipsia, and drowsiness since the previous day. He was diagnosed with type 1 diabetes 2 years previously. He mentions that he ran out of insulin 2 days ago. Vital signs at admission are: BP 106/67 mmHg, heart rate 123 beats per minute, respiratory rate 32 breaths per minute, temperature 37.1°C (98.8°F). On mental status examination, he is drowsy. Physical examination reveals Kussmaul breathing (deep and rapid respiration due to ketoacidosis) with acetone odour and mild generalised abdominal tenderness without guarding and rebound tenderness. Initial laboratory data are: blood glucose 25.0 mmol/L (450 mg/dL), arterial pH 7.24, pCO2 25 mmHg, bicarbonate 12 mmol/L (12 mEq/L), WBC count 18.5 × 10^9/L (18,500/microlitre), sodium 128 mmol/L (128 mEq/L), potassium 5.2 mmol/L (5.2 mEq/L), chloride 97 mmol/L (97 mEq/L), serum urea 11.4 mmol/L (32 mg/dL), creatinine 150.3 micromol/L (1.7 mg/dL), serum ketones strongly positive.

A

DKA

780
Q

It is now well recognised that new-onset type 2 diabetes can manifest with DKA. These patients are obese and have undiagnosed hyperglycaemia, impaired insulin secretion, and insulin resistance. However, after treatment of the acute hyperglycaemic episode with insulin, beta-cell function and insulin effects improve, so these patients are able to discontinue insulin therapy and may be treated orally or by diet alone, with 40% remaining insulin-independent 10 years after the initial episodes of DKA. These patients do not have the typical autoimmune laboratory findings of type 1 diabetes. [2] This type of diabetes has been labelled as ‘type 1 and 1/2’ or ‘type 1 and a half’ diabetes, ‘Flatbush’ diabetes, or ‘ketosis-prone’ diabetes. Conversely, an extreme hyperosmolar state similar to hyperosmolar hyperglycaemic state (HHS) has been reported in combination with DKA in type 1 diabetes.

A

DKA

781
Q

Define HF in a child

A

Heart failure (HF) results from structural or functional cardiac disorders that impair the ability of the ventricle(s) to fill with and/or eject blood. The presentation of pediatric HF is diverse because of the numerous underlying cardiac etiologies and varying clinical settings.

782
Q

Epidemiology of HF in a child

A

12-15k in US

783
Q

Aetiology/RF/Causes of HF in a child

A

Ventricular dysfunction:
Cardiomyopathy
Myocarditis
MI
Cardiac ischaemia - Kawasaki, coronary vasculitis
Arrhthmogenic - complete heart block with bradycardia, supra ventricular tachycardia, ventricular tachycardia
Drug/toxin - anthracycline
Non-cardiac: sepsis, RF, OSA, CF, interstitial lung disease, HIV
SLE
Congenital heart disease

Preserved ventricular contractility:
VOLUME OVERLOAD:
L-R shunting - VSD, PDA, ASD, AVSD
Valvular insufficiency - AR, MR, PR
Non-cardiac - AV malformation, fluid overload
PRESSURE OVERLOAD:
LS - AS, Aortic coarc, HTN
RS - PS, PHTN
784
Q

Sx of HF in a child

A

Infants – The most common symptoms are tachypnea and diaphoresis during feeds, easy fatigability, irritability, decreased volume of feeds, and poor weight gain. Failure to thrive may result in delayed motor milestones.

Young children – In young children, symptoms may include gastrointestinal symptoms (abdominal pain, nausea, vomiting, and poor appetite), failure to thrive, easy fatigability, and recurrent or chronic cough with wheezing. These symptoms be mistaken for common childhood illnesses such as gastroenteritis, reflux, asthma, or even behavioral issues.

Older children – Older children may present with exercise intolerance, anorexia, abdominal pain, wheezing, dyspnea, edema, palpitations, chest pain, or syncope

Physical examination — Physical findings vary depending on the cardiac output, and degree of volume overload and pulmonary congestion, and/or systemic venous congestion.

Tachycardia
Poor perfusion -> cool, mottled extremities, dec CAP refill, dec peripheral pulses, dec BP
Gallop rhythm (S3) - dec CO/volume overload
Pulmonary findings - tachypnoea, nasal flaring, recession, tracheal tug, grunting
Wheeze/rales
Signs of systemic congestion - hepatosplenomegaly, JVP, peripheral oedema

Associated signs: High BP in arms, low in extremities (coarc)
Systolic murmur in HOCM, AS, MR, LR shunting
Thrill with long-standing cardiomyopathy

785
Q

Ix for HF in a child

A

CxR
ECG
Echo

BNP + NT-proBNP
Troponin
FBC

Chemistry:
Hyponatraemia may be seen
RFTs
Electrolytes needed before commencing diuretics/ACEi
LFTs can be elevated 

Cardiace catheterisation

786
Q

Rx of HF in a child

A

Diuretics to reduce congestion
Inotropes to increase contractility
Vasodilators to reduce after load

Furosemide - 1–2 mg/kg
Digoxin 8–10 µg/kg/day
ACEi - Captopril is preferred in neonates (0.4–1.6 mg/kg/day in 3 divided doses) and infants (0.5–4 mg/kg/day in three divided doses). Enalapril is the first choice for those older than 2 years of age (0.1–0.5 mg/kg/day in two divided doses).

Spironolactone 1 mg/kg/day and the target maximum dose is 2 mg/kg/day

BBs - Metoprolol (0.1–0.2 mg/kg/dose twice daily and increased to 1 mg/kg/dose twice daily)

Inotropes - dopamine 5–20 mcg/kg/min and dobutamine 5–20 mcg/kg/min.

Consider ECMO if reversible cardiopulmonary arrest

Cardiac transplantation

787
Q

Define Trisomy 13 / Patau syndrome

A

Patau syndrome is a syndrome caused by a chromosomal abnormality, in which some or all of the cells of the body contain extra genetic material from chromosome 13. The extra genetic material disrupts normal development, causing multiple and complex organ defects.

788
Q

Epidemiology of Trisomy 13 / Patau syndrome

A

Patau syndrome affects somewhere between 1 in 10,000 and 1 in 21,700 live births

In England and Wales during 2008–09, there were 172 diagnoses of Patau syndrome (trisomy 13), with 91% of diagnoses made prenatally. There were 111 elective abortions, 14 stillbirth/miscarriage/fetal deaths, 30 outcomes unknown, and 17 live births.

789
Q

Aetiology of Trisomy 13 / Patau syndrome

A

3 copies of chromosome 13

790
Q

RFs for Trisomy 13 / Patau syndrome

A

Increased maternal age at pregnancy

31yrs average

791
Q

Sx of Trisomy 13 / Patau syndrome

A

NB - While many of the physical findings are similar to Edwards syndrome there are a few unique traits, such as polydactyly. .

Polydactyly
Cyclopia, proboscis 
Trigegr digits
Low set ears
Prominent heel
ROCKERBOTTOM FEET
Omphalocele - abdo contents visible in sack
Overlapping of fingers over thumb
Cleft palate 

EYES: cataracts, small eyes, retinal detachment/dysplasia,

Nervous system: Intellectual disability, Microcephaly, meningomyelocele

Abnormal genitalia, kidney defects, VSD, PDA, dextrocardia, single umbilical artery.

The most common characteristics of this syndrome are problems such as late development, mental disability, multiple malformations, cardiomyopathy, and kidney abnormalities. The most common physical signs for Patau Syndrome are the decreasing of muscle tone, small hands, small ears, small head and mouth, as well as wide and short hands with short fingers. Physical development for children affected by Patau Syndrome occurs more slowly than children without Patau syndrome. However, children affected by Patau Syndrome should still undergo regular physical activity, even though muscle development may occur more slowly.

792
Q

Ix for Trisomy 13 / Patau syndrome

A

Clinical findings

Fetal chromosome testing - trisomy 13

While many of the physical findings are similar to Edwards syndrome there are a few unique traits, such as polydactyly.

793
Q

Rx of Trisomy 13 / Patau syndrome

A

Medical management of children with Trisomy 13 is planned on a case-by-case basis and depends on the individual circumstances of the patient. Treatment of Patau syndrome focuses on the particular physical problems with which each child is born. Many infants have difficulty surviving the first few days or weeks due to severe neurological problems or complex heart defects. Surgery may be necessary to repair heart defects or cleft lip and cleft palate. Physical, occupational, and speech therapy will help individuals with Patau syndrome reach their full developmental potential. Surviving children are described as happy and parents report that they enrich their lives

794
Q

Complications / prognosis of Trisomy 13 / Patau syndrome

A

80% die within first year of life

In a retrospective Canadian study of 174 children with trisomy 13, median survival time was 12.5 days. One and ten year survival was 19.8% and 12.9% respectively

795
Q

Define trisomy 18 / Edwards syndrome

A

Edwards syndrome, also known as trisomy 18, is a genetic disorder caused by a third copy of all or part of chromosome 18. Many parts of the body are affected. Babies are often born small and have heart defects. Other features include a small head, small jaw, clenched fists with overlapping fingers, and severe intellectual disability.

796
Q

Epidemiology of trisomy 18 / Edwards syndrome

A

1 in 5k births
More females survive
Many of those affected die before birth
1 year survival 5-10%

797
Q

Aetiology of trisomy 18 / Edwards syndrome

A

Edwards syndrome is a chromosomal abnormality characterized by the presence of an extra copy of genetic material on the 18th chromosome, either in whole (trisomy 18) or in part (such as due to translocations). The additional chromosome usually occurs before conception.

798
Q

RFs for trisomy 18 / Edwards syndrome

A

Increased maternal age

799
Q

Sx of trisomy 18 / Edwards syndrome

A
Renal malformations
Structural heart - VSD, ASD, PDA
Omphalocele - intestines protruding outside body
Oesophageal atresia
Intelectual disabilities
Contractures from birth (athrogryposis)

Microcephaly, prominent occiput, low set malformed ears, ABNORMALLY SMALL JAW, micrognathia, cleft palate/lip, upturned nose, narrow eye lid. PALPEBRAL fissures, WIDE SPACED EYES, ptosis, clenched hands, underdeveloped thumbs, best radius, webbing of 2nd/3rd toes, rocker bottom/clubfoot, undescended vesicles in males.

FREQUENTLY SEE choroid plexus cysts in utero

800
Q

Rx of trisomy 18 / Edwards syndrome

A
Renal malformations
Structural heart - VSD, ASD, PDA
Omphalocele - intestines protruding outside body
Oesophageal atresia
Intelectual disabilities
Contractures from birth (athrogryposis)

Microcephaly, prominent occiput, low set malformed ears, ABNORMALLY SMALL JAW, micrognathia, cleft palate/lip, upturned nose, narrow eye lid. PALPEBRAL fissures, WIDE SPACED EYES, ptosis, clenched hands, underdeveloped thumbs, best radius, webbing of 2nd/3rd toes, rocker bottom/clubfoot, undescended vesicles in males.

FREQUENTLY SEE choroid plexus cysts in utero

801
Q

Rx of trisomy 18 / Edwards syndrome

A

Supportive

802
Q

Complications / prognosis of trisomy 18 / Edwards syndrome

A

Number of live birth per year estimated at 37

Major causes of death include apnea and heart abnormalities. It is impossible to predict an exact prognosis during pregnancy or the neonatal period

Half do not survive 1st week

Median lifespan 15 days

8-12% survive longer than 1 year

803
Q

Define trisomy 21 / downs syndrome

A

Down’s syndrome (DS), or trisomy 21, is the most common genetic form of intellectual disability, occurring in 1 in 691 births. In addition to having characteristic physical features, short stature, and hypotonia, individuals with DS have a distinctive cognitive and behavioural profile. They also have a higher frequency of associated medical conditions, including congenital heart defects, and gastrointestinal, thyroid, haematological, audiological, and visual abnormalities.

Most common genetic cause of cognitive or intellectual disability, with a prevalence of 1 in 691 births.

Characteristic physical features include brachycephaly with a flat occiput; epicanthal folds and upslanting palpebral fissures; Brushfield spots in the iris; low nasal bridge; low-set ears; broad neck; and small hands with transverse crease.

Higher frequency of congenital and acquired medical conditions, including congenital heart defects, audiological, vision, GI, haematological, and thyroid issues.

Global developmental delay, variable IQ ranging from mild to moderate intellectual disability, and age-related pattern of neurobehavioural phenotype.

Early interventional therapies (e.g., speech therapy, physiotherapy, occupational therapy) and individualised educational programmes/resources are essential to maximise the potential of a person with Down’s syndrome.

804
Q

Epidemiology of trisomy 21 / downs syndrome

A

Incidence = 1 in 691 births

Standard trisomy 21, the presence of an extra chromosome 21, is caused by chromosome non-disjunction and occurs in 95% of cases.

In about 4% to 5% of individuals with DS, the trisomy results from a chromosome translocation, and the remaining 1% of cases of DS are mosaic with a combination of typical and trisomic cells.

805
Q

RFs for trisomy 21 / downs syndrome

A

Advanced maternal age
Previous child with DS - 1% risk
Parental Karyotype with a translocation

806
Q

Aetiology of trisomy 21 / downs syndrome

A

Standard trisomy 21, the presence of an extra chromosome 21, is caused by chromosome non-disjunction and occurs in 95% of cases.

In about 4% to 5% of individuals with DS, the trisomy results from a chromosome translocation, and the remaining 1% of cases of DS are mosaic with a combination of typical and trisomic cells.

807
Q

Sx of trisomy 21 / downs syndrome

A
COMMON
RFS
Common physical findings 
- Brachycephaly with flat occiput
- Epicanthal folds
- Upslanting palpebral fissures
- Short nose
- Protruding tongue
- Small mouth
- Extra skin on neck
- Single palmar crease
- a wide space between first and second toes with vertical plantar creases; and hypoplasia of iliac wings
ALSO
Hypotonia
Congenital HD
Gross motor delay
Language delay
Cognitive disability 
Hearing loss
Autism spectrum disorder

Congenital GI: duodenal / anal stenosis/atresia
Constipation / hirschprungs
Feeding difficulties
Behavioural difficulties

808
Q

Ix for trisomy 21 / downs syndrome

A

Chromosomal karyotyping - trisomy 21, Robertsonian translocation, or mosaicism

CHECK
Echo, abdo xray, hearing test, TFTs, FBC, visual/dental/developmental,
Coeliac ABs
Polysomnography

809
Q

Rx of trisomy 21 / downs syndrome

A

50% have congential HD - evaluation by a paediatric cardiologist, including echocardiogram, is recommended in all newborns with DS

Early intervention therapies:
Developmental delay
Occupational therapy
SALT

Individualised education plan

810
Q

Prognosis / complications of trisomy 21 / downs syndrome

A

Av age 50-60yo

Complications:
Congenital HD
Duodenal atresia
Short stature
AML
ALL
Dementia
Myeloproliferative disorder 
Epilepsy 
Depression
Intellectual disability 
OSA
Hearing loss
Resp infections
Obesity 
Visual abnormalities/dental 
Behavioural problems
Hirschsprungs, coeliacs
Autsism SD
Antlantoaxial instability 
Joint dislocations
811
Q

A girl is born at term after a normal pregnancy and delivery. On examination she is noted to be hypotonic with hyper-extensible joints, small ears, and a small mouth. She has a protruding tongue, a broad neck, an upward slant to the eyes with epicanthal folds, and a single palmar crease.

A

Downs syndrome

812
Q

The result of an amniocentesis in a pregnant 39-year-old mother reveals a chromosomal karyotype of 47,XX,+21.

A

Downs syndrome

813
Q

DS is typically diagnosed during the newborn period based on the characteristic physical presentation. However, the diagnosis may be challenging in certain racial/ethnic groups or in individuals with mosaicism. For example, black children with DS typically may not have Brushfield spots, a protruding tongue, or extra nuchal skin. Children with DS diagnosed after the newborn period may present with developmental delay, hypotonia with hyper-extensible joints, or DS-associated medical conditions (e.g., atrioventricular septal defects, hearing loss, or Hirschsprung’s disease).

A

Downs syndrome

814
Q

Define Noonan syndrome

A

Noonan syndrome (NS) is a relatively common, autosomal-dominant inherited disorder that is predominantly characterised by short stature, subtle facial dysmorphisms, chest deformity, congenital heart disease, and variable degrees of developmental delay.

Coagulation defects, cryptorchidism (undescended testes) in men, and lymphatic dysplasia are not uncommon.

Relatively common, autosomal-dominant inherited disorder.

Characteristic phenotype includes short stature, chest deformity, congenital heart defects, and unusual facial features.

Boys frequently present with cryptorchidism and manifest delayed puberty.

Caused by activating mutations in multiple genes in the Ras/mitogen-activated protein kinase (MAPK) signal transduction pathway. The most commonly implicated gene is PTPN11.

Treatment focuses on the individual symptom, and may include surgery for undescended testes in boys, optimisation of cardiac function, and growth hormone treatment for short stature.

The majority of patients lead normal lives. Prognosis is largely dependent on the type and severity of cardiac disease, which may occur in 50% to 80% of cases.

The syndrome is typically caused by gain-of-function (activating) mutations multiple genes in the Ras/mitogen-activated protein kinase (MAPK) signal transduction pathway.

815
Q

Aetiology of Noonan syndrome

A

The syndrome is typically caused by gain-of-function (activating) mutations multiple genes in the Ras/mitogen-activated protein kinase (MAPK) signal transduction pathway.

816
Q

Epidemiology of Noonan syndrome

A

1 in 1000-2500
M=F
No ethnic predominance

817
Q

RFs for Noonan syndrome

A

STRONG
FHx - AUTOSOMAL DOMINANCE INHERITANCE

WEAK
Advanced paternal age

818
Q

Sx of Noonan syndrome

A

COMMON
Short stature
Dysmorphic face - wide-spaced and down-slanting eyes with vivid blue or blue-green irides; low-set, posteriorly rotated ears; an inverted triangular-shaped face with small chin; and a broad or webbed neck.

Undescended testes are presenting features in most boys (60% to 69%), and may be associated with delayed puberty.

Pulmonary valve stenosis and/or dysplasia, septal defects, and other less common structural cardiac defects are cardinal features of NS.

Male delayed puberty

Easy bruising/bleeding

UNCOMMON
20% have lymphatic anomaly 
Café au lait patches, lentigines, nevi, and keratosis are seen.
Sparse/absent eyebrows/lashes
Splenomegaly 

OTHER COMMON
During pregnancy, the most common features suggesting a diagnosis of NS are polyhydramnios (excess of amniotic fluid) and cystic hygroma (cystic lymphatic lesion).
Other features noted on ultrasound include scalp oedema, increased nuchal translucency, pleural or pericardial effusion, ascites, and/or hydrops (fluid in body cavities).

Chest - inferior excavatum and subtle superior carinatum

Developmental delay
Skeletal anomalies - cubitus valgus (an increased carrying angle of the forearm) is present in 50% of cases, and short fingers with blunt fingertips are seen in up to one third of cases.
Joint hyperextensibility occurs in 30% of cases.
Less common skeletal anomalies include talipes equinovarus (clubfoot), joint contractures, scoliosis, and radioulnar synostosis (fusion of radius and ulna).

Muscle weakness

819
Q

Ix for Noonan syndrome

A

ECG - wide QRS complexes, with a predominantly negative pattern in the left precordial leads (62%); left axis deviation, and giant Q waves
Echocardiogram - congenital HD

FBC
Coagulation profile (possible prolonged)
Molecular gene testing - May be necessary when the diagnosis is equivocal, or for family reasons (e.g., parental diagnosis, determination of the risk for recurrence, or reproductive planning).
Renal US - renal malformation is suspected (e.g., malformations such as duplex collecting system, distal ureteric stenosis, renal hypoplasia, unilateral renal agenesis, or unilateral renal ectopia); this may be present in up to 10% of cases
Abdo US - may show splenomegaly. (myelodyslasia)

820
Q

Rx of Noonan syndrome

A

Specialist assessment and consult

Cryptorchidism - Urology referral
Congenital heart - CV assessment and evaluation for surgery
Poor growth - recombinant growth hormone (somatropin) + endocrinology referral

821
Q

Prognosis / complications of Noonan syndrome

A

Majority can lead normal lives.

Prognosis is largely dependent on the type and severity of cardiac disease. This may occur in up to 75% of cases and is the most common cause of death for people with the syndrome.

Complications:
Failure to thrive
Hearing loss
Leukaemia
Solid tumours
Opthalmic abnormalities
Learning difficulties 
Seizures 
Musculoskeletal problems
AI disease (systemic lupus erythematosus, autoimmune thyroiditis, coeliac disease, primary antiphospholipid syndrome, autoimmune hepatitis, vitiligo, and autoimmune thyroiditis._
822
Q

A newborn boy is examined after delivery and found to have a murmur, cryptorchidism, a broad chest with widely spaced nipples, excess nuchal skin, and puffy hands and feet. The family feels that he looks a little different from their first child, with his tall forehead, small chin, and droopy eyelids. An echocardiogram is ordered and reveals pulmonary stenosis with some mild thickening of the ventricular walls.

A

Noonan

823
Q

A 4-year-old girl is referred to her paediatrician because of small stature, some delays in language development, and mild hypotonia. Physical examination reveals height below the third centile and weight at the third centile, while head circumference is at the fiftieth centile. There is cubitus valgus and a broad chest with widely spaced nipples. Speech developed at around age 2 years and is still slightly unclear, although the mother understands most words. Hearing appears clinically normal.

A

Noonan

824
Q

Congenital structural heart defects and hypertrophic cardiomyopathy may present in the antenatal period, the newborn period, or later in childhood. In pregnancy, polyhydramnios (excess of amniotic fluid), increased nuchal translucency, or a cystic hygroma (cystic lymphatic lesion) may be the first signs of Noonan syndrome, and hydrops fetalis (abnormal accumulation of fluid in body cavities) is an occasional finding. Short stature may be present at birth or postnatal in origin. Infants may present with failure to thrive and mild-to-severe feeding problems that usually resolve by 18 months. The facial features of NS are often subtle, but are perhaps most florid in newborns and adolescents. These features are difficult to recognise in adults, and many adults are only diagnosed after the birth of a child with more obvious features.

A

Noonan

825
Q

Define Prader-Willi

A

Prader–Willi syndrome (PWS) is a genetic disorder due to loss of function of specific genes. In newborns, symptoms include weak muscles, poor feeding, and slow development. Beginning in childhood, the person becomes constantly hungry, which often leads to obesity and type 2 diabetes. Also, mild to moderate intellectual impairment and behavioral problems are typical. Often, the forehead is narrow, hands and feet are small, height is short, skin is light in color, and most of the affected are unable to have children.

826
Q

Epidemiology of Prader-Willi

A

1 in 10-30k

827
Q

Aetiology of Prader-Willi

A

About 74% of cases occur when part of the father’s chromosome 15 is deleted.

In another 25% of cases, the person has two copies of chromosome 15 from their mother and none from their father.

As parts of the chromosome from the mother are turned off, they end up with no working copies of certain genes.

PWS is not generally inherited, but instead the genetic changes happen during the formation of the egg, sperm, or in early development

828
Q

RFs for Prader-Willi

A

NONE

829
Q

Sx of Prader-Willi

A

APPEARANCE:

Prominent nasal bridge
Small hands and feet with tapering of fingers
Soft skin, which is easily bruised
Excess fat, especially in the central portion of the body
High, narrow forehead
Thin upper lip
Downturned mouth
Almond-shaped eyes
Light skin and hair relative to other family members
Lack of complete sexual development
Frequent skin picking
Stretch marks
Delayed motor development

Uterus and birth:

Reduced fetal movement
Frequent abnormal fetal position
Occasional polyhydramnios (excessive amniotic fluid)
Often breech or caesarean births
Lethargy
Hypotonia
Feeding difficulties (due to poor muscle tone affecting sucking reflex)
Difficulties establishing respiration
Hypogonadism
Thin upper lip

Childhood:

Delayed milestones/intellectual delay
Excessive sleeping
Strabismus (crossed eyes)
Scoliosis (often not detected at birth)
Cryptorchidism
Speech delay
Poor physical coordination
Hyperphagia (overeating) begins between the ages of 2 and 8, and continues on throughout adulthood.
Excessive weight gain
Sleep disorders
Delayed puberty
Short stature
Obesity
Extreme flexibility

Adulthood:

Infertility (males and females)
Hypogonadism
Sparse pubic hair
Obesity
Hypotonia (low muscle tone)
Learning disabilities/borderline intellectual functioning (but some cases of average intelligence)
Prone to diabetes mellitus
Extreme flexibility

5%: IQ above 85 (high to low average intelligence)
27%: IQ 70–85 (borderline intellectual functioning)
39%: IQ 50–70 (mild intellectual disability)
27%: IQ 35–50 (moderate intellectual disability)
1%: IQ 20–35 (severe intellectual disability)
<1%: IQ <20 (profound intellectual disability)

PWS is frequently associated with a constant, extreme, ravenous, insatiable appetite, which persists no matter how much the patient eats, often resulting in morbid obesity.

830
Q

Ix for Prader-Willi

A

It is traditionally characterized by hypotonia, short stature, hyperphagia, obesity, behavioral issues (specifically obsessive–compulsive disorder-like behaviors), small hands and feet, hypogonadism, and mild intellectual disability

Genetic testing - DNA-based methylation testing to detect the absence of the paternally contributed PWS/AS region on chromosome 15q11-q13.

831
Q

Rx of Prader-Willi

A

No cure

Physio - to improve muscle strength
SALT
Physical activity
May need psych input for behavioural problems.

Prescription of daily recombinant GH injections are indicated for children with PWS. GH supports linear growth and increased muscle mass, and may lessen food preoccupation and weight gain.

NB beware of OSA

832
Q

Prognosis of Prader-Willi

A

People with PWS usually reach adulthood and are able to function in a group home setting, performing vocational work, or attending community college classes.

However, they do need a significant amount of support from their families and from school, work, and residential service providers. Even those with IQs in the normal range need lifelong diet supervision and protection from food availability.

833
Q

Complications of Prader-Willi

A

Complications that could affect the quality of life and potentially shorten life expectancy include those relating to hypogonadism, behavioral or psychological issues, and morbid obesity.

834
Q

Define Turner’s syndrome

A

A chromosomal disorder involving a complete or partial absence of the second sex chromosome in phenotypic females, with characteristic features such as short stature and premature ovarian failure.

Chromosomal abnormality involving a complete or partial absence of the second sex chromosome, occurring in approximately 1 in 2500 live female births.

Characteristic clinical features include short stature and premature ovarian failure in a phenotypic female.

Variable phenotype; obvious stigmata such as neck webbing affect only 20% to 30% of patients.

Haplo-insufficiency for X- or Y-encoded pseudo-autosomal genes largely responsible for the phenotype.

Intelligence is normal, with verbal skills generally greater than performance or visual-spatial skills.

835
Q

Epidemiology of Turner’s syndrome

A

1 in 2500

All races
<10% diagnosed antenatally

20% are detected in infancy owing to the presence of lymphoedema, neck webbing, or congenital heart defects.

The largest proportion of detection is at 10 to 16 years of age, owing to a combination of marked short stature and delayed puberty. Another 10% are diagnosed in adulthood, owing to secondary amenorrhoea.

836
Q

Aetiology of Turner’s syndrome

A

Sporadic

Loss of all or a part of a sex chromosome during early embryogenesis usually results in the development of a female fetus with developmental anomalies. Monosomy for the Y-chromosome is non-viable. Those 46,XY fetuses that lose a Y chromosome relatively late in development may have enough Y chromosome input to drive partial or complete male development. However, these patients are identified as males, and are considered under the diagnosis of Disorders of Sexual Differentiation rather than Turner’s syndrome. Males with 45,X cell lines have variable degrees of short stature, delayed puberty or infertility, and risk for cardiovascular disorder.

837
Q

RFs for Turner’s syndrome

A

None

838
Q

Sx of Turner’s syndrome

A

COMMON

Poor growth
Short stature
Delayed/absent puberty
Primary amenorrhoea
Congenital HDs: Aortic coarctation and bicuspid aortic valves are the most common abnormalities.
Skeletal: wide carrying angle, scoliosis, short fourth metacarpals or metatarsals, and, rarely, a Madelung deformity (prominent distal ulna)

UNCOMMON

Webbed neck
Peripheral lymphoedema

OTHER COMMON

Low-set or malrotated ears, down-sloping eyes, ptosis or hooded eyes, and low posterior hairline are pathognomonic. A high-arched palate, although common, is not specific for Turner’s syndrome.

Secondary amenorrhoea
Multiple melanocytic naevi
Recurrent otitis media
Systolic ejection murmur/click
Poor social skills

UNCOMMON
Upper extremity HTN (coarc)
Dystrophic/hyperconvex nails

839
Q

Ix for Turner’s syndrome

A

Karyotyping - at least 10% of the cells with a complete or partial loss of a sex chromosome. Abnormalities include non-mosaic 45,X; mosaic 45,X (proportion of cells with a normal chromosome constitution); and fragmented X or Y (Xp deletions, isoXq chromosomes, Xq deletions, and ring X or Y chromosomes with substantial interstitial deletions).

CONSIDER

Echo/MRI- bicuspid aortic valve; coarctation of the aorta; dilated aorta; partial anomalous pulmonary veins; left heart hypoplasia

FSH - high
Renal US - horseshoe or single kidney, or duplicated collecting system
TFTs - Autoimmune thyroid disease (Hashimoto’s thyroiditis and, less commonly, Graves’ disease) is a common complication in Turner’s syndrome.
LFTs - About 30% to 40% of patients develop a complication of ‘Turner’s hepatitis’.
Fasting glucose + HBA1c
Serum lipids - elevated (common in turners)
IgA + IgA TTG - coeliac common

840
Q

Rx of Turner’s syndrome

A

Supportive - healthy diet, oestrogen therapy, statin Rx fo high risk CVD, surveillance of congenital HDs

Annual screening should be carried out, including TFTs for autoimmune hypothyroidism, LFTs to screen for ‘Turner’s hepatitis’, BP measurements for HTN, fasting glucose to identify patients with diabetes mellitus (fasting glucose and HbA1c to identify patients likely to develop diabetes), lipids to monitor dyslipidaemia, and hearing screen to identify those with sensorineural or other types of deafness. Tissue transglutaminase IgA (if normal IgA levels) should be measured every 3 to 5 years to screen for coeliac disease. Baseline bone mineral density should be assessed at the transition to adult care and re-evaluated at menopause or sooner if indicated.

Poor growth - somatropin recombinant / oxandrolone

Low dose oestrogen to induce pubertal development. - estradiol transdermal / estradiol

ADD cyclic progesterone once there is breakthrough bleeding

CV specialist assessment

Women with Turner’s syndrome should normally continue on HRT after induction and development of puberty. HRT should continue until about 50 years of age, after which further oestrogen therapy may be given depending on individuals’ risk/benefit considerations.

Low bone mass should be managed with oestrogen replacement hormone therapy, adequate calcium in diet, normal vitamin D levels, and weight-bearing activities, not bisphosphonates.

Most women with Turner’s syndrome are infertile, but spontaneous menses and pregnancies may occur in 2% to 3%.

PREGNANCY COUNSELLING
There is increased incidence of a fetus with aneuploidy when using their own oocytes. Pregnancy exacerbates underlying metabolic, hypertensive, and cardiovascular problems and may be catastrophic for women with Turner’s syndrome; therefore, girls over the age of 10 years need close monitoring with regard to potential fertility and education on reproductive issues and sexual behaviours.

Moreover, infertile adults with Turner’s syndrome may be interested in pregnancy via assisted reproduction using donor oocytes. Such pregnancies may have an unusually high risk for maternal death from aortic dissection or rupture, and from pre-eclampsia and its complications.

841
Q

Prognosis of Turner’s syndrome

A

Epidemiological studies based on European registry data collected mainly in the late 20th century consistently report increased morbidity and mortality in adults with Turner’s syndrome, owing to complications of congenital heart disease, ischaemic heart disease, diabetes mellitus, and osteoporosis.

842
Q

Complications of Turner’s syndrome

A
T2DM
Dyslipidaemia
Hearing loss
Osteoporosis
AD
HTN
Neurocognitive defects
Hashimotos
Graves
Hepatitis
Gonadoblastoma
843
Q

Define Kleinfelter’s

A

Klinefelter syndrome (KS) also known as 47,XXY or XXY, is the set of symptoms that result from two or more X chromosomes in males.

The primary features are infertility and small testicles.

Often, symptoms may be subtle and many people do not realize they are affected. Sometimes, symptoms are more prominent and may include weaker muscles, greater height, poor coordination, less body hair, breast growth, and less interest in sex. Often it is only at puberty that these symptoms are noticed.

Intelligence is usually normal; however, reading difficulties and problems with speech are more common.

Symptoms are typically more severe if three or more X chromosomes are present (XXXY syndrome or 49,XXXXY).

844
Q

Epidemiology of Kleinfelter’s

A

Random

1-2 per 1000

845
Q

Aetiology of Kleinfelter’s

A

Older mother

846
Q

RFs for Kleinfelter’s

A

The extra chromosome is retained because of a nondisjunction event during paternal or maternal meiosis I (gametogenesis). Nondisjunction occurs when homologous chromosomes, in this case the X and Y or two X sex chromosomes, fail to separate, producing a sperm with an X and a Y chromosome or an egg with two X chromosomes. Fertilizing a normal (X) egg with this sperm produces an XXY offspring (Klinefelter). Fertilizing a double X egg with a normal sperm also produces an XXY offspring (Klinefelter).

847
Q

Sx of Kleinfelter’s

A
Weak muscles
Reduced strength
Tall height
Reduced muscle control
Less facial / pubertal hair
Wider hips
May develop gynaecomastia

Infertility (hypogonadism)

XXY males are also more likely than other men to have certain health problems that typically affect females, such as autoimmune disorders, breast cancer, venous thromboembolic disease, and osteoporosis

XXY males may sit up, crawl, and walk later than other infants; they may also struggle in school, both academically and with sports

848
Q

Ix for Kleinfelter’s

A

The standard diagnostic method is the analysis of the chromosomes’ karyotype on lymphocytes

In the past, the observation of the Barr body was common practice, as well.

Other methods may include research of high serum levels of gonadotropins (follicle-stimulating hormone and luteinizing hormone), presence of azoospermia, determination of the sex chromatin

10% detected prenatally via chorionic villus sampling or amniocentesis.

849
Q

Rx of Kleinfelter’s

A

Testosterone replacement
- implants possible

Surgical breast removal

The use of behavioral therapy can mitigate any language disorders, difficulties at school, and socialization. An approach by occupational therapy is useful in children, especially those who have dyspraxia.

By 2010, over 100 successful pregnancies have been reported using IVF technology with surgically removed sperm material from males with KS.

Microdissection testicular sperm extraction in adult men with Klinefelter syndrome reported success rates up to 45%

850
Q

Prognosis / complications of Kleinfelter’s

A

A 1985 publication identified a greater mortality mainly due to diseases of the aortic valve, development of tumors, and possible subarachnoid hemorrhages, reducing life expectancy by about 5 years.

Later studies have reduced this estimated reduction to an average of 2.1 years

851
Q

Define Toddler’s Diarrhoea

A

Toddler’s diarrhoea is a common cause of persistent (chronic) diarrhoea in young children. It mainly affects children between the ages of 1 and 5 years and is more common in boys. Toddler’s diarrhoea is not serious and the child is well. The diarrhoea will go as the child becomes older. The diet of young children is often not ideal and is thought to contribute to the cause. The diarrhoea will often stop if the child: has a good amount of fat in the diet (whole milk, etc); does not drink too much fruit juice or squash; has meals that include a normal amount of fibre (but not a high-fibre diet).

852
Q

Epidemiology of Toddler’s Diarrhoea

A

1-5yos

M>F

853
Q

RFs for Toddler’s Diarrhoea

A

Poor diet
Low fruit and veg
Low fat
Low fibre

854
Q

Aetiology of Toddler’s Diarrhoea

A

It is thought that the balance of fluid, fibre, undigested sugars and other undigested foods that reach the colon may be upset in affected children. This can increase the amount of fluid (water) that is kept in the colon rather than being absorbed into the body. In young children, even a slight increase in fluid left in the colon can cause stools to become more frequent and runny than normal. As the child grows, the colon becomes more efficient and the condition goes.

Toddler’s diarrhoea is not due to poor absorption (malabsorption) of food or to a serious bowel problem. It is also not due to an intolerance of a type of food.

855
Q

Sx of Toddler’s Diarrhoea

A
3-10 loose stools per day
Visualisation of vegetable in food
Smelly + pale
Soon after eating
Mild tummy pain (unusual)
NORMAL GROWTH
NORMAL PLAY/UNBOTHERED
NORMAL EXAM
856
Q

Ix for Toddler’s Diarrhoea

A

Clinical

Stool sampling possible

857
Q

Rx of Toddler’s Diarrhoea

A

Usually dissipates by age 5

4 Fs diet

FAT
More common in children eating a low fat diet. The diet of preschool children should have about 35-40% fat

FLUID + FRUIT JUICE
Do not give children too much fruit juice or squash.
Fruit juices contain various sugars (carbohydrates). Some types of sugar are not digested or absorbed and so get to the large bowel (colon). Here they may act to keep water in the bowel and cause watery stools. Clear apple juice seems to be the worst, as it contains a lot of certain sugars. Cloudy juices that contain some fibre are not as bad.
The sugar in juice and squash contains a lot of calories. This can reduce the appetite for normal meals. Therefore, the child tends to eat less fat and fibre at normal mealtimes. Some children seem to obtain most of their daily calories from juice and don’t eat very much solid food.

FIBRE
- very low- or high-fibre intakes may make symptoms worse in some children

858
Q

Define UTI in children

A

Paediatric urinary tract infection (UTI) is defined as a common bacterial infection involving the lower urinary tract (cystitis), the upper urinary tract (pyelonephritis), or both, causing illness in children. Recognising and treating these infections promptly and accurately is important. UTI is associated with pyelonephritis, which has potential sequelae, including renal scarring. Untreated UTI can also lead to hypertension and end-stage renal disease.

Asymptomatic bacteriuria is the presence of bacteria in urine obtained in asymptomatic children (usually girls) on routine screening or incidentally during other investigations. Antibiotic treatment does not seem to help eliminate the bacteria, reduce recurrence, or prevent kidney damage.

A common diagnosis among infants and children; if missed, can lead to renal scarring, hypertension, and end-stage renal disease.

Non-specific signs and symptoms may herald UTI, and practitioners should have a high index of suspicion in a febrile infant.

An appropriately obtained urine specimen can confirm the diagnosis and pathogen; urine culture and antimicrobial susceptibility testing will define the appropriate antibiotic for treatment.

Of children <6 years of age with first-time UTI, 25% have vesicoureteral reflux (VUR) and, of those, 25% have significant VUR (grade IV or V), placing them at risk for renal scarring.

Infection can recur in young infants and those with voiding dysfunction in the absence of urinary reflux.

859
Q

Epidemiology of UTI in children

A

8% girls, 2% of boys have had a UTI by age 7

8% of children can verbalise Sx

Prevalence in children with undifferentiated fever = 5%

W>B

White girls with fever >39 w/o potential source - 30% will have UTI

860
Q

Aetiology of UTI in children

A

Bacterial infections are the most common cause.

Escherichia coli is the cause of 85% to 90% of paediatric UTIs.
Proteus mirabilis may be found in 30% of boys with uncomplicated cystitis.
Staphylococcus saprophyticus is found in adolescents of both sexes with acute UTI.
Staphylococcus aureus is the most common cause of renal abscess.
Pseudomonas species, Serratia marcescens , Citrobacter species, and Staphylococcus epidermidis may cause low-virulence infections in patients with malformation or dysfunction of the urinary tract.
Other organisms causing UTI in children include Klebsiella aerogenes and Enterococcus species.
Adenovirus infections are a rare cause of UTI.

While bacteria are the most common infectious agents causing UTIs worldwide, co-infection with parasites, such as Schistosoma species, may significantly contribute to morbidity of the genitourinary tract.

861
Q

RFs for UTI in children

A
STRONG
Age <1
Female
White
Uncircumcised 
Previous UTI 
Voiding dysfunction - frequency/urgency/withholding manoeuvres
Vesicoureteral reflux
Sexual activity
No Hx Breast feeing
Obstructive anomalies / Hx surgery 

WEAK
Constipation
Immunosuppression

862
Q

Sx of UTI in children

A
COMMON
RFs
Fever >39
Irritability 
Poor feeding
Suprapubic tenderness

UNCOMMON
Costovertebral angle tenderness

OTHER COMMON
Foul smelling urine
Dysuria
Increased frequency
Abdominal/flank pain
UNCOMMON
Vomiting
Ill appearance
Gross haematuria 
New-onset urinary incontinence
863
Q

Ix for UTI in children

A

Urine Dip - positive leukocyte esterase and/or positive nitrite
Urine MC+S
Blood culture - positive for infecting organism

CONSIDER
Fungal urine culture if immunosuppressed
Renal US to look for any anatomical abnormalities of the urinary tract
DMSA scan - pyelonephritis or renal scarring: focal or diffuse areas of decreased uptake
Voiding cystourethrogram - if vesicoureteral reflux is present: contrast seen ascending out of the bladder into the upper urinary tract

CT/MRI possible in extreme situations, ie pyrexia of unknown origin to search for a renal abscess and phlegm

864
Q

Rx of UTI in children

A

Neonates/infants <6w
IV ampicillin + gent / ampicillin + cefotaxime

Others: STABLE
Oral ABX: Cefixime 16 mg/kg/day orally given in divided doses every 12 hours on day one, followed by 8 mg/kg/day

OR

amoxicillin/clavulanate

Or

trimethoprim/sulfamethoxazole

IF UNSTABLE
Ampicillin + Gentamicin IV
Ciprofloxacin

865
Q

Complications of UTI in children

A
ABx rash - STOP
Renal abscess
Phlegmon (lobar nephronia)
Sepsis in adult
ABx related colitis
Renal scarring
HTN
CKD
866
Q

Prognosis of UTI in children

A

The prognosis is good overall, although there is a risk of recurrent infection after a first UTI. In patients with urinary tract comorbidity, progression of renal dysfunction is likely.

867
Q

A 3-week-old uncircumcised boy presents with fever to 38.5°C (101.3°F) and poor feeding. He is fussy, with poor capillary refill. Mild jaundice is noted. His abdomen is distended and diffusely tender.

A

UTI

868
Q

An uncommon presentation of UTI in young infants may be with late-onset jaundice or failure to thrive. Toddlers rarely present with urinary symptoms; non-specific symptoms are much more common.

A

UTI

869
Q

An uncommon presentation of UTI in young infants may be with late-onset jaundice or failure to thrive. Toddlers rarely present with urinary symptoms; non-specific symptoms are much more common.

A

UTI

870
Q

Define biliary atresia

A

Biliary atresia is a progressive idiopathic, necroinflammatory process that may involve a segment or the entire extrahepatic biliary tree. Even with appropriate, timely surgical intervention, it is often an unrelenting inflammatory process.

A serious disease requiring prompt early diagnosis preferably before 6 to 8 weeks of age.

A fibro-obliterative obstruction of the extrahepatic biliary tree progressing to intrahepatic ducts, which can develop in utero or during the neonatal period.

Presentation may include jaundice, pale stools, or hepatomegaly.

Biliary atresia should be excluded in any baby with jaundice associated with pale stools, with jaundice persisting beyond 14 days of age, or if the conjugated bilirubin is higher than 34.2 micromol/L (>2 mg/dL) or 20% of the total.

Hepatoportoenterostomy is warranted in infants less than 100 days of age and without evidence of end-stage liver disease, but many go on to require liver transplantation within a few years.

Most common complications or sequelae are growth failure, portal hypertension, cholangitis, and ascites. Long-term management focuses on optimisation of growth and nutrition.

This condition is fatal without surgery and is the leading indication for paediatric liver transplantation. At least 70% of children with biliary atresia will undergo liver transplantation, 50% of them by 2 years of age.

871
Q

Epidemiology of biliary atresia

A

1 in 18k births in europe
F>M
Highest prevalence in asia

It is the most common cause of neonatal jaundice requiring surgical intervention and the most common indication for liver transplantation in children.

872
Q

Aetiology of biliary atresia

A

Unknown. Several theories:

Viral infection/toxin exposure: numerous studies exploring a viral aetiology have focused on reovirus, rotavirus, and other hepatotropic viruses.

Defects in morphogenesis: potentially caused by a toxin at a critical time in embryological development.

Genetic predisposition: the condition is unlikely to be inherited by Mendelian genetics, but genetic factors likely contribute. The increased association with histocompatibility antigen HLA-B12 (haplotypes A9-B5 and A28-B35) and its early onset suggest a genetic susceptibility to an acquired insult.

Defects in antenatal circulation: bile ducts receive their blood supply exclusively from the hepatic arterial circulation.

Immune or autoimmune dysregulation: animal models of the condition support the role of autoimmunity. Autoreactive T cells specific to bile duct epithelia are sufficient to produce bile duct inflammation in mice, causing a similar phenotype.

873
Q

RFs for biliary atresia

A
WEAK
Genetic predispostion
Viral infection
Environmental / behavioural exposures
Maternal DM
874
Q

Sx of biliary atresia

A
COMMON
Neonatal jaundice
Acholic stool (white->tan->yellow)
Dark urine
Bruising

UNCOMMON
Hepatomegaly
Ascites

875
Q

Ix for biliary atresia

A

1st:
Serum total + conj bilirubin - conjugated bilirubin >34.2 micromol/L (>2.0 mg/dL) or >20% of total bilirubin
Newborn Guthrie/screen - usually normal (but CF may coexist)
PT time / INR - N
FBC - in advanced disease; low platelets, low WBC
LFTs - elevated GGT supports biliary disease
Abdo US - liver usually normal texture, possibly enlarged, unlikely ductal dilation; absent or multiple spleen; ascites, triangular cord sign possible

CONSIDER
Hepatobiliary scintigraphy
Liver biopsy
Cholangiogram
ERCP
876
Q

Rx of biliary atresia

A

Without End-stage liver disease:

Hepatoportoenterostomy

Ursodeoxycholic acid: 20-30 mg/kg/day orally given in 2 divided doses. Thought to be hepatoprotective and facilitates bile flow.

Liver transplantation

End stage liver disease:

Liver transplantation

ONGOING
ABx prophylaxis for first year of life
Vitamin supplementation - ADEK

877
Q

Complications of biliary atresia

A
Growth failure
Cholangitis
Portal HTN
GI bleed
Vitamin deficiency
Bone fractures
Osteomalacia
Osteopenia
Ascites
Bile lakes/bilomas
Spontaneous bacterial peritonitis
Xanthoma
Hepatopulmonary HTN
878
Q

Prognosis of biliary atresia

A

First-line treatment is hepatoportoenterostomy (HPE), which provides transplant-free survival for at least 2 years in about half of children.

The focus of care following HPE is fat-soluble vitamin replacement, nutrition rehabilitation, prevention of cholangitis, and minimisation of the sequelae of portal hypertension. Overall, it is thought that 70% or more of affected children will ultimately require liver transplantation.

879
Q

A 6-week-old girl of African-American origin is seen at routine follow-up by the paediatrician. No specific concerns are raised by the mother. The baby was full term and average weight and length for gestational age. She is taking breast milk without problems. On physical examination there is faint scleral icterus and the urine in the nappy was yellow staining

A

Biliary atresia

880
Q

Accurate and rapid diagnosis may be problematic, as symptoms and signs may be difficult to spot. Presentation may include prolonged neonatal jaundice, pale stools, and hepatomegaly. Jaundice may be difficult to detect in dark-skinned infants. Mild scleral icterus may be the only noticeable feature. There are few clues in the history unless the mother comments on the pale colour of the child’s stool or notices jaundice.

A

Biliary atresia

881
Q

Define oesophageal atresia and tracheo-oesophageal fistula

A

Oesophageal atresia (OA) and tracheo-oesophageal fistula (TOF) are congenital malformations that result from defective separation of the oesophagus and trachea. A blind-ending upper oesophageal pouch with a fistulous connection between the distal oesophageal segment and the trachea is the most common type.

Occurs in 1 of every 2500 to 4500 live births.

There are no known predisposing factors; however, the condition often presents with other congenital anomalies, including VACTERL complex (vertebral defects, anorectal anomalies, cardiac defects, tracheo-oesophageal abnormalities, radial and renal abnormalities, and limb anomalies).

Surgical intervention is required as a newborn.

Patients may have long-term complications, such as GORD, tracheomalacia, and chronic respiratory infections.

882
Q

Epidemiology of oesophageal atresia and tracheo-oesophageal fistula

A

1 in 2500-4500 births

The majority of infants have Gross type C, comprising 85% to 90% of all cases.

The second most common type is a pure atresia (Gross A), which comprises 4% to 7% of all cases. Gross type D occurs 3% of the time. A much rarer form, the H-type fistula (Gross E), occurs in approximately 2% to 3% of cases. This type of fistula may be undiagnosed until later in life. Type B occurs only 1% of the time.

883
Q

Aetiology of oesophageal atresia and tracheo-oesophageal fistula

A

Abnormal separation during 4th week in utero.

Type A

Pure atresia (4% to 7%)

Type B

Proximal fistula with distal atresia (1%)

Type C

Proximal atresia with distal fistula (85% to 90%)

Type D

Proximal and distal fistula (3%)

Type E

H-type fistula (2% to 3%)
Tracheo-oesophageal fistula without oesophageal atresia

884
Q

RFs for oesophageal atresia and tracheo-oesophageal fistula

A

WEAK
Trisomy 18 (ed) or 21
FHx

885
Q

Sx of oesophageal atresia and tracheo-oesophageal fistula

A

COMMON
Maternal Hx of polyhydramnios
Inability to swallow secretions
Inability to pass NG tube

OTHER COMMON
Laboured respiratory 
Coughing
Choking
VACTERL association

UNCOMMON
Cyanosis

886
Q

Ix for oesophageal atresia and tracheo-oesophageal fistula

A

Antenatal US / MRI diagnoses 37%
Xray chest/abdo - nasogastric tube coiled in upper pouch

If suspect H type fistula patients present with evidence of recurrent aspiration. Perform barium swallow / oesophagoscopy - contrast in trachea / fistula visualised

887
Q

Rx of oesophageal atresia and tracheo-oesophageal fistula

A

TYPE A
1 - Stabilisation and gastrostomy (enteral feeds, allows oesophagus to grow, repaired at 3months)
2 - Oesophageal replacement

TYPE B+D
Suction catheter and surgery (performed in 24-48hrs life)

TYPE C
Stabilisation and surgical correction

TYPE E
NBM + surgical division of fistula

888
Q

Complications of oesophageal atresia and tracheo-oesophageal fistula

A
Failure to thrive
GORD, dysphagia, dysmotility
Chest wall malformation/scoliosis
Oesophageal stricture
Tracheomalacia
889
Q

Prognosis of oesophageal atresia and tracheo-oesophageal fistula

A

Patients with isolated OA/TOF without commonly associated congenital abnormalities (i.e., cardiac and chromosomal) have a good prognosis and survive to live healthy lives. Motility disorders and respiratory complications may occur.

890
Q

A newborn presents at birth with mild respiratory distress and increased oral secretions. An attempt to pass a suction catheter meets with resistance. An attempt to pass a replogle tube also meets with resistance.

A

Oesophageal atresia and tracheo-oesophageal fistula

891
Q

Up to 37% of patients are now diagnosed antenatally. [2] Fetal ultrasound shows polyhydramnios and often no stomach bubble.

If the infant presents with a pure atresia (no fistula), the initial findings at delivery are similar. The nasogastric tube coils in the upper oesophageal pouch; however, there is no air in the bowel as there is no connection to the airway. Associated anomalies may be present, such as cardiac, renal, limb, and spinal abnormalities.

A

Oesophageal atresia and tracheo-oesophageal fistula

892
Q

Define cleft lip and palate

A

An oro-facial cleft is an abnormal opening secondary to developmental failure in utero. Cleft lip with or without cleft palate can be unilateral or bilateral, and occurs in a variety of combinations with variable expression in the nose, lip, alveolus, and primary and secondary palates. Diminutive expressions of cleft lip and/or palate are known as microform, occult, minor, or forme fruste (aborted form).

Oro-facial clefts are among the most common birth defects.

Cleft lip with or without cleft palate is approximately twice as common as isolated cleft palate.

The majority of cleft lip deformities are associated with a varying degree of nasal deformity.

Oro-facial clefts can occur in isolation or as a component of an identifiable syndrome.

The aetiology of cleft lip and palate involves a variety of genetic and environmental factors that result in variable expressions of oro-facial clefting.

Comprehensive care involves a multi-disciplinary cleft team with specialists from fields such as facial plastic surgery, genetics, nursing, speech-language pathology, orthodontics, dentistry, oral surgery, audiology, and paediatrics.

893
Q

Epidemiology of cleft lip and palate

A

1 in 600-750 births

Two-thirds of oro-facial clefting involve the lip and/or palate, and nearly one third involve the palate alone. The majority of cleft lip and/or palate cases are unilateral (80%), and are more common on the left side. Midline clefts of the nose and/or lip are rare deformities.

Isolated cleft palate occurs twice as often in females, while cleft lip and/or palate occurs more frequently in males.

894
Q

Aetiology of cleft lip and palate

A
Genetic factors
More than 300 associated syndromes, EG Pierre robins
Environmental factors:
Teratogens (eg phenytoin)
Nutritional deficiencies 
Perinatal multivitamin non-adherence
Maternal tobacco / alcohol
895
Q

RFs for cleft lip and palate

A
STRONG
Genetic predisposition 
Dysmorphic syndrome 
Anticonvulsant drugs
Maternal tobacco use

WEAK
Maternal alcohol consumption
Folic acid deficiency

896
Q

Sx of cleft lip and palate

A
Visualisation of:
Bilateral cleft lip/palate
Unilateral cleft ip/palate
Isolated cleft palate
Microform cleft lip 
Isolated sub-mucous cleft palate - Identified by a diastasis of the midline palatal musculature (zona pellucida), a notched hard palate on palpation, and a bifid uvula
\+ve AN US
897
Q

Ix for cleft lip and palate

A

Audiogram - hearing loss

898
Q

Rx of cleft lip and palate

A
Specialised feeding until 3 months age
Pre-surgical lip taping from 3-7 months
Corrective surgery 10-14 months
SALT
Skeletal maturity: Orthodontics and orthognathic surgery for dentofacial malocclusion
899
Q

Complications / prognosis of cleft lip and palate

A
Failure to thrive
Oro-nasal fistula
Wound infection
Speech impairment
Tympanic membrane perforation
Palatal fistula 

A child born with a cleft lip with or without cleft palate will require a multi-disciplinary approach to management throughout his or her development. In the absence of an associated syndromic condition; however, children with cleft lip and/or palate can have excellent aesthetic and functional outcomes.

One study found that boys with right-sided cleft lips had more brain structure abnormalities seen on MRI than did those with left cleft lips or healthy controls, suggesting that both gender and the side of the cleft lip may provide insights into the co-existing neurodevelopmental differences of children with unilateral cleft lip.

900
Q

An expectant mother (gravida 2, para 1) presents to a cranio-facial clinic following an antenatal ultrasound at 18 weeks’ gestation that suggests the possibility of a cleft lip in the fetus.

A

Cleft lip and palate

901
Q

A newborn infant with a bilateral cleft lip and palate who has passed newborn hearing screening presents with difficulty nursing and feeding using a bottle.

A

Cleft lip and palate

902
Q

Define congenital diaphragmatic hernia

A

Congenital diaphragmatic hernia (CDH) is a birth defect of the diaphragm. The most common type of CDH is a Bochdalek hernia; other types include Morgagni hernia, diaphragm eventration and central tendon defects of the diaphragm. Malformation of the diaphragm allows the abdominal organs to push into the chest cavity, hindering proper lung formation.

The Bochdalek hernia, also known as a postero-lateral diaphragmatic hernia, is the most common manifestation of CDH, accounting for more than 95% of cases.

It involves three major defects:

A failure of the diaphragm to completely close during development
Herniation of the abdominal contents into the chest
Pulmonary hypoplasia

903
Q

Epidemiology of congenital diaphragmatic hernia

A

Mortality of 40-62%

1-5/10k

904
Q

Aetiology of congenital diaphragmatic hernia

A

It involves three major defects:

A failure of the diaphragm to completely close during development
Herniation of the abdominal contents into the chest
Pulmonary hypoplasia

905
Q

Sx of congenital diaphragmatic hernia

A

Often diagnosed before birth

Respiratory failure due to pulmonary HTN + Hypoplasia

Dec lung volume

Presence of abdominal contents in chest

906
Q

Ix for congenital diaphragmatic hernia

A

US in utero usually

907
Q

Rx of congenital diaphragmatic hernia

A

Orogastric tube
Intubation
ECMO can be used (reduces PHTN)

Hernia stitched closed / patch used

Organs restated

Diaphragm eventration is typically repaired thoracoscopically, by a technique called plication of the diaphragm.

908
Q

Complications / prognosis of congenital diaphragmatic hernia

A

CDH is a life-threatening pathology in infants and a major cause of death due to two complications: pulmonary hypoplasia and pulmonary hypertension

Congenital diaphragmatic hernia has a mortality rate of 40–62%, with outcomes being more favourable in the absence of other congenital abnormalities. Individual rates vary greatly dependent upon multiple factors: size of hernia, organs involved, additional birth defects, and/or genetic problems, amount of lung growth, age and size at birth, type of treatments, timing of treatments, complications (such as infections) and lack of lung function.

909
Q

Define cryptorchidism

A

Cryptorchidism, or undescended testis, is when one or both testes are not present within the dependent portion of the scrotal sac.

Diagnosis is made on physical examination when one or both testes are not present within the dependent portion of the scrotal sac.

Approximately 70% of cryptorchid testes are palpable within the upper portion of the scrotum or inguinal canal, whereas the other 30% are not palpable, suggesting either an intra-abdominal location, testicular nubbin, or anorchia.

Referral to a specialist should be made by 6 months corrected gestational age and surgical correction should be performed within the next year. Testes that remain undescended by 6 months are unlikely to descend spontaneously.

Boys with newly acquired (ascended) testicles should be referred to a surgical specialist.

Delayed or lack of treatment has been associated with a higher incidence of testicular cancer and reduced fertility from the affected testis.

Immediate specialist consultation should be obtained in any patient with bilateral non-palpable testicles.

Patients affected with both cryptorchidism and hypospadias should be screened with karyotype for a disorder of sex development (DSD).

910
Q

Epidemiology of cryptorchidism

A

2-8% WW
Decreases to 1-2% (may spontaneously resolve with the neonatal peak of testosterone by 3 months)

Two-thirds of cases are unilateral, while the remainder are bilateral.

911
Q

Aetiology of cryptorchidism

A

Unknown

Hormonal: patients may have abnormalities in the pathways/signalling of testosterone, mullerian inhibiting substance, insulin-like 3 hormone or its receptor LGR8, epidermal growth factor, and/or oestrogens.

Environmental or maternal toxins: organochlorines, environmental oestrogens, phthalate esters, and pesticides have all been linked to increased risk of cryptorchidism, though current literature is inconclusive.

Maternal alcohol consumption, analgesic consumption, and smoking have also been associated with an increased risk, though these data are conflicting.

Gestational diabetes and/or obesity may be related to the development of cryptorchidism.

Genetic: up to 23% of cases have been associated with familial clustering, suggesting an underlying genetic mutation as the aetiology in these patients.

Mechanical: problems with development of the gubernaculum or cremasteric muscle fibres, a patent processus vaginalis, or impaired intra-abdominal pressure have also been hypothesised to contribute to cryptorchidism.

Neuromuscular: abnormalities of the calcitonin gene-related peptide of the genitofemoral nerve or the cremasteric nucleus have been postulated to cause cryptorchidism.

912
Q

RFs for cryptorchidism

A

STRONG
FHx [23% of cases]
Prematurity
LBW/SGA

WEAK
Environmental exposures
Maternal alcholo use
GDM
Hx inguinal surgery
913
Q

Sx of cryptorchidism

A

COMMON
Malpositioned / absent testes
Palpable cryptorchid testes
Non-palpable testes

OTHER COMMON
Testicular asymmetry
Scrotal hypoplasia
Retracile testis

UNCOMMON
Ascending cryptorchidism
Hypospadias
Micropenis
Surgical scar (can be iatrogenic)
914
Q

Ix for cryptorchidism

A

Clinical

If testis is non-palpable the current recommendation is examination under anaesthesia, followed by diagnostic laparoscopy (or open surgery) if still non-palpable.

915
Q

Rx of cryptorchidism

A

Retractile testes:
Annual followup

Undescended, prepubertal, no hypospadias:
Orchiopexy is the treatment of choice for a palpable testicle(s) that has not descended into the dependent portion of the scrotum by 6 months of age.
Surgical therapy should ideally occur prior to 12-18 months of age.

Bilateral non-palpable
Surgical exploration + endocrine referral

916
Q

Prognosis of cryptorchidism

A

92-95% surgical success rate

The chance of later neoplasia in a boy born with unilateral undescended testicle is approximately 1 in 120, and with bilateral undescended testicles the chance is approximately 1 in 44. The higher the testis is in the abdomen, the higher the risk appears to be.

917
Q

Complications of cryptorchidism

A

Infertility

Malignancy

918
Q

A 4-month-old boy is noted on routine medical evaluation to have the left testis palpable in the scrotal sac, but the right scrotal sac is empty. The right testis is palpable in the upper portion of the scrotum, but cannot be pulled down into the scrotum.

A

cryptorchidism

919
Q

Acquired cryptorchidism may occur in patients with previously documented normal testicular position. These patients are later noted to have testes that have ascended into the high portion of the scrotum or the inguinal canal. The incidence and aetiology of this phenomenon are unclear. Boys with testicles that descended in the first months of life may be at highest risk. Retractile testes are located in a suprascrotal position but can be pulled down without pain into the scrotum and remain there after traction is released. This is considered a normal variant in position due to the cremasteric reflex, and these patients generally do not undergo treatment but should be followed for acquired cryptorchidism.

A

cryptorchidism

920
Q

Define mesenteric adenitis

A

Mesenteric lymphadenitis refers to inflammation of the mesenteric lymph nodes and is considered present if a cluster of three or more lymph nodes, each measuring 5 mm or greater, is detected in the right lower quadrant mesentery. This process may be acute or chronic, depending on the causative agent, and it causes a clinical presentation that is often difficult to differentiate from acute appendicitis in children.

921
Q

Epidemiology of mesenteric adenitis

A

Numerous organisms have been cultured from mesenteric lymph nodes and blood, such as beta-hemolytic streptococcus, Staphylococcus species, Escherichia coli, Streptococcus viridans, Yersinia species (responsible for most cases currently), Mycobacterium tuberculosis, Giardia lamblia, and non– Salmonella typhoid. Viruses, such as coxsackieviruses (A and B), rubeola virus, and adenovirus serotypes 1, 2, 3, 5, and 7, have also been implicated.
Mesenteric node involvement can also be part of infectious Epstein-Barr virus (EBV), acute human immunodeficiency virus (HIV) infection, and catscratch disease (CSD).
The frequent association of this condition, especially in children with upper respiratory tract infection, has popularized a theory that swallowed pathogen-laden sputum may be the primary source of infection.
Fecal-oral transmission occurs in Y enterocolitica infection and may present as a common source outbreak. This infection has also been associated with meat, milk, and water contamination. Rarely, person-to-person or zoonotic contacts with fecal carriers can lead to infection.

922
Q

Aetiology of mesenteric adenitis

A

Up to 20% of patients undergoing appendectomy have been found to have non-specific mesenteric adenitis

M=F

923
Q

Sx of mesenteric adenitis

A
Onset = insideous or sudden
Abdo pain
Fever
Diarrhoea
Anorexia
Concomitant/antecedent URTI
N+V (generally precedes pain)
Hx of ingestion of raw pork (endemic Yersinia eg Belgium)
Fever 38-38.5
Flushed
RLQ tenderness
W/WO rebound
Voluntary guaring
Rhinorrhoea
Hyperaemic pharynx
Toxic appearance
Cervical/peripheral lymphadenopathy
924
Q

Ix for mesenteric adenitis

A

FBC - leucocytosis
Chemisty - consisted with N+V IE metabolic alkalosis and uraemia/creatinine.

Stool culture if diarrhoea
Blood culture
Urinalysis to exclude UTI

US performed to detect appendicitis
CT possible but not usually worth performing
MRI may be warranted

925
Q

Rx of mesenteric adenitis

A

Empiric, broad-spectrum antibiotics may be used in moderately to severely ill patients and should cover Yersinia strains, commonly causative in mesenteric adenitis.

Patients with volume depletion, significant electrolyte imbalance, and/or sepsis require hospital admission.

Surgery is usually indicated in suppuration and/or abscess, with signs of peritonitis, or if acute appendicitis cannot be excluded with certainty.
At laparotomy, the diagnosis is generally clear. An appendectomy should be performed in view of the tendency for recurrence of lymphadenitis and the difficulty in differentiating adenitis from appendicitis.

Metronidazole usually given

926
Q

Complications/prognosis of mesenteric adenitis

A

The prognosis is good. Typically, complete recovery can be expected without specific treatment. Death is rare.

Mesenteric lymphadenitis generally is a benign disease, but patients with sepsis may have a fatal outcome.

Complications of mesenteric lymphadenitis include the following:
Volume depletion and electrolyte imbalance in patients with severe diarrhea, nausea, and vomiting
Abscess formation
Peritonitis (rare)
Sepsis
In cases where the underlying pathogen is Y enterocolitica, some patients may develop arthralgias. These symptoms typically develop 1 month after the initial episode of diarrhea and usually resolve after 1-6 months. A rash on the legs and/or trunk, erythema nodosum, may also appear and is also self-limited.

927
Q

Complications/prognosis of mesenteric adenitis

A

The prognosis is good. Typically, complete recovery can be expected without specific treatment. Death is rare.

Mesenteric lymphadenitis generally is a benign disease, but patients with sepsis may have a fatal outcome.

Complications of mesenteric lymphadenitis include the following:
Volume depletion and electrolyte imbalance in patients with severe diarrhea, nausea, and vomiting
Abscess formation
Peritonitis (rare)
Sepsis
In cases where the underlying pathogen is Y enterocolitica, some patients may develop arthralgias. These symptoms typically develop 1 month after the initial episode of diarrhea and usually resolve after 1-6 months. A rash on the legs and/or trunk, erythema nodosum, may also appear and is also self-limited.

928
Q

Define Juvenile idiopathic arthritis (JIA)

A

A collection of chronic paediatric arthropathies characterised by onset before 16 years of age and the presence of objective arthritis (in one or more joints) for at least 6 weeks. Arthritis of joints is defined by swelling or effusion, increased warmth, and/or painful limited movement with or without tenderness.

Juvenile idiopathic arthritis (JIA) is the most common chronic arthropathy of children and includes several subtypes (including oligoarticular, polyarticular, and systemic onset).

Affects 1 in 1000 children and can present at any age.

Diagnosis is made clinically. Laboratory and radiographic testing provide classification and prognostic information but are not diagnostic.

Intra-articular corticosteroids offer good control if only a few joints are affected. Methotrexate is the most commonly used disease-modifying agent. Agents that block inflammatory cytokines (e.g., tumour necrosis factor alpha inhibitors, interleukin-1, and interleukin-6) are used in more resistant cases. Physiotherapy, occupational therapy, and psychologyp form an important aspect of management.

Around 10% to 20% of children with JIA are at risk of developing anterior uveitis. All children with a diagnosis of JIA must undergo regular ophthalmological examinations to detect and manage inflammation.

929
Q

Epidemiology of Juvenile idiopathic arthritis (JIA)

A

JIA is the most common chronic arthropathy and most common chronic rheumatic disorder of childhood.

M:F 1:2

Mainly oligoarticular

Mainly females <6yo

Rheumatoid factor (RF)-negative polyarticular JIA is the next most common subtype and is also usually seen in young female children. RF-positive polyarticular JIA is typically seen in teenage girls. Black, Indian, and Native American children are more likely to have polyarticular JIA. Enthesitis-related JIA has male predominance, usually in boys after the age of 6 years.

Systemic-onset juvenile idiopathic arthritis (SoJIA) comprises about 10% of all JIA and affects boys and girls equally.

930
Q

RFs of Juvenile idiopathic arthritis (JIA)

A
STRONG
Female
HLA polymorphism
<6yo
FHx autoimmunity

WEAK
ABx exposure in childhood

931
Q

Aetiology of Juvenile idiopathic arthritis (JIA)

A

The exact aetiology is unknown. However, direct and indirect evidence points to the role of aberrant immune responses, suggesting that JIA is an autoimmune disorder.

Breast-feeding may be protective

Maternal smoking antenatally

932
Q

Sx of Juvenile idiopathic arthritis (JIA)

A

COMMON
RFs
>6weeks duration
Joint pain
Joint swelling - commonly at the knees in oligoarticular JIA
Fever - high spiking fevers often seen, interspersed with normal temperatures

OTHER COMMON
Morning stiffness - gets better throughout the day
Limp (worse in morning)
Limp
Limited motion
Rash - salmon coloured, non-pruritic, elicited by scratching the skin (koebner’s phenomenon).
Enthesitis

UNCOMMON
Limb length discrepancy
Uveitis
Rheumatoid nodules

933
Q

Ix for Juvenile idiopathic arthritis (JIA)

A

FBC (exclude malignancy) - N / reduced haemoglobin / elevated platelets
ESR - N / elevated
CRP - N / elevated
ANA - +ve or negative
RF - +ve / -ve
Anti-CCP - +ve / -ve
Chlamidya test - exclude reactive arthritis
Ferritin - could be abnormal
US affected joints - abnormal
MRI - synovial fluid; synovial thickening and/or synovial enhancement

MRI may be indicated in patients with monoarticular disease to rule out other diagnoses such as pigmented villonodular synovitis or synovial haemangiomas. It can also be used to monitor cartilage injury.

934
Q

Rx of Juvenile idiopathic arthritis (JIA)

A

ACUTE
Physiotherapy and occupational therapy are encouraged in conjunction with medicines. Patients are encouraged to participate in activities such as swimming and cycling, and weight-bearing activities are indicated for those with a greater risk of low bone mineral density.

NSAID:
Naproxen / ibuprofen / celecoxib / indomethacin

ONGOING
Methrotrexate / sulfsalazine
Oral corticosteroids - prednisolone. Move to intra-articular -> IV

TNFa inhibitor, interleukin receptor antagonist or fusion protein

935
Q

Prognosis of Juvenile idiopathic arthritis (JIA)

A

Reports suggest that 40% to 60% of patients achieve remission or inactive disease.

However, outcome is variable depending on disease subtype, and long-term outcome is best predicted by disease characteristics at 5 years follow-up than at onset.

936
Q

Complications of Juvenile idiopathic arthritis (JIA)

A
Leg length discrepancy, micrognathia
Joint erosion
C-spine fusion and c1/2 subluxation
Sacroiliac joint and spine ankylosis
Uveitis
Macrophage activation syndrome
937
Q

A 3-year-old girl presents with stiffness and limp of several weeks’ duration. The onset was insidious and her parents do not recall any specific injury or prior infections. Her parents mention that one of her knees is swollen and cannot be straightened, although it is not especially painful. Her symptoms are particularly bad in the mornings when she wakes, but her gait improves as the day goes on. She has not had any fevers, rashes, or other constitutional symptoms.

A

Juvenile idiopathic arthritis (JIA)

938
Q

Oligoarticular JIA describes arthritis in four or fewer joints and typically affects young children (often younger than 6 years of age and female). It most often affects the large joints such as the knee, ankle, wrist, and/or elbow joints. It can be associated with an eye disease called uveitis, which affects up to 20% of young people with this type of JIA.

Rheumatoid factor (RF)-negative polyarticular JIA typically affects young girls and usually presents with symmetrical stiffness, swelling, and pain in several joints, often involving small joints of the hands and feet.

RF-positive polyarticular JIA typically affects older girls and usually presents with symmetrical symptoms affecting several joints, often involving the small joints of the hands and wrists.

Systemic-onset juvenile idiopathic arthritis (SoJIA) may present with arthritis in one or more joints in addition to daily high spiking fevers, and evanescent, truncal, salmon-coloured, macular rashes. Arthritis does not need to be present initially for diagnosis.

Enthesitis-related JIA typically (but not exclusively) affects boys over the age of 6 years and usually presents with asymmetrical arthritis, enthesitis, and sacro-iliac joint involvement.

Psoriatic JIA usually presents with arthritis and definite histories or family histories of psoriasis (in first-degree relative), nail changes, and/or dactylitis; psoriatic rash may/may not be present.

Undifferentiated JIA may present with features of more than one subtype.

A

Juvenile idiopathic arthritis (JIA)

939
Q

Define reactive arthritis

A

An inflammatory arthritis that occurs after exposure to certain gastrointestinal and genitourinary infections. The classical triad of post-infectious arthritis, non-gonococcal urethritis, and conjunctivitis is frequently described but found only in a minority of cases and not required for diagnosis.
ReA belongs to the family of spondyloarthropathies, which share similar clinical, radiographic, and laboratory features that include spinal inflammation and an association with HLA-B27. They include psoriatic arthritis, ankylosing spondylitis, arthritis related to inflammatory bowel disease, and undifferentiated spondyloarthropathy.

Patients may give a history of an antecedent genitourinary or dysenteric infection 1 to 4 weeks before the onset of arthritis.

Presenting features include systemic symptoms such as fever, peripheral and axial arthritis, enthesitis (inflammation where tendons insert into bone), dactylitis (swelling of an entire finger or toe), conjunctivitis and iritis, and skin lesions including circinate balanitis and keratoderma blennorrhagicum.

The peripheral arthritis in reactive arthritis (ReA) is usually an asymmetrical oligoarticular arthritis affecting the large joints of the lower limb, although monoarticular and polyarticular arthritis can also occur.

There is no specific test for diagnosing ReA. Rather, a group of tests is used to confirm the suspicion in someone who has clinical symptoms suggestive of an inflammatory arthritis in the post-venereal or post-dysentery period.

Treatment is aimed at symptomatic relief and preventing or halting further joint damage. Typical agents include non-steroidal anti-inflammatory drugs (NSAIDs), corticosteroids, and disease-modifying antirheumatic drugs (DMARDs).

Approximately 30% to 50% of patients will go on to develop some form of chronic ReA.

940
Q

Epidemiology of reactive arthritis

A

30-40 per 100k

The attack rate of ReA after a C trachomatis infection ranges from about 4% to 8%

post-dysentery ReA ranges from 1.5% to about 30%.

One study found that the arthritis persisted at 2 years in nearly half (47%) of affected patients.

941
Q

Aetiology of reactive arthritis

A

The most commonly implicated bacterial species are Chlamydia , Salmonella , Campylobacter , Shigella , and Yersinia species, although ReA has been described after many other bacterial infections.

942
Q

RFs for reactive arthritis

A

STRONG
Male
HLA-B27
Preceding GI / chlamidya infection

943
Q

Sx of reactive arthritis

A

Presence RFs
Peripheral arthritis - The arthritis is most commonly an asymmetrical oligoarthritis. However, polyarticular and monoarticular arthritis can occur.

There is usually a predilection for the larger joints in the lower extremity. The inflammation often leads to painful, swollen, warm, red, and stiff joints, especially in the morning. Swelling of entire digits (fingers or toes), termed dactylitis, is a specific finding within the spondyloarthropathies arising from flexor tenosynovitis. Usually, patients are defined as having chronic reactive arthritis (ReA) when joint symptoms have been present for more than 6 months.

Axial (spinal) arthritis

OTHER COMMON

Constitutional symptoms - Fever, fatigue, and weight loss are common.
Enthesitis

UNCOMMON
Mucous membrane pains ulcers
Keratoderma blennorrhagicum
Circinate balanitis 
Ocular manifestations: Ocular inflammation in the form of conjunctivitis can cause redness, tearing, and a sterile purulent discharge during the acute period. Anterior uveitis (iritis), associated with HLA-B27 positivity, causes symptoms of pain, redness, and photophobia. Iritis often becomes chronic.
Aortitis possible
944
Q

Ix for reactive arthritis

A

ESR - High
CRP - High
ANA - negative
RF - Negative
Plain xray - sacroiliitis or enthesopathy
Arthrocentesis with synovial fluid analysis - rule out gout

Consider
HLA-B27 - +ve or -ve. Kinda useless
NAAT for chlamidya - may be positive for Chlamydia trachomatis or Neisseria gonorrhoeae

945
Q

Rx of reactive arthritis

A

ACUTE

  1. NSAIDs
  2. Corticosteroids

ONGOING
DMARDs - Sulfsalazine 500mg

946
Q

Complications of reactive arthritis

A

Secondary osteoarthritis
Iritis / events
Keratoderma blenorrhagicum

947
Q

Prognosis of reactive arthritis

A

Approximately 50% of patients can expect symptoms to resolve within the first 6 months. However, 30% to 50% of patients will develop chronic reactive arthritis.

948
Q

A 21-year-old male student presents with a 4-week history of a painful, hot, and swollen left knee, low back pain with bilateral buttock pain, and left heel pain. He denies trauma and states the symptoms began acutely. He denies any fever or any other significant arthralgias. Further review of symptoms indicates the patient was treated for a chlamydia infection after he developed dysuria approximately 8 weeks ago (he was treated with a single dose of azithromycin). He admits to unprotected sexual intercourse with a new partner 2 days before the onset of his dysuria. In addition to pain and swelling, the patient reports that he has developed morning stiffness in the left knee and low back that last more than 1 hour. He continues to have episodes of dysuria. The findings of physical examination are significant for a large effusion of the left knee with warmth. Range of motion is slightly diminished. There is tenderness to palpation of the left heel at the site of the Achilles’ insertion. Laboratory findings are significant for an ESR of 35 mm/hour, and both FBC and uric acid level are within normal limits. He is HLA-B27 positive, rheumatoid factor negative, and ANA negative. The synovial fluid analysis is negative for crystals, with a total nucleated cell count of 22,000 cells/microlitre (65% neutrophils). A urethral swab was positive by PCR for Chlamydia trachomatis 9 weeks ago at initial presentation of dysuria. There was no evidence of gram-negative diplococci on Gram stain.

A

reactive arthritis

949
Q

Define septic arthritis

A

Septic arthritis is defined as the infection of 1 or more joints caused by pathogenic inoculation of microbes. It occurs either by direct inoculation or via haematogenous spread.

A septic joint will be painful, hot, swollen, and restricted.

Diagnosis is based on the opinion of the clinician experienced in the management of musculoskeletal disease.

In all cases of suspected native joint sepsis, the joint should be aspirated.

In all cases of suspected joint sepsis, empirical antibiotic therapy should be commenced once appropriate cultures have been taken.

950
Q

Epidemiology of septic arthritis

A

The estimated incidence of septic arthritis in developed countries is 6 cases per 100,000 population per year.

In patients with underlying joint disease or with prosthetic joints the incidence increases approximately 10-fold, to 70 cases per 100,000 of the population.

951
Q

Aetiology of septic arthritis

A

The predominant causative organisms of septic arthritis are staphylococci or streptococci. These organisms account for 91% of cases.

Risk factors for the development of joint sepsis include rheumatoid arthritis or osteoarthritis, joint prostheses, intravenous drug abuse, alcohol use diabetes, previous intra-articular corticosteroid injection, and the presence of cutaneous ulcers

Community-associated MRSA is becoming more common in many parts of the world and should be suspected in patients recently discharged from hospital, nursing home residents, those with leg ulceration, and those with indwelling urinary catheters.

952
Q

RFs for septic arthritis

A
STRONG
Underlying joint disease - rheumatoid arthritis, osteoarthritis, or crystal arthritides
Joint prosthesis 
IVDU
DM
Cutaneous ulcers

WEAK
Alcohol use disorder
Hx intra-articular corticosteroid injection

953
Q

Sx of septic arthritis

A

COMMON
Hot, swollen, tender restricted joint

OTHER COMMON
Low socio-E
Hx IVDU, DM, cutaenous ulcers
Prosthetic joint
Hx other arthritis
Short Hx Sx
Fever

UNCOMMON
Hx alcohol use
Hx Corticosteroid injection

954
Q

Ix for septic arthritis

A

Synovial fluid gram stain and culture - presence of microorganisms
Synovial fluid WCC - High
Blood culture
WCC
ESR - can be high
CRP = high
Plain radiograph - may reveal degenerative changes
USS - may show the presence of an effusion to guide aspiration

955
Q

Rx of septic arthritis

A

Suspected G+ve = vancomycin IV
Suspected G-ve = cephalosporin ie ceftriaxone, cefotaxime

IV ABx for 2 weeks
Oral Abs for subsequent 4 weeks

956
Q

Prognosis of septic arthritis

A

Delayed or inadequate treatment can lead to irreversible joint destruction and subsequent disability. There is also an estimated case fatality rate of 11%.

A survey in the UK revealed that certain factors might be associated with a poor prognosis including older age, pre-existing joint disease, and the presence of a joint prosthesis.

957
Q

Complications of septic arthritis

A

ABx allergic reaction
Osteomyelitis
Joint destruction

958
Q

A 55-year-old woman presents with a 1-week history of pain and swelling in her left wrist. She was diagnosed with rheumatoid arthritis at the age of 36 years but the rest of her joints are currently asymptomatic. Her rheumatoid arthritis is well controlled on her current medication. On examination her left wrist is found to be hot, swollen, tender, and highly restricted in its range of movement. There is no sign of inflammation in any of her other joints. She has a temperature of 37.5˚C (99.5˚F).

A

septic arthritis

959
Q

A 25-year-old man who is a known intravenous drug abuser presents with a 5-day history of pain and swelling in his right leg. On examination there are multiple sites of intravenous puncture. His right leg is swollen from the knee downwards. There is a large effusion on the right knee together with significant cellulitic changes of the overlying skin.

A

septic arthritis

960
Q

Define glucose-6-phosphate dehydrogenase (G6PD) deficiency

A

Glucose-6-phosphate dehydrogenase (G6PD) deficiency is an inherited condition in which patients are excessively susceptible to the development of haemolytic anaemia. Affected people lack the ability to tolerate biochemical oxidative stress, and red cell haemolysis is the most important clinical consequence. Almost all patients are completely asymptomatic. However, infections, drugs, or ingesting broad beans can precipitate haemolysis. Newborns may have prolonged and/or severe neonatal hyperbilirubinaemia.

An inherited (X-linked) enzyme deficiency that is common among populations originating from parts of the world where malaria is or was common: that is, sub-Saharan Africa, Asia, the Mediterranean region, and the Middle East.

Most patients are asymptomatic.

Most common manifestations are prolonged or severe neonatal jaundice and haemolytic anaemia in association with infection or following consumption of broad beans or certain drugs.

Diagnosis is made from blood tests including smear review and the specific enzyme assay.

Patients should be advised to avoid precipitant factors, especially oxidative drugs and broad beans.

961
Q

Epidemiology of glucose-6-phosphate dehydrogenase (G6PD) deficiency

A

It is one of the most common enzyme deficiency states in the world, and one of the most frequent inherited disorders.

400ml WW

The highest prevalence is in people of African, Mediterranean, and Asian heritage. The prevalence in Kurdish Jews is 50%-70%.

G6PD confers a degree of protection against malaria

962
Q

Aetiology of glucose-6-phosphate dehydrogenase (G6PD) deficiency

A

The gene for G6PD is located at the end of the long arm of the X chromosome. The deficiency is thus seen in hemizygous males (who have a mutation on their sole X chromosome) and in homozygous females (who have a mutation on each of their X chromosomes; very rarely seen in most X-linked diseases, but frequent in populations where G6PD deficiency is common [e.g., Africa]).

963
Q

RFs for glucose-6-phosphate dehydrogenase (G6PD) deficiency

A

STRONG
Male
Neonate
Ethnic origin in the Mediterranean, sub-Saharan Africa, Middle East, or Southeast Asia
FHx
Oxidative drug exposure: (e.g., dapsone, primaquine, sulphonamides, caution aspirin)
Infection: Bacterial (e.g., lobar pneumonia, typhoid) and viral (e.g., hepatitis) infections have been particularly implicated.
Synonyms for fava beans include broad beans, fever beans, and English dwarf beans.

964
Q

Sx of glucose-6-phosphate dehydrogenase (G6PD) deficiency

A
COMMON
RFs
Jaundice
Pallor
Dark urine

OTHER COMMON
Nausea

UNCOMMON
Splenomegaly

965
Q

Ix for glucose-6-phosphate dehydrogenase (G6PD) deficiency

A

FBC - anaemia and normochromic red cell index
Reticulocyte count - raised
Urinalysis - haemoglobinuria
Unconjugated bilirubin - high
LDH - high
Haptoglobin - Binds free haemoglobin; low plasma values suggest intravascular haemolysis.
Peripheral blood smear - anisocytosis, abnormal forms, bite cells, HEINZ BODIES

Genetic testing possible but rarely needed

966
Q

Rx of glucose-6-phosphate dehydrogenase (G6PD) deficiency

A

Acute haemolysis
Supportive care + folic acid 5mg

Haemoglobin <70g/L = blood transfusion

Haemoglobin <70g/L + renal impairment = blood transfusion and dialysis

NEONATES WITH PROLONGED INDIRECT HYPERBILIRUBINAEMIA
Phototherapy + exchange transfusion

Splenectomy may be considered for those with significant extravascular haemolysis, marked splenomegaly, or persistent severe anaemia that is interfering with growth, development, or normal activity. This may result in a significant decrease in haemolysis.

Cholecystectomy may be considered at the same time if gallstones are present.

Patients who are about to undergo splenectomy should receive appropriate immunisations (against pneumococcus, meningococcus, and Haemophilus influenzae ) and start long-term antibiotic prophylaxis to protect against infection by capsulated bacterial organisms.

967
Q

Prognosis of glucose-6-phosphate dehydrogenase (G6PD) deficiency

A

Almost all patients are asymptomatic and develop problems only when they encounter a haemolytic challenge in response to oxidative stress. The prognosis for complete recovery is excellent.

Patients may benefit from receiving an information sheet on the condition specifying that they must avoid triggers for oxidative stress, including certain drugs (e.g., dapsone, primaquine, sulphonamides, and aspirin), and that they may develop haemolysis after eating broad beans and at times of infection.

968
Q

Complications of glucose-6-phosphate dehydrogenase (G6PD) deficiency

A

Kernicterus
Cataract
Gallstones
Increased susceptibility to sepsis

969
Q

Patients (usually boys) are at an increased risk of developing neonatal jaundice. This usually presents 2 to 3 days after birth as exaggerated physiological jaundice with minimal anaemia. However, in some cases the jaundice and anaemia are severe, and treatment with exchange blood transfusion and phototherapy may be required. Haemolytic anaemia can also occur (also typically in males) in the setting of the recovery phase of an acute infection (bacterial or viral; typically pneumonia or typhoid fever). A few patients (exclusively males) with severe deficiency enzyme variants have severe neonatal jaundice and chronic haemolytic anaemia with splenomegaly. Females are usually asymptomatic but may develop mild drug-induced jaundice.

A

glucose-6-phosphate dehydrogenase (G6PD) deficiency

970
Q

Define long bone fracture

A

A fracture is an abnormal disruption in the continuity of a bone and is often referred to as a broken bone. For the purposes of this monograph, long bones are defined as the humerus, radius, ulna, femur, tibia, and fibula. This monograph focuses on extra-articular fractures (i.e., fractures that do not extend into a joint).

This monograph covers long bone fractures and includes fractures of the humerus, radius, ulna, femur, tibia, and fibula.

Acute fractures usually have a dramatic presentation, whereas stress fractures often present more subtly.

Acute fractures in older people often occur with relatively low-energy trauma, whereas in younger patients with previously healthy bone, they usually result from higher-energy trauma.

Associated injuries should be diligently searched for and neurovascular status should be evaluated.

At least two 90° orthogonal x-rays (e.g., anteroposterior and lateral) should be obtained, with inclusion of the joints proximal and distal to the site of suspected injury.

Proper immobilisation, analgesia, and timely orthopaedic referral as appropriate can greatly enhance patient comfort and ensure optimal outcome. Some non-displaced long bone fractures can be treated conservatively, but consultation with an orthopaedist is generally recommended.

Potential life-threatening complications include acute compartment syndrome, fat embolism, and haemorrhage.

971
Q

Epidemiology of long bone fracture

A

The tibia is the most frequently fractured long bone in humans.

21 per 1000 (for all types)

As global standards of living and lifespan have increased, so too have osteoporotic fractures, as have fractures among those with chronic disease. In patients with chronic renal failure, risk factors include female sex, advanced age, and diabetes, whereas black people and those with increased BMI seemed to be protected. Among dialysis patients, long bone fracture is associated with increased risk of stroke, pulmonary embolism, congestive heart failure, pneumonia, and septicaemia.

972
Q

Aetiology of long bone fracture

A

Among otherwise healthy adults, high-energy trauma (e.g., motor vehicle accidents), sport injuries, falls, and assaults are the main causative factors.

Humeral shaft fractures:

Commonly result from direct trauma to the humerus and falls onto the outstretched hand. [17] [18] Less commonly, extreme muscle contraction, electrocution injury, or seizure may lead to humeral shaft fracture. [19]
Proximal humeral shaft fractures:

Typically seen in older people after a fall on the outstretched hand. Direct trauma and seizures may also lead to these fractures. [20]
Humeral stress fractures:

Primarily occur as a result of overuse among throwing athletes. Gymnasts, weightlifters, and other athletes who place repetitive high-impact or -torque loads on the humerus have also been known to sustain these injuries.
Radial and ulnar shaft fractures:

Radial shaft fractures usually result from a fall onto the outstretched or pronated wrist, or from a direct blow. High-torque forces from twisting injuries can also cause such injuries.
Isolated fractures of the mid shaft of the ulna, often called nightstick fractures, usually result from a person trying to ward off a blow from a heavy, blunt object (e.g., a night stick or truncheon). If the ulnar fracture involves the proximal third of the shaft, there may be associated dislocation of the radial head at the elbow (Monteggia fracture/dislocation). Monteggia fractures are usually due to a fall onto the outstretched hand, with the elbow extended and pronated. [21]
Concomitant fractures of both the radius and the ulna are usually the result of high-energy trauma from a blow, fall, or motor vehicle accident.
Radial and ulnar stress fractures:

Primarily occur among athletes who repetitively load the bones with high forces (e.g., gymnasts). [22] [23]
Femoral shaft fractures:

Generally caused by high-energy trauma, such as a motor vehicle accident, or fall from a height.
Spiral fractures of the femoral shaft may occur as a result of a twisting injury.
Comminuted and open fractures may occur from gunshot wounds or other forms of high-energy penetrating trauma.
Femoral stress fractures:

Commonly seen in athletes and runners.
Tibial and fibular shaft fractures:

Tibial shaft fractures may result from direct trauma (usually causes transverse or comminuted fractures) or indirect twisting forces (usually causes spiral or oblique fractures). [24]
High-energy trauma may result in simultaneous fracture of both the tibial and fibular shafts.
Isolated fibular fractures are typically caused by a direct blow to the outer aspect of the leg or from an external rotation force at the ankle.
Tibial and fibular stress fractures:

Stress fractures are relatively common in impact athletes, such as runners, basketball players, and military personnel. [25] [26]
Stress fractures of the fibula are uncommon but typically occur in runners and ballet dancers.

973
Q

RFs for long bone fracture

A
STRONG
Direct trauma
Indirect trauma
Osteoporosis
Bone tumour
Age >70
Age <30
Prolonged corticosteroids
Low BMI
Hx of fall
Prior fracture
Seizures
Long term bisphosphonate use
WEAK
Chronic RF
DM
Male sex (acute fractures)
Female sex (stress and insufficiency)
974
Q

Sx of long bone fracture

A
Pain
Swelling 
Ecchymosis
Expanding haematoma
Impaired limb function
Inability to bear weight
Point tenderness
Deformity 
Guarding 
Wound overlaying site of injury
Signs of ischaemic limb -Includes: loss of distal pulses, pallor, increased pain with passive stretch of tissues distal to fracture site, paraesthesias, and poikilothermia.
Hypotension/hypovolaemic shock
OTHER
Altered nerve sensation
Impaired motor function
Bony crepitus
Callus
975
Q

Ix for long bone fracture

A

X-ray limb 2 plains

FBC, blood typing, and cross-matching (major trauma) - may show acute drop in haematocrit

976
Q

Rx for long bone fracture

A

Orthopaedic consultation
Immobilisation
Analgesia
Elevation and ice

Consider ABx, tetanus toxoid

977
Q

Complications / prognosis of long bone fracture

A
VTE
Infection
Compartment syndrome
Fat embolism
Malunion
Delayed union
Adhesive capsulitis
Non-union
Avascular necrosis
978
Q

A 20-year-old man is brought in on a stretcher, complaining of severe pain in his right leg and inability to walk after being struck on the shin by a baseball bat. He is anxious and in moderate distress due to pain. Examination reveals an ecchymotic and oedematous right shin, with disruption of the normal contour of the underlying tibia. No wound or blistering is noted. There is point tenderness and crepitus to palpation at the mid tibia, and the patient guards any movement of the right lower extremity. Knee and ankle examination is limited by guarding but does not show gross instability. Distal pulses are faint and the right foot is somewhat cool and pale, with decreased sensation to light touch and pinprick.

A

Long bone fracture

979
Q

Complications / prognosis of long bone fracture

A
VTE
Infection
Compartment syndrome
Fat embolism
Malunion
Delayed union
Adhesive capsulitis
Non-union
Avascular necrosis 

With recent advances in emergency transport and orthopaedic surgical technique, many patients with high-energy trauma now survive their injuries and go on to complete or near complete recovery. Naturally, the nature, severity, and number of injuries dictate the prognosis for a given person.

Humeral fractures
Proximal humerus fractures usually heal well, but disruption of the vascular supply can lead to avascular necrosis (AVN) of the humeral head. Shaft (diaphyseal) fractures are often complicated by neurovascular injury (i.e., radial nerve injury) and non-union.

Radial and ulnar fractures
Appropriate surgical intervention leads to satisfactory outcome for most of these injuries.

Femur fractures
Proximal injuries may lead to femoral head AVN and may require hip arthroplasty. Massive bleeding, neurovascular injury, fat embolism syndrome, venous thromboembolic disease, and other complications may occur. If the patient survives the initial trauma, the outlook is generally good.

Tibial and fibular fractures
Operative management of tibial shaft fractures generally leads to good outcomes. Non-displaced fractures generally heal well with conservative treatment. Open fractures are associated with high risk of infection, and compartment syndromes must be detected and treated early. Isolated fibular fractures usually heal well with non-operative management. Healing of non-operatively treated tibial shaft fractures has been shown to be delayed among smokers [94] and (with transverse tibial fractures) among alcohol abusers.

980
Q

Acute long bone fractures are usually associated with sudden onset of moderate to severe pain, swelling, and impaired function. Open fractures are often obvious, but sometimes an apparently minor surface wound belies severe injury below. A patient with head injury, other distracting injuries, or intoxication may not manifest the classic signs of pain and limb dysfunction. New deformity in a limb implies a fracture and/or dislocation. Blistering of the skin associated with marked oedema also signifies underlying fracture. Stress fractures typically have a gradual, more subtle course, often with no to minimal abnormalities on inspection. Pathological or insufficiency fractures may be the first clue that an underlying process (e.g., cancer, osteoporosis) is present. Patients with dementia may exhibit withdrawal from attempted pressure or motion to the affected area, lack of use of the involved extremity, and non-specific signs such as decreased appetite, new or worsened incontinence, or depression.

A

Long bone fracture

981
Q

Acute long bone fractures are usually associated with sudden onset of moderate to severe pain, swelling, and impaired function. Open fractures are often obvious, but sometimes an apparently minor surface wound belies severe injury below. A patient with head injury, other distracting injuries, or intoxication may not manifest the classic signs of pain and limb dysfunction. New deformity in a limb implies a fracture and/or dislocation. Blistering of the skin associated with marked oedema also signifies underlying fracture. Stress fractures typically have a gradual, more subtle course, often with no to minimal abnormalities on inspection. Pathological or insufficiency fractures may be the first clue that an underlying process (e.g., cancer, osteoporosis) is present. Patients with dementia may exhibit withdrawal from attempted pressure or motion to the affected area, lack of use of the involved extremity, and non-specific signs such as decreased appetite, new or worsened incontinence, or depression.

A

Long bone fracture

982
Q

Define delayed puberty / constitutional delay

A

Puberty is an interval characterised by the acquisition of the secondary sexual characteristics, accelerated linear growth, increase in the secretion of sex hormones, maturation of gonads (testes in boys; ovaries in girls), and the potential for reproduction. It is typically complete within 2 to 5 years. Delayed puberty is defined as the lack of any pubertal signs by the age of 13 years in girls and 14 years in boys.

Defined as the lack of any pubertal signs by the age of 13 years in girls and 14 years in boys. More common in boys.

May be functional (constitutional delay, underlying chronic disease, malnutrition, excessive exercise) or organic, due to either a lack of serum gonadotrophin production or action (hypogonadotrophic hypogonadism), or gonadal insufficiency with elevated gonadotrophins (hypergonadotrophic hypogonadism).

Most patients seek medical assistance because of slow growth rather than slow pubertal development.

Careful assessment of height and pubertal stage is crucial for evaluation of the underlying cause.

The distinction between organic gonadotrophin deficiency and constitutional delay of puberty is not easy and is often resolved only with time.

Patients with constitutional delay are typically observed. Sex-steroid treatment is reserved for those with psychosocial maladaptation, and consists of a short course of sex steroids to induce puberty.

Patients with an organic cause for delay are given sex-steroid therapy to induce puberty and are most likely to require lifelong hormone replacement therapy after puberty is complete.

983
Q

Epidemiology of delayed puberty / constitutional delay

A

The exact prevalence and incidence of children presenting with a delay in pubertal development is unknown. The majority of patients are boys. The prevalence of primary amenorrhoea in the US is <0.1%, [1] that of Turner’s syndrome is 1 in 2500 live-born females, [2] [3] and that of Klinefelter’s syndrome is 1 in 1500 live-born males.

984
Q

Aetiology of delayed puberty / constitutional delay

A

The vast majority of patients presenting with delayed puberty have only a temporary delay due to a central inhibition of the hypothalamo-pituitary axis that resolves with time and serial observation (functional delay).

Constitutional delay is the most common cause of temporary delayed sexual maturation in boys. It tends to occur within families and is associated with short stature that is appropriate for the skeletal age. [12] It may occasionally be difficult to distinguish from organic gonadotrophin deficiency.
A temporary or reversible delay in puberty may also occur due to any chronic illness in children (e.g., cystic fibrosis, inflammatory bowel disease, coeliac disease, poorly controlled diabetes mellitus), excessive physical exercise, malnutrition and eating disorders (anorexia nervosa), and prolonged stress.
Organic causes may be due to a hypothalamo-pituitary cause or due to gonadal abnormality.

Hypothalamo-pituitary disorders
Congenital gonadotrophin deficiency can be isolated or be associated with combined pituitary hormone deficiencies. It can also be associated with other systemic features such as anosmia in Kallman’s syndrome, [13] midline brain disorders in septo-optic dysplasia and holoprosencephaly, [14] adrenal hypoplasia, [15] or with other syndromes such as Prader-Willi, Bardet-Biedl, and CHARGE syndromes.
Several genes have been linked to the pathogenesis of congenital gonadotrophin deficiency, including KAL1, FGFR1, GPR54, Kiss-1, GnRHR, PROK2, PROKR2, NELF, DAX1, LH and FSH beta-subunit, leptin, leptin receptor, and prohormone convertase 1 (PC1) genes. [16]
Acquired forms of gonadotrophin deficiency are associated with intracranial trauma, tumours, surgery, or radiotherapy. [17] Histiocytosis, sickle cell disease, and iron overload (associated with transfusion) can result in permanent gonadotrophin deficiency. [18]
Gonadal disorders
Congenital disorders include cryptorchidism/anorchia.
Chromosomal disorders include Klinefelter’s syndrome (XXY), [19] Turner’s syndrome (45XO), [20] XY gonadal dysgenesis, and 45X/46XY mixed gonadal dysgenesis.
Acquired causes include testicular torsion, polyglandular autoimmune diseases, [21] chemotherapy, viral infections (such as mumps), pelvic or abdominal radiation, [22] and testicular or ovarian surgery.

985
Q

RFs for delayed puberty / constitutional delay

A

STRONG

FHx delayed puberty
Congenital pituitary structural abnormalities
Gene mutations
Chromosomal disorders [Klinefelter's syndrome (XXY), Turner's syndrome (45XO), XY gonadal dysgenesis, and 45X/46XY mixed gonadal dysgenesis are associated with dysgenetic gonads and hypergonadotrophic hypogonadism.]
Prader willi
Eating disorders
Chronic systemic illnesses
Malnutrition
Intense exercise
Cryptorchidism
Pituitary surgery 
Adrenal hypoplasia 
WEAK
Chemo/radio
Histiocytosis 
Sickle cell disease
Iron overload
986
Q

Sx of delayed puberty / constitutional delay

A

RF presence
Boy test size <3ml [4ml, longitudinal length >2.5cm = onset of puberty]
Girls - absent breast development
Absent pubic/axillary hair - Pubic and axillary hair, acne, and body odour develop as a result of androgens secreted from the adrenal gland. The onset of axillary hair occurs in mid-puberty.
Absence of menarche
Absent growth spurt
Anosmia (kallmans)

OTHER
Short statur
Dysmorphic features

987
Q

Ix for delayed puberty / constitutional delay

A

Non-dominant wrist x-ray - helps estimate skeletal age - delayed bone age seen
Basal LH/FSH - low in hypogonadotrophic hypogonadism; elevated in hypergonadotrophic hypogonadism

Consider
LHRH stimulation test - low FSH and LH levels post-stimulation in hypogonadotrophic hypogonadism
MRI brain - if basal gonadotrophins are low: normal or pituitary tumour, forebrain defects, olfactory hypoplasia
Karyotype - turners/kleinfelters
Serum ovarian autoantibodies - +ve in autoimmune endocrinopathy
TFTs - Hypothyroidism can cause hypogonadotrophic hypogonadism.
Serum prolactin - hyperprolactinaemia delays puberty
Chek other pituitarty hormones - panhypopituitarism

988
Q

Rx of delayed puberty / constitutional delay

A

CONSITUTIONAL DELAY - observe + monitor
MALE: Can give Testosterone / oxandrolone (increase growth velocity)
FEMALE: Short course of estradiol
[add cyclical progesterone to promote endothelial shedding]

TREAT ANY UNDERLYING CHRONIC ILLNESSES (EG COELIAC)

Persistent hypogonadism post-puberty = testicle / breast implants

989
Q

Prognosis / complications of delayed puberty / constitutional delay

A

Overall outlook relates to the underlying cause of delayed puberty rather than to the delay itself.

Patients with a temporary delay, such as those with constitutional delay in puberty, have an excellent prognosis and achieve normal gonadal function post-puberty without testosterone (or oestrogen) replacement therapy.

Similarly, patients with a chronic illness, malnutrition, or intense exercise typically recover normal gonadal function after resolution of the illness or exercise.

Patients with a permanent cause, such as organic gonadotrophin deficiency, Turner’s syndrome, Klinefelter’s syndrome, or previous pituitary surgery for a craniopharyngioma, require lifelong hormone therapy.

Osteoporosis
Psychological problems
Skin irritation from gels and patches
Polycythaemia

990
Q

Prognosis / complications of delayed puberty / constitutional delay

A

Overall outlook relates to the underlying cause of delayed puberty rather than to the delay itself.

Patients with a temporary delay, such as those with constitutional delay in puberty, have an excellent prognosis and achieve normal gonadal function post-puberty without testosterone (or oestrogen) replacement therapy.

Similarly, patients with a chronic illness, malnutrition, or intense exercise typically recover normal gonadal function after resolution of the illness or exercise.

Patients with a permanent cause, such as organic gonadotrophin deficiency, Turner’s syndrome, Klinefelter’s syndrome, or previous pituitary surgery for a craniopharyngioma, require lifelong hormone therapy.

Osteoporosis
Psychological problems
Skin irritation from gels and patches
Polycythaemia

991
Q

A 15-year-old boy is concerned because he is much shorter than his peers and is being bullied at school. He is reluctant to go swimming with his friends as they are all much taller than him. His 13-year-old sister is also taller than him. On examination his height is on the 2nd centile, and mid-parental height is on the 50th centile. He is pre-pubertal with testicular volumes of 3 mL bilaterally. X-ray of his wrist reveals a bone age of 12.8 years

A

Delayed puberty

992
Q

The parents of a 14-year-old girl present with concerns that she is short. On examination her height is well below the 0.4th centile and her weight is on the 25th centile. Her previous growth records indicate that her height at 2 years was on the 75th centile and at 8 years had fallen to the 25th centile. The parents are both measured at the visit and the mid-parental height is calculated to be on the 91st centile. She has slightly large ears with minimal webbing of the neck. Her birth was uneventful at 39 weeks; she weighed 3 kg and has remained well. She struggles with maths but is above average in reading and writing.

A

Delayed puberty

993
Q

Define precocious puberty

A

Puberty is an interval characterised by the acquisition of the secondary sexual characteristics, accelerated linear growth, increase in the secretion of sex hormones, maturation of gonads (testes in boys; ovaries in girls), and the potential for reproduction. It is typically complete within 2 to 5 years. Precocious puberty should be considered when secondary sexual characteristics appear before 8 years in girls and 9 years in boys.

It also results in accelerated skeletal development with a reduced adult height.

Should be considered when secondary sexual characteristics appear before 8 years of age in girls and 9 years in boys.

Results in accelerated skeletal development with a reduced adult height, and may have a psychosocial impact.

Two forms exist: gonadotrophin-dependent (GDPP; due to premature activation of the hypothalamo-pituitary-gonadal axis) and gonadotrophin-independent (GIPP; due to autonomous secretion of sex steroids).

History should be directed dependent on whether puberty is consonant or disconsonant (i.e., whether the pattern of endocrine change is the same as in normal puberty or not).

Treatment of GDPP is usually straightforward with gonadotrophin-releasing hormone agonists.

GIPP is more difficult to treat; may require ketoconazole, or aromatase inhibitors and anti-androgens.

Treatment should be stopped once an acceptable age of puberty is reached.

994
Q

Epidemiology of precocious puberty

A

1 in 5000

Gonadotrophin-dependent precocious puberty (GDPP) or central precocious puberty affects girls 10 times more commonly than boys.

995
Q

Aetiology of precocious puberty

A

Causes can be divided into those that are central or gonadotrophin-dependent, in which there is premature activation of the hypothalamo-pituitary-gonadal axis; and those due to premature activation of the ovaries, testes, or adrenal glands independent of gonadotrophin secretion (gonadotrophin-independent precocious puberty [GIPP]).

Gonadotrophin-dependent precocious puberty (GDPP):

Idiopathic (90% of females)
Brain neoplasms
Cranial radiotherapy
Neurodisability 
Post-traumatic head injury 
Child adoption/sexual abuse 

Gonadotrophin-independent precocious puberty (GIPP):

Ovarian causes: follicular cysts of the ovary, granulosa cell tumours, Leydig cell tumours, and gonadoblastoma.
Testicular causes: Leydig cell tumours and a defect of luteinising hormone (LH) receptor function (testotoxicosis or familial GIPP). The latter is caused by an activating mutation in the LH receptor gene.
Adrenal causes: 21-hydroxylase congenital adrenal hyperplasia (CAH) in males results in GIPP. CAH in females presents with signs of virilisation (e.g., pubic and axillary hair and clitoromegaly) but no breast development. Other adrenal causes include Cushing’s syndrome and an adrenal virilising tumour.
McCune-Albright syndrome (MAS)
Exposure to exogenous hormones such as the contraceptive pill or testosterone gels may be responsible for early pubertal development in some patients. Oestrogenic agents in cosmetics and food products have also been implicated in causing an earlier age of puberty; ‘epidemics’ of premature thelarche (isolated breast development) in some geographic areas may be linked to an environmental exposure to oestrogens.
HCG tumour

996
Q

RFs of precocious puberty

A
STRONG
Brain tumour
Cranial irradiation
McCune Albright syndrome
Gonadal tumours 
Congenital adrenal hyperplasia 
WEAK
\+ve FHx
Exposure to exogenous hormones
Head injury 
NF1
Hydrocephalus 
Congenital pituitary abnormalities
997
Q

Sx of precocious puberty

A
COMMON
Boys testes >4ml / 2.5cm
Girls breast development
Pubic/axilliary hair
Menarche
Increased growth velocity
Tall stature

OTHER UNCOMMON

Cafe au lat
Sx of autonomous endocrine hyperfunction
Polyostotic fibrous dysplasia
Facial dysmorphism
Clitoromegaly 
Eye abnormalities
Motor deficits
Abnormal head size
998
Q

Ix for precocious puberty

A

Bone age assessment - enhanced
Basal LH/FSH - low in gonadotrophin-independent precocious puberty (GIPP); elevated in gonadotrophin-dependent precocious puberty (GDPP)
Serum testosterone/oestrogen - high
US pelvis - ovarian cyst(s), tumours, uterine enlargement ± endometrial echo
LHRH stimulation - elevated gonadotrophins in GDPP; suppressed gonadotrophins in GIPP

999
Q

Rx of precocious puberty

A

GDPP
GnRH agonist - Continuous exposure to GnRH suppresses puberty, as it is only pulsatile exposure that triggers pubertal progression. GnRH agonists are the only effective treatment modality for GDPP. Leuprorelin, triptorelin, or goserelin depot preparations
+/- somatotrophin - (Treatment with gonadotrophin-releasing hormone (GnRH) agonists can lead to a reduction in the growth rate due to a reduction in GH and insulin-like growth factor 1 (IGF1) concentrations.)
Cyproterone is a peripherally-acting anti-androgen that suppresses both gonadotrophin and gonadal function.

Gonadotrophin independent:
Ketoconazole acts as an inhibitor of steroid synthesis and suppresses both gonadal and adrenal steroid production, and is typically a first-line treatment. It helps decrease testosterone levels and achieve good growth. Ketoconazole may cause severe liver injury and adrenal insufficiency.
Cyproterone is a peripherally-acting anti-androgen that suppresses both gonadotrophin and gonadal function and may be used as alternative first-line therapy.

Patients may need additional pharmacotherapy to prevent the synthesis or action of gonadal steroids. Options include aromatase inhibitors (e.g., testolactone, letrozole, anastrozole), and anti-androgens (e.g., spironolactone or bicalutamide).

CAH
Adjust hydrocortisone Rx, GNRH agonists

1000
Q

Prognosis / complications of precocious puberty

A

Short stature

Psychological problems

1001
Q

Define haemophilia

A

Haemophilia is a bleeding disorder, usually inherited with an X-linked recessive inheritance pattern, which results from the deficiency of a coagulation factor. Haemophilia A results from the deficiency of clotting factor VIII. Haemophilia B results from the deficiency of clotting factor IX. Acquired haemophilia is a separate non-inherited condition. It is much rarer than congenital haemophilia and has an autoimmune-related aetiology with no genetic inheritance pattern.

A bleeding disorder, usually inherited, characterised by the deficiency of coagulation factor VIII or IX.

Occurs almost exclusively in males due to an X-linked pattern of inheritance.

Graded as mild, moderate, or severe, based on factor VIII or IX level.

Musculoskeletal bleeding is the most common type of haemorrhage.

Treatment consists of coagulation factor VIII or IX replacement.

A major complication of treatment is the development of inhibitory antibodies against infused factor VIII or IX.

1002
Q

Epidemiology of haemophilia

A
A = 1 in 5000
B = 1 in 30,000

X linked
Therefore, boys and/or men are exclusively affected, although many female carriers (approximately 30%) have clotting factor levels in the haemophilia range due to lyonization (random inactivation of the normal X chromosome) and may have bleeding symptoms requiring appropriate management. Rare cases of girls and/or women with severe haemophilia are described because of extreme lyonization, homozygosity, mosaicism, or Turner syndrome. Acquired haemophilia is rare, with an incidence of acquired haemophilia A in the UK of 1.48 per million per year.

1003
Q

RFs for haemophilia

A

STRONG
FHx
Male sex (congenital)
Age >60 (acquired)

Autoimmune disorders, inflammatory bowel disease, diabetes, hepatitis, pregnancy, postpartum, or malignancy (acquired haemophilia)

1004
Q

Aetiology of haemophilia

A

X linked
Therefore, boys and/or men are exclusively affected, although many female carriers (approximately 30%) have clotting factor levels in the haemophilia range due to lyonization (random inactivation of the normal X chromosome) and may have bleeding symptoms requiring appropriate management. Rare cases of girls and/or women with severe haemophilia are described because of extreme lyonization, homozygosity, mosaicism, or Turner syndrome. Acquired haemophilia is rare, with an incidence of acquired haemophilia A in the UK of 1.48 per million per year.

1005
Q

Sx of haemophilia

A

Hx recurrent / severe bleeding (spontaneous or trauma induced)
Onset may be DELAYED by several days
Recurrent nasal / oral mucosal bleeding
Bleeding into muscles = hallmark = Presents with pain and swelling of the involved area, commonly extremities, with decreased range of motion, erythema, and increased local warmth
Prolonged bleeding following heel prick
Haemoarthrosis

UNCOMMON
3-5% present with intracranial bleeding - Symptoms and signs are not specific, but include hypoactivity, decreased oral intake, irritability, bulging/tense fontanelle, seizures, and pallor.

OTHER COMMON
Fatigue (anaemia signs)
Menorrhagia (may present if woman carrier)
Extensive cutaneous purport (acquired haemophilia)

UNCOMMON
GI bleeding + haematuria
Distended/painful abdomen
Pallor, tachycardia, tachypnoea, hypotension

1006
Q

Ix for haemophilia

A
FBC - rule out thrombocytopenia
APTT - prolonged 
Mixing study  - differentiates between acquired and inherited haemophilia - pts plasma mixed with normal plasma. In acquired the prolonged APTT cannot be corrected.
Plasma F8/9 deficiency = decreased/absent in A/B
PT time - Normal [VWD]
Plasma VWF assay - N
Factor 5/7 assay - N
Bleeding time - checks platelet function
Serum AST/ALT - N
Plain Xray - if haemoarthrosis
1007
Q

Rx for haemophilia

A

HAEMOPHILIA A
1. Desmopressin (increases production of F8 (as well as VWF)
+ Antifibronolytic - tranexamic acid / aminocaproic acid
2. F8 concentrate - octocog alfa / efmoroctocog alfa
+ antifibrinolytic

HAEMOPHILIA B
1. Coagulation factor IX (recombinant): haemophilia B: consult specialist for guidance on dose
+ antifibrinolytic

FAILURE:
Can move up to high-dose factor concentrates

Recurrent haemoarthosis:
Radioactive synovectomy

Acquired Rx:
Replace 8/9
Potentially use bypassing agents
+ IMMUNOSUPPRESION WITH PREDNISOLONE

1008
Q

Complications / prognosis of haemophilia

A
Compartment syndrome
Allergic reaction to infused product
Joint damage
Blleding/life threatening haemorrhage
Devopment of Rx related inhibitors to factor 8/9
Blood borne infections

Advances in treatment over the last several decades have allowed a near-normal lifestyle and lifespan for most patients with haemophilia A and B. Preservation of joint function may be achieved, even in patients with severe haemophilia A or B, with the use of prophylaxis (intravenous factor replacement given for at least 45 weeks/year in anticipation of, and to prevent, bleeding). However, the high cost of prophylaxis has prevented its implementation in developing countries. Therefore, complications of haemophilia, such as chronic joint arthropathy, are still a common occurrence, primarily in those who are not receiving prophylactic factor infusions or have developed inhibitors to factor VIII or factor IX.

1009
Q

Define ITP

A

Immune thrombocytopenic purpura (ITP) is a clinical syndrome in which a decreased number of circulating platelets (thrombocytopenia) (see the image below) manifests as a bleeding tendency, easy bruising (purpura), or extravasation of blood from capillaries into skin and mucous membranes (petechiae). Although most cases of acute ITP, particularly in children, are mild and self-limited, intracranial hemorrhage may occur when the platelet count drops below 10 × 109/L (< 10 × 103/µL); [1] this occurs in 0.5-1% of children, and half of these cases are fatal.

1010
Q

Epidemiology of ITP

A

3 per 100k
Children + >60yos
Peaks in winter
Persistence or chronicity occurred in 36% of children compared with 67% of adults. In adults, 18% of ITP cases were secondary, with malignancy the main cause.

1011
Q

Aetiology/RFs of ITP

A

In children, most cases of immune thrombocytopenic purpura (ITP) are acute, manifesting a few weeks after a viral illness. In adults, most cases of ITP are chronic, manifesting with an insidious onset, and occur in middle-aged women. These clinical presentations suggest that the triggering events may be different. However, in both children and adults, the cause of thrombocytopenia (destruction of antibody-coated platelets by mononuclear macrophages) appears to be similar.

RFs
Child
>60
Winter
Viral illness preceding 
Malignancy (adults)
Child - M>F
Elderly - F>M
1012
Q

Sx of ITP

A

ITP is a primary illness occurring in an otherwise healthy person. Signs of chronic disease, infection, wasting, or poor nutrition indicate that the patient has another illness. Splenomegaly excludes the diagnosis of ITP.
An initial impression of the severity of ITP is formed by examining the skin and mucous membranes, as follows:
Widespread petechiae and ecchymoses, oozing from a venipuncture site, gingival bleeding, and hemorrhagic bullae indicate that the patient is at risk for a serious bleeding complication
If the patient’s blood pressure was taken recently, petechiae may be observed under and distal to the area where the cuff was placed and inflated
Suction-type electrocardiograph (ECG) leads may induce petechiae
Petechiae over the ankles in ambulatory patients or on the back in bedridden ones suggest mild thrombocytopenia and a relatively low risk for a serious bleeding complication
Findings suggestive of intracranial hemorrhage include the following:
Headache, blurred vision, somnolence, or loss of consciousness
Hypertension and bradycardia, which may be signs of increased intracranial pressure
On neurologic examination, any asymmetrical finding of recent onset
On fundoscopic examination, blurring of the optic disc margins or retinal hemorrhage

1013
Q

Ix for ITP

A

FBC - isolated thrombocytopenia
Blood smear - normal morphology of RBC/WBCs, normal platelet morphology but reduced numbers. May see large platelets

Many children with acute ITP have an increased number of normal or atypical lymphocytes on the peripheral smear, reflecting a recent viral illness. Clumps of platelets on a peripheral smear prepared from ethylenediaminetetraacetic acid (EDTA)–anticoagulated blood are evidence of pseudothrombocytopenia.

In children, bone marrow examination is not required except in patients with atypical hematologic findings, such as immature cells on the peripheral smear or persistent neutropenia..

No single laboratory result or clinical finding establishes a diagnosis of ITP; it is a diagnosis of exclusion. The differential diagnosis includes such other causes of thrombocytopenia as leukemia, myelophthisic marrow infiltration, myelodysplasia, aplastic anemia, and adverse drug reactions. Pseudothrombocytopenia due to platelet clumping is also a diagnostic consideration.

1014
Q

Rx of ITP

A

ITP has no cure, and relapses may occur years after seemingly successful medical or surgical management.
Most children do not require rx - will resolve spontaneously

Treatments possible:

  1. Oral prednisone, IV methylprednisolone, or high-dose dexamethasone may be used
  2. IVIG
  3. Rituximab
1015
Q

Complications / prognosis ITP

A

More than 80% of children with untreated immune thrombocytopenic purpura (ITP) have a spontaneous recovery with completely normal platelet counts in 2-8 weeks. Fatal bleeding occurs in 0.9% upon initial presentation.

Most cases of intracranial hemorrhage occur in patients whose platelet counts are less than 10 × 109/L (< 10 × 103/µL).

This situation occurs in 0.5-1% of cases in children, and half of those are fatal

1016
Q

An 18-month-old boy presents with left ankle swelling and pain. He has limited range of motion at the ankle and has difficulty walking. Over the last year, he has presented with significant haematomas at immunisation sites. He also had prolonged bleeding after heel prick for neonatal screening tests.

A

Haemophilia

1017
Q

A 6-year-old boy presents with prolonged bleeding after trauma to the oral cavity.

A

Haemophilia

1018
Q

The age of presentation and bleeding frequency are influenced by the severity of the condition. Most patients are diagnosed as children. Less commonly, haemophilia may present in a newborn child. Signs of intracranial haemorrhage in a newborn child may include hypoactivity, decreased oral intake, and a bulging/tense fontanelle. Some patients may go undiagnosed until adulthood, particularly those with mild or even moderate haemophilia who have had no significant haemostatic challenges in their lives. Haemophilia is usually an inherited disorder. However, an acquired form occasionally occurs, typically in the elderly. Acquired haemophilia may present with bleeding into the skin (purpura), soft tissues, and mucous membranes. Musculoskeletal bleeding is less common than in congenital haemophilia. There is an association between acquired haemophilia and other conditions such as autoimmune disorders, pregnancy, and malignancy.

A

Haemophilia

1019
Q

Define ID anaemia in children

A

Anaemia in childhood is defined as a haemoglobin (Hb) concentration below established cut-off levels.

These levels are <11 g/dL in children aged 6-59 months, <11.5 g/dL in children aged 5-11 years and 12 g/dL in older children (aged 12-14)[1].

1020
Q

Aetiology of ID anaemia in children

A

WW = dietary mainly

1021
Q

Sx of ID anaemia in children

A

May be asymptomatic.
Fatigue.
Shortness of breath.
Left upper quadrant pain (if there is associated splenomegaly).
Right upper quadrant pain (secondary to cholelithiasis in haemolytic anaemia).
Failure to thrive.
Symptoms related to underlying disease pathology - eg, acute pain in sickle cell crises and chronic/recurrent diarrhoea, which can suggest a malabsorption syndrome such as coeliac disease.

Cardiovascular system - look for exertional tachycardia, a systolic flow murmur. Gallop rhythm, cardiomegaly and hepatomegaly are signs of congestive cardiac failure.
Plot weight, height and head circumference - in chronic anaemia, growth can be affected, usually with preservation of head circumference.
Pallor - examine conjunctivae, nail beds, palmar creases.
Petechiae and bruising - may be a sign of thrombocytopenia secondary to marrow aplasia or malignancy or of vasculitis.
Splenomegaly - this occurs in chronic haemolysis and malignancy.
Papulovesicular lesions on the feet - this may indicate hookworm infestation.
Dysmorphic features of Fanconi’s anaemia - small stature, small head, frontal bossing, absent thumbs, hyperpigmented skin.
Jaundice - this may be present in haemolytic anaemias.

1022
Q

Ix of ID anaemia in children

A

FBC - dec Hb Hct
Low MCV - ID
Reticulocyte count - H
Iron, ferritin and total iron binding capacity levels - Low, Low, High
Check coeliac
Blood film - anisocytosis, poikilocytosis, hypochromic cells, pencil cells

1023
Q

Rx of ID anaemia in children

A

Ferrous sulfate - For Child 6–17 years
200 mg 2–3 times a day

The haemoglobin concentration should rise by about 100–200 mg/ 100 mL (1–2 g/litre) per day or 2 g/100 mL (20 g/litre) over 3–4 weeks. When the haemoglobin is in the normal range, treatment should be continued for a further 3 months to replenish the iron stores. Epithelial tissue changes such as atrophic glossitis and koilonychia are usually improved, but the response is often slow.

1024
Q

Complications / prognosis of ID anaemia in children

A

Most children have a good capacity for tolerating low Hb levels and the morbidity of childhood anaemia is usually related to the primary disease process rather than the anaemia itself.

1025
Q

Define sickle cell anaemia

A

Sickle cell anaemia (sickle cell disease) is caused by an autosomal-recessive single gene defect in the beta chain of haemoglobin (HbA), which results in production of sickle cell haemoglobin (HbS). Other forms of sickle cell disease may occur if HbS is inherited from one parent and another abnormal haemoglobin or beta thalassaemia is inherited from the other parent (e.g., HbSB-thal or HbSC). Sickle cell anaemia is associated with varying degrees of anaemia, red cell haemolysis, and obstruction of small blood capillaries causing painful crises, damage to major organs, and increased vulnerability to severe infections. Sickle cell trait occurs if HbS is inherited from one parent and the normal HbA from the other.

Disease of red blood cells caused by an autosomal-recessive single gene defect in the beta chain of haemoglobin (HbA), which results in sickle cell haemoglobin (HbS).

Sickle cells can obstruct blood flow and break down prematurely, and are associated with varying degrees of anaemia.

Obstruction of small blood capillaries can cause painful crises, damage to major organs, and increased vulnerability to severe infections.

Associated with lifelong morbidity and reduced life expectancy.

All infants are screened, with findings confirmed by haemoglobin electrophoresis, full blood count, reticulocyte count, and peripheral blood smear.

Treatment goals include symptom control (including pain management), and prevention and management of complications.

1026
Q

Aetiology of sickle cell anaemia

A

Sickling occurs when red blood cells containing sickle haemoglobin become rigid and distorted into a crescent shape. In sickle cell anaemia, valine replaces glutamic acid at the sixth amino acid of the beta globin chain, as a result of a recessive single gene mutation. Valine can fit into the hydrophobic pocket of another haemoglobin molecule, causing the haemoglobin to polymerise within the red cell, thus forming long stiff fibres of haemoglobin tetramers.

Sickle cell trait occurs when a child inherits a sickle gene from one parent and a normal gene from the other parent. Sickle cell disease occurs when a child inherits a sickle gene from each parent.

Sickle beta-thalassaemia occurs when a child inherits a sickle gene from one parent and a beta-thalassaemia gene from the other parent. In patients with sickle beta-thalassaemia, a mutation of the beta gene blocks production of the normal beta globin chain (beta 0) or reduces its production (beta +).

Haemoglobin SC disease occurs when a child inherits a sickle gene from one parent and haemoglobin C mutation from the other parent. The haemoglobin C mutation is an amino acid substitution of lysine for glutamic acid at the 6th position of the beta globin chain. Those with haemoglobin SC disease make no haemoglobin A.

1027
Q

Epidemiology of sickle cell anaemia

A

1 in 2000 births
8% black people carry the gene
Prevalence is 10% to 30% in sub-Saharan Africa

1028
Q

RFs for sickle cell anaemia

A

Sickle cell is an autosomal-recessive trait. Sickle cell anaemia occurs in 1 in 4 pregnancies when both parents carry the sickle cell gene.

1029
Q

Sx of sickle cell anaemia

A

COMMON
Parents with SCD/trait
Persistent pain in skeleton, chest or abdomen
Diactylitis

OTHER COMMON
High temperature
Pneumonia like syndrome
Bone pain
Visual floaters
Tachypnoea
Failure to thrive
Pallor
Jaundice 
Tachycardia
Lethargy 
UNCOMMON
Maxillary hypertrophy with overbite 
Protuberant abdomen 
Cardiac systolic murmur (in infants secondary to anaemia)
Shock - splenic sequestration
1030
Q

Ix for sickle cell anaemia

A

DNA Assay - replacement of both beta haemoglobin subunits with HbS is diagnostic for sickle cell anaemia (HbSS); presence of 1 normal beta haemoglobin subunit and 1 HbS is diagnostic for sickle cell trait

Cellulose acetate electrophoresis - sickle cell anaemia 75% to 95% HbS, HbA is notably absent; sickle cell trait 40% HbS, <2% HbF, 60% HbA

HPLC - High-performance liquid chromatography (HPLC)

Peripheral blood smear - presence of nucleated red blood cells, sickle-shaped cells, and Howell-Jolly bodies

FBC + reticulocyte - some degree of anaemia occurs in most patients

Iron studies - serum iron, transferrin, ferritin levels, and serum iron-binding capacity are normal or elevated

Do CXR if suspecting ACS

1031
Q

Rx of sickle cell anaemia

A

This can be achieved through: early diagnosis, pneumococcal immunisation, antibiotic prophylaxis with penicillin in children under 5 years of age, nutritional counselling, and prompt treatment when infections do occur.

Counselling of patient and/or parents to avoid triggers (e.g., dehydration, cold and high altitudes, and strenuous exercise).

Hydroxycarbamide (>2 years old) - (NB complication neutropenia)

L-glutamine (>5yo) in sickle cell anaemia is unclear, but it is thought to decrease susceptibility of sickle erythrocytes to oxidative damage by raising the redox potential within these cells

Blood transfusions
BM transplantation

____________________

Vaso-occlusive crisis Rx:
Analgesia
Antihistamine (opiate itchiness)
Blood transfusion
ABx

____________________

Acute chest syndrome Rx:
Oxygen + incentive spirometry
Analgesia 
Antihistamine
BS-ABx
Blood transfusion
Hydration
1032
Q

Complications of sickle cell anaemia

A
Anaemia
Opioid dependence 
Iron overload
Liver complications
Cholelithiasis
Avascular necrosis of hip/shoulder
Dactylitis
Leg ulcers
CV manifestations
Priapism
Splenic / pulmonary sequesteration
Pulmonary HTN
Renal abnormalities
Growth/development delay 
Stroke
Proliferative retinopathy 
Transient red cell aplasia
Malaria
1033
Q

Prognosis of sickle cell anaemia

A

Chronic organ damage secondary to sickle cell disease results in many medical complications, although some prophylactic treatments can reduce the incidence of these. Without bone marrow transplantation (the only potentially curative treatment), median age at death is 42 years for men and 48 years for women in patients with sickle cell anaemia (SS), and 60 years for men and 68 years for women with haemoglobin SC disease.

1034
Q

A 6-month-old boy with no previous medical problems presents with fever and painful swelling of the hands and feet. His parents are concerned because he has been inconsolable for 6 hours. The infant has been refusing bottles and has needed fewer nappy changes over the last 2 days.

A

SCD

1035
Q

A 24-year-old woman with known sickle cell disease presents with a 3-day history of cough productive of white sputum, nausea, and poor appetite. She also has chest and hip pain unalleviated by paracetamol or ibuprofen.

A

SCD

1036
Q

Most cases are diagnosed through neonatal screening programmes. Very young children may present with jaundice, haemolysis, or splenic sequestration crisis. For children older than 4 months, presentation may include swelling of the joints, especially dactylitis, leukocytosis in the absence of infection, protuberant abdomen (often with umbilical hernia), cardiac systolic flow murmur, and maxillary hypertrophy with overbite. For all patients, acute painful episodes and haemolysis are characteristic of the disease. Patients with haemoglobin SC disease may have fewer painful episodes but are at higher risk of retinal haemorrhage. In patients born before widespread newborn screening was available, this can be the first presentation of the disease.

A

SCD

1037
Q

Define Thalassaemia

A

Thalassemias are inherited blood disorders characterized by abnormal hemoglobin production. Often there is mild to severe anemia (low red blood cells).

Anemia can result in feeling tired and pale skin. There may also be bone problems, an enlarged spleen, yellowish skin, and dark urine. Slow growth may occur in children.

There are two main types, alpha thalassemia and beta thalassemia. The severity of alpha and beta thalassemia depends on how many of the four genes for alpha globin or two genes for beta globin are missing.

Diagnosis is typically by blood tests including a complete blood count, special hemoglobin tests, and genetic tests. Diagnosis may occur before birth through prenatal testing.

1038
Q

Epidemiology of Thalassaemia

A
280ml WW
440k severe disease
It is most common among people of Italian, Greek, Middle Eastern, South Asian, and African descent
M=F
Confers malaria protection
1039
Q

Aetiology of Thalassaemia

A

Both α- and β-thalassemias are often inherited in an autosomal recessive manner.

For the autosomal recessive forms of the disease, both parents must be carriers for a child to be affected. If both parents carry a hemoglobinopathy trait, the risk is 25% for each pregnancy for an affected child.

The α-thalassemias involve the genes HBA1 and HBA2,inherited in a Mendelian recessive fashion. Two gene loci and so four alleles exist. It is also connected to the deletion of the 16p chromosome. α Thalassemias result in decreased alpha-globin production, therefore fewer alpha-globin chains are produced, resulting in an excess of β chains in adults and excess γ chains in newborns. The excess β chains form unstable tetramers (called hemoglobin H or HbH of 4 beta chains), which have abnormal oxygen dissociation curves.

Beta thalassemias are due to mutations in the HBB gene on chromosome 11, also inherited in an autosomal, recessive fashion. The severity of the disease depends on the nature of the mutation and on the presence of mutations in one or both alleles.
Mutated alleles are called β+ when partial function is conserved (either the protein has a reduced function, or it functions normally but is produced in reduced quantity) or βo, when no functioning protein is produced.
The situation of both alleles determines the clinical picture:

β thalassemia major (Mediterranean anemia or Cooley anemia) is caused by a βo/βo genotype. No functional β chains are produced, and thus no hemoglobin A can be assembled. This is the most severe form of β-thalassemia;
β thalassemia intermedia is caused by a β+/βo or β+/β+ genotype. In this form, some hemoglobin A is produced;
β thalassemia minor is caused by a β/βo or β/β+ genotype. Only one of the two β globin alleles contains a mutation, so β chain production is not terribly compromised and patients may be relatively asymptomatic.

1040
Q

RFs of Thalassaemia

A

Carrier / affected parents

Italian, Greek, Middle Eastern, South Asian, and African descent

1041
Q

Sx of Thalassaemia

A
Iron overload
Infection (especially with splenectomy)
Bone deformities 
Splenomegaly 
Reduced growth 
Heart problems: Diseases, such as congestive heart failure and abnormal heart rhythms, may be associated with severe thalassemia.
1042
Q

Ix for Thalassaemia

A

Thalassemia can be diagnosed via a complete blood count, hemoglobin electrophoresis, and DNA testing.

1043
Q

Rx of Thalassaemia

A

Genetic counselling
Warn that traits may be misdiagnosed as ID anaemia

Severe thalassaemia - regular blood transfusions
Iron chelation therapy - deferoxamine, deferiprone, or deferasirox

BM transplantation

1044
Q

Prognosis / complications of Thalassaemia

A

The major causes of morbidity and mortality in thalassemia are anemia and iron overload. The severe anemia resulting from this disease, if untreated, can result in high-output cardiac failure; the intramedullary erythroid expansion may result in associated skeletal changes such as cortical bone thinning. The long-term increase in red-cell turnover causes hyperbilirubinemia and bilirubin-containing gallstones.
Increased iron deposition resulting from lifelong transfusions and enhanced iron absorption results in secondary iron overload. This overload causes clinical problems similar to those observed with primary hemochromatosis (eg, endocrine dysfunction, liver dysfunction, cardiac dysfunction).

1045
Q

Define rickets

A

Rickets refers to changes caused by deficient mineralisation at the growth plate of long bones. Osteomalacia refers to impaired mineralisation of the bone matrix. Rickets and osteomalacia usually occur together while the growth plates are open. Osteomalacia can also occur after the growth plates have fused. Rickets can manifest in childhood at the distal forearm, knee, and costochondral joints, as these are sites of rapid bone growth, where large quantities of calcium and phosphorus are required for mineralisation. Characteristic features include widening of the bones at the wrists and knees, bowing of the legs, spine deformities, fractures, bone pain, and dental abnormalities.

Rickets is deficient mineralisation at the growth plate of long bones, resulting in growth retardation. If the underlying condition is not treated, bone deformity occurs, typically causing bowed legs and thickening of the ends of long bones.

Rickets only occurs in growing children before fusion of the epiphyses, and typically affects the wrists, knees, and costochondral junctions.

Rickets occurs primarily as a result of a nutritional deficiency of vitamin D, but can be associated with nutritional deficiencies of calcium or phosphorus. Hypophosphataemic rickets is a common genetic cause of rickets.

The mainstay of treatment is to correct vitamin D deficiency and to ensure adequate calcium intake.

Vitamin D deficient rickets can be prevented in many cases by ensuring that children and pregnant women have sufficient vitamin D and calcium intake.

1046
Q

Epidemiology of rickets

A

5 per 100k
Majority are black or breat fed
Peak incidence is 6-18months

Globally, nutritional deficiencies are the leading cause of rickets, followed by vitamin D-dependent, vitamin D-resistant, and renal rickets.

Surprisingly, in the sunniest areas of the world, rickets is still a major health problem. Reasons for this include burka-wearing practices, avoidance of exposure of any skin to sunlight, and foods unfortified with vitamin D. Darker-pigmented people also require more sun exposure than lighter-skinned people, to convert bioactive vitamin D.

1047
Q

Aetiology of rickets

A

Vitamin D metabolism is dependent on sunlight and enzymatic conversion in the liver and kidneys. Ultraviolet light converts 7-dehydrocholesterol to colecalciferol (vitamin D3). Colecalciferol may also be obtained from the diet. Colecalciferol is converted to calcidiol (25-hydroxyvitamin D) by vitamin D 25-hydroxylase in the liver. Calcidiol is converted to calcitriol (1,25-dihydroxyvitamin D) by 1-alpha-hydroxylase in the kidney. Ergocalciferol (vitamin D2) is obtained from the diet and is metabolised in a similar manner as colecalciferol.

The main reasons for inadequate vitamin D supply in infants from Western countries are prolonged breastfeeding without vitamin D supplementation and concomitant avoidance of sun exposure. At latitudes above 40º north or south, no effective synthesis of colecalciferol occurs in winter months, so deficiency is more common.

The recommended intake of vitamin D to prevent deficiency in normal infants and young children was 200 units/day. This has been updated to 600 units/day by the US Institute of Medicine. Human breast milk typically contains <25 units vitamin D per litre unless the mother receives vitamin D supplementation.

Phosphate-deficient rickets is almost always caused by renal phosphate wasting. Fibroblast growth factor-23 (FGF-23) is a hormone secreted by osteoblasts, osteoclasts, and osteocytes that decreases phosphate reuptake in the kidney. Genetic hypophosphataemic rickets, oncogenic hypophosphataemic rickets, and rickets associated with McCune-Albright syndrome result from excessive blood levels of FGF-23. Hereditary hypophosphataemic rickets with hypercalcuria, Fanconi syndrome, and renal tubular acidosis type II are caused by mutations in ion transporter genes. Nutritional deficiency of phosphate is a rare cause of rickets. Phosphate is abundant in most diets but may not be sufficient in growing infants, or it may be precipitated in the stomach by antacids.

Medications that can cause calcium and phosphorus deficiency include loop diuretics and corticosteroids, and phenytoin can cause target organ resistance to calcitriol.

1048
Q

RFs for rickets

A
STRONG
6-18m
Dec sunlight exposure
Breast feeding
Calcium deficiency 
Phosphate deficiency
FHx
Antacids, loop diuretics, corticosteroids, anticonvulsants 

WEAK
Darker skin

1049
Q

Sx of rickets

A
Bone pain
Growth retardation
Delayed achievment of motor milestones
Bony deformities
Muscle weakness
UNCOMMON
Carpopedal spasm - hypocalcaemia/phosphataemia
Numbness parasthesias - hypocalcaemia
Tetany - hypocalcaemia
Seizures - hypocalcaemia
1050
Q

Ix for rickets

A

X-ray - widening of the epiphyseal plate, loss of definition of the zone of provisional calcification at the epiphyseal/metaphyseal interface, cupping, splaying, and fraying of the metaphysis; Looser’s zone (pseudofracture)

Serum calcium - may be decreased in hypocalcaemic rickets; normal in hypophosphataemic rickets

Serum phosphate - may be decreased in hypocalcaemic rickets; decreased in hypophosphataemic rickets

Serum PTH - high with hypocalcaemic rickets and normal with hypophosphataemic rickets

25-hydroxyvitamin D levels (calcidiol) - low in vitamin D-deficient rickets

ALP + LFTs - alkaline phosphatase is elevated in rickets

Serum urea + creatinine - raised in rickets caused by kidney disease

Urinary calcium and phosphorus - calcium is decreased and phosphorus is increased in hypocalcaemic rickets; calcium is normal and phosphorus is high in hypophosphataemic rickets

1051
Q

Rx of rickets

A

Symptomatic hypocalcaemia - hospitalise

Vit D def rickets/calcium deficient:
Calcium + ergocholecalciferol / cholecalciferol

Pseudovitamin D deficient:
(Occurs due to a defect in 1-alpha-hydroxylase, the enzyme that is responsible for the conversion of 25-hydroxyvitamin D into the active metabolite.)
Calcitriol

Replace phosphate via sodium phosphate in x-linked hypophosphataemic rickets, alongside calcitriol

1052
Q

Complications / prognosis of rickets

A

Fractures
Hungry bone syndrome - Worsening of hypocalcaemia after the start of vitamin D therapy for hypocalcaemic rickets may occur.

Possible adverse effects of vitamin D therapy include hypercalciuria, hypercalcaemia, nephrocalcinosis, and intra-ocular calcifications.

Calcium-deficiency rickets: vitamin D deficiency
Most children with vitamin D-deficient rickets will respond well to vitamin D and calcium, although response rates are higher with intramuscular than with oral treatment. [4]

Calcium-deficiency rickets: calcium deficiency
Calcium supplementation leads to relief of bone pain within 1 month, with improved mobility. Wrist enlargement may resolve within 6 months, although knee deformity may not resolve spontaneously. [33]

Calcium-deficiency rickets: pseudovitamin D deficiency
A physiological dose of calcitriol generally promotes complete healing of the bone disease and resolution of the biochemical abnormalities.

Calcium-deficiency rickets: vitamin D resistance
Not all patients respond to oral treatment, but metabolic and bone abnormalities may improve with intravenous calcium. [34]

Hypophosphataemic rickets: X-linked, autosomal dominant, autosomal recessive, McCune-Albright syndrome
Many patients have impaired growth and bone healing despite treatment. Outcomes are better when treatment is started in early infancy, but skeletal development remains abnormal and early growth deficits may be permanent. [35] [36]

Hereditary hypophosphataemic rickets with hypercalciuria
Patients may respond to treatment. Spontaneous improvement in the renal phosphate-wasting defect has been reported in later life. [37]

Hypophosphataemic rickets: tumour-induced
Surgical removal of the tumour can cure rickets, but not all children have a complete response. [36]

Management of deformity
Untreated rickets can cause permanent bone deformity and lead to stunted growth. Surgical intervention may be necessary to repair severe bony abnormalities.

1053
Q

A 24-month-old girl has failure to thrive and an unusual gait. She has bowed legs, thick wrists, and dental caries. Her weight (8 kg) and height (72.5 cm) are below the 3rd percentiles for her age. Her diet consists predominantly of breastfeeding 5 times daily. The patient’s antenatal, delivery, and post-natal history are unremarkable. She lives with her parents. Laboratory studies reveal elevated total alkaline phosphatase and elevated intact parathyroid hormone level. The 25-hydroxyvitamin D level is decreased. Plain x-rays of the patient’s knees and wrists demonstrate osseous changes including metaphyseal cupping and flaring, epiphyseal irregularities, and widening of the physeal plates.

A

rickets

1054
Q

A 13-year-old girl presents to the emergency department with 4-day history of knee pain. She cannot recall any injury to account for her symptoms. She has localised tenderness of the distal femur with no swelling, warmth, or erythema. The knee joint itself is unremarkable, with no effusion and full range of movement. Plain x-rays of the knee show generalised reduced bone density with cortical thinning, coarse trabecular pattern, and a Looser’s zone (pseudofracture). Blood testing shows hypocalcaemia, low levels of vitamin D, elevated levels of parathyroid hormone, and high levels of alkaline phosphatase. Liver and renal chemistries are otherwise unremarkable.

A

rickets

1055
Q

Other presentations of rickets include hypoplasia of the dental enamel; delay in the closure of the fontanelles; parietal and frontal bossing; craniotabes (soft skull bones); rachitic rosary (enlargement of the costochondral junction, visible as beading along the anterolateral aspects of the chest); and a Harrison’s groove, caused by the muscular pull of the diaphragmatic attachments to the lower ribs. Hypocalcaemic rickets may affect the musculoskeletal system, with decreased muscle tone, leading to delayed achievement of motor milestones, and result in predisposition to infections. Hypocalcaemia may also present with carpopedal spasms, tetany, or seizures.

Hypophosphataemic rickets has highly variable clinical characteristics and may not be diagnosed until adulthood. Adults may present with short stature, osteomalacia, bowed legs, lower leg pain, arthritis, stress fractures, dental caries, dental abscesses, and calcification of the tendons and ligaments (enthesopathy).

A

rickets

1056
Q

Define obesity in children

A

Obesity is a condition of excessive body fat or adiposity that exceeds healthy limits. The most widely accepted method to screen for excess adiposity is calculation of body mass index (BMI). Abnormal BMI cut-offs in children are determined by age- and sex-specific percentiles based on growth charts, as the amount of body fat changes with age and differs between boys and girls. Worldwide, the definition of overweight and obesity changes; however, a BMI >85th percentile is defined as overweight or at risk for overweight in the US and UK, a BMI ≥95th percentile is variably defined as obesity or overweight, and a BMI >99th percentile is defined as severe obesity. For children under 2 years of age, BMI percentiles are not available; thus, obesity may be defined as a weight ≥95th percentile for height.

1057
Q

Epidemiology of obesity in children

A

Behavioural and environmental factors are primarily responsible for the dramatic increase in obesity in the past two decades, although genes play an important role in regulation of body weight.

Calculating body mass index (BMI) is the most widely accepted method of screening for obesity in children. Abnormal BMI cut-offs in children are determined by age- and sex-specific percentiles.

The dramatic increase in childhood obesity has led to a marked increase in the diagnosis of impaired glucose tolerance and type 2 diabetes mellitus in children.

Preventing excessive weight gain in children is of paramount importance in confronting the obesity epidemic, as obesity is difficult to treat at all ages, and obese children tend to become obese adults.

The mainstay of treatment is lifestyle modification to improve diet and increase physical activity. Pharmacotherapy and bariatric surgery may be considered as an adjunct to lifestyle modification in severely obese adolescents.

1058
Q

Aetiology of obesity in children

A

Obesity in children is multifactorial. Interactions among factors such as genetic predisposition, behavioural and cultural practices, and environmental influences lead to discordant energy balance, with energy intake exceeding energy expenditure, eventually leading to obesity in predisposed people. Many factors typically co-exist in a person, making it difficult to determine the impact of any one factor independently of the others on the development of obesity. Behavioural and environmental changes must play an important role even in genetically predisposed children, as the prevalence of obesity in children has increased dramatically over the last 30 years despite a low likelihood of a rapid change in the genetic makeup of the population.

1059
Q

RFs for obesity in children

A
STRONG
Obese parents
Rapid weight gain in infancy
Weight gain in early childhood
Black / hispanic ethnicity
Poor socio-E
Sedentary lifestyle
WEAK
IUGR
Maternal GDM
Poor dietary choices
Screen time >2-3hrs/day
Sleep deprivation
1060
Q

Sx of obesity in children

A

BMI >95th centile
Increased waist-hip ratio
Increased skin fold thickness
Hypertension

1061
Q

Ix for obesity in children

A

Fasting blood glucose - screen DM 5.6-6.9 = impaired glucose tolerance
Serum lipids - to screen for dysplipidaemia
LFTs - N / elevated transaminases if NAFLD

1062
Q

Rx of obesity in children

A

BMI <95th centile = lifestyle modifications and intense exercise

95-99th centile
2-11 - lifestyle mods
12-18yo: orlistat / metformin

SUPER LAST LINE RARELY DONE:

Bariatric surgery is reserved for the most obese adolescents who have been involved in a behavioural treatment programme for ≥6 months.

It should only be considered in children who have attained Tanner 4 or 5 pubertal development and final, or near final, height with BMI ≥40 with mild comorbidities, or BMI >35 with extreme comorbidities. Children must be able to adhere to healthy diet and activity to be eligible for surgery.

The surgical approaches used most often are laparoscopic adjustable gastric banding, the Roux-en-Y gastric bypass, and vertical sleeve gastrectomy.

1063
Q

Complications of obesity in children

A
obesity in adults	long term	high

type 2 diabetes	long term	high

impaired glucose tolerance	long term	high

metabolic syndrome	long term	high

cardiovascular disease	long term	high

hypertension	long term	high

acanthosis nigricans	long term	high

hyperlipidaemia	long term	high

post-surgical complications	long term	high

polycystic ovary syndrome	long term	high

obstructive sleep apnoea	long term	medium

non-alcoholic fatty liver disease	long term	medium

psychosocial morbidities	long term	medium

cholelithiasis	long term	medium

intertrigo	long term	medium

furunculosis	long term	medium

obesity hypoventilation syndrome	long term	low

slipped capital femoral epiphysis	long term	low

Blount's disease	long term	low

asthma	long term	low

constipation	variable	medium

gastro-oesophageal reflux	variable	medium

pseudotumor cerebri
1064
Q

Prognosis of obesity in children

A

Treatment of obesity at any age is a challenge. Even with successful weight loss, children are at risk of rebounding back to or going above their previous weight. Family involvement in the weight loss regimen is imperative for successful weight loss. Obese children are at high risk of becoming obese adults, and obesity in adults is a serious health risk. Approximately 80% of children who are obese at 10 to 15 years of age will remain obese as adults.

1065
Q

Define depression in children

A

The Diagnostic and Statistical Manual of Mental Disorders, Fifth Edition (DSM-5) categorises depressive disorders in children into the following categories: major depressive disorder (MDD), persistent depressive disorder (dysthymia), disruptive mood dysregulation disorder, premenstrual dysphoric disorder, substance/medication-induced depressive disorder, depressive disorder due to another medical condition, other specified depressive disorder, and unspecified depressive disorder. This topic focuses on MDD and persistent depressive disorder (dysthymia). MDD in children is a more severe form of depressive disorder, and is characterised by at least 5 depressive symptoms, with 3 levels of severity: mild, moderate, and severe. Persistent depressive disorder is a more chronic form of depressive disorder, which is characterised by a chronic sad or irritable mood, lasting for at least 1 year, with 2 or more additional depressive symptoms.

Characterised by sad or irritable mood, anhedonia, decreased capacity to have fun, decreased self-esteem, sleep disturbance, social withdrawal or impaired social relationships, and impaired school performance.

One of the most common paediatric psychiatric disorders, especially among girls during adolescence.

At-risk children should be screened for depression. It is crucial to make an accurate diagnosis, based on a comprehensive assessment and review of the history, with input from multiple sources.

The safety of the child and others, and the duration and severity of depression, need to be evaluated carefully to help determine the appropriate level of care and treatment modality. Treatment is typically with active monitoring, specific psychotherapies, antidepressants, or a combination of these therapies.

There is an increased risk for substance abuse, suicide attempts, and completed suicide. Suicidality needs to be assessed at each healthcare encounter.

Following recovery, relapse or recurrence rate is high in the absence of continuation treatment.

1066
Q

Epidemiology of depression in children

A

prevalence = under 13 - 2.8% and among children between 13 and 18 years - 5.6%.

M=F until adolescence when F>M

1067
Q

Aetiology of depression in children

A

Childhood depression is likely to be caused by both genetic and environmental factors, and by their interactions. It is estimated that up to 40% of variance can be explained by genetic factors, and the remaining variances can be explained by environmental factors and by the interactions between genetic factors and the environment.

1068
Q

RFs for depression in children

A
STRONG
Fix
Other parental psychopathology
Personal Hx
Stress/trauma
Female
Sexual minority status
Hx chronic illness
Post natal status
Neighbourhood and social instability 
Immunosuppressive medications (corticosteroids)
Substance use/abuse
1069
Q

Sx of depression in children

A
Sad/irritable
Dec interest / anhedonia
Significant functional impairment 
No Hx manic/hypomanic episode
No Hx recent bereavement
Dec concentration
Insomnia/hypersomnia
Change of appetite/weight
Fatigue 
Worthlessness/guilt
Hopelessness
Psychomotor agitation/retardation
Somatic complaints
Social withdrawal / change of friends

Suicidal ideation
Increased substance abuse

1070
Q

Ix for depression in children

A

Clinical diagnosis

CONSIDER
Checking TSH/T4
FBC
Urine drug / pregnancy test 
B12/folate
Vitamin D
1071
Q

Rx of depression in children

A

AT risk of harm to self or others = MHC assessment

Mild:
Psychotherapy
Melatonin/dipenhydramine to help sleep

SSRI:
Fluoxetine = the choice
Sertraline
Citalopram

Once remission is achieved, whether it is after the first medication or psychotherapy treatment or after multiple treatment trials, treatment is continued for 6 to 12 months to avoid relapse, at the same dose used for acute treatment. Recommendation is 6 months for first episode, 12 months for recurrent episode

Following the period of continuation therapy, 1 to 2 years of maintenance treatment may be needed for children who are at risk of having recurrent depression (multiple episodes, chronic depression, comorbid disorders).

1072
Q

Complications / prognosis of depression in children

A

Treatment induced mania/disinhibition/agitation
Suicidal behaviour
Aggression
Substance abuse

About 50% to 60% of young people respond to first psychotherapy or medication treatment. Between 40% and 50% of young people who do not respond to at least one medication trial respond when switching to a new antidepressant, or switching to a new antidepressant plus psychotherapy.

Recurrence is high

Up to 70% of young people have a recurrence of an episode within 5 years of remission.

1073
Q

A 15-year-old girl, at a private school, presents with poor concentration. She lives with her biological mother and a 13-year-old sister. Her mother describes her as an outgoing and straight-A student until about 2 months ago. Her grades have slipped from As to Cs, and she has been feeling sad and irritable. She has started avoiding her friends, and has been worrying about her appearance and her grades. She states that she feels dumb, and that her classmates don’t like her. Recently, she started to think that life was not worth living, and wished she would fall asleep and never wake up. Her boyfriend broke up with her about 3 months ago. The last time she felt this sad was 5 years ago when her parents divorced.

A

Depression in children

1074
Q

A 9-year-old boy presents with a change in his behaviour over the past 4 weeks, from being an outgoing child who loved school to frequently complaining of stomach aches and refusing to go to school. He lives with his biological parents and a 5-year-old sister. He is attending a local school. His parents say that he has been unkind to his 5-year-old sister, and frequently screams at her. He used to like to play outside after school, but recently has stayed in his room a lot and played video games. He cannot identify any precipitants, but his parents recall that his mother was hospitalised for surgery about 3 months ago.

A

Depression in children

1075
Q

Depression in children and adolescents may sometimes present as ‘acting out’, aggression, and defiance. Depression in younger children can present as somatic complaints and school refusal. Young people with a chronic medical illness may present with decreased concern about their medical illness and/or decreased compliance with medical treatment. Careful interviews with both the child and the parents are important to discover potential causes of the presentation, other depressive symptoms, and other concurrent conditions (both medical and psychiatric) that may exacerbate the depression.

A

Depression in children

1076
Q

Define generalised seizures in children

A

With the revised and updated classification by the International League Against Epilepsy (ILAE) Commission on Classification and Terminology, generalised seizures are now understood to originate at some point within the brain and rapidly engage bilaterally distributed networks. This can include both cortical and subcortical structures and may not necessarily involve the entire cortex.

Seizures may occur as a stand-alone event or they may recur, in which case the term ‘epilepsy’ is used. This topic addresses generalised epilepsies. Previous terminology of ‘secondarily generalised’ seizure (i.e., a focal-onset seizure that spread to involve the rest of the body) has now changed to ‘focal to bilateral tonic-clonic seizure’. It is also recognised that if the onset of a seizure is unwitnessed or cannot be described, this would be an ‘unknown onset tonic-clonic seizure’.

Seizures may occur as a stand-alone event or may recur (epilepsy).

Aetiology can be structural, genetic, infectious, metabolic, immune, or unknown.

An attempt should be made to identify the type(s) of epilepsy and the epilepsy syndrome by recognising a pattern of seizure types, clinical features, and EEG characteristics.

Detailed history is of paramount importance in the diagnosis, as key diagnostic factors lie in the history as opposed to ancillary investigations.

Main treatment options will depend on the epilepsy syndrome and include anticonvulsants, a ketogenic diet, vagus nerve stimulation, and surgery as well as consideration of lifestyle factors.

1077
Q

Epidemiology of generalised seizures in children

A

The average annual rate of new cases of epilepsy throughout the world is approximately 5 to 10 cases per 10,000 children and adolescents.

It is estimated that 0.4% to 0.8% of all children will develop a type of epilepsy by 11 years of age.

The incidence of epilepsy depends on age: it is highest in the first year of life, falls between 1 and 10 years of age (40 to 50 per 100,000 per year), and falls further in adolescence (20 per 100,000 per year).

1078
Q

Aetiology of generalised seizures in children

A

Generalised seizures may be genetic (previously referred to as ‘idiopathic’), or may be due to an underlying metabolic or immune disease, or less commonly may occur in the presence of a structural abnormality of the brain (in which case there may also be focal seizures), or their aetiology may be unknown. The International League Against Epilepsy (ILAE) uses the following aetiological classification system. [1]

Genetic

Many epileptic syndromes with generalised seizures have a genetic component; this is usually not a specific inheritance pattern, but rather a complex inheritance often with age-dependent penetrance. [10] For some of these syndromes, linkage to specific chromosomes has been noted. [11] Although the exact molecular mechanisms of febrile seizures are yet to be understood, underlying mutations have been found in genes encoding the gamma-aminobutyric acid A receptor and the sodium channel (e.g., sodium channel, voltage-gated, type I, alpha subunit [SCN1A]). [12]
The previous term of ‘idiopathic generalised epilepsies’, which referred to childhood absence epilepsy, juvenile myoclonic epilepsy, juvenile absence epilepsy, and epilepsy with generalised tonic-clonic seizures alone, has now been replaced with ‘generalised genetic epilepsies’. When the generalised epilepsy cannot be classified as one of these four syndromes or evidence is lacking for a genetic basis, then the epilepsy can be classified as having an unknown basis.
Structural/metabolic

The previous term ‘symptomatic generalised epilepsy’, often used to refer to epilepsy with rapid bilateral network involvement due to an underlying cause, is now replaced by the term ‘(developmental) epileptic encephalopathy’, which can consist of generalised, focal, or a combination of both types of seizures.
Generalised seizures in such a context can be due to a known or suspected disorder of the central nervous system (CNS), can occur in certain metabolism and neurodegenerative disorders (epilepsy is then part of a generalised encephalopathy), or may be due to infection or post-infective causes, post-traumatic causes, abnormalities of brain development, chromosomal syndromes, birth asphyxia, malignancy, neurocutaneous syndromes, or autism spectrum disorder.
An example is Ohtahara syndrome, which is often caused by various brain malformations. It has also been described in association with some metabolic disorders; however, in a minority of cases the cause is not found.
Similarly, the aetiology of epileptic spasms and Lennox-Gastaut syndrome is heterogeneous; in a number of cases an underlying structural abnormality of the brain is found.
Immune

Seizures result directly from an immune disorder in which seizures are the core symptom of the disorder. An immune aetiology should be suspected when there is evidence of autoimmune-mediated CNS inflammation. Examples include anti-NMDA (N-methyl-D-aspartate) receptor encephalitis and anti-LGI1 encephalitis.
Infectious

Seizures result directly from a known infection in which seizures are the core symptom of the disorder. The most common aetiology worldwide is where epilepsy occurs as a result of an infection. An infectious aetiology refers to a patient with epilepsy, rather than with seizures occurring in the setting of acute infection such as meningitis or encephalitis. Examples include neurocysticercosis, tuberculosis, HIV, cerebral malaria, subacute sclerosing panencephalitis, cerebral toxoplasmosis, and congenital infections such as Zika virus and cytomegalovirus.
Unknown

These are seizures where the cause is not known; no assumption should be made about their aetiology.

1079
Q

RFs of generalised seizures in children

A
STRONG
Genetic predisposition or Hx
Perinatal asyphyxia
Metabolic/neurodegenerative disorders 
Head trauma
Structural CNS abnormalities
WEAK
Autism
CNS infection
Neurocutaneous syndromes
Hx of febrile seizures
1080
Q

Sx of generalised seizures in children

A

Staring spells/inattention
Tonic-clonic
Brief arrhythmic muscular jerking movements
Eyes rolling into back of head
Intercurrent illness
Unexplained falls
Incontinence
Tongue biting
Post ictal phenomena - some patients may experience post-ictal phenomena such as sleepiness, headaches, amnesia, or confusion.
Precipitated by fatigue/lack of sleep/light/noise
Developmental delay

Neurocutaneous stigmata:

Tuberous sclerosis: depigmented macules, shagreen patches, periungual fibromas, adenoma sebaceum, and ash-leaf macules.

Neurofibromatosis type 1: cafe au lait spots, axillary and inguinal freckling, and plexiform neurofibromas.

Sturge-Weber syndrome: port-wine coloured haemangiomas on the face or trunk.

1081
Q

Ix for generalised seizures in children

A

EEG - Standard diagnostic test. Measurement of electrical activity in the brain recorded from electrodes placed on scalp. Typically run for 20 minutes. - abnormal rhythms specific to epilepsy syndrome

BG - obligatory part of evaluation of any child with decreased consciousness level.

Metabolic panel - N
FBC - may be raised WCC if preceding illness
ECG - N

Consider MRI/CT head - no underlying pathology in idiopathic generalised seizures; may reveal cause in patients with symptomatic epilepsy

1082
Q

Rx of generalised seizures in children

A

ONGOING SEIZURE:
If seizure lasts more than 5 minutes, or separate seizures happen in clusters, pharmacological intervention should be considered. First-line options include rectal diazepam or buccal/intranasal midazolam.
Phenytoin possible.

Anticonvulsants:
Sodium valproate
Ethosuximide
Lamotrigine

1083
Q

Complications of generalised seizures in children

A
Worsening seizures
Developmental delay
Status epilepticus
Sudden death in epilepsy 
Evolution into other epilepsy syndromes
Psychopathology 
Cardiovascular disease
1084
Q

Prognosis of generalised seizures in children

A

There is a good prognosis in 70% to 80% of children with idiopathic epilepsy and late onset of seizures, and in those without neurological dysfunction.

Rapid response to therapy is an important predictor of lasting remission.

Immediate treatment does not reduce the risk of death in patients with a first unprovoked tonic-clonic seizure, but death risk is higher (by 6% at 10 years and 10% at 20 years) in those who have not entered 5-year remission.

1085
Q

A 10-year-old girl presents after having had a generalised tonic-clonic seizure while at school the previous day. It lasted approximately 2 minutes and she was incontinent of urine during the episode. Afterwards she complained of headache and feeling tired. She had been well prior to this episode and there is no family history of epilepsy. General physical examination including neurological assessment on the day after the seizure were both normal. An ECG was done, which was normal and showed a normal QTc interval.

A

Generalised seizures

1086
Q

A 15-year-old boy presents with a history of having had two seizures. He is healthy and has no relevant past medical history. There is no family history of epilepsy. Both episodes happened early in the morning and were self-limiting. Jerking of the whole body and all four limbs lasted <5 minutes, and he was sleepy for several hours after the episodes. His general examinations, including blood pressure, a random blood sugar, and an ECG, were normal.

A

Generalised seizures

1087
Q

Other presentations
While generalised tonic-clonic seizures are easily recognisable, in some instances a generalised seizure is not obvious. Tonic-clonic seizures that occur exclusively at night may only come to light when the patient shares a bedroom or if the accompanying noise wakes up a family member. Absence seizures are characterised by a sudden interruption in the child’s activities, often with a blank stare or inattentions. Atonic seizures are characterised by a brief loss of muscle tone causing what used to be referred to as ‘drop attacks’, whereby the patient suddenly and limply falls to the ground. Myoclonic seizures are characterised by brief, arrhythmic muscular jerking movements. Clonic seizures consist of rhythmic, muscular jerking movements with or without impaired consciousness. Tonic seizures consist of tonic extension or flexion of the extremities.

A

Generalised seizures

1088
Q

Define migraine in children

A

Migraine is defined by the International Headache Society (IHS) as a recurrent headache that occurs with or without aura and lasts for 2 to 48 hours. It is usually unilateral in nature, of gradual onset (15 to 30 minutes), pulsating in quality, of moderate or severe intensity, and is aggravated by routine physical activity. Nausea, vomiting, photophobia, and phonophobia are common accompanying symptoms.

Migraine has a high prevalence in children (10%) and is a significant source of morbidity.

Careful consideration of the broad differential diagnosis is important when evaluating a child with headache.

The expectations for the success of treatment should take account of the level to which psychological factors are contributing to symptoms.

Not all treatments (acute or prophylactic) work for every patient.

Spontaneous remission with increasing age may occur in 25% to 50% of children.

1089
Q

Epidemiology of migraine in children

A

Studies in developed countries suggest that migraine is the most common diagnosis among children presenting with headache to a medical practitioner.

The mean age at onset is 7.2 years for boys and 10.9 years for girls, with 20% of children experiencing their first attack before the age of 5 years.

It occurs in equal proportions of boys and girls before puberty, but becomes predominantly female (3:1) thereafter.

1 per 20 children

1090
Q

Aetiology of migraine in children

A

Central neuronal hyperexcitability underlies susceptibility to and development of migraine episodes.

The broad evidence base for this suggests a multi-factorial causation, with amino acids, magnesium depletion, calcium channels, and controlling genes all being implicated.

1091
Q

RFs of migraine in children

A
STRONG
FHx migraine
Chocolate cheese and citrus fruits
Stress
Hormonal changes 
WEAK
Alcohol and caffeine
Volume depletion
Analgesic overuse 
Sleep disturbance 
Bright/flickering lights
Periodic syndromes
1092
Q

Sx of migraine in children

A

Gradual onset (typically 15mins)
Normal physical examination
Aura
N+V
Visual disturbance - diplopia, blurred vision, and photophobia may be associated symptoms.
Phonophobia
Occipital headache, ataxia, diplopia, blurred vision, vertigo, and tinnitus
Unilateral motor/sensory signs - hemiplegic migraine
Ptosis - dilated pupils
Generalised seizure

1093
Q

Ix for migraine in children

A

Clinical

CONSIDER CT/MRI ONLY IF:
The presence of hypertension, bradycardia, altered consciousness level, or any focal neurological deficit suggests a more serious injury or alternative diagnosis and is an indication for urgent central nervous system (CNS) imaging.

1094
Q

Rx of migraine in children

A

Acute episode:
1.
Analgesia - paracetamol, ibuprofen, codeine phosphate
Antiemetic - cyclizine

2.
Sumatriptan nasal
Rizatriptan

Prophylaxis, consult neurologist:

  1. Propranolol
  2. Pizotifen
  3. Topiramate
  4. . Amitripyline/gabapentin/carbemazepine
  5. Verapamil / indometacin
1095
Q

Complications/prognosis of migraine in children

A

Neg effect of schooling

There are no reliable data regarding the prognosis of paediatric migraine diagnosed by International Headache Society (IHS) criteria. Spontaneous remission after puberty may occur, and migraine headaches may cease before the age of 25 years in 23% of people, although by the age of 50 years >50% of people remain affected. This study pre-dated the IHS diagnostic criteria.

There is also evidence that migraine that develops during adolescence tends to continue in adult life, although attacks tend to be less frequent and severe in later life.

1096
Q

Define tic disorders

A

Tics are brief, sudden, repetitive movements and/or sounds. They are divided into motor tics (e.g., blinking, shrugging shoulders, grimacing, or jerking) or phonic tics (e.g., throat clearing, sniffing, grunting, or more complex utterances). Patients describe an inner urge or a local premonitory sensation, which is then relieved by performing the tic. The tic can be voluntarily suppressed for short periods of time. Tics increase with stress, anxiety, and excitement, and decrease with distraction. Tic disorders may be primary (idiopathic or inherited) or secondary to certain conditions (e.g., head trauma, encephalitis).

Brief, sudden, repetitive movements and/or sounds that increase with stress, anxiety, or excitement.

Occur most commonly in children and adolescents, with boys more frequently affected than girls.

The exact pathophysiological mechanisms are unknown, but the disorders are likely to be due to disturbances of the cortico-striatal-thalamo-circuitry.

Comorbid conditions such as anxiety, obsessive-compulsive disorder, and ADHD are common.

Tics are often intermittent and mild in children and may not require treatment. Treatment is indicated only for those patients whose tics are interfering with activities of daily living or social interactions, or who experience significant tic-related physical pain.

1097
Q

Epidemiology of tic disorders

A

Children > adults
Male > Female
Increased frequency in special education populations

Provisional tic disorders (lasting <1 year) appear to be very common in childhood and have been estimated to occur in up to 20% of primary school children.

Population-based surveys indicate that the prevalence of chronic tic disorders is 1% to 3%.

Tourette’s syndrome (multiple motor and at least one phonic tic for >1 year) has a population-based prevalence in children of 0.3% to 0.9%, 3:1 M:F, onset 6-7yo

1098
Q

Aetiology of tic disorders

A

Primary tics

‘Transient tics’/ Provisional motor or phonic tics (<1 year)
Persistent (chronic) motor or phonic tics (>1 year)
Tourette’s syndrome (idiopathic or inherited)
Tic disorder without an identifiable cause (e.g., with tic onset in a person over age 18 years).
Secondary tics can be due to:

Infections (e.g., encephalitis, Creutzfeldt-Jakob disease, and Sydenham’s chorea). There is conflicting evidence concerning an association between tics and recurrent group A streptococcal infections.
Substance abuse (stimulant abuse being the most common)
Medicine (e.g., lamotrigine) [18]
Toxins (e.g., carbon monoxide)
Head trauma
Stroke
Metabolic and endocrine (chorea gravidarum, thyrotoxicosis)
Neurodegenerative (Huntington’s disease, Wilson’s disease).

1099
Q

RFs for tic disorders

A
STRONG
Age at onset <18
Male
FHx
FHx OCD/ADHD/Depression
Antenatal maternal smoking
Hx OCD / ADHD

WEAK
Autism spectrum disorder

1100
Q

Sx of tic disorders

A
RFs
Observed tics
Premonitory sensation
Ability to suppress tics
Symptoms improve when distracted
Otherwise normal near exam
Variable severity over time
Worse with stress/anxiety/excitement
1101
Q

Ix for tic disorders

A

Clinical - investigations are only required in the setting of abnormal neurological examination

CONSIDER

MRI - Indicated in patients with an abrupt onset of tics associated with other neurological symptoms, unusual patterns of tics (unilateral or associated with dystonia), onset in adulthood, or an abnormal neurological examination suggestive of secondary causes of tics.

EEG - Indicated only if the history does not help to differentiate between seizures and tics.

1102
Q

Rx of tic disorders

A

Patients and their parents should be reassured about the benign nature of tics.

They should also be aware of the tendency for tics to increase in times of stress, anxiety, and excitement.

The family and the school should try not to focus on the presence of the tics. A child with a tic disorder should not be punished for carrying out their tics.

Interference with daily life:
CBT
Habit reversal therapy HRT

Medication:
clonidine
topiramate

Atyptical neuroleptics [dopamine antagonists]:
Aripiprazole
Olanzapine
Respiradone

1103
Q

Complications / prognosis of tic disorders

A
Behavioural problems
Musculoskeletal problems 
Neuroleptic induced:
akathisia
dystonic reactions
Tardive dyskinesias
Parkinsonisms
Neuroleptic malignant syndrom
1104
Q

A 7-year-old boy presents with a 4-month history of eye blinking and sniffing. He states that his nose feels ‘weird’, but when he blinks or sniffs, the sensation goes away. His symptoms have not improved with the use of antihistamines. He is able to control the blinking and sniffing, but only for a few seconds. He blinks and sniffs more frequently when he is tired or nervous, and less frequently when he is concentrating on a task. There is no significant medical history. He currently takes no medications and he had not been on any other medication prior to the onset of symptoms. His family history is negative for asthma, allergies, seizures, or tics. On physical examination, he has normal growth parameters and vital signs. His general physical examination and neurological examination are unremarkable, except for frequent eye blinking and sniffing that is suggestible and distractible.

A

tic disorders

1105
Q

A 10-year-old boy presents with a history of intermittent barking and head jerking for over 1 year. The head jerking has now become so frequent that it causes neck pain. He sometimes also repeats what others say. At home, his mother has noticed that he spends a long time bathing and he frequently washes his hands. These behaviours worsen with stress. His medical and family histories are unremarkable. He does not take medications. His development was normal, but he has always struggled in school and has had problems with maintaining attention. He has normal growth parameters and vital signs. His physical examination is unremarkable and his neurological examination is normal except for occasional barking noises and movements of his neck, as if to look up, accompanied by shoulder shrugging.

A

tic disorders

1106
Q

Although most commonly found in children, tic disorders may also present in adults. In many such cases, more trivial tics may have been present but unrecognised during childhood. Far less commonly, tic symptoms may first present in adulthood and require comprehensive evaluation to exclude underlying medical illness or toxicity.

A

tic disorders

1107
Q

Define infantile spasm

A

Infantile spasms are the characteristic seizure type of West’s syndrome (infantile spasms, developmental plateau, and hypsarrhythmia). Age of onset is typically from 1 month to 1 year with a median age of 3 to 5 months. The seizures are characterised by an initial contraction phase followed by a more sustained tonic phase. The contractions may be flexor, extensor, or mixed flexor-extensor, with sudden, usually bilateral, symmetrical contractions of the neck, trunk, and limbs. There may be an associated brief loss of consciousness. The seizures may be subtle, consisting of brief head nods, or be precipitated by sleep onset and offset or by feeding, leading to diagnostic confusion with gastro-oesophageal reflux. They typically occur in clusters or runs of 20 to 100 spasms at a time, with visible distress. Aetiology is heterogeneous and, in a significant proportion, remains unidentified.

A hypsarrhythmic electroencephalogram pattern is characteristic, although not universally present. Onset of infantile spasms is often followed by plateauing or regression of subsequent development. West’s syndrome is thus the archetypal infantile epileptic encephalopathy. It is considered that rapid identification and treatment of the spasms improves prognosis.

1108
Q

Epidemiology of infantile spasm

A

1-2 per 4000 births incidence

M>F slightly

1109
Q

Aetiology of infantile spasm

A

More than half of those with infantile spasms have an identifiable underlying disorder such as periventricular leukomalacia, hypoxic-ischaemic encephalopathy, neuronal migration disorders, metabolic defects, and genetic syndromes such as Down syndrome. Many children who would previously have been classified as idiopathic or cryptogenic are now being diagnosed with variants in genes associated with early infantile epileptic encephalopathy (EIEE). Over 65 EIEE genes have now been identified. One third of children with tuberous sclerosis will suffer infantile spasms, and these children comprise 10% of those with infantile spasms

1110
Q

RFs of infantile spasm

A

STRONG
Brain malformation
Neurocutaneous syndrome (tuberous sclerosis)
Antenatal/perinatal vascular event
Intrauterine / perinatal infection (include herpes simplex, cytomegalovirus, toxoplasmosis, and HIV)
Inherited metabolic disorder
Genetic disorders (downs)

WEAK
FHx
Neonatal sinovenous thrombosis 
Postnatal brain injury 
Brain tumours (astrocytoma and ganglioglioma)
1111
Q

Sx of infantile spasm

A

COMMON
RFs
Spasms - Usually symmetrical, bilateral, brief contractions of the axial muscle groups and may involve the flexors, extensors, or both. They usually occur in clusters and are often seen on waking or falling asleep. Isolated spasms can occur. Spasms can be asymmetrical and may manifest only as head drops or abnormal eye movements. Occasionally spasms are associated with feeding.

Head nodding 
Neurodevelopmental delay / regression
Ash leaf macules (Sx of tub sclerosis)
Onset 3-12m
Abnormal eye movements 
Abnormal tone
1112
Q

Ix for infantile spasm

A

EEG - hypsarrhythmia or modified hypsarrhythmia
Plasma glucose - rule out hypo

To rule out other causes check:
Calcium
Magnesium 
LFTs - Elevated creatine kinase, aspartate aminotransferase, alanine aminotransferase, total bilirubin, alkaline phosphatase in CMV/toxoplasmosis
RFTs
Blood/urine cultures 
FBC - N
1113
Q

Rx of infantile spasm

A

Prednisolone (1st line)
Tetracosactide (synthetic ACTH)
Cortiocotrophin
Vigabatran

Refractory to all above:
Try Pyridoxine

Still refractory:
Try Clonazepam/nitrazepam
Or ketogenic diet

1114
Q

Complications / prognosis of infantile spasm

A

Lennox-Gastaut syndrome (an epilepsy syndrome)
Other epilepsy syndromes
Developmetnal delay
Reduced life expectancy

Adverse prognostic indicators include:

Age at onset under 3 months
Presence of pre-existing neurological impairment or developmental delay
Co-occurrence of other seizure types
Lack of response to adrenocorticotropic (ACTH) therapy.
Predictors of a favourable outcome are: [65]

No apparent aetiology
Age at onset at least 4 months
Absence of atypical spasms and focal seizures
Absence of asymmetrical electroencephalogram abnormalities
Prompt treatment and an early and sustained response to treatment
Normal psychomotor development before the onset of spasms.

1115
Q

A 7-month-old boy presents with brief episodes of head drops and flexion of the body, followed by tonic extension of arms and legs. These movements last 1 to 2 seconds and occur in several clusters per day, with 15 to 30 spasms per cluster. The head drops are mostly seen on awakening or just before falling asleep. He was born at 41 weeks’ gestation with a birth weight of 4050 grams and Apgar scores of 7 at 1 minute and 8 at 5 minutes. He had achieved normal developmental milestones but has regressed since the onset of seizures.

A

infantile spasm

1116
Q

Infantile spasms may present either with subtle head nods or merely with eye fluttering. Spasms may be asymmetrical or unilateral, when a lesional or structural cause should be suspected. [4] [5] The obvious distress following flexion of the body and stiffened clenched fists often leads to misdiagnosis as infantile colic or gastro-oesophageal reflux disease.

A

infantile spasm

1117
Q

Define intraventricular haemorrhage

A

Intraventricular haemorrhage is a haemorrhage that occurs into the ventricular system of the brain. It is relatively rare in adult surgical practice and when it does occur, it is typically associated with severe head injuries. In premature neonates it may occur spontaneously. The blood may clot and occlude CSF flow, hydrocephalus may result.

In neonatal practice the vast majority of IVH occur in the first 72 hours after birth, the aetiology is not well understood and it is suggested to occur as a result of birth trauma combined with cellular hypoxia, together the with the delicate neonatal CNS.

1118
Q

Epidemiology of intraventricular haemorrhage

A

In neonatal practice the vast majority of IVH occur in the first 72 hours after birth, the aetiology is not well understood and it is suggested to occur as a result of birth trauma combined with cellular hypoxia, together the with the delicate neonatal CNS.

1119
Q

Rx of intraventricular haemorrhage

A

Is largely supportive, therapies such as intraventricular thrombolysis and prophylactic CSF drainage have been trialled and not demonstrated to show benefit. Hydrocephalus and rising ICP is an indication for shunting.

1120
Q

Complications of intraventricular haemorrhage

A
Complications of GM/IVH include the following:
Obstructive hydrocephalus
Nonobstructive hydrocephalus
Posthemorrhagic hydrocephalus
Developmental impairment
Cerebral palsy
Seizures

GM/IVH remains a significant cause of both morbidity and mortality in infants who are born prematurely. Sequelae of GM/IVH include short- and long-term complications and can result in life-long neurologic deficits, specifically cerebral palsy, developmental delay, and seizures. GM/IVH is diagnosed primarily through the use of brain imaging studies, usually cranial ultrasonography, as shown below. Because GM/IVH can occur without clinical signs, screening and serial examinations are necessary for the diagnosis.

1121
Q

Sx of intraventricular haemorrhage

A

In the majority of patients, GM/IVH is asymptomatic and diagnosed by surveillance ultrasonography.

The presentation of GM/IVH widely varies. Most infants are asymptomatic or demonstrate subtle signs that are easily overlooked. GM/IVH is then subsequently found on surveillance sonography.
One subgroup of infants with GM/IVH presents with the following:
A sudden unexplained drop in hematocrit levels
Possible physical findings related to anemia (eg, pallor, poor perfusion) or hemorrhagic shock
Another subgroup of infants with GM/IVH presents with extreme signs, including the following:
A sudden and significant clinical deterioration associated with anemia, metabolic acidosis, glucose instability, respiratory acidosis, apnea, hypotonia, and stupor is present.
Physical findings related to these signs include poor perfusion, pallor or an ashen color, irregularities of respiratory pattern, signs of respiratory distress including retractions and tachypnea, hypotonia, and altered mental status (eg, decreased responsiveness, coma).
Additional neurologic signs, such as fullness of the fontanelles, seizures, and posturing, may also be observed.
Progression can be rapid and may result in shock and death.
Between the two extremes of presentation, infants may demonstrate varying degrees of neurologic and systemic signs. Those with symptoms are more likely to have a more serious grade of GM/IVH.

1122
Q

Define muscular dystrophy

A

Muscular dystrophies are progressive, generalised diseases of muscle, most often caused by defective or specifically absent glycoproteins (e.g., dystrophin) in the muscle membrane. All muscular dystrophies are characterised by ongoing degeneration and re-generation of muscle fibres. The most common and rapidly progressive muscular dystrophy is Duchenne muscular dystrophy (DMD). This is X-linked and diagnosed by the finding of absent dystrophin on muscle biopsy. Other X-linked muscular dystrophies include Becker muscular dystrophy (BMD), which is characterised by diminished quantity or quality of dystrophin in muscle biopsy specimens. BMD is a milder form of DMD. Diagnosis can be confirmed using multiplex ligation-dependent probe amplification or full sequencing of the dystrophin gene. This topic will mainly discuss DMD.

Duchenne muscular dystrophy (DMD) is highlighted as the most common and most rapidly progressive muscular dystrophy, with most patients losing the ability to walk by 12 years of age and requiring ventilatory support by 25 years of age.

Before the use of cardioprotective drugs and respiratory muscle aids, respiratory complications were responsible for 80% of deaths and cardiac complications were responsible for approximately 20%. Survival past 30 years of age is no longer rare, with 10% to 40% of patients with DMD surviving to age 40 in centres that use non-invasive means of respiratory support.

The development of acute respiratory failure, previously considered inevitable, can be preventable for most patients.

Severe scoliosis and the need for corrective surgery can be avoided, and the need for mechanical ventilation delayed, by early and aggressive use of corticosteroids.

With a few exceptions, all generalised muscle disease can be managed using the principles for managing DMD.

1123
Q

Epidemiology of muscular dystrophy

A

DMD 1 per 3500
BMB 1 per 25000

Two-thirds of cases of DMD are inherited from a mother carrying a genetic mutation on the X chromosome at the Xp21 position. One third of cases result from spontaneous mutations. Girls or women with DMD are rare.

Less-frequent muscular dystrophies include myotonic dystrophy, which has a prevalence rate of 93.1 per million and incidence of about 1 per 30,000 inhabitants of northern Italy.

1124
Q

Aetiology of muscular dystrophy

A

Duchenne/Becker muscular dystrophies are caused by a mutation of the dystrophin gene located on the small arm (p) of the X chromosome at the Xp21 position. The dystrophin gene, the largest in the human genome with 79 exons, has the highest spontaneous mutation rate. Spontaneous mutation results in one third of DMD cases, whereas maternal-fetal transmission results in the remaining two-thirds.

Myotonic dystrophy is caused by a defective gene located on chromosome 19, in which a triplet repeat sequence of DNA is abnormally expanded many times. A second form of myotonic dystrophy exists with a gene defect on chromosome 3q. Clinical severity increases as the number of nucleotide triplet repeats exceeds 40 and potentially reaches into the thousands.

1125
Q

RFs for muscular dystrophy

A

FHx

Male

1126
Q

Sx of muscular dystrophy

A

Imbalance of lower limb strength
Lower extremity musculotendinous contractors
- Because the hip extensors are weaker than the hip flexors, the patient must keep the centre of gravity behind the hip to avoid jack-knifing. Likewise, jack-knifing occurs at the knees if the centre of gravity is behind the knees, because the knee extensors are weaker than the hamstrings.

Thus, the patient tends to walk with increased lordosis and on the toes to keep the centre of gravity behind the hips and in front of the knees.

Delayed motor milestones
Calf hypertrophy 
Ambulation difficulties 
Diminished muscle tone + reflexes
Normal sensation

OTHER COMMON

Gowers sign 
Toe walking
Hypotonia
Hyperactivity 
Urinary / bowel incontinence
Mental retardation
1127
Q

Ix for muscular dystrophy

A

Serum CK - 50-100x the normal level is consistent with DMD
Genetic testing - Xp21 mutation may be present in DMD and Becker muscular dystrophy

CONSIDER
EMG - myopathic reading with fast firing, short duration but polyphasic and decreased amplitude motor units with early recruitment in the affected muscles

Muscle biopsy - absence of dystrophin with Duchenne muscular dystrophy; diminished quantity or quality of dystrophin with Becker muscular dystrophy; other specific defective or absent glycoproteins in the membrane of the muscle wall with other muscular dystrophies

1128
Q

Rx of muscular dystrophy

A

Ambulatory:
Prednisolone
Physiotherapy, exercise and psychological support
Surgery for contractures

LV-ejection fraction <50%
Give carvedilol / lisinopril

May need surgery for scoliosis

Will eventually need ventilation - expiratory and inspiratory assistance

Maintenance of optimal nutrition - 15-20% will lose ability to swallow and may need PEG feed

1129
Q

Prognosis of muscular dystrophy

A

Survival past 30 years of age is no longer rare, with 10% to 40% of patients with DMD surviving to age 40 in centres that use non-invasive ventilatory support.

1130
Q

Complications of muscular dystrophy

A
Resp failure
Loss of mobility 
Osteoporosis from glucocorticoid therapy 
WL/malnutrition 
Sexual dysfunction 
Cardiac failure 
Hypersomnolence and morning headaches
1131
Q

A 4-year-old boy presents with a history of ambulation delayed until 18 months of age, toe walking, calf hypertrophy, and proximal hip girdle muscle weakness. His paediatrician, considering a mild static encephalopathy, did not request screening for myopathy but referred him to an orthopaedic surgeon, who found that his creatine kinase levels were greatly elevated, indicating need for referral to a neurologist. His siblings, a boy of 6 years and a girl of 7 years, are apparently well.

A

muscular dystrophy

1132
Q

Unusual presentations of dystrophinopathies include asymptomatic elevation of serum creatine kinase (CK), exercise intolerance, dilated cardiomyopathy, malignant hyperthermia, quadriceps myopathy, speech delay, and Xp21 muscular dystrophy appearing in Turner’s syndrome (i.e., X chromosome monozygote females). For some patients with sub-clinical DMD the diagnosis is first suspected by family history or the presence of elevated liver enzymes, the reason for which is unclear. These enzymes may include alanine aminotransferase and aspartate aminotransferase. Because inheritance is X-linked, the overwhelming majority of patients are male. At least 7% of female carriers of DMD are manifesting carriers with mild symptoms or signs of DMD. Some carriers are diagnosed after an incidental finding of elevated serum CK. Occasionally, manifesting carriers can be affected sufficiently to require ventilator use. Symptomatic disease in girls can be explained by Turner’s syndrome, skewed X chromosome inactivation, translocation of the mutated gene to an autosome, or uniparental disomy (when both copies of a chromosome pair have originated from one parent). Dystrophinopathy, as a cause of clinical myopathy, is not rare among girls or women, with two studies showing an incidence of nearly 10%. [1] Most symptomatic girls or women present in childhood with proximal muscle weakness, but new weakness in adulthood, myalgias, cramps, and fatigue have been reported as initial symptoms. Congenital muscular dystrophies, rarely confused with DMD, cause proximal muscle weakness at birth and usually presents with hypotonia at birth or soon thereafter. Static to rapidly progressive forms have been described. [2] These patients usually develop predominantly proximal muscle weakness, joint contractures, and occasionally facial muscle weakness. Bulbar musculature is generally spared. About 50% of patients never achieve the ability to walk. Scoliosis and chronic alveolar hypoventilation can cause early and severe problems for all children with muscular dystrophy.

A

muscular dystrophy

1133
Q

Define tuberous sclerosis complex

A

Tuberous sclerosis complex (TSC) is an autosomal-dominant, neurocutaneous, multi-system disorder characterised by cellular hyperplasia, tissue dysplasia, and multiple organ hamartomas. The disease has 2 genetic loci: TSC1, found on chromosome 9q34; and TSC2, found on chromosome 16p13.
The clinical phenotype can result from a mutation in either of these genes.

An autosomal-dominant, neurocutaneous, multi-system disorder characterised by cellular hyperplasia, tissue dysplasia, and multiple organ hamartomas.

The primary recognised neurocutaneous features include dental pits, hypopigmented macules (ash leaf spots), facial angiofibromas, and ungual fibromas.

The most often identified clinical presentations are epilepsy (infantile spasms), autism and cognitive impairment, and neonatal cardiac rhabdomyomas.

Cardiac rhabdomyomas regress in early childhood and are generally asymptomatic, whereas renal angiomyolipomas and giant cell astrocytomas progress in adolescence and adulthood. The renal complications are second only to the central nervous system complications as a cause of significant morbidity.

Lymphangioleiomyomatosis of the lung associated with tuberous sclerosis is unique in that it is identified in females only and arises from metastases from renal angiomyolipomas.

1134
Q

Epidemiology of tuberous sclerosis complex

A

TSC can be identified in all ethnic groups and is equally identified in both sexes. Population studies have estimated a prevalence of 1 in 6000 to 9000 people. Although TSC is an autosomal-dominant inherited disorder, up to 65% to 75% of people affected with TSC have had spontaneous mutations as the underlying genetic cause. An estimated 40,000 Americans and at least 2 million people worldwide are affected with TSC.

1135
Q

Aetiology of tuberous sclerosis complex

A

There are 2 recognised genetic loci that produce TSC: the TSC1 gene, found on chromosome 9q34; and the TSC2 gene, found on 16p13.

The majority of affected patients have mutations on the TSC2 gene (80% to 90%); however, each genetic locus may produce the same phenotypic expression with individual variation.

1136
Q

RFs for tuberous sclerosis complex

A

Genetic predisposition:
35% to 40% of affected people have autosomal-dominant inheritance. However, the majority (60% to 65%) of affected people have sporadic occurrence.

1137
Q

Sx of tuberous sclerosis complex

A
COMMON
FHx
Epilepsy 
Cardiac rhabdomyoma (identified at 22 weeks of gestation)
Renal angiomyolipomas
Lymphangioleiomyomatosis of the lung 
Cerebral subependymal calcified nodules
Giant cell atrocytomas
Facial angiofibromas 
Cephalic plaques
Periungal fibromas
Hypomelanotic patches (ash leaf spots)
Shagreen patch
Numerous dental enamel pits and gingival fibromas
OTHER 
Autism 
Cognitive impairment
Behavioural problems
Retinal nodular Hamartomas
Multiple hamartomatous colonic polyps
1138
Q

Ix for tuberous sclerosis complex

A

Usually clinical

Genetic testing - mutation of TSC1 or TSC2 gene
Neurodevelopmental testing
Echocardiography - rhabdomyomas
Renal USS/CT - angiomyolipomas
Chest CT - lymphangioleiomyomatosis
Cranial CT/MRI - calcified subependymal nodules, tubers, subependymal giant cell astrocytoma, cortical dysplasia, migration lines

1139
Q

Rx for tuberous sclerosis complex

A

Specific management for each lesion

1140
Q

Complications of tuberous sclerosis complex

A
Chronic renal/resp failure
Sleep disturbances
Sudden death
IC haemorrhage 
Anxiety/depression
1141
Q

Prognosis of tuberous sclerosis complex

A

TSC is a genetically and clinically heterogeneous disorder. Each patient will face specific and individual challenges related to the effects that the disease places upon them.

1142
Q

Six-month-old adopted fraternal twins present to their paediatrician for a first healthcare visit. There is no available family history. Each child is healthy and twin A appears developmentally on target, whereas twin B is not as vocal, is less visually interactive, is not sitting, and has been noted to have frequent startles. Their examinations are normal; however, both twins have several hypopigmented macules. Upon careful inspection under Wood’s lamp illumination, there is 1 large macule (3 cm) and 4 smaller ones (<1.5 cm) on the skin of each child. Ophthalmological examination reveals 2 small retinal hamartomas in the eyes of twin B. A brain CT scan of each child identifies calcified subependymal nodules, and an MRI shows these as well as several cortical tubers. An electroencephalogram (EEG) is normal for twin A, but is severely abnormal in twin B, demonstrating hypsarrhythmia with concurrent infantile spasms. Each child has a normal echocardiogram and renal ultrasound. They each have a definite diagnosis of TSC.

A

Tuberous sclerosis complex

1143
Q

A healthy 28-year-old woman presents for a routine fetal ultrasound at 6 months of gestation. The ultrasound identifies a cardiac mass in the fetus. She later undergoes a fetal MRI, which discloses 2 cardiac masses within the ventricular septum not obstructing cardiac outflow. The infant is born at term without perinatal complications. The cardiac masses remain asymptomatic and an electrocardiogram (ECG) is normal. There are no hypomelanotic macules. The infant undergoes brain MRI, which identifies periventricular nodules and cortical tubers. Ophthalmological examination is normal, as is a subsequent renal ultrasound. A definite diagnosis of TSC is made. The mother is examined and is found to have facial angiofibromas, no hypomelanotic macules, and a normal ophthalmological examination. Subsequent brain MRI is normal, but she has several bilateral renal angiomyolipomas and is diagnosed with definite TSC. There are no other maternal family members with symptoms or signs of TSC.

A

Tuberous sclerosis complex

1144
Q

TSC may present in people not otherwise aware that they carry the gene for the disorder. In infancy, a diagnosis of autosomal dominant (adult) polycystic kidney disease (APKD) may be made on the basis of bilateral flank/abdominal masses, palpable on examination, and then identified with renal imaging such as ultrasound. These are multiple large cysts with little remaining normal renal parenchyma. APKD can be part of a large, contiguous gene deletion syndrome identified in 3% to 5% of TSC patients, because the gene for APKD is adjacent to and continuous with the TSC2 gene (1 of 2 recognised genetic loci that produce TSC). Similarly, in children, an initial evaluation for developmental delays and/or autism may reveal hypomelanotic macules, which should prompt further investigation and a definitive diagnosis of TSC. The incidence of cognitive impairment in patients with TSC approaches 30% to 40%, and up to 25% have autism. When people with TSC and cognitive impairment are examined for the presence of autism spectrum disorders, the incidence increases to over 60%. Even less usual presentations of TSC occur, including spontaneous pneumothorax in young women with previously unrecognised lymphangioleiomyomatosis of the lung. Occasionally, patients may undergo brain imaging and the hallmark calcified subependymal nodules will be incidental findings, leading to the diagnosis of TSC in the absence of epilepsy.

A

Tuberous sclerosis complex

1145
Q

Define neurofibromatosis

A

Neurofibromatosis (NF) is a group of three conditions in which tumors grow in the nervous system. The three types are neurofibromatosis type I (NF1), neurofibromatosis type II (NF2), and schwannomatosis. In NF1 symptoms include light brown spots on the skin, freckles in the armpit and groin, small bumps within nerves, and scoliosis. In NF2 there may be hearing loss, cataracts at a young age, balance problems, flesh colored skin flaps, and muscle wasting. The tumors are generally non-cancerous.

The cause is a genetic mutation in certain genes. These can be inherited from a person’s parents, or in about half of cases spontaneously occur during early development. The tumors involve supporting cells in the nervous system rather than the neurons. In NF1 the tumors are neurofibromas (tumors of the peripheral nerves), while in NF2 and schwannomatosis tumors of Schwann cells are more common. Diagnosis is typically based on the signs and symptoms and occasionally supported by genetic testing.

1146
Q

Epidemiology of neurofibromatosis

A

NF1 - 1 in 3500
NF2 - 1 in 25000

NF1 normal life expectancy
NF2 increased risk early death

1147
Q

Aetiology of neurofibromatosis

A

Neurofibromatosis is an autosomal dominant disorder, which means only one copy of the affected gene is needed for the disorder to develop. If one parent has neurofibromatosis, his or her children have a 50% chance of developing the condition as well. The severity of the condition of the parent does not affect the child; the affected child may have mild NF1 even though inherited from a parent with a severe form of the disorder

1148
Q

RFs for neurofibromatosis

A

50% chance of being affected with affected parent (Autosomal dominant)

1149
Q

Sx of neurofibromatosis

A

In NF1 the tumors are neurofibromas (tumors of the peripheral nerves), while in NF2 and schwannomatosis tumors of Schwann cells are more common.

NF1:
Six or more light brown dermatological spots (“café au lait spots”)
At least two neurofibromas
At least two growths on the eye’s iris (litchi noduels)
Abnormal growth of the spine (scoliosis)

In NF2 there may be hearing loss, cataracts at a young age, balance problems, flesh colored skin flaps, and muscle wasting.

Neurofibromatosis type I, in which the nerve tissue grows tumors (neurofibromas) that may be benign, may cause serious damage by compressing nerves and other tissues.[8]
Neurofibromatosis type II, in which bilateral acoustic neuromas (tumors of the vestibulocochlear nerve or cranial nerve 8 (CN VIII) also known as schwannoma) develop, often leading to hearing loss.[9]
Schwannomatosis, in which painful schwannomas develop on spinal and peripheral nerves.[10]

1150
Q

Ix for neurofibromatosis

A
Radiograph
MRI or CT scan
EEG
Slit-lamp examination
Genetic testing
Histology
1151
Q

Rx of neurofibromatosis

A

Surgical removal of tumors is an option, however the risks involved should be assessed first. With regard to OPG (optic pathway gliomas), the preferred treatment is chemotherapy. However, radiotherapy isn’t recommended in children who present with this disorder. It is recommended that children diagnosed with NF1 at an early age have an examination each year, which allows any potential growths or changes related to the disorder to be monitored

1152
Q

Prognosis / complications of neurofibromatosis

A

In most cases, symptoms of NF1 are mild, and individuals live normal and productive lives. In some cases, however, NF1 can be severely debilitating and may cause cosmetic and psychological issues. The course of NF2 varies greatly among individuals. In some cases of NF2, the damage to nearby vital structures, such as other cranial nerves and the brain stem, can be life-threatening. Most individuals with schwannomatosis have significant pain. In some extreme cases the pain will be severe and disabling.

1153
Q

Define mumps

A

Mumps is an acute systemic infectious disease caused by an RNA paramyxovirus. It is spread by respiratory droplets, and, before the advent of the mumps vaccine, it affected school-aged children commonly. Classically, mumps causes swelling of the parotid glands; the involvement of other salivary glands, meninges, gonads, and pancreas is also common.

Hallmark of infection is swelling and inflammation of one or both of the parotid glands (parotitis).

More serious complications of mumps, such as meningitis, encephalitis, and orchitis, can occur in the absence of parotitis, potentially delaying the accurate diagnosis of the disease.

Diagnosis can usually be made based on the characteristic clinical features of parotitis and prodrome alone. Laboratory diagnosis is based on serological or salivary confirmation, detection of viral nucleic acid, or isolation of the virus from body fluids.

Treatment is symptomatic, and the disease can be prevented largely by routine vaccination.

The mumps vaccine is used almost universally in developed countries, as part of MMR (measles, mumps, and rubella) vaccine. Sporadic outbreaks have occurred in highly vaccinated populations, leading to suggestions that the vaccine’s efficacy may not be as high as originally thought and that vaccination policy should be reviewed, perhaps to include 3 doses.

1154
Q

Epidemiology of mumps

A

In countries without vaccination against mumps virus, the incidence remains high, with epidemic peaks every 2 to 5 years

In the UK, the cases that do occur are generally in the older, unvaccinated population. In England in 2017, 1550 cases of mumps were reported and over 70% of these were in people over 15 years of age

There is no difference in the prevalence of mumps between men and women. However, men are more likely to have complications, particularly neurological.

1155
Q

Aetiology of mumps

A

Spread is by respiratory droplets, direct contact, or contaminated fomites.

The incubation period is usually 14 to 18 days, and peak contagion occurs 1 to 2 days before the onset of symptoms. Viral shedding has been isolated in saliva as early as 7 days before and until 9 days after clinical symptoms. Vaccination status does not seem to impact the duration of viral shedding.

The mumps virus is a single-stranded RNA virus and a member of the Paramyxovirus genus. It is an enveloped virus composed of a matrix protein and 2 surface glycoproteins. The glycoproteins, known as haemagglutinin-neuraminidase and fusion protein, enable viral absorption and fusion of the virion membrane with the host cell membrane, respectively.

The envelope is composed of a lipid bilayer membrane, which makes the virus susceptible to ether and alcoholic disinfectants.

Twelve mumps virus genotypes have been described, designated A to L. Their prevalence varies by geographic location. Humans serve as the only natural host for mumps.

1156
Q

RFs for mumps

A

STRONG
Unvaccinated - One dose of the live attenuated mumps vaccine protects against mumps infection in 96% of people, but immunity wanes with time.
International traveller

WEAK
Immunosuppression
Healthcare worker 
Close contact living
Vaccine failure
1157
Q

Sx of mumps

A

COMMON
RFs
Parotitis
Hx missed / no vaccination

OTHER COMMON
Constitutional symptoms
Epididymo-orchitis
Oophoritis
Aseptic meningitis 

UNCOMMON
Mastitis
Encephalitis
Deafness

1158
Q

Ix for mumps

A

1st:
Salivary mumps IgM

Consider:
FBC + Dif - possible leukocytosis in mumps meningitis, orchitis, or pancreatitis

LP - CSF cell count: lymphocytosis in mumps meningitis and encephalitis; possible pleocytosis; CSF pressure, glucose, protein: generally normal

Serum mumps IgG/IgM

Reverse transcriptase PCR
Serum amylase if also pancreatitis
Viral culture of CSF, saliva, urine
CT head - normal in uncomplicated disease, but focal lesions may be present; excludes other pathology

1159
Q

Rx of mumps

A

Isolation and supportive care

Ibuprofen + paracetamol (antipyretics/analgesia)

Public Health England also recommends exclusion from school or work for 5 days.

1160
Q

Prognosis of mumps

A

Mumps is a self-limiting disease that often resolves within 1 to 2 weeks. Most people recover from mumps without any long-term residual complications. One attack of mumps usually confers life-long immunity.

Pregnancy
Mumps in pregnancy has not been shown to cause an increased risk of severe congenital abnormalities or low birth weight. A study in the 1960s showed increased risk of miscarriage with mumps infection during the first trimester, whereas a more recent study had no increased rate of miscarriage.

1161
Q

Complications of mumps

A
Epididymo-orchitis
Aseptic meningitis
Oophoritis
Mumps encephalitis
Pancreatitis
Myocardial involvement
Arthritis
Deafness
1162
Q

A 20-year-old university student with no significant past medical history presents to her doctor with the complaint of painful bilateral swelling near her jaw and cheek. She describes an “earache” and says it is difficult to eat, swallow, and talk. She remembers a low-grade fever and headache several days before the onset of swelling. She does not recall having a second dose of MMR before starting university.

A

Mumps

1163
Q

Mumps infection is asymptomatic in 15% to 20% of cases. [3] Orchitis is the most common symptomatic complication of mumps, occurring in 20% of post-pubertal males with mumps. [1] Oophoritis develops in 5% of post-pubertal females, and mastitis can also be seen. [2]

Aseptic meningitis is the most frequent extra-salivary complication, although often asymptomatic. Pleocytosis of the cerebrospinal fluid can develop in as many as half of all people with symptomatic mumps, but true symptoms of meningeal irritation occur in only 5% to 25% of people. Aseptic meningitis occurs 3 times more frequently in males than females and can occur before, during, or after an episode of parotitis. [4] In some series, up to 50% of people developed mumps meningitis without parotitis. [5]

Other less common CNS presentations of mumps infection include encephalitis (approximately 0.1% of people with mumps), deafness, cerebellar ataxia, facial palsy, transverse myelitis, and Guillain-Barre syndrome. [6] Pancreatitis occurs in approximately 4% of mumps infections, usually with a relatively mild course.

A

Mumps

1164
Q

Define measles

A

Measles is a highly infectious disease caused by measles virus, characterised by a maculopapular rash, cough, coryza, conjunctivitis, and a pathognomonic enanthem (Koplik’s spots) with an incubation period of about 10 days.

Preventable by immunisation but high levels of coverage are required to prevent outbreaks of disease from occurring.

No specific treatment for measles is available except for supportive care.

Complications of measles are more common in immunocompromised and poorly nourished individuals and include pneumonia, laryngotracheitis, otitis media, and encephalitis.

1165
Q

Epidemiology of measles

A

In the pre-vaccine era, measles was ubiquitous. In the UK, there were 160,000 to 800,000 cases reported every year, with peaks every 2 years.

5-9yos

Measles vaccine was introduced in the 1960s, leading to a more than 99% reduction in the number of reported measles cases. Two doses of the measles vaccine are recommended to ensure immunity.

1166
Q

Aetiology of measles

A

Measles is caused by a spherical RNA virus of the genus Morbillivirus and family Paramyxoviridae.

It is relatively large, with an RNA genome.

Measles virus is heat labile. Humans are the natural host but monkeys can become infected. Laboratory strains can infect mice and hamsters.

Measles virus is transmitted via droplets and infects epithelial cells of the nose and conjunctivae.

1167
Q

RFs for measles

A

STRONG
Exposure
No prior immunisation
Failed vaccine response

Mathematical modelling suggests that immunisation rates of 92% to 95% are required to interrupt measles transmission.

After 2 doses of vaccine about 99% will be protected.

1168
Q

Sx of measles

A

COMMON
Exposure
Un-immunised

PRODROME
Fever
Cough
Coryza
Conjunctivitis 

THEN
Macropapular rash + resolution of fever

1169
Q

Ix for measles

A

1st
Measles specific IgM and IgG ELISA

Consider
Acute + convalescent sera for measles specific IgG and IgM
Measles RNA detection via PCR
Fluorescent antibody detection

1170
Q

Rx of measles

A

Supportive
Paracetamol/ibuprof

Respiratory support in the event of pneumonia and neurological support in the event of encephalitis should be given.

Vitamin A supplementation:

Consider vitamin A in children who: are up to 2 years of age; are hospitalised with measles; have complications of measles; have immunodeficiency; have clinical evidence of vitamin A deficiency; have impaired intestinal absorption and malnutrition; or have recently emigrated from areas where measles mortality rates are high.

1171
Q

Complications of measles

A

Pneumonia
Otitis media
Encephalitis
Subacute sclerosing panencephalitis (SSPE)

1172
Q

Prognosis of measles

A

Most individuals recover from measles without sequelae and death is rare in developed countries. In developing countries, however, mortality rates vary from 2% to 15%. Malnourished, vitamin A-deficient, and immunocompromised children are most susceptible to morbidity, mortality, and complications such as pneumonia.

1173
Q

Atypical measles occurs in individuals who have been immunised previously and are exposed to wild-type measles. Most cases occur in those who received killed measles vaccine. The onset is acute, with high fever, headache, abdominal pain, myalgia, and dry cough. Coryza and conjunctivitis are not prominent and Koplik’s spots are rare. The rash, unlike that of natural measles, begins in the extremities and proceeds to the head. It may be especially prominent on the wrists and ankles and includes the palms and soles.

A

Measles infection

1174
Q

Modified measles occurs in partially immune individuals. It has a shorter prodromal period, Koplik’s spots are rare, and the rash progresses similarly to typical measles but does not coalesce. Unusual manifestations of measles include pneumonia, otitis media, myocarditis/pericarditis, and encephalitis. Measles is more severe in immunocompromised and poorly nourished individuals. Subacute sclerosing panencephalitis is a rare, fatal neurological disorder occurring about 7 to 10 years after natural measles infection, possibly as a result of persistent measles infection within the central nervous system.

A

Measles infection

1175
Q

A 17-month-old previously healthy but unimmunised child develops fever, malaise, and upper respiratory symptoms, including cough, runny nose, and conjunctivitis, that worsen over several days. The fever increases gradually up to 39°C to 40°C (103°F to 104°F) over several days and the cough worsens. Photophobia is present. Whitish lesions on erythematous buccal mucosa are visible at this time. Two days later, an erythematous, maculopapular rash appears on the head and spreads from head to toe over the next 3 days. The rash takes on a brownish coloration and confluent appearance over the next few days. The fever resolves on the third day of the rash, which persists for about a week as does the cough.

A

Measles infection

1176
Q

Define rubella

A

This topic focuses on postnatal rubella (German or 3-day measles), a mild, self-limiting, systemic infection caused by rubella virus. Up to one half of all cases are asymptomatic. The common manifestations of symptomatic infection include mild fever, a generalised rash, lymphadenopathy, conjunctivitis, and arthralgias or arthritis. Maternal infection in pregnancy, particularly early in gestation, may cause spontaneous abortion, fetal death, or a wide spectrum of anatomical and laboratory anomalies (congenital rubella syndrome).

Treatment of symptomatic infection is largely supportive, as the illness is self-limiting.

The most important consequence of rubella infection is congenital rubella syndrome, which may result from infection during pregnancy. Specialty consultation is strongly recommended for pregnant women with exposure to rubella.

Rubella immunisation programmes have eliminated endemic spread of the virus in the UK and the Americas; most cases are imported or associated with imported infections.

1177
Q

Epidemiology of rubella

A

Previously 5-10yo peak incidence
<10 people per year
Most cases now affect adolescents and young adults.

1178
Q

RFs for rubella

A

Incomplete immunisation
Exposure to infectious contact
International travel

1179
Q

Aetiology of rubella

A

Rubella is caused by rubella virus, which is a togavirus and the only member of the genus Rubivirus

Humans are the only natural host. The virus has a positive-stranded RNA genome and a glycolipid envelope. There is only 1 antigenic type. Rubella virus is readily inactivated by chemical agents, low pH, heat, and cold.

Rubella is transmitted from human to human only by direct or droplet contact with infected body fluids, most commonly nasopharyngeal secretions.

Patients may shed infectious virus from 7 to 30 days after infection (from 1 week before to 2 weeks after the onset of rash). However, infants with congenital rubella syndrome may be contagious for >1 year.

14 day incubation

Rubella vaccines are reported to be approximately 97% effective in preventing disease after a single dose.

1180
Q

Sx of rubella

A

COMMON
RFs
Macropapular rash often first presentation - usually begins on the face and spreads from the head to the feet. Occasionally, there may be a petechial component to the rash or palatal petechiae. The rash persists for an average of 3 to 4 days.

Fever (low grade)
Arthralgias
Lympahenopathy
Incomplete immunisation

PRODROME 
Malaise 
Coryza / pharyngitis
Arthritis
Conjuncitivits
1181
Q

Ix for rubella

A

Rubella serology - IgM: positive in acute serum; IgG: seroconversion or 4-fold rise between acute and convalescent titres

FBC - N / Occasionally thrombocytopenia may be present. This is thought to have an immunological basis.

Consider
Viral culture
Reverse transcriptase PCR

1182
Q

Rx of rubella

A

PREGNANT
Specialist referral

Human IM IG

There is insufficient evidence to assess the effectiveness of polyclonal immunoglobulin administered to pregnant women for the prevention of congenital rubella in the fetus.

Routine administration of immunoglobulin for post-exposure prophylaxis of rubella in pregnancy is not recommended, but this option may be considered if a pregnant woman will not consider termination of the pregnancy.

NON PREGNANT

Postnatal rubella is generally a mild, self-limiting condition that requires only symptomatic therapy. No specific antiviral therapy is available. A brief course of an NSAID, if no contraindication exists, may be helpful for patients with arthritis.

1183
Q

Complications of rubella

A

Thombocytopenia
Encephalitis
Congenital rubella syndrome
Neurological complications

1184
Q

Prognosis of rubella

A

Postnatal rubella is generally a mild illness that resolves spontaneously over several days to 1 week. The most important consequence of postnatal rubella infection is congenital rubella syndrome.

Arthritis
Arthralgias and arthritis may persist for weeks or months or recur, particularly in adult women. However, the overall prognosis for joint function is good.

Pregnant women
Maternal infection in non-immune women during pregnancy, particularly early in gestation, may cause spontaneous abortion, fetal death, or a wide spectrum of anatomical and laboratory anomalies (congenital rubella syndrome).

1185
Q

A 35-year-old man presents with a 3-day history of low-grade fever, malaise, headache, and aching knees. That morning he developed a rash on his face, which has now spread to his chest and arms. His physical examination is notable for mild conjunctival injection, mild bilateral posterior auricular lymphadenopathy, and a discrete erythematous papular rash on his face, trunk, and upper arms. The patient is a business traveller from Nigeria who arrived in the United States a week prior to the onset of his illness. He is unaware of his immunisation status and reports that a co-worker with whom he had close contact had a similar rash recently.

A

rubella

1186
Q

A 2820-gram female infant is born to a 22-year old primigravidas mother at approximately 38 weeks’ gestation following an uncomplicated pregnancy. The baby has mild hepatosplenomegaly, numerous purplish, firm, non-blanching skin nodules, scattered petechiae, and a grade 3 continuous murmur audible at the left infraclavicular area. The baby’s mother emigrated from Vietnam during the sixth month of her pregnancy; she cannot recall having been immunised in childhood.

A

rubella

1187
Q

Postnatal rubella infection may be complicated by overt arthritis. This is more common in adults and females and may persist for weeks to months. Thrombocytopenia occurs in approximately 1 out of 3000 cases, most commonly in children. Serious haemorrhagic complications may occur. Rare complications include neurological disorders, myocarditis, pericarditis, hepatitis, and bone marrow failure.

A

rubella

1188
Q

Define erythema infectiosum / parvovirus B19

A

Erythema infectiosum is a childhood illness caused by infection with parvovirus B19. A ‘slapped cheek’ appearance with a lacy eruption on the torso and extremities is the most common presentation. Parvovirus B19 infection can also be associated with arthropathy and a purpuric cutaneous eruption. Select populations may be at risk for chronic anaemia or transient aplastic crises. Fetal complications include hydrops fetalis and intrauterine fetal demise.

The classic childhood presentation is a ‘slapped cheek’ appearance followed by a reticular, erythematous eruption that is predominantly found on the extremities and may be preceded by mild systemic symptoms. Adults, more than children, may report arthritis and arthralgias.

Most cases do not require specific treatment beyond symptomatic therapy and reassurance.

Infection in pregnant women may result in fetal anaemia, hydrops fetalis, or intrauterine death.

Persistent infection, lasting longer than approximately 3 weeks and accompanied by chronic anaemia, may occur in people who are immunosuppressed (e.g., patients with HIV, people receiving chemotherapy or immunosuppression following transplant, or patients with congenital immunodeficiencies).

People with a high RBC turnover/destruction (e.g., those with hereditary spherocytosis, sickle cell disease, thalassaemia, iron deficiency anaemia) may develop transient aplastic crisis.

1189
Q

Epidemiology of erythema infectiosum / parvovirus B19

A

Any age
Any time
Often late winter / early spring
Direct contact / respiratory droplet / even blood
Infection most common in school age children -> their family

Seroprevalence for parvovirus B19 increases with age and by 15 years old, over 50% of adolescents have antiparvovirus antibodies.

1190
Q

Aetiology of erythema infectiosum / parvovirus B19

A

Erythema infectiosum is caused by parvovirus B19, a single-stranded, non-enveloped DNA virus. Humans are the only known hosts. Members of the family parvoviridae are frequent causes of infection in veterinary medicine but animal parvoviruses are not a cause of human infection.

Parvovirus B19 infects erythrocyte progenitor cells, using the erythrocyte P antigen (globoside) as its receptor.

Initial respiratory tract infection and replication is followed by a viraemia after 4 to 14 days, which ends with the appearance of anti-B19 IgM. IgG appears approximately 1 week later and coincides with the appearance of the typical exanthem and arthralgias.

1191
Q

RFs for erythema infectiosum / parvovirus B19

A

STRONG
Close contact to infected

WEAK
Immunodeficiency

1192
Q

Sx of erythema infectiosum / parvovirus B19

A

COMMON
Close contact to infected
Slapped cheek - sparing nasal ridge and peri-oral areas
Macules and papules appear 1 to 4 days after initial facial rash.
Lacy reticular erythema lasts for 1 to 3 weeks although it may persist for longer.

The generalised exanthem may intensify with increased body temperature; parents should be cautioned that this does not reflect the recrudescence of the disease.

Arthralgia
Prodrome - fever, ehadache, pharyngitis, coryza, abdominal pain

Persistent parvovirus: Sx of anaemia

1193
Q

Ix for erythema infectiosum / parvovirus B19

A

Clinical - typical appearance of the classic ‘slapped cheek’ rash with a lacy, reticular exanthem on the extremities and torso

Consider
FBC - check for anaemia
Reticulocyte count - decreased reticulocyte count
Serology - positive IgM antibodies for parvovirus B19
PCR - presence of parvovirus B19 DNA

1194
Q

Rx of erythema infectiosum / parvovirus B19

A

ACUTE
Paracetmol / supportive / ibuprofen

As in any viral illness, maintaining hydration and appropriate rest is indicated.

Persistent (>3w)
In immunosuppressed patients, discontinuation of immunosuppressive therapy or institution of antiretroviral therapy in patients with HIV may terminate persistent parvovirus B19 infection and thus resolve the anaemia.

RBC transfusion may be required in the interim to prevent complications of anaemia.

normal immunoglobulin human: adults: 400 mg/kg/day intravenously for 5 days

1195
Q

Complications of erythema infectiosum / parvovirus B19

A

Parvovirus B19-induced fetal anaemia may result in high-output CHF and intrauterine death may occur.

Transient aplastic crisis, causing a decrease in haemoglobin levels with absent to low reticulocytes during acute infection and resolving within days to weeks, is seen in patients with increased red-cell turnover or destruction.

1196
Q

Prognosis of erythema infectiosum / parvovirus B19

A

Erythema infectiosum is self-limited and resolves without sequelae in the majority of cases. Parents of children should be advised that the generalised exanthem may persist for several weeks and may intensify with increased body temperature (bathing, exertion), but this does not represent recrudescence of the disease. In the immunosuppressed population, persistent B19 infection with resultant anaemia may occur.

1197
Q

A 6-year-old child presents to the clinic in February with bright red macules on his cheeks and a lacy, reticular eruption on his extremities and torso. His mother reports that last week he did have symptoms of a mild cold. Other than the exanthem, the child appears healthy

A

erythema infectiosum / parvovirus B19

1198
Q

In adults with parvovirus B19, self-limited symmetric polyarthropathy of the small joints is often the most common presenting complaint and may or may not be accompanied by the typical exanthem. In addition to erythema infectiosum, another exanthem caused by parvovirus B19 is that of papular purpuric gloves and socks syndrome (PPGSS). This eruption is classically characterised by painful oedema, pruritus, and sharply marginated petechiae and purpura of the hands and feet. PPGSS may be associated with fever, mucocutaneous lesions or morphologically similar lesions outside the typical distribution. Unlike erythema infectiosum, this eruption is associated with concurrent viraemia and infectivity. Immunodeficient patients (e.g., patients with HIV, people receiving chemotherapy or immunosuppression following transplant, or patients with congenital immunodeficiencies) may not present with classic erythema infectiosum, but may develop chronic anaemia/pure red cell aplasia due to persistent parvovirus B19 infection. Complications including transient aplastic crisis may occur in people with increased RBC turnover/destruction (e.g., hereditary spherocytosis, sickle cell disease, thalassaemia, iron deficiency anaemia) and infection in pregnant women may result in severe fetal anaemia and hydrops fetalis. Rare reports of nephritis, hepatitis, and neurological disease in association with parvovirus B 19 infection have been reported. In addition, parvovirus has been attributed to cardiomyopathy in children.

A

erythema infectiosum / parvovirus B19

1199
Q

Define roseola infantum

A

Roseola infantum (also called exanthema subitum, sixth disease) is a common early childhood febrile illness, usually characterised by 3 to 5 days of high fever followed by onset of rash that appears with defervescence. The rash consists of asymptomatic pink-red macules and papules. Febrile seizures may occur. Roseola is usually caused by human herpesvirus (HHV)-6B, but occasionally by HHV-7 and rarely by other viruses. HHV-6A (associated with thyroiditis), HHV-6B (roseola), and HHV-7 (roseola) are roseoloviruses (within the betaherpesviruses subfamily), and all establish latency. HHV-6A and HHV-6B can occasionally integrate into the host chromosome. Reactivation of roseoloviruses may occur with immunosuppression.

Roseola is a common febrile illness of early childhood; it is usually caused by human herpesvirus (HHV)-6B and occasionally by HHV-7.

Roseola is usually characterised by 3 to 5 days of fever followed by onset of a morbilliform rash that appears with defervescence. The lesions are discrete 3- to 5-mm pink-red macules and papules that commonly begin on the neck and trunk and spread to the extremities.

Roseola is usually a benign self-limited illness that has been associated with febrile seizures.

1200
Q

Epidemiology of roseola infantum

A

6-24 months of age predominantly
Rarely diagnosed before 3 months age or AFTER 4 years of age

Human herpesvirus (HHV)-6B is found worldwide. Over 90% of children are seropositive for HHV-6B by 24 months of age.

About 65% of children are seropositive for HHV-7 by 36 months of age.

While more than 90% of children with primary HHV-6B infection have fever, only a minority develop classic roseola rash (reported as 23% in one study).

Most have an undifferentiated febrile illness without rash.

1201
Q

Aetiology of roseola infantum

A

Roseola infantum is caused primarily by human herpesvirus (HHV)-6, sometimes by HHV-7, and rarely by other viruses, including coxsackievirus, echovirus, adenovirus, and parainfluenzavirus. The incubation period ranges from 1 to 2 weeks.

HHV-6A (associated with thyroiditis), HHV-6B (roseola), and HHV-7 (roseola) are the 3 species of the roseolovirus genus, which is within the betaherpesviruses subfamily.

Human herpesvirus (HHV)-6B and HHV-7 are most likely to be spread through respiratory secretions in asymptomatic contacts. HHV-6B and HHV-7 DNA may be found in saliva for extended periods of time following primary infection. These viruses are double-stranded DNA viruses and are trophic for CD4+ T lymphocytes.

1202
Q

RFs for roseola infantum

A

Age <2

Immunosuppression

1203
Q

Sx of roseola infantum

A

COMMON
RFs
High fever 40deg, Sudden onset

Typically peaks in early evening and persists for 3 to 5 days.

Exanthem - the typical exanthem presents at the time of defervescence and consists of 3- to 5-mm pink-red macules and papules on the trunk, neck, and proximal extremities, and occasionally on the face

OTHER COMMON
Diarrhoea
Nagayama's spots - enanthem composed of red papules on the soft palate and uvula has been described.
Tympanic membrane inflammation
URT symptoms 

UNCOMMON
Seizures - Up to 15% of children will present with a seizure episode, and primary HHV-6 infection has been associated with approximately one-third of first-time childhood febrile seizures.
Peri-orbital oedema
Bulging anterior fontanelle
Cervical/occipital/post-auricular lymphadenopathy

1204
Q

Ix for roseola infantum

A

Roseola can almost always be diagnosed based on the classic presentation of a previously healthy infant, 9 to 12 months of age, with a sudden onset of high fever for 3 to 4 days, followed by development of discrete red macules and papules on the trunk. For patients with this classic presentation, a clinical diagnosis can be made based on physical examination findings and history (usually at the time of defervescence). Laboratory investigation is seldom necessary.

Can do viral culture / antibody detection

1205
Q

Rx of roseola infantum

A

Antipyretics / oral hydration

1206
Q

Complications of roseola infantum

A

Seizures
Reactivation of latent virus
Progression of HIV disease

1207
Q

Prognosis of roseola infantum

A

The large majority of patients experience an acute febrile illness that generally resolves without sequelae. There is very little risk of recurrence in healthy individuals. However, up to 15% of children with roseola experience seizures. For example, in one series, 13% of children with roseola experienced seizures, which may be prolonged or recurrent.

Following a primary infection, the virus becomes latent in peripheral blood mononuclear cells, and reactivation after solid-organ or bone marrow transplantation has been associated with morbidity.

1208
Q

A previously healthy 9-month-old infant presents with a 4-day history of irritability and high fever in the range of 39°C to 40°C (102°F to 104°F), peaking in the early evening. On day 4 of illness his fever and irritability resolved, and he then developed a rash consisting of 2- to 5-mm red macules on his trunk that spread to his proximal extremities. The rash faded over a day and he has remained well.

A

roseola infantum

1209
Q

Febrile seizures occur in 10% to 15% of infants with roseola. Other findings include a bulging anterior fontanelle; oropharyngeal inflammation and ulcers; mild posterior cervical, post-auricular, or occipital lymphadenopathy; tympanic inflammation; cough; rhinorrhoea; and mild diarrhoea. Encephalitis can occur, but very rarely.

A

roseola infantum

1210
Q

Define acute varicella-zoster / chicken pox

A

Varicella (chickenpox), one of the childhood exanthemas, is caused by the human alpha herpes virus, varicella zoster. Varicella-zoster virus (VZV) is an exclusively human virus. The incubation period is about 14 days (range 9 to 21 days). Varicella is characterised by fever, malaise, and a generalised pruritic, vesicular rash. The disease normally presents in childhood and is usually self-limiting. Adverse outcomes are more common in immunocompromised people, adolescents, adults, and pregnant women.

Adults, pregnant women, immunosuppressed patients, and neonates are at high risk of complications, including pneumonia, neurological sequelae, hepatitis, secondary bacterial infection, and death.

Patients in high-risk categories should receive treatment with antiviral therapy.

While most countries in Europe do not currently vaccinate children against varicella, vaccination strategies differ widely within the EU, with a few countries incorporating the vaccine into routine childhood vaccination, and others recommending it to susceptible adolescents and adults. In the US, varicella vaccine is currently recommended for immunocompetent children and susceptible adults (e.g., healthcare workers, those occupationally exposed to children, people admitted to hospital, military recruits).

Patients with high risk for severe disease who have had significant exposure to the virus and in whom the vaccine is contraindicated (i.e., neonates, pregnant women, immunocompromised people, and those receiving high-dose systemic immunosuppressive therapy) may receive immunoprophylaxis or post-exposure antiviral prophylaxis.

1211
Q

Epidemiology of acute varicella-zoster / chicken pox

A

90% of unimmunised get infected
Infection occurs at different ages in different parts of the world.
India/caribbean/SEasia - 20-30yos
UK/US - <10yo

The virus is more prevalent in temperate climates, and outbreaks are more common in late winter and spring.

Estimated hospital admission rates for varicella in developed countries range from 2 to 6 per 100,000 people and appear to be higher in African-American people and non-white Hispanic people.

1212
Q

Aetiology of acute varicella-zoster / chicken pox

A

Varicella is caused by primary infection with the human alpha herpes virus, varicella zoster, in a non-immune host. Clinical disease is a manifestation of the second viraemic phase of the virus. Exposure to varicella-zoster virus (VZV) initiates production of host antibodies and cell-mediated immune responses, which are important for early control and limiting dissemination of primary varicella infection. After initial presentation, the virus then establishes lifelong latency in cranial nerves and dorsal root ganglia. In up to one third of cases, VZV may re-activate later in life to produce shingles (herpes zoster).

Varicella occurs when a susceptible person is exposed to varicella-zoster virus (VZV) either by direct contact with lesions or through airborne spread from respiratory droplets.

Patients remain infectious for at least 5 days and until all lesions have crusted over. The incubation period is typically 14 days.

1213
Q

RFs for acute varicella-zoster / chicken pox

A

Age 1-9yo
Exposure
Unimmunised status
Occupational exposure

1214
Q

Sx of acute varicella-zoster / chicken pox

A

COMMON

Fever <38-41
- Secondary bacterial infection should be considered in patients with persistent (i.e., >3 days) or recurrent fever.

Vesicular rash
- The rash usually first appears centrally (on the face, scalp, or torso), before spreading to the extremities. Lesions first appear as macules and quickly develop into fluid-filled vesicles.

As the disease progresses, early lesions will begin to scab over as new peripheral lesions develop. This appearance of lesions in ‘crops’ (i.e., different stages of acuity/healing) is characteristic of varicella.

Can see vesicles on mucous membranes

OTHER COMMON
Pruritis Headache
Fatigue/malasie
Sore throat
Tachycardia
1215
Q

Ix for acute varicella-zoster / chicken pox

A

Clinical

Ultrasound is recommended for all women who develop varicella in pregnancy, to screen for fetal consequences of infection.

Could do:
PCR
Viral culture
Tzanck smear 
Latex agglutination
ELISA
Complement fixation 
US - pregnant women -
1216
Q

Rx of acute varicella-zoster / chicken pox

A

Low risk - supportive, paracetamol, ibuprofen

Can give dipenhydramine oral/topical to aid itching BUT -> Antihistamine treatment for varicella in children has been associated with ataxia, urinary retention, and other adverse effects. In addition, a warning has been issued against the use of some cough and cold medicines (many of them antihistamines) in children under the age of 2 years. Risks may outweigh benefits in young children.

MODERATE RISK

Those at moderate risk of severe disease include: patients 13 years of age and over; those with concurrent chronic skin disease (e.g., atopic dermatitis); those with concurrent underlying pulmonary disease; those receiving salicylate therapy; and those receiving short course or intermittent oral corticosteroids.
ORAL ACICLOVIR

HIGH RISK / SEVERE DISEASE

Those at high risk of severe disease include: immunocompromised patients (e.g., organ transplant, chemotherapy, HIV infection); neonates; those taking chronic oral corticosteroids or high-dose systemic immunosuppressants; pregnant women.
IV ACICLOVIR

NB ABx can be considered if evidence of bacterial infection at sites of vesicles

1217
Q

Complications of acute varicella-zoster / chicken pox

A

Secondary bacterial infection - Staphylococcus aureus and group A streptococcus

Varicella caused:

  • Pneumonitis
  • Encephalitis
  • Cerebral ataxia
  • Meningitis
  • IC vasculitis
  • Hepatitis

SEVERE infection in newborn
Congential varicella syndrome - Presents as cicatricial skin lesions, limb hypoplasia or paresis, microcephaly, and ophthalmic lesions. Highest risk 13-20 weeks gestation
Reye’s syndrome - Associated with the use of aspirin or other salicylates. Presents with vomiting, encephalopathy, and metabolic disturbances such as hyperammonaemia and elevated liver enzymes.
Herpes Zoster

1218
Q

Prognosis of acute varicella-zoster / chicken pox

A

Typically, varicella is a self-limiting disease. After initial infection and clinical syndrome, no follow-up is necessary. In up to one third of infected people, varicella-zoster virus reactivates later in life as shingles or herpes zoster.

1219
Q

A 36-year-old man undergoing chemotherapy for non-Hodgkin’s lymphoma presents with fever, shortness of breath, haemoptysis, and a diffuse rash. His family recalls that he had a fever the previous day, and that the rash started on his chest and progressed rapidly. In a review of recent exposures, his wife recounts that she was told that a child who visited their home later developed ‘chickenpox’. His current medications are levofloxacin and an antidepressant. A review of his medical history indicates negative serological tests for varicella-zoster virus prior to starting chemotherapy, and his family does not recall him receiving the varicella vaccine. On examination he has a temperature of 40.1°C (104.2°F), a heart rate of 145 bpm, and an O2 saturation of 83%. Lung examination demonstrates bilateral crackles, and the patient has diffuse vesicular lesions, some of which appear to be haemorrhagic. Initial laboratory testing indicates a low haematocrit and platelets, a low absolute lymphocyte count (<100 cells/mL), and mild transaminitis. A chest x-ray demonstrates ground glass opacities or diffuse small nodular infiltrates.

A

acute varicella-zoster / chicken pox

1220
Q

Define hand-foot-and-mouth disease

A

A common childhood viral infection that is most commonly caused by a coxsackievirus, characterised by low-grade fever, painful oral ulcers, and vesicles on the hands and feet. The disease has no relation to foot-and-mouth disease, which affects cattle and other cloven-hoofed animals.

Diagnosis is usually clinical, based on a typical history and characteristic clinical features.

Infection typically resolves spontaneously within 10 to 14 days, and treatment is mainly supportive.

In East and South-east Asian countries, enterovirus 71 (EV71) is responsible for a more severe version of the disease with serious complications; however, this is uncommon in the US and Europe

1221
Q

Epidemiology of hand-foot-and-mouth disease

A

Coxsackievirus infection is extremely contagious. It affects males and females equally and is most common in children <10 years of age. However, infection in older children, adolescents, and adults can occur. Epidemics due to the more virulent strains of enterovirus 71 have occurred most often in East and South-east Asian countries such as Taiwan and Malaysia. These have resulted in severe illness with significant associated morbidity and mortality.

1222
Q

Aetiology of hand-foot-and-mouth disease

A

Hand-foot-and-mouth disease is due to infection by some of the enteroviruses.

Coxsackie A16 infection is the most common cause, but A4 through to A7, A9, and A10 infections also occur.

The infection occurs when a susceptible person is exposed to the virus by means of direct contact with nose and throat discharges, saliva, vesicle fluid, or faecal material from an infected person.

The virus may persist in faecal material for up to 1 month. After contact, the virus spreads to regional lymph nodes within 24 hours and viraemia rapidly follows, with spread of the virus to the oral mucosa and skin causing the vesicular rash. The incubation period is 4 to 7 days; however, there may be a prodromal period of 3 to 4 days. Lesions in the mouth heal within 1 week, and lesions on the hands and feet may last for up to 10 days.

The genome is made of a single-stranded linear molecule of RNA.

1223
Q

RFs of hand-foot-and-mouth disease

A

STRONG
Immunosuppression
Age <10
Family/school contacts

Infection is spread by direct contact with nose and throat discharges, saliva, vesicle fluid, or faecal material from an infected person. The virus may persist in faecal material for up to 1 month.

1224
Q

Sx of hand-foot-and-mouth disease

A

COMMON
RFs
Low grade fever <38.5
Oral vesicles - The oropharynx is inflamed, with scattered papules, macules, or vesicles, or more commonly yellowish ulcers on an erythematous base are present on the tongue, pharynx, buccal mucosa, gingiva, and occasionally the lips. Lesions heal within 1 week.

Rash/vesicles on hands and feet - The rash consists of small oval or linear grey-white vesicopustules. The vesicles are flaccid and thin-walled, with an erythematous halo. They may occasionally be painful or pruritic. They tend to ulcerate and become crusted. Lesions may last up to 10 days.

OTHER
Malaise
Sore mouth
Loss of appetite
Sore throat
UNCOMMON
Rash/vesicles on buttocks
Abdominal pain
Diarrhoea
Cough
Tongue erythema
Arthralgia
1225
Q

Ix for hand-foot-and-mouth disease

A

Viral culture - Only used if exposure to enterovirus 71 (EV71) is a possibility (e.g., exposure in East or South-east Asia).

FBC - Only used if exposure to enterovirus 71 (EV71) is a possibility (e.g., exposure in East or South-east Asia) -elevated WBC count, atypical lymphocyte

1226
Q

Rx of hand-foot-and-mouth disease

A

Analgesia/antipyretics
Lidocaine topical
Maintaining adequate fluid and nutritional intake is important but may be difficult in patients with oral ulcers.

Evidence SE Asia travel + severe:
The complications that can occur with enterovirus 71 (EV71) infection include aseptic meningitis, encephalitis, encephalomyelitis, pulmonary oedema, pulmonary haemorrhage, myocarditis, a polio-like syndrome, and death.

1227
Q

Complications of hand-foot-and-mouth disease

A

Volume depletion
Secondary bacterial infection

EV71 associated:
polio like syndrome
Aseptic meningitis
Myocarditis
Pulmonary oedema/haemorrhage
Congenital E71 infection
Spontaneous miscarriage
1228
Q

Prognosis of hand-foot-and-mouth disease

A

Spontaneous recovery with no complications is normal, and therefore follow-up is not usually required. However, infection with enterovirus 71, which occurs most often in East and South-east Asian countries, has caused more severe illness, with significant associated morbidity and mortality; one study from Taiwan found that 4% of patients had long-term sequelae, and death occurred in 8% of patients.

1229
Q

Define hernias in children

A

Inguinal hernia is a type of ventral hernia that occurs when an intra-abdominal structure, such as bowel or omentum, protrudes through a defect in the abdominal wall. Most hernias that are present at birth or in childhood are indirect inguinal hernias. Other less common types of ventral hernias include umbilical, epigastric, and incisional hernias.

1230
Q

Epidemiology of hernias in children

A

Although the exact incidence of indirect inguinal hernia in infants and children is unknown, the reported incidence ranges from 1-5%. Sixty percent of hernias occur on the right side. Premature infants are at increased risk for inguinal hernia, with incidence rates of 2% in females and 7-30% in males. Approximately 5% of all males develop a hernia during their lifetime.

Inguinal hernia appears to occur equally among races. Umbilical hernias, on the other hand, appear to be more common in blacks than in other races.

Increased in mal

1231
Q

Aetiology of hernias in children

A

The cause of inguinal hernia in children can be termed an abnormality of embryologic development of the fetus. However, some children may present with an acquired form of inguinal hernia, also called a direct inguinal hernia. In this type of hernia, weakness of the inguinal floor is present, which allows for protrusion of viscera from the abdominal cavity. The hernia sac is composed of the peritoneal fold that contains the hernia.
Anatomically speaking, indirect and direct inguinal hernias differ in that the direct hernia bulges through the inguinal floor medial to the inferior epigastric vessels and the indirect hernia arises lateral to the inferior epigastric vessels. Either hernia may cause fullness or a palpable bulge in the inguinal region, and distinguishing between the two types on the basis of physical examination findings may be difficult. The clinician may assume, until proven otherwise, that the pediatric patient with an inguinal hernia has indirect inguinal hernia.

The following are associated with an increased risk of inguinal hernia:
Prematurity and low birth weight (Incidence approaches 50%.)
Urologic conditions
Cryptorchidism
Hypospadias
Epispadias
Exstrophy of the bladder
Ambiguous genitalia
Patent processus vaginalis, which may be present because of increased abdominal pressure due to ventriculoperitoneal shunts, peritoneal dialysis, or ascites
Abdominal wall defects
Gastroschisis
Omphalocele
Family history
Meconium peritonitis
Cystic fibrosis
Connective tissue disease
Mucopolysaccharidosis
Congenital dislocation of the hip
Ehlers-Danlos syndrome
Marfan syndrome
Cloacal exstrophy
Fetal hydrops
Liver disease with ascites
Ventriculoperitoneal shunting for hydrocephalus
1232
Q

RFs for hernias in children

A

Prematurity = increased risks
MALE

The following are associated with an increased risk of inguinal hernia:
Prematurity and low birth weight (Incidence approaches 50%.)
Urologic conditions
Cryptorchidism
Hypospadias
Epispadias
Exstrophy of the bladder
Ambiguous genitalia
Patent processus vaginalis, which may be present because of increased abdominal pressure due to ventriculoperitoneal shunts, peritoneal dialysis, or ascites
Abdominal wall defects
Gastroschisis
Omphalocele
Family history
Meconium peritonitis
Cystic fibrosis
Connective tissue disease
Mucopolysaccharidosis
Congenital dislocation of the hip
Ehlers-Danlos syndrome
Marfan syndrome
Cloacal exstrophy
Fetal hydrops
Liver disease with ascites
Ventriculoperitoneal shunting for hydrocephalus
1233
Q

Sx of hernias in children

A
Bulge at internal / external ring / scrotum
Visible swelling 
May be intermittent
Pain/discomfort or not
Commonly occurs after crying/straining
May resolve in sleeping/atnight
May be delayed descent of a testicle 

INCARCERATED: Fussy/poor feeding/inconsolable
Erythematous/discolouration

In cases of incarceration, ischemic necrosis develops, and intestinal perforation may result, representing a true medical emergency. When an incarceration is encountered, an attempt should be made to reduce it manually if the patient has no signs of systemic toxicity (eg, leukocytosis, severe tachycardia, abdominal distention, bilious vomiting, discoloration of the entrapped viscera).

Hernia and hydrocele: In boys, differentiating between a hernia and a hydrocele is not always easy. Transillumination has been advocated as a means of distinguishing between the presence of a sac filled with fluid in the scrotum (hydrocele) and the presence of bowel in the scrotal sac.

Silk sign: When the hernia sac is palpated over the cord structures, the sensation may be similar to that of rubbing 2 layers of silk together. This finding is known as the silk sign and is highly suggestive of an inguinal hernia.

Femoral hernia: A femoral hernia can be very difficult to differentiate from an indirect inguinal hernia. Its location is below the inguinal canal, through the femoral canal. The differentiation is often made only at the time of operative repair, once the anatomy and relationship to the inguinal ligament is clearly visualized. The signs and symptoms for femoral hernias are essentially the same as those described for indirect inguinal hernias.

1234
Q

Ix for hernias in children

A

No laboratory studies are needed in the assessment of a patient with a suspected inguinal hernia and/or hydrocele.

Imaging studies are generally not indicated to assess for inguinal hernia. However, ultrasonography can be helpful in the assessment of selected patients.

1235
Q

Rx of hernias in children

A

Generally, a surgical consultation should be made at the time of diagnosis, and repair (on an elective basis) should be performed very soon after the diagnosis is confirmed. Parents may be instructed on the application of gentle pressure on the bulge of an inguinal hernia to prevent incarceration until the elective operative repair is performed.

Umbilical hernias: Most umbilical hernias do not cause any symptoms and do not require surgical repair until approximately age 5 years. For that reason, almost all umbilical hernias in young children and infants are managed by simple observation.

1236
Q

Prognosis/complication of hernias in childrens

A

An incarcerated or strangulated inguinal hernia can result in severe complications and even death. An incarcerated or strangulated inguinal and/or femoral hernia may also result in significant sequela, depending on which visceral structure is involved in the hernia sac. Such sequela can range from life-threatening complications to gonadal dysfunction, including intestinal necrosis and perforation, intestinal obstruction, intestinal stricture, testicular necrosis, testicular atrophy, ovarian necrosis, ovarian atrophy, and tubal stricture.

1237
Q

Define HIV in child

A

Since the first cases of human immunodeficiency virus (HIV) infection were identified, the number of children infected with HIV has risen dramatically in developing countries, the result of an increased number of HIV-infected women of childbearing age in these areas. HIV is a retrovirus and can be transmitted vertically, sexually, or via contaminated blood products or IV drug abuse. Vertical HIV infection occurs before birth, during delivery, or after birth.

1238
Q

Aetiology of HIV in child

A

HIV can be transmitted vertically, sexually, or via contaminated blood products or IV drug abuse. Vertical HIV infection occurs before birth, during delivery, or after birth. With infection before birth (period 1), the fetus can be hematologically infected by means of transmission across the placenta or across the amniotic membranes, especially if the membranes are inflamed or infected.

Most vertical infections occur during delivery (period 2), and many factors affect the risk of infection during this period (see Deterrence/Prevention). In general, the longer and the greater amount of contact the neonate has with infected maternal blood and cervicovaginal secretions, the greater the risk of vertical transmission. Premature and low-birthweight neonates appear to have an increased risk of infection during delivery because of their reduced skin barrier and immunologic defenses.

Postnatal vertical transmission (period 3) occurs with the ingestion of HIV in the breast milk.

1239
Q

RFs for HIV in child

A

Infected mother

1240
Q

Sx of HIV in child

A

Unusually frequent and severe occurrences of common childhood bacterial infections, such as otitis media, sinusitis, and pneumonia
Recurrent fungal infections, such as candidiasis (thrush), that do not respond to standard antifungal agents: Suggests lymphocytic dysfunction
Recurrent or unusually severe viral infections, such as recurrent or disseminated herpes simplex or zoster infection or cytomegalovirus (CMV) retinitis; seen with moderate to severe cellular immune deficiency
Growth failure
Failure to thrive
Wasting
Failure to attain typical milestones: Suggests a developmental delay; such delays, particularly impairment in the development of expressive language, may indicate HIV encephalopathy
Behavioral abnormalities (in older children), such as loss of concentration and memory, may also indicate HIV encephalopathy

Candidiasis: Most common oral and mucocutaneous presentation of HIV infection
Thrush in the oral cavity and posterior pharynx: Observed in approximately 30% of HIV-infected children
Linear gingival erythema and median rhomboid glossitis
Oral hairy leukoplakia
Parotid enlargement and recurrent aphthous ulcers
Herpetic infection with herpes simplex virus (HSV): May manifest as herpes labialis, gingivostomatitis, esophagitis, or chronic erosive, vesicular, and vegetating skin lesions; the involved areas of the lips, mouth, tongue, and esophagus are ulcerated
HIV dermatitis: An erythematous, papular rash; observed in about 25% of children with HIV infection
Dermatophytosis: Manifesting as an aggressive tinea capitis, corporis, versicolor, or onychomycosis
Pneumocystis jiroveci (formerly P carinii) pneumonia (PCP): Most commonly manifests as cough, dyspnea, tachypnea, and fever
Lipodystrophy: Presentations include peripheral lipoatrophy, truncal lipohypertrophy, and combined versions of these presentations; a more severe presentation occurs at puberty
Digital clubbing: As a result of chronic lung disease
Pitting or nonpitting edema in the extremities
Generalized cervical, axillary, or inguinal lymphadenopathy

1241
Q

Ix for HIV in child

A

Because of the persistence of the maternal HIV antibody, infants younger than 18 months require virologic assays that directly detect HIV in order to diagnose HIV infection
Preferred virologic assays include HIV bDNA polymerase chain reaction (PCR) and HIV RNA assays. The HIV PCR DNA qualitative test is usually less expensive.
Further virologic testing in infants with known perinatal HIV exposure is recommended at 2 weeks, 4 weeks, and 4 months
An antibody test to document seroreversion to HIV antibody–negative status in uninfected infants is no longer recommended.

In older children and adults, an enzyme-linked immunosorbent assay (ELISA) to detect HIV antibody, followed by a confirmatory Western blot (which has increased specificity), should be used to diagnose HIV infection.
Rapid HIV tests, which provide results in minutes, simplify and expand the availability of HIV testing. Their sensitivity is as high as 100%, but they must be followed with confirmatory Western blotting or immunofluorescence antibody testing, as with conventional HIV antibody tests.

1242
Q

Rx of HIV in child

A

Combination ART with at least 3 drugs from at least 2 classes of drugs is recommended for initial treatment of infected infants, children, and adolescents because it provides the best opportunity to preserve immune function and delay disease progression. Drug combinations for initial therapy in ART-naive children include a backbone of 2 NRTIs plus 1 NNRTI or 1 PI.

Nucleoside or nucleotide reverse transcriptase inhibitors (NRTIs)
Protease inhibitors (PIs)
Nonnucleoside reverse transcriptase inhibitors (NNRTIs)
Fusion inhibitors
CCR5 coreceptor antagonists (entry inhibitors)
HIV integrase strand transfer inhibitors

In addition to antiretroviral drugs (ARDs), other types of medication are required as appropriate for specific infections or malignancies. For example, P jiroveci pneumonia prophylaxis is recommended in patients who are HIV positive and younger than 1 year and in older children based on CD4+ counts.

1243
Q

Prognosis of HIV in child

A

The nutritional status of the child and the diligence with which viral replication is controlled are paramount in determining the outcome of most children with HIV disease.
Aggressive treatment of opportunistic infections prevents the more deleterious effects of secondary disease from progressing and further weakening the patient. The social setting and the stressors to which children are exposed have also been linked to the progression of the disease.

Hematologic disturbances, such as anemia, thrombocytopenia, and neutropenia, increase the risk of complications and death. Resolution of anemia improves the prognosis, and treatment of anemia with erythropoietin improves survival. Neutropenia significantly increases the risk of bacterial infection, and treatment of neutropenia with granulocyte colony-stimulating factor substantially decreases the risk of bacteremia and death.
Infection with Mycobacteriumavium complex (MAC) hastens death, especially in patients with coexisting anemia (defined as a hematocrit < 25%).

1244
Q

Define paediatric UC

A

Ulcerative colitis (UC) is a disease characterized by remitting and relapsing inflammation of the large intestine.

Many patterns of presentation are possible within the pediatric age group. The hallmark symptoms of UC include abdominal cramping, diarrhea, and bloody stools, but physical symptoms vary with extent, duration, and severity of the disease. UC affects the rectum, with contiguous involvement that can include the entire large intestine.

Extraintestinal manifestations of UC, such as joint pain, ophthalmic conditions, and hepatobiliary disease may occur in some patients.

1245
Q

Epidemiology of paediatric UC

A

2 per 100k age 10-19

20-25% of all cases UC occur in persons aged 20 years or younger and the disease is increasing among young children

UC incidence peaks between adolescence and early adulthood (ie, in people aged 15-30 y). A smaller peak occurs in patients aged 60-80 years.

1246
Q

Aetiology of paediatric UC

A

Unclear

UC is probably an autoimmune disease initiated by an inflammatory response to colonic bacteria. Immune dysfunction has been postulated as a cause, with limited evidence. UC might also be linked to diet, although diet is thought to play a secondary role. Food or bacterial antigens might affect the already damaged mucosal lining, which has increased permeability.

1247
Q

RFs fo paediatric UC

A

STRONG
FHx
HLA-B27
Infection

WEAK
NSAIDS
Not smoking/former smoker

1248
Q

Sx of paediatric UC

A

Rectal bleeding
Diarrhoea
Abdominal pain

Mild disease is observed in 50-60% of patients. This presentation involves insidious onset of diarrhea, later associated with hematochezia. No systemic findings of fever, weight loss, or hypoalbuminemia are observed. Mild UC is typically confined to the distal colon and responds well to therapy.
Moderate disease is observed in 30% of patients and is characterized by bloody diarrhea, cramps, urgency to defecate, and abdominal tenderness. Associated systemic findings, such as anorexia, weight loss, low-grade fever, and mild anemia, are present.
Severe disease occurs in approximately 10% of patients. This presentation involves more than 6 bloody stools per day, abdominal tenderness, fever, anemia, leukocytosis, and hypoalbuminemia. Patients with severe colitis may experience life-threatening complications, including severe hemorrhage, toxic megacolon, or intestinal perforation.

Less than 5% of children with UC present with predominantly extraintestinal manifestations, such as growth failure, arthropathy, skin manifestations, or liver disease.

Abdo tenderness
Associated liver signs if involvement
Arthralgia

Unlike what is seen in some CD patients, perianal examination in UC patients should not reveal any evidence of fistulae or abscesses; chronic diarrhea may lead to perianal erythema, fissuring, or hemorrhoids, however.
Sexual development may be delayed in patients with UC, but this finding is more common in patients with CD.

1249
Q

Ix for paediatric UC

A

Colonoscopy
(opportunity for “pill cam”)
- Continuous
- Mucosa typically appears erythematous, friable, and granular and has lost the normally visible vascular markings
Biopsy - polymorphonuclear leukocytes near the base of the crypts. Cryptitis describes aggregation of polyps in the crypt epithelium, and the term crypt abscess is used when polyps have accumulated in the lumen of the crypt.

ANCA assay results are positive in 60-80% of UC patients. (mainly pANCA)

FBC - ID anaemia

Stool culture - rule out infection

ESR + CRP - high

Can do:
Upper GI series, barium enema, CT, MRI - but these are all contraindicated if colonoscopy is conclusive

1250
Q

Rx of paediatric UC

A

MILD
Sulfsalazine/mesalamine
- Oral or suppository
Probiotics

MOD-SEVERE
Oral/IV Corticosteroids can be given short term - not indicated LT
Azathioprine or 6-mercaptopurine useful if steroid-dependent [SEs of pancreatitis, hepatitis, and bone marrow suppression]

FULMINANT
Treatments for fulminant disease include cyclosporine (nephrotoxic), tacrolimus (nephrotoxicity, bone marrow suppression, and neurologic symptoms), and infliximab (hepatosplenic T-cell lymphoma).

Total proctocolectomy

In children, elective colectomy is indicated when refractory disease significantly interferes with their growth and nutrition or with their ability to maintain a normal lifestyle (ie, attend school) or when dysplasia or malignancy is detected.

1251
Q

Prognosis / complications of paediatric UC

A

Although many children with UC have a mild course and respond well to therapy, severe complications can arise in patients with ulcerative colitis during acute exacerbations of disease or as the disease progresses.

Toxic megacolon
Colonic malignancy 
Pyoderma gangrenous 
Uveitis, arthritis
PSC
Cholelithiasis 
Thromboembolic disease
1252
Q

Define paediatric crohns

A

Crohn disease, a chronic inflammatory bowel disease (see the image below) that was once considered rare in the pediatric population, is currently recognized as one of the most important chronic diseases that affect children and adolescents. Approximately 20-30% of all patients with Crohn disease present when they are younger than 20 years. In addition to the common GI symptoms (diarrhea, rectal bleeding, and abdominal pain), children with Crohn disease often experience growth failure, malnutrition, pubertal delay, and bone demineralization.

1253
Q

Epidemiology of paediatric crohns

A

Incidence of 3.5-100k in 10-19yos
IN PAEDS M>F

Crohn disease is more common in whites than in blacks and is rare in Asian and Hispanic children. Rates are higher in people of Jewish descent, particularly Ashkenazi Jews and Jews of middle European origin compared with Sephardic or eastern European Jews.

1254
Q

Aetiology of paediatric crohns

A

The pathogenesis of Crohn disease is multifactorial. After a triggering event occurs in a genetically susceptible individual, an altered immune response leads to chronic inflammation of the intestine. Although the etiology of the precipitating event is unknown, luminal bacteria or specific antigens are thought to be involved.

1255
Q

RFs for paediatric crohns

A

STRONG
White ancestry
15-40 or 60-80
FHx

WEAK
Cigarette smoking
High refined sugar diet
OCP
Not breast fed
NSAIDS
1256
Q

Sx of paediatric crohns

A

With upper GI tract involvement, nausea, vomiting, and abdominal pain dominate as presenting symptoms.
Children with Crohn disease of the small intestine usually present with evidence of malabsorption, including the following:
Diarrhea
Abdominal pain
Growth deceleration
Weight loss
Anorexia
Initially, manifestations of malabsorption may be quite subtle. The onset of growth failure is usually insidious and may precede GI symptoms by years.
Colonic Crohn disease may be clinically indistinguishable from ulcerative colitis (UC), with manifestations that include the following:
Bloody mucopurulent diarrhea
Cramping abdominal pain
Urgency to defecate
Perianal involvement in Crohn disease may produce the following:
Simple skin tags, fissures, abscesses, and fistulae
Painful defecation
Bright-red rectal bleeding
Perirectal pain, erythema, or discharge
Physical examination findings
Vital signs are usually normal, although tachycardia may be present in anemic patients
Chronic intermittent fever is a common presenting sign
Body weight and height may reveal weight loss and growth delay
The most sensitive indicator of growth abnormalities is a decrease in growth velocity, which may be observed before the major percentile lines on standard growth curves are crossed
Abdominal findings may vary from normal to those of an acute abdomen; diffuse abdominal tenderness is common; fullness or a discrete mass may be palpable, typically in the right lower quadrant
Perianal disease (eg, skin tags, abscesses, fistulae, fissures) is present in approximately 45% of patients
Tanner staging may indicate pubertal delay, which may precede the onset of intestinal symptoms
The most common cutaneous manifestations are erythema nodosum and pyoderma gangrenosum
Skin examination may also reveal pallor in patients with anemia or jaundice in those with concomitant liver disease
The most common ocular findings are episcleritis and anterior uveitis
The most common extraintestinal manifestations of Crohn disease are arthritis and arthralgia, typically involving the large joints (eg, hips, knees, ankles)

1257
Q

Ix for paediatric crohns

A

Laboratory data for Crohn disease are nonspecific, as follows:
The CBC may show hypochromic microcytic anemia, from iron deficiency due to GI blood loss, or normocytic anemia of chronic disease
levels of acute-phase reactants (ESR and CRP) are often elevated in patients with Crohn disease but may be normal
Hypoalbuminemia is common
Additional common deficiencies include iron and micronutrients (eg, folic acid, vitamin B-12, serum iron, total iron binding capacity, calcium, and magnesium)
Stool studies should be obtained to rule out bacterial or parasitic infection
Excretion of fecal calprotectin is increased with colorectal inflammation [1] ; on ELISA, the cutoff level is 50 µg/g feces.

Endoscopy
Colonoscopy with several colonic and terminal ileal biopsies is considered a standard diagnostic procedure
Upper endoscopy, or esophagogastroduodenoscopy (EGD), should be part of the first-line investigation
Video capsule endoscopy is increasingly being used to evaluate for small-bowel Crohn disease in children [4]

Imaging studies
A single-contrast upper GI radiologic series with small-bowel follow-through (SBFT) can be used to evaluate the small intestine, which cannot be reached during endoscopy
Magnetic resonance enterography (MRE) [2] and computed tomography enterography (CTE) are as sensitive and specific as SBFT for detection of small bowel inflammation and may be more accurate for detection of extraenteric complications, including fistulae and abscesses [3]
MRI is especially useful in the evaluation of pelvic and perianal disease
Abdominal ultrasonography (US) can be used to investigate intestinal disease and to rule out gallbladder and kidney stones

The microscopic findings in intestinal biopsy samples from pediatric patients with Crohn disease consist of edema, inflammation (mononuclear and polymorphonuclear), cryptitis and crypt abscesses, architectural crypt changes, and transmural extension of the inflammation (see the image below).

The presence of granulomas may be helpful in differentiating between UC and Crohn disease, but granulomas are present in only about 30% of biopsy specimens obtained from patients with Crohn disease.

1258
Q

Rx of paediatric crohns

A

tep-up approach
Patients with mild disease are treated with preparations of 5-aminosalicylic acid (5-ASA), antibiotics, and nutritional therapy
If no response occurs or if the disease is more severe than was initially thought, corticosteroid and immunomodulatory therapy with 6-mercaptopurine (6-MP) or methotrexate (MTX) is i
Infliximab is effective in patients who have an inadequate response to conventional therapy and in patients who have fistulizing Crohn disease [5]
Adalimumab is a safe and effective substitute for patients who are allergic to infliximab or develop high titers of human antichimeric antibodies (HACA) [6]
Surgery is considered when medical therapy fails

Indications for surgery include the following:
Intractable disease with growth failure
Obstruction or severe stenosis
Abscess requiring drainage
Perianal fistulae
Intractable hemorrhage
Perforation
Features of surgery are as follows:
Recurrence of disease at the anastomotic site is common after resection
Surgical treatment for Crohn disease is not curative
Laparoscopic techniques are becoming the standard of care

1259
Q

Complications / prognosis of paediatric crohns

A

Although Crohn disease may have a sizable effect on the life of a child or adolescent, with appropriate treatment and support, the prognosis is good, and the risk of a fatal outcome is extremely low.

Death from Crohn disease is extremely rare in children and adolescents. Severe and complicated Crohn disease may result in prolonged hospitalizations, multiple surgical procedures, growth failure, malnutrition, pubertal delay, and poor quality of life.

1260
Q

Define NEC

A

Necrotizing enterocolitis (NEC), which typically occurs in the second to third week of life in premature, formula-fed infants, is characterized by variable damage to the intestinal tract, ranging from mucosal injury to full-thickness necrosis and perforation (see the image below). NEC affects close to 10% of infants who weigh less than 1500 g, with mortality rates of 50% or more depending on severity, but may also occur in term and near-term babies.

1261
Q

Epidemiology of NEC

A

PRETERM - Presents during first several weeks of birth. Usually onset = when would have been born.

black > white

Incidence = 0.3-2.4 cases per 1000 live births

1262
Q

Aetiology of NEC

A

Although the exact etiology of necrotizing enterocolitis (NEC) remains unknown, research suggests that it is multifactorial; ischemia and/or reperfusion injury, exacerbated by activation of proinflammatory intracellular cascades, may play a significant role.

Cases that cluster in epidemics suggest an infectious etiology. Gram-positive and gram-negative bacteria, fungi, and viruses have all been isolated from affected infants; however, many infants have negative culture findings.

1263
Q

RFs for NEC

A

Prematurity
Anything that can cause ischaemia IE Maternal HTN/Pre-eclampsia/cocaine OR PDA/congenital HD

Numerous medications have been implicated as a risk factor in NEC. Xanthine derivatives, such as theophylline and aminophylline, slow gut motility and produce oxygen free radicals during their metabolism to uric acid. Indomethacin, used to treat patent ductus arteriosus, may cause splanchnic vasoconstriction leading to impaired intestinal integrity. Vitamin E, used to treat retinopathy of prematurity, is known to impair leukocyte function and has been associated with NEC. Inhibitors of gastric acid secretion alter the pH of the intestinal milieu, which subsequently affects the intestinal flora.

1264
Q

Sx of NEC

A

Initial symptoms may be subtle and can include 1 or more of the following:
Vomiting
Diarrhea
Delayed gastric emptying
Abdominal distention, abdominal tenderness, or both
Ileus/decreased bowel sounds
Abdominal wall erythema (advanced stages)
Hematochezia
Systemic signs are nonspecific and can include any combination of the following:
Apnea
Lethargy
Decreased peripheral perfusion
Shock (in advanced stages)
Cardiovascular collapse
Bleeding diathesis (consumption coagulopathy)
Physical findings in patients with NEC can be primarily GI, primarily systemic, indolent, fulminant, or any combination of these. Gastrointestinal signs can include any or all of the following:
Increased abdominal girth
Visible intestinal loops
Obvious abdominal distention and decreased bowel sounds
Change in stool pattern
Hematochezia
Palpable abdominal mass
Erythema of the abdominal wall
Systemic signs can include any of the following:
Respiratory failure
Decreased peripheral perfusion
Circulatory collapse

1265
Q

Ix for NEC

A

WBC – Moderate to profound neutropenia (absolute neutrophil count [ANC] < 1500/μL) strongly suggests established sepsis
Hematocrit and hemoglobin – Blood loss from hematochezia and/or a developing consumptive coagulopathy can manifest as an acute decrease in hematocrit; an elevated hemoglobin level and hematocrit may mark hemoconcentration due to notable accumulation of extravascular fluid
Platelet count – Thrombocytopenia may be present

Other laboratory findings
Blood culture is usually negative
Hyponatremia – An acute decrease in serum sodium (< 130 mEq/dL) is alarming
Low serum bicarbonate (< 20) may be seen in babies with poor tissue perfusion, sepsis, and bowel necrosis
Reducing substances may be identified in the stool of formula-fed infants
A breath hydrogen test may be positive
Arterial blood gas levels may indicate the infant’s need for respiratory support and can provide information on the acid-base status

Abdominal radiography

An AP and a left lateral decubitus view are essential for initial evaluation
Should be performed serially at 6-hour or greater intervals, depending on presentation acuity and clinical course, to assess disease progression

Characteristic findings on AP views include an abnormal gas pattern, dilated loops, and thickened bowel walls
A fixed and dilated loop that persists over several examinations is especially worrisome
Scarce or absent intestinal gas is more worrisome than diffuse distention that changes over time

Other radiographic findings include the following:
Pneumatosis intestinalis – Pathognomic of NEC
Abdominal free air – Ominous; patients usually require emergency surgical intervention
Portal gas – A poor prognostic sign
Distended loops of small bowel – Common but nonspecific
Intraperitoneal free fluid

Can do USS but very operator dependent

1266
Q

Rx of NEC

A

Stop enteral feedings
Perform nasogastric decompression
Initiate broad-spectrum antibiotics (eg, ampicillin, gentamicin, and clindamycin or metronidazole)

Bell stages IA and IB – suspected disease
NPO diet and antibiotics for 3 days
IV fluids, including total parenteral nutrition (TPN)

Bell stages IIA and IIB – definite disease
Support for respiratory and cardiovascular failure, including fluid resuscitation
NPO diet and antibiotics for 14 days
Consider surgical consultation
After stabilization, provide TPN while the infant is NPO

Bell stage IIIA – advanced disease
NPO for 14 days
Fluid resuscitation
Inotropic support
Ventilator support
Obtain surgical consultation
Provide TPN during the period of NPO
Surgical intervention
1267
Q

Complications/prognosis of NEC

A

The mortality rate in NEC ranges from 10% to more than 50% in infants who weigh less than 1500 g, depending on the severity of disease, compared with a mortality rate of 0-20% in babies who weigh more than 2500 g. Extremely premature infants (1000 g) are still particularly vulnerable, with reported mortality rates of 40-100%. One study comparing mortality rates for term versus preterm infants reported rates of 4.7% for term infants and 11.9% for premature babies.

The improvement in treatment efficacy in infants with NEC is underscored by the fact that if patients with pan involvement are excluded, the survival in surgically treated infants with NEC is 95%.

The 2 most common complications are intestinal stricture and short-gut syndrome.

Also:
Cholestatic liver disease
Recurrent NEC
Neurodevelopmental disorders